Divine Intervention Step 2CK Podcasts Notes - Read Only File - PDFCOFFEE.COM (2024)

Divine Intervention Step 2CK Podcasts Notes - Read Only File LAST UPDATE: October 25, 2020 Official Errata To submit any errors please use this document: Click to report errors here! Announcements (10/25/2020): ● ● ●

Apologies for the delay, I will be making updates to the main doc on November 1st. Checklists with a timestamp for each episode available here Thank you for your contributions!!

Podcast Episodes Pending: ●

Ep 269 [Ortho Series 1: Bone Tumors] ○

Ep 270 [Rapid Review Series 43] ○

Thank you! -> Google Doc - Ep 263 [Clutch CNS Infection]

Ep 264 [Clutch Comprehensive USMLE Heart Failure] ○

Thank you! -> Google Doc - Ep 262 [Clutch TB]

Ep 263 [Clutch CNS Infection] ○

Help us transcribe this episode here -> Ep 267 [USMLE & normal changes in Elderly]

Ep 262 [Clutch TB] ○

Help us transcribe this episode here -> Ep 270 [Rapid Review Series 43]

Ep 267 [USMLE & normal changes in Elderly] ○

Help us transcribe this episode here -> Ep 269 [Ortho Series 1: Bone Tumors]

Thank you! -> Google Doc - Ep 264 [Clutch Comprehensive USMLE Heart Failure]

Ep 268 [Clean SP 4 - (Palliative Care 2)] ○

Thank you! this episode here -> Ep 268 [Clean SP 4 - (Palliative Care 2) ]

Questions, comments, or feedback regarding this file? Refer to the thread on Reddit here, or reach out to u/Coffee_Beast directly on Reddit.

Common abbreviations you’ll find on the following pages: #1 RF = number one risk factor NBS = next best step MCC = most common cause MCCOD = most common cause of death Tx = treatment Dx = diagnosis MI = most important MIPF = most important prognostic factor MIRF = most important risk factor MSC = most serious complication ML = most likely MLM = most likely mechanism MLCP = most likely clinical presentation NBSIM = next best step in management POOP = pain out of proportion

Table of Contents Episode 17: Peds Diseases of The Pediatric Population Part 1.

7 1

Episode 19: Neurology Shelf review part 1

10

Episode 21: Comprehensive Pediatric 3rd Year Shelf Review

16

Ep 22 [OBGYN Shelf Review]

20

Episode 26: USMLE Radiology

37

Episode 29: 3rd Year Medicine Shelf Review Part 1

38

Episode 30: Comprehensive Medicine Shelf Review Session 2.

52

Episode 31: Comprehensive Medicine Shelf Review. Session 3.

79

Episode 32: Comprehensive Medicine Shelf Review Session 4

115

Episode 36: Ophthalmology

140

Episode 37: Risk Factors

144

Episode 41: Antibiotic Guide

146

Episode 45: Neurology Clerkship Shelf Review Part 2

153

Episode 46: Neurology Clerkship Shelf Review Part 3

159

Episode 47: Neurology Clerkship Shelf Review Part 4

162

Episode 48: Neurology Clerkship Shelf Review Part 5

165

Episode 49: Neurology Clerkship Shelf Review Part 6

171

Episode 58: Neurology Clerkship Shelf Review Part 7

179

Episode 59: Neurology Clerkship Shelf Review Part

188

Episode 65: Ventilator Physiology for the USMLEs

195

Episode 94: Rapid Review, Series 1, Peds

196

Episode 95: Rapid Review, Series 2, Peds

198

Episode 97: “Most Important ”

200

Episode 100: The Clutch Micro

201

Episode 102: The “Clutch” cancer podcast (IN PROGRESS)

206

Episode 102: The “Clutch” cancer podcast (Version 2)

210

Episode 104: ACLS, Arrhythmias, and HY Cardiac Pharm

219

Episode 111: The “Clutch” Pharmacology Podcast Round 1

221

Episode 112: The “Clutch” Pharmacology Podcast Round 2

233

Episode 118: Confusing Breast Pathologies

248

Episode 119: Rapid Review Series 3 (IM)

248

Episode 120: Rapid Review Series 4 (IM/Peds)

250 2

Episode 121: Rapid Review Series 5 (IM/Peds)

252

Episode 123: Ethics I

254

Episode 125: Rapid Review Series 6 (IM)

256

Episode 126: Rapid Review Series 7 (IM)

258

Episode 127: Rapid Review, Series 8 OB/GYN

259

Episode 128: Rapid Review Series 9 (IM)

260

Ep 129: (Targeted Pulmonary Review)

262

Episode 130 : Rapid Review Series 10 (Peds)

270

Episode 131: Rapid Review Series 11 (OB, Psych, IM, Neuro)

273

Episode 132: Weird topics of Social Sciences

275

Episode 132: Weird topics of Social Sciences (Version 2)

279

Episode 134: Rapid Review Series 12 (IM/ Cardiology)

286

Episode 135: The Clutch Electrolytes

287

Episode 137: Next Best Step In Management Series 1

293

Episode 138: The Clutch Genetic Disease

298

Episode 141: Rapid Review Series 13 (IM)

303

Episode 143: Biostats Review

308

Episode 145: Rapid Review, Series 14, Surgery

319

Episode 153: Rapid Review, Series 15, Surgery

321

Episode 156: Rapid Review, Series 16, OB/GYN

323

Episode 158: Rapid Review, Series 17, Surgery

323

Episode 159: Rapid Review Series 18 (IM)

324

Episode 161: The Clutch Antibody

327

Episode 163: Rapid Review, Series 19, Surgery (Abdomen)

330

Episode 164: The Clutch Toxicology Podcast (Drugs of abuse, overdose, withdrawal, toxidromes)

333

Episode 166: Rapid Review Series 20 (Psych)

339

Episode 167: Rapid Review Series 21 Cardio

340

Episode 169: Clutch Nephrotic/Nephritic Syndrome

343

Episode 169: Clutch Nephrotic/Nephritic Syndrome (Version 2)

345

Episode 173: The Clutch Immunodeficiency Diseases Podcast

348

Episode 174: Rapid Review Series 22 (IM)

352 3

Episode 175: Rapid Review Series 23 (Psych)

354

Episode 177: Rapid Review Series 24 (OGBYN)

356

Episode 180: Hematology

358

Episode 181: Comprehensive NBME Emergency Medicine Shelf Review Series 1

360

Episode 182: Comprehensive NBME Emergency Medicine Shelf Review Series 2

373

Episode 183: Comprehensive NBME Emergency Medicine Shelf Review Series 3

376

Episode 184: NBME weird

389

Episode 187: Rapid Review Series 25

394

Episode 189: Rapid Review Series 26

397

Episode 195: Rapid Review Series 27

401

Episode 196: Rapid Review Series 28

404

Episode 197: Bias in Biostatistics

406

Episode 198: Clutch Hypertensive Integrations.

408

Episode 199: Rapid Review Series 29

411

Episode 202: Rapid Review Series 30

414

Episode 203: Leukemia and Lymphoma

417

Episode 204: Military Part 1

419

Episode 206: Family Medicine Shelf Review Series 1

420

Episode 207: Geriatrics

425

Episode 207: Geriatrics (Version 2)

426

Episode 208: Transfusion Reactions

428

Episode 209: Family Medicine Shelf Review Series 2

430

Episode 210: Rapid Review Series 31

431

Episode 211: Rapid Review Series 32

433

Episode 212: Family Medicine Shelf Review Series 3 (GI)

436

Episode 213: Family Medicine Shelf Review Series 4 (GI)

439

Episode 214: Family Medicine Shelf Review Series 5 - GI

441

Episode 214: Family Medicine Shelf Review Series 5 - GI (Version 2)

445

Episode 215: Acetylcholine and The NBME

446

Episode 217: Family Medicine Shelf Review Series 6 - Pulm

449

Episode 217: Family Medicine Shelf Review Series 6 - Pulm (Version 2)

453 4

Episode 219: Rapid Review Series 34

454

Episode 220: Rapid Review Series 34

457

Episode 221: Trauma

461

Episode 223: NBME Peds Shelf Add-On: The HY Newborn

462

Episode 224: Genetic Diseases 2: Chromosomes

467

Episode 225: Rapid Review Series 35

472

Episode 226: Iron labs

474

Episode 227: Rapid Review Series 36 (OBGYN + Others)

475

Episode 228: CLEAN-SP 1 Palliative

477

Episode 230: CLEAN-SP 2 Quality/Safety

478

Episode 231: Military Part 2

479

Episode 232: Vasculitis

481

Episode 233: Shock

483

Episode 233: Shock (Version 2)

485

Episode 234: CLEAN-SP 2 Medication/Transition of Care

486

Episode 237: (HIV)

488

Episode 238: Rapid Review Series 37

490

Episode 239: OBGYN Risk Factors

492

Episode 240: Rapid Review Series 38 (Ortho and OBGYN)

493

Episode 242: Dermatology Part (1 of 3)

495

Episode 243: Water Soluble Vitamins

499

Ep 244: (Cardiac Valvular Disorders)

501

Ep 245: (Rapid Review Series 39)

506

Ep 246: (Derm Part 2 of 3)

508

Ep 247: (Rapid Review Series 40)

513

Ep 248: [New Free 120 Q1-10 (2020)]

516

Ep 249: (Blood Oxygen Content and the USMLEs)

519

Ep 250: [HY Vaccine]

522

Ep 251: [The HY Thyroid Podcast]

526

Ep 252 [Post Exposure Prophylaxis and the NBMEs]

530

Ep 253 [Starling Forces and The NBMEs]

532 5

Ep 255 [Legendary Step 2CK/3 Pharmacology]

533

Ep 256 [Rapid Review Series 41]

537

Ep 257 [Clutch Bilirubin Podcast]

539

Ep 258 [Rapid Review Series 42]

542

Ep 259 [Clutch Incontinence Podcast]

545

Ep 261 [USMLE Derm Part 3 of 3]

547

Ep 265 [Male Reproductive Pathologies and the USMLEs]

550

----------------------------------------------------------------------------------------------------------------------------

Episode 17: Peds Diseases of The Pediatric Population Part 1. GI PROBLEMS IN KIDDOS ● 7 yo Asian male w/ chronic watery diarrhea + stool positive for reducing sugars + positive hydrogen breath test → lactose intolerance = lactase deficiency ○ Tx: ■ Infant: soy-based formula 6

● ●

Child: Avoid lactose 23 mo w/ bowed LE (vit D def) + diffuse blistering rash over patellae (dermatitis herpetiformis) + diffuse muscle wasting + MCV 60L (iron deficiency), serum Ca+ 7.1L → celiac disease ○ Dx: ■ tTG-IgA *but IgA deficiency is common ■ If IgA deficient → check gliadin Ab (IgG) ■ Duodenal bx: flattening of microvilli, intraepithelial lymphocytes ○ Tx: ■ Avoid gluten ■ Vitamins ■ For dermatitis herpetiformis: dapsone ● Caution in G6PD deficiency IgA deficiency ○ Anaphylactic transfusion reaction (type I hypersensitivity rxn) 8 mo drinks cow milk, causes bloody stool → milk protein allergy ○ Outgrown by age 1 ○ Tx: formula with casein hydrolysate 5 yo kid w/ chronic constipation, immigrant from South America, rectal exam is accompanied by an explosive expulsion of poop → Hirschsprung's disease ○ Failure of neural crest cell migration to distal colon ○ Dx: distal colon biopsy w/ absence of Auerbach’s plexus ○ Tx: resect affected bowel, end-to-end anastomosis ○ Association: Down syndrome ○ Mimic: Trypanosoma cruzi (Chagas) destroys enteric nervous system → megacolon 18 mo w/ recent URI, now with severe abdominal pain & bloody diapers + RLQ mass → intussusception ○ Pressure necrosis of bowel if not corrected quickly ○ Lead point for telescoping: Mecklel’s diverticulum, hyperplasia of Peyer’s patches in terminal ileum ○ Dx: ■ US w/ target sign ■ Air enema (diagnostic & therapeutic) ○ Tx: surgery is air enema not effective ○ If a kid gets septic after air enema? Think bowel perforation, do XR (see free air under diaphragm), give abx & straight to OR 4 yo with 12 hrs of severe abdominal pain and n/v + WBC 13k + US w/ echogenic material (likely fecolith) in the RLQ and fat stranding ○ Dx: Appendicitis ■ How to dx: US (Kid) ○ PE findings (challenging / not always given in Peds population vignettes) ■ McBurney’s point tenderness ■ Psoas sign ■ Obturator sign (pain with internal rotation of hip) ■ Rovsing’s sign (palpation of LLQ → pain in RLQ) ○ Tx: IVF, broad-spectrum abx (e.g. amp + gent + clinda/metro OR ceftriaxone + metro), lap appy ○ Complications: abscess, perforation

GENETICS REVIEW ● Myotonic dystrophy → trinucleotide repeat disorder 7

● ● ● ● ● ●

○ CTG repeat Fragile X Syndrome → trinucleotide repeat disorder ○ CGG repeat Angelman → maternal deletion or paternal uniparental disomy (both chromosomes from dad) Prader-Willi Syndrome → paternal deletion or maternal uniparental disomy (both chromosomes from mom) Beckwith Wiedemann Syndrome → imprinting disorder of chromosome 11, can be caused by paternal uniparental disomy Trisomy 21 → maternal nondisjunction or unbalanced robertsonian translocation Mom passing a disease to all her kids → mitochondrial inheritance ○ Ex: MELAS (Mitochondrial Encephalopathy, Lactic Acidosis, and Stroke-like episodes) ○ Ex: MERRF (Myoclonic epilepsy with ragged red fibers) ○ Ex: Leber's hereditary optic neuropathy & other optic neuropathies

CHROMOSOME DISORDERS ● Down’s syndrome = Trisomy 21 ○ Genetic pathophysiology? ■ maternal nondisjunction (failure to separate in meiosis I or II) ■ Robertsonian translocation (RT) ○ Early onset neuro dz? Alzheimer dz (presenilin is on chromosome 21) ○ Classic hematologic malignancy? ALL (TdT positive) ■ “ALL fall Down” ○ Considerations before beginning sports? Cervical XR to screen for atlanto-axial instability ○ Hand findings? Single palmar crease ○ Common cardiac defect? Endocardial cushion defects (endocardial cushion derived from neural crest) ○ Common GI defects? ■ Hirschprung’s ■ duodenal atresia (bilious vomiting, double bubble) ■ pyloric stenosis (first 8 weeks of life, non-bilious vomiting, olive-shaped mass, erythromycin exposure increases risk) ○ Maternal quad screen findings ■ High beta-hcg & inhibin A ■ Low AFP & estriol ■ “HIGH” → Hcg & inhibin are high ○ How many chromosomes are found in a RT carrier (balanced RT)? 46 ○ How many chromosomes are found in a patient with Downs Syndrome from a RT (unbalanced RT)? 46 ● Prominent occiput + overlapping digits + clenched fists + rocker bottom feet → Edward syndrome (trisomy 18) ● Microcephaly + cleft lip/palate + polydactyly + rocker bottom feet → Patau syndrome (trisomy 13) ● 5 yo M with a long face, large ears, and large testicl*s → Fragile X syndrome ○ CGG repeat ○ Inheritance? X-linked, trinucleotide repeat ■ Genetic anticipation (worsens with each generation) ○ Dx: FMR1 genetic testing ○ Psych association: ADHD, Autism ○ Neuro association: mild intellectual disability, seizures ■ Most common cause of inherited ID in the US! 8

● ●

Tall male + Gynecomastia + Infertility + no facial hair → Kleinfelter syndrome ○ Karyotype? 47XXY Female with a low posterior hairline, webbed neck, congenital lymphedema or cystic hygroma, and short stature → Turner syndrome ○ Karyotype? 45X ○ Aortic pathologies? Bicuspid aortic valve → early AS, coarctation of the aorta → hypertension in arms & hypotension in legs ○ Renal pathology? Horseshoe kidney ○ How can the short stature be corrected? Give growth hormone ○ How can the development of secondary sexual characteristics be promoted at puberty? Give estrogen ○ What is the “kind” of hypogonadism associated with this disorder? hypergonadotropic hypogonadism (streak ovaries → no estrogen → no negative feedback → high LH/FSH) ○ As an aside, what is the male equivalent (similar findings) of Turner’s syndrome? Noonan syndrome ■ Is this disorder only present in males? NO! ■ What is the mechanism of inheritance? AD ■ ■

Genetic mutation? PTPN11 mutation What is one good “cardiac clue” on exams to help with differentiating this counterpart from Turner’s syndrome? RVOT problems (e.g. pulmonic stenosis)

IMPRINTING DISORDERS ● Male with short stature, obesity, small testicl*s & penis, huge appetite → Prader-Willi syndrome ○ Neonate: hypotonic, feeding difficulties, cryptorchidism ○ Paternal chromosome is usually “on”, maternal chromosome is imprinted ○ Deletion of paternal chromosome ■ “Prader has no papa” ○ Maternal uniparental disomy ● Female with ataxia, small face, uncontrollable laughter → Angelman syndrome ○ Maternal chromosome is usually “on”, maternal chromosome is imprinted ○ Deletion of maternal chromosome ■ “M” in AngelMan for Mom ○ Paternal uniparental disomy VACTERL / CHARGE / CATCH-22 ASSOCIATIONS ● VACTERL association ○ V = vertebral ○ A = anal atresia ○ C = cardiac defect ○ TE = TE fistula (often with esophageal atresia) ○ R = renal defect ○ L = limb defect ● CHARGE association ○ C = coloboma (keyhole defect in iris) ○ H = heart defect ○ A = atresia of the choanae (blue but pinks up when they cry) ○ R = retardation 9

○ G = GU anomalies ○ E = ear anomalies ● CATCH-22 association? DiGeorge syndrome = velocardiofacial syndrome ○ Failure of 3rd and 4rd pharyngeal pouches, derived from endoderm, to develop normally ○ Cardiac abnormality (commonly interrupted aortic arch, truncus arteriosus and tetralogy of Fallot) ○ Abnormal facies (e.g. micrognathia) ○ Thymic aplasia (no thymic shadow on newborn CXR) → T cell deficit → viral & fungal deficits ○ Cleft palate & cognitive deficits ○ Hypoparathyroidism → hypocalcemia → seizure and/or QT prolongation ○ 22q11 deletion ----------------------------------------------------------------------------------------------------------------------------

Episode 19: Neurology Shelf review part 1 ●

Brain tumor that causes loss of “vertical gaze” → pinealoma (causes Parinaud’s syndrome) ○ Pathophys? Compression of superior colliculus, which is the vertical conjugate gaze center

13 mo child with history of hypopigmented macules + seizures. → tuberous sclerosis ○ Hypopigmented macules = ash leaf spots ○ Associated seizure syndrome? West syndrome (infantile spasms) ■ EEG finding? Hypsarrhythmia ■ Tx? ACTH or vigabatrin

Most common primary brain tumor in kids → pilocytic astrocytoma ○ Histology? Rosenthal fibers ○ Marker? GFAP (glial cell marker)

Medulloblastoma ○ Location? Cerebellar vermis ○ Histology? Homer-Wright rosettes

Brain tumor presenting as hydrocephalus → ependymoma ○ Histology? Perivascular pseudorosettes or ependymal rosettes

Brain tumor causing visual problems in kid → craniopharyngiomas ○ Derived from? Rathke’s pouch ○ Imaging? Calcified suprasellar mass ○ Complications? ■ Bitemporal hemianopsia (2/2 optic chiasm compression) ■ Anterior pituitary hormone deficiencies (2/2 pituitary stalk/gland compression)

MC cause of brain tumor in adults → metastasis ○ Location? Grey-white junction ○ Imaging? Multiple well-circ*mscribed lesions

Most common primary brain tumor in adults → GBM ○ Imaging? Butterfly-shaped mass that crosses corpus callosum + edema + central necrosis ○ Marker? GFAP (glial cell marker) ○ MC brain tumor excluding mets 10

Bilateral acoustic neuromas ○ Pathophys? Schwannoma of CN8 ○ Location? Cerebellopontine angle ■ #1 MC tumor at cerebellopontine angle → acoustic neuroma ■ #2 MC tumor at cerebellopontine angle → meningioma ○ Tumor marker? S-100 ○ Associated syndrome? NF-2

Brain tumor that presents as parasagittal mass along falx cerebri → meningioma ○ Histology? Psammoma bodies (“laminated calcifications”)

2 yo child with abdominal mass that crosses midline + myoclonus + weird eye movements + calcified mass on imaging → neuroblastoma ○ Alternate location: posterior mediastinum ○ Associated syndrome? ■ NF-1 ■ Beckwith-Weideman syndrome ● Neuroblastoma / Wilms tumor / hepatoblastoma ● Hemihypertrophy ● Macroglossia ● Enlarged abdominal organs ● Abdominal wall defects ● Hypoglycemic seizure in newborn ○ Pathophys? hyperplasia of beta cells of pancreatic islets 2 yo child with abdominal mass that does not cross midline + no calcifications → Wilms tumor Frontal lobe tumor → oligodendroglioma ○ Histology? fried egg appearance Brain tumor that produces EPO → hemangioma blastoma ○ Associated syndrome? VHL syndrome ○ Treat because they can cause a life-threatening hemorrhage!

● ● ●

● ●

Recent viral illness + vertigo + tinnitus → labyrinthitis Feeling like room is spinning with positional changes + nystagmus with provocative maneuvers → BPPV (caused by otolith displacement) ○ Diagnostic maneuver? Dix-Hallpike ○ Tx? Epley / Semont maneuver

Anesthesia over medial thigh + weak thigh adduction → obturator nerve injury ○ Nerve roots? L2-L4

Anesthesia over lateral thigh → lateral femoral cutaneous nerve injury

7 yo kid with ataxia + hypertrophic cardiomyopathy + LE hyporeflexia → Freidrich’s ataxia ○ Pathohpys? ■ GAA repeats → LOF mutation in frataxin gene (iron binding protein) → iron overload causes oxidative damage 11

○ ○ ○

● Destruction of dorsal columns of spinal cord → hyporeflexia ● Cerebellar destruction → ataxia Inheritance? AR ■ Unlike other trinucleotide repeat disorders, which are AD Repeat? GAA MC cause of death? CHF 2/2 hypertrophic cardiomyopathy

35 yo F with a BMI of 35 with visual difficulty and severe intermittent headaches. She takes tetracycline for acne. → idiopathic intracranial hypertension (“pseudotumor cerebri”) ○ Imaging/Procedure Mgmt: ■ 1st step? Fundoscopic exam → shows papilledema ■ 2nd step? Head CT → is normal ■ 3rd step? LP → shows elevated OP ○ Tx? ■ Weight loss ■ Acetazolamide (carbonic anhydrase inhibitor) ■ Serial LPs ■ VP shunt ○ Tx for severe visual difficulties? optic nerve fenestration (relieves pressure) ○ Triggers? ■ Tetracyclines (i.e. female patient getting acne tx) ■ Vit A derivatives

Management of ischemic stroke (caused by blockage w/i blood vessel) ○ Initial imaging? Non-contrast head CT to DDx ischemic stroke vs. hemorrhagic stroke b/c blood looks just like contrast ☺ ○ Next imaging? MRI ○ Time window for tPA administration? 3-4.5 hrs ■ Up to 6 hrs if you can inject tPA directly into involved vessel ○ HY contraindications to TPA administration ■ Bleeding disorder ■ GI bleeding ■ Recent brain surgery ○ Blood pressure management? ■ Permissive HTN (BP up to 220/120) in patients with ischemic strokes only in pt’s that cannot NOT get TPA ● Why? Increased BP keeps vessels perfused, which optimizes blood flow to ischemic penumbra of brain ■ If getting tPA, BP < 185/110 ■ If hemorrhagic stroke with high BP, can lower BP with labetalol, nitroprusside, nicardipine

78 yo has 15 min episode of complete vision in one eye that has since resolved. → amaurosis fugax (TIA) ○ Classic presentation? Painless loss of vision in one eye, “curtain coming down” ○ Dx? ■ Non-contrast head CT ■ Brain MRI 12

○ ○

○ ●

■ Echo (to look for origin of clot) ■ Carotid US NBSIM? Probably carotid US on shelf exam Prevention of further episodes? ■ Aspirin ■ Aspirin + dipyridamole ■ If aspirin contraindicated, clopidogrel Indications for carotid endo ■ >70% stenosis ■ Symptomatic #1 modifiable RF for a stroke? HTN

6 yo M with difficulty walking + uses arms to “walk up” his legs/thighs (Gower’s sign) + hypertrophy of calves bilaterally. → duch*enne muscular dystrophy ○ Gene? DMD ○ Mutated protein? Dystrophin (stabilizes sarcolemma to cytoskeleton) ○ Inheritance? X-linked recessive ○ Dx? ■ Increased creatine kinase levels ■ Genetic testing for dystrophin gene mutation ○ Tx? ■ Steroids ■ High-calorie nutrition ○ MC cause of death? Respiratory failure or cardiac causes Muscular dystrophy + intelligence + life expectancy in the 50s → Becker muscular dystrophy ○ Pathophys? Have some functional dystrophin ○ Gene? DMD ○ Mutated protein? Dystrophin ○ Inheritance? X-linked recessive Ataxia + confusion + ophthalmoplegia → Wernicke encephalopathy ○ Reversible ○ Imaging? Hemorrhagic infarct of mamillary bodies ○ Pathophys? Vitamin B1 deficiency ○ Function of which enzyme affected? Transketolase in pentose phosphate pathway ■ Decreased transketolase activity ○ Tx? IV thiamine then glucose ○ Note: “ophthalmoplegia” means eye problem; pt’s with Wernicke-Korsakoff can have any eye problem, e.g. nystagmus, lateral gaze palsy, etc. Complication of untreated Wernicke encephalopathy? Korsakoff syndrome ○ Irreversible ○ Sxs: ataxia + confusion + ophthalmoplegia + amnesia + confabulation 40 yo F with diplopia or droopy eyelids worse at end of day + difficulty swallowing + weird speech → myasthenia gravis ○ Pathophys? Antibodies to post-synaptic nicotinic ACh receptor ○ Dx? anti-AChR or anti-MUSK Ab testing ■ NOT edrophonium test/Tensilon test (no longer standard of care) 13

○ ○

Tx? AChE inhibitors (e.g. pyridostigmine) Associated malignancy finding? Thymoma (anterior mediastinal mass)

NBSIM for patients with myasthenia gravis? CT chest to look for thymoma ○ b/c removal of thymoma can resolve myasthenia gravis

Smoker with proximal muscle weakness that improves with use → LEMS ○ Pathophys? Antibodies to presynaptic voltage-gated Ca++ channel

Lateral medullary syndrome (Wallenberg syndrome) causes dysphonia, dysphagia, and loss of gag reflex ○ Blood vessel occluded? PICA occlusion ○ Loss of pain and temperature on left face → L trigeminal nerve ○ Loss of pain and temperature on right side of the body→ L spinothalamic tract ■ Located in lateral brainstem ■ Decussates at anterior white commissure → contralateral sxs ○ Absent gag reflex, dysphonia, dysphagia → CN9/10 ■ Medulla ○ Vertigo → CN8 ■ Pons/Medulla ○ Ptosis and miosis on left (Horner’s) → L sympathetic tract to superior cervical ganglion ■ Located in lateral brainstem

Medial medullary syndrome causes tongue deviation to ipsilateral side ○ Blood vessel occluded? Anterior spinal artery occlusion ○ Right sided paralysis → L corticospinal tract ■ Runs through medial brainstem ■ Decussates at level of medullary pyramids ○ Tongue deviation to the left. → L CN12 ■ Tongue deviation to ipsilateral side

14

A 35 yo African American female with 3-day history of eye pain. Funduscopic exam is notable for conjunctival erythema and miosis. CBC is notable for increased ACE levels and Ca2+ of 12.9. CXR with bilateral lymphadenopathy. What is the cause of her eye symptoms? → optic neuritis 2/2 sarcoidosis ○ Exam finding? APD + pain with eye movements (this is optic neuritis!) ○ Another condition a/w optic neuritis? Multiple sclerosis

35 yo F with tremors in her hands bilaterally that are worsened by stretching out her hand. Better with alcohol. → benign essential tremor ○ Inheritance? AD ○ Tx? ■ Beta blocker (e.g. propranolol) ■ Barbiturates (e.g. primidone) ● Mechanism? Increase duration of opening of chloride channels → hyperpolarization

Benign essential tremor: ▪ ▪ ▪

Bilateral Worse with activity Tx: propranolol

Parkinson’s disease tremor: ▪ ▪ ▪

Unilateral/asymmetric Worse at rest Tx: levodopa/carbidopa; dopamine agonists; 15

COMT inhibitors; NMDA antagonists; MAO-B inhibitors; antimuscarinics

-------------------------------------------------------------------------------------------------------------------------------

Episode 21: Comprehensive Pediatric 3rd Year Shelf Review

● ●

CrouP treated with raCemiC ePinephrine Seizure disorders by age range ○ Baby w/ hypsarrhythmia → West Syndrome (tuberous sclerosis) ○ KId 2-6 yo → Lennox-Gastaut ○ Teenager → JME

16

Associations with eosinophils ○ Erythema toxicum neonatorum ○ Chlamydia pneumonia Empiric tx of neonatal sepsis per WHO? Ampicillin + gentamicin

Peritonsillar abscess

Retropharyngeal abscess

Teens Overall not-sick kiddo “Hot potato” voice Uvular deviation

Kid < 5 yo Toxic-appearing kid Neck asymmetry Inability to extend neck Widened prevertebral tissue on lateral neck XR

● ● ● ● ● ● ● ● ● ●

ID, low BW, microcephaly, high pitched cat like cry → cri-du-chat Obesity, infantile hypotonia, ID, small extremities, Cr 15 deletion → Prader-Willi Happy mood, ID, inappropriate laughter, ataxia, Cr 15 deletion (mom) → Angelman syndrome “Elfin facies”, short upward turned nose, long philtrum → Williams syndrome X-linked, CGG repeat, anticipation, long face, large ears, ID, macroorchidism → Fragile X Short palpebral fissures, thin upper lip, smooth philtrum, microcephaly, VSD → FAS Tricuspid valve displaced downward, RV hypoplasia → Ebstein’s anomaly (Li toxicity( Lateral neck mass that does not move with swallowing, derived from ectoderm → branchial cleft cyst Midline neck mass, moves with swallowing, may have ectopic thyroid tissue → thyroglossal duct cyst Hypopigmented macules identified with Wood’s lamp, shagreen patches, adenoma sebaceum (face), infantile spasms, subependymal nodules in brain, cardiac rhabdomyooma, renal angiomyolipoma → tuberous sclerosis

Severe fasting hypoglycemia, lactic acidosis, hepatomegaly, gout, may have kidney sxs (since this also does gluconeogenesis), no muscle sxs, give cornstarch → Von Gierke disease (GSD type 1) ○ Deficiency of? Glucose-6-phosphatase ○ V for “very bad” ○ Also remember the two L’s (liver only & lactic acidosis) Glycogen storage disease associated with heart failure and death at < 2 yo → Pompe disease (GSD type 2) ○ Deficiency of? Lysosomal alpha-glucosidase = acid maltase ○ P for “pump”

17

○ Mild hypoglycemia, hepatomegaly, muscle sxs, no lactic acidosis → Cori disease (GSD 3) ○ Deficiency of? Debranching enzyme ○ C for “combined” (muscle & liver sxs) No lactic acidosis, muscle weakness, no hepatomegaly → McArdle’s dz (GSD type 5) ○ Deficiency? glycogen phosphorylase (removes one glucose from the chain) ○ M for “muscle”

High-yield lymphadenopathy ○ Anterior cervical → strep ○ Posterior cervical → mono ○ Posterior auricular & suboccipital → rubella ○ Unilateral → Kawasacki

Ostium primum defects associated with Down’s ○ Ostium primum is down by the endocardial cushions so this makes sense

18

● ● ●

Transposition of the great vessels associated with (pre-existing) maternal diabetes ○ “Egg on a string” on CXR Truncus arteriosus associated with DiGeorge Tricuspid atresia ○ Need ASD + VSD OR ASD + PDA ○ LAD on EKG Tetralogy ○ Squatting helps during Tet spells because it increases SVR (which decreases the R to L shunting)

----------------------------------------------------------------------------------------------------------------------------

Ep 22 [OBGYN Shelf Review] 22 y/o G1P1 visits her obstetrician 1 week after delivering a 6 lb 4 oz boy. Delivery unremarkable. Apgars * and ( at 1 and 5 minutes. Plans to have her next baby when she graduates from graduate school 2 years from now. In addition to routin screening for post partum depression, what is the next best step in the management of this patient? - Best answer is injectable bc you don’t want to give estrogen in this OCP-> this will cause difficulty breastfeeding. MOA’s- progestin only contraceptives- protect ag endometrial cancer, thicken cervical mucus. CI - If you have a malignancy that is progestin receptor + (lobular carcinoma in situ, invasive lobular carcinoma)- avoid progestin containing contraceptives. Side effect of injectables- can cause weight gain, reversible decrease in bone mineral density Treat mastitis with dicloxacillin Breast feeding = high prolactin state. Prolactin-> Shutting down HPG axis-> less production of “stuff” that causes breast cancer-> = decreased risk of breast cancer, decreased ovulation (destroying ovarian epithelium less)-> less risk of ovarian cancer. Helps to lose pregnancy weight. Breast feeding contraindications - If lady has active TB, HIV- do not breast feed - Galactosemia- galactose 1 phosphate uridyl transferase deficiency - definite CI to breastfeeding - Mastitis- NOT a breast feeding contraindication. She SHOULD breast feed bc this will help clear the infection Q2 -

-

22 y/o female is tearful 3 days after delivery = post partum blues. Just reassure. Usually see in first 2 weeks of delivery. 22 y/o is brought to ED by husband 3 days after delivery. Sees a radio taped to her head, she claims she is receiving information from outer space about how sacrificing her baby will help solve world hunger. = postpartum psychosis. This is an indications for involuntary hospitalization, prescribe antipsychotics. High risk of infanticide if you do not involuntarily hospitalize her 22 year old female comes to 3 week post partum appt. She looks disheveled, and occasionally has thoughts of hurting the baby, which she feels remorseful about. She is breastfeeding, but no longer enjoys activities she loves before she got pregnant. She is accompanied by supportive husband. = post 19

partum depression. See this more than 2 weeks after delivery, can see in immediate period after delivery as well. Q3 4 days after c section of a 9 lb female, a 33 y/o G2P2002 complains of abdominal pain. Vitals are normal (BP120/80, HR 99, Resp 19). She is febrile. She has diffuse lower abdominal tenderness and foul smelling lochia. C section was due to arrest in active pahse of labor. Most important risk factor in this presentation? = endometritis, biggest risk factor is c section (more so than maternal infection prior to delivery) - tx = clind + gent - Do not confuse endometritis with chorioamnionitis. More important risk factor here is prolonged rupture of membranes. This will be a patient with fever during labor, fetal tachycardia, foul smelling lochia. Txamp + gent. Amp covers listeria. Give IV oxytocin to help deliver baby faster. - Listeria- if you see a lady with a stillborn and abscesses all over the body, this may be granulomatosis infanticeptica = congenital listeriosis. Can kill the baby in utero. - Rupture of membrane >18 hr- give GBS ppx Diagnosing and dating pregnancy - Check BhCG in urine or serum- serum shows up a week before the urine - When is checking bhcg the NBSIM? - Woman has ammenorrhea - s/s ectopic pregnancy - Date - First day of LMP + 7 days, subtract 3 months, add 1 year - GTPAL - G4P3104 - G = gravidity, how many times you have been pregnancy - T = # of term pregnancies - P= # preterm pregnancy - A = # abortions - L = # living children Q4 Do you want to be a pregnant millionaire?? - blue/purple vagin* = chadwick’s sign - Blotchy pigmentation of the face = melasma - Arterial BP in first 20 weeks- should go down bc of high levels of progesterone, a smooth muscle relaxant-> decrease in SVR. may get higher after first 20 weeks, but shouldnt get higher than before you were pregnant - Plasma volume goes up. Think of birth as a bloody experience- the body prepared for this by boosting up moms plasma vol by 50% - SVR- decreases bc progestin - Oncotic pressure- decreases. Albumin goes down in the blood too - CO- increases bc increased volume = increased preload - Decreases when supine- uterus is compressing the IVC - Nonreassuring fetal heart tone- tell mom to lay in LLD position - RBC mass- goes up by 30% - Hematocrit - a concentration measure. RBC mass is not increasing as much as plasma volume, so hematocrit is decreased- this is Physiologic anemia of pregnancy - Coagulation factors- increase. You can think of this as mum is bound to bleed out a lot during birth, so lets make it easier to clot to prevent massive blood loss 20

-

-

-

-

-

In general, every protein in the bloodstream increases during pregnancy, with the notable exception of Albumin Gastric motility- high levels of progestin-> smooth muscle relaxation-> decreased motility, increased gastric emptying time, GERD Acid base- see alkalosis. If mom is alkalotic relative to the fetus, it creates a gradient for the fetus to dump CO2 into mom’s circulation across the blood placental barrier. Tidal volume- increases Ureter size- consider progestin-> dilation. This could contribute to increased stasis + increased UTI’s in pregnancy BUN/Cr- both decrease. If elevated, this is a concerning sign - If blood volume is increasing, your hydrostatic pressure is increasing in the glomerulus - GFR + cr clearance should go up Glucose and protein in the urine- increase. Transport maximum for glucose in the PCT goes down in pregnancy. Very mild proteinuria is fine. Pituitary size- should double in size. If lady bleeds out a lot during delivery, hypoperfusion of pituitary can occur, = ischemic stroke of pituitary gland (sheehan’s syndrome). Presentation = failure to lactate after pregnancy Size of thyroid- increases. You need thyroid for proper fetal development - Estrogen- increases synthesis of thyroxine binding globulin, so TBG goes up, and total thyroid hormone levels will go up, free thyroid levels are normal. BHcg - doubles every 2 days- increases until 10 weeks, then levels off/ tapers somewhat

Q5 32 y/o female 36 w gestation comes to ED because she hasn’t felt her baby move for 6 hours. Physical exam shows 37 cm from tip of pubis to top of uterus, no fetal heart tones detected on doppler u/s. Rest of exam is within normal limits. BP 105/78, HR 78, RR 16, mildly febrile. In addition to patient centered counseling and physician empathy, NBSIM? = induction of labor with IV oxytocin. Do not postpone delivery- retained fetal products can lead to DIC. NEVER do a c section- risk of surgery isnt worth it. Q6 25 y/o primigravida at 10 weeks gestation presents for a first prenatal visit. Maternal exam and VS wnl. Maternal hgb is 11 g/dL. WBC = 6,000. Pap smear 5 months ago is unremarkable. Rubella titers WNL. HBV Surface antibody +, surface antigen negative. Core antibody negative. HIV/syphillis/chlamydia all negative. NBSIM?. Urinalysis- screen for ax bacteriuria. Progestin effect causing ureter dilation and stasis-> increased risk UTI’s. Asx bacteruria can progress to pyelo if unnoticed. tx - normal non preg female- wouldnt treat asx bacteriuria unless it was complicated. In preg woman- tx with nitrofurantoin, amoxicillin, phosphomycin. If pyelo, give IV ceftriaxone. Pyelo can increase risk of preterm delivery- maintain this lady on nitrofurantoin for the rest of the preg. After treating, perform a test of cure. - Do not give preg ladies excessive Vitamin A or live vaccines. Give folate. Q7 What is the most appropriate time frame for the following interventions/prenatal tests - rh d immune globulin in an rh negative female - 28 w - GBS swab - 35-37 w - Screen for gestational diabetes - 1 hour gtt, confirmatory is 3hr. - 24-28 ish w 21

-

- Tx- diet, exercise, insulin is probably first line though - ACOG- beginning to feel more comfortable using metformin and glyburide Chorionic villus sampling - 10-13 w Screening u/s for neural tube defects - 18-20 Amniocentesis, quad screen - After 15 w In the question stem, mom will be concerned about her baby’s risk of chromosomal problem etc- will give w of pregnancy in the q- use this to rule answers out. ***doing an invasive procedure that mixes mom and baby blood like this- give Rh d immunoglobulin! More info on prenatal testing - Elevated AFp- think neural tube defect. Also see elevated acetylcholinesterase in the amniotic fluid. May also occur in abdominal wall defect, i.e. omphalocele/gastroschisis - MCC- incorrect dating - See increased level of inhibin A + bhcg, afp/estriol is decreased - think Downs. Arises from maternal nondisjunction, robertsonian translocation - See decreased inhibin A, bhcg, AFP/estriol all low. Kid has rocker bottom feet, overlapping digits and toes. - Quad screen is + -Get an u/s - Lecithin : sphingomyelin is 2+, this is a sign of lung maturity.

If a lady has a history of 2nd trim painless preg loss, i.e. from cervical insufficiency- can place a cerclage. But, cerclage means CI to exercise Gestational DM - From human plactental lactogen- a diabetogenic hormone. Its job is to create hyperglycemia in mom so that she can shunt more energy to the fetus. GDM can cause macrosomia, polyhydramnios (AF >25)(baby will have hyperglycemia, and will make more urine). When baby is delivered, no longer is exposed to hyperglycemia. Their pancreas will keep making insulin-> triggers hypoglycemia-> seizures. Also can have seizures from hypocalcemia - Baby Cqx from GDM vs DMII mom - Chronic DMII - Kids have cardiac probs, i.e. hypertrophic Cm - Fusion of legs (sirenomelia) - Sacrum doesnt develop right= caudal regression syndrome - Hypocalcemic seizures - >4500g - recommend elective c section Q8 Recommended weight gain from preg - BMI underweight- gain 28-40 lb - BMI 23- gain 25-35 lb - BMI overweight- gain 15-25 lb - BMI obese- 11- 20 lb Q9

22

-

-

39 year old female with mood swings, amenorrhea for the past year. Has severe episodes of intense sweating and palpitations once or twice daily. Meds include levothyroxine, on physical, see skin hyperpigmentation. = premature ovarian failure - Ovaries stop working-> estrogen goes down-> FSH/LH goes up because there is no negative feedback - Tx menopause- dyspareunia- lubricant, vagin*l estrogen to build back up vagin*l epithelium. - vagin*l estrogen is CI if Hx of Estrogen responsive malignancy - Osteoporosis (estrogen increases synthesis of osteoprotegrin- this binds up RANKL which decreases activation of osteoclasts.). Supplement Ca, VD, bisphosphonates (first line in osteoporosis), raloxifene (a SERM in the breast and uterus, agonist in bone!) - Obesity is a protective factor for osteoporosis. Anorexics will have increased risk of osteoporosis. - Obesity will also increase risk of osteoarthritis - Severe menopausal sx- (use short course 6 months) HRT, used combined estrogen and progestin to have less problems. - CI- If lady has estrogen responsive malignancy, hx of strokes, migraine with aura, hepatic adenoma 19 y/o female presents with intense anal pruritis. Physical exam reveals erythema of the vulvovagin*l area. She recently completed a course of ciprofloxacin for lobar pneumonia. She has a hx of DMI and pH of vagin*l secretions of 4.3. KOH prep shows spores and structures resembling hyphae = candida - Trich and gardnerella- pH >4.5. - BV from gardnerella- see fishy smell, KOH shows clue cells. Take metronidazole, counsel not to take alcohol concurrently- dont want disulfiram effect from inhibition of acetaldehyde dehydrogenase - Dont need to treat partner for candida or gardnerella - Trich- treat partner as well to prevent recurrence. Tx with metro - Punctate hemorrhages (strawberry cervix), frothy green foul smelling discharge - Metro can treat GET GAP: Giardia, Entamoeba, Trich, Gardnerella, Anaerobes belows the diaphragm, Protozoal infections - Risk factors for candida- immunosuppression, diabetes, recent antibiotic use - Tx- antifungal cream, or oral fluconazole

Q10 34 y/o female at 10 weeks gestation is brought to ED by husband, who is concerned about her severe vomiting for the past week. Physical exam shows skin tenting, >20 mmHg change in BP from supine to standing position, weighs 131 lb (recorded in chart at 140 lb 3 months ago). Further testing would reveal- a hypokalemic hypochloremic metabolic alkalosis (hyperemesis gravidarum) - She is losing a lot of acid with vomiting, becomes hypochloremic. Losing acid = alkalotic. Volume down = RAAS activation-> principle cells ENAC transporter is working a lot-> excrete more K in the urine-> hypokalemia. If you increase activity of the proton ATPase pump, the a intercalated cells of the nephron, see alkalosis due to this as well because you are dumping more protons in the urine. - She has lost more than 5% of her prepregnancy weight- this qualifies her for hyperemesis gravidarum. - Tx - ondansetron, metochlopromide. Replace electrolytes, NBME likes thiamine - Eat small frequent meals, take vitamin b6 and doxylamine Q11 23

26 year old female presents to obstetrician for yearly check up. PMH hypertension controlled with captopril and LSM. vitals WNL. BMI 23. Mirena IUD implanted 18 months ago, wants it removed. Got married 3 months ago, plans to have kids. Pap smear 2 years ago. In addition to routine guidance, and recommendations and appropriate vitamin supplementation, NBSIM? = switch captopril to alphamethyldopa. - pap smear- good bc its q3 years from 21-29 years - Captopril- not good for pregnant woman, its a teratogen. So are ARBs - Chronic hypertension- HTN before you are pregnancy - Gestational hypertension- >140/>90 after 20 w pregnancy, no proteinuria - Tx- “Hypertensive moms Love nifedipine”- Hydralazine, Methyldopa, Labetalol. Nifedipine - Pre eclampsia- gestational hypertension + proteinuria - Pathophys is unk- recognize phrases like endothelial dysfunction, formation of abnormal blood vessels - Risk factors- most important = prior hx of preE - PreE q/ severe features- they have some kind of end organ dysfunction, i.e. increasing BUN, Cr, LFTs, or BP is >160/>110 - Consider Mg at this pt for seizure prophylaxis. - Check DTR’s- decreased is a telltale sign of Mg toxicity-> could lead to resp depression, CV collapse and die - Rescue agent for Mg toxicity= calcium gluconate - PreE w/ seizure= Eclampsia - Tx- #1 mg. If mg doesnt work, give short acting benzo. - Definitive treatment- delivery - HELLP - Hemolysis, elevated LFT, low platelets Q12 Most common teratogen for each birth defect - Hypoplasia of RV, downward displacement of tricuspid valve (atrialization of RV), mom is treated for bipolar. - = epstein anomaly due to lithium toxicity - Firstline med for mania in pregnancy is haloperidol - Tooth discoloration - Tetracyclines - Cartilage damage - Fluoroquinolones - Stippling of epiphysis, dont use this anticoagulant in a pregnant lady with DVT - Warfarin - Clear cell adenocarcinoma of the vagin* - DES (diethylsilbestrol)- assoc with t shaped uterus - Renal probs in the fetus - ACEi/ARBs - Gray baby syndrome - Chloramphenicol- because neonates have very low levels of UDP glucuronysyl transferase, can not metabolize the drug - Kernicterus - TMP-SMX - Smooth philtrum, microcephaly, - FAS 24

-

IUGR, hypoplastic nails, microcephaly, cleft lip - Phenytoin- fetal hydantoin syndrome Most dangerous anti epileptic/highest risk of neural tube defect - Valproic acid

G13 21 y/o female comes to PCP with severe facial acne. LMP 1 w ago. Stable relationship with BF of 3 months, use condoms inconsistently. See open and closed comedones clustered around patient’s lower face. Trials of tetracycline, benzoyl perized, and combined OCP yielded no positive results. Smokes 2 cigarettes a day. Requests prescription of isotretinoin. Most likely CI to isotretinoin = unprotected sex control. - Isotretinoin prescription requires 2 forms of birth control - 35 y/o female that smokes/hx of migraine with auras, hx thromboembolic disease, hepatic adenomaany of these- OCP with estrogen isnt a good option - See jittery newborn that is inconsolable- think opioid withdrawal- give methadone, taper - Cocaine use in pregnancy can cause abruptio placentae- painful third trimester bleeding - Use acetaminophen in pregnancy- using cox inhibitors can trigger premature closure of the ductus arteriosus. G14 29 y/o G3P2 becomes unresponsive 15 min after delivering a 3900 g newborn. BP- 40/palpable, HR 40, RR 6, O2 sat 78%. See blood oozing from a venipuncture site = amniotic fluid embolism Q15 Breast triggers - 44 year old male treated for class 3 HF with gynecomastia - Spironolactone- is an aldosterone receptor antagonist, but also has androgen receptor blocking activity - 32 y/.o 4w postpartum has fever, breast tenderness, and erythema- Mastitis, tx with antistaph penicillins, i.e. dicloxacilin - 22y/o with multiple mobile breast masses, become more painful and wax/wane in size with cycle - = fibrocystic changes - 23 y/o female with solitary, mobile, well defined nontender breast mass fibroadenoma - Bloody nipple discharge - Intraductal papilloma - 49 y/o female with large breast mass, core needle bx reveals structures of leaf like projections on histology - Phyllodes tumor - BRCA mutation assoc with male breast cancer - BRCA 2 - Most common location of breast cancers - Upper outer quadrant - Whats the receptor positivity most commonly found in LCIS - ER/PR+ - Most common invasive breast cancer - Invasive ductal carcinoma - 55y/o female that presents with a breast mass, physical exam reveals peau d’orange - Inflammatory breast cancer 25

-

-

-

-

55y/o presents with an eczematoid scaly appearing lesion around the nipple - Paget’s disease of the breast Is there a difference in survival between lumpectomy + radiation or mastectomy for early stage breast cancer - NO NBSIM in female with DCIS with + sentinel LN bx - Do axillary LN dissection ER/PR + breast cancers tx - Tx- SERMS, aromatase inhibitors (anastrozole) Treat Her2Neu+ malignancies - Trastuzumab 45 y/o female with hx breast cancer tx had JVD, EF of 35% - Due to trastuzumab. This is reversible - If due to doxorubicin/daunorubicin- HF will be irreversible - Preventable with iron chelator dexraroxane 55 y/o female with 10 year hx of lymphedema from axillary LN dissection has weight loss, purplish necrotic ulcerating mass on L arm - Lymphangiosarcoma Breast cancer is floridly high yield- its very scenario based. The answers can vary from different sources. These answers are primarily from his best judgement from ACOG and online, but you should use your best research and email Divine questions

**core needle bx = excisional bx if the mass if small, well circ*mscribed Q16A - 45 y/o female has a palpable L breast mass she noticed when she took a shower 1 week ago. - NBSIM = mammogram first. Since she is older than 30, you do mammo first rather than u/s - Diagnostic mammo is negative-> do a core needle biopsy Q16B - 25 y/o female with breast pain- LMP started 3 days ago. PE shows multiple mobile breast masses. NBSIM- reassure, schedule follow up for a few weeks Q16C - 25 y/o presents to yearly gyn appt with complaint of a palpable breast mass she noticed while taking a shower. - See immobile, painless mass located close to the nipple. No FMH of breast cancer. NBSIM-get u/s - See immobile, painless mass located upper outer quadrant of breast. U/S shows solid mass. NBSIM- get a core needle bx - See immobile, painless mass located upper outer quadrant of breast. U/S shows cystic mass. NBSIM- get an FNA, send fluid for cytology, get a f/u u/s in a few weeks - If FNA were bloody, send it for cytology and do a core needle bx - If non bloody cyst recurs, get a core needle bx Q16D - American cancer society guidelines- screening mammos starting at 40 y/o annually. Divine says this is more reliable - USPFTF guideline- start at 50, get q2y. - Women with BRCA mutations get these mammos and also MRIs of the breast. Can do a prophylactic total abdominal hysterectomy and bilateral salpingo oophorectomy to prophylaxis ag breast, ovarian, and endometrial cancer. 26

Q17 33y/o G2P1 female presents to L&D at 34w gestation due to consistent uterine contractions. Last preg required classical c section due to severe obesity. As resident walks into the room, pt begins to complain of severe abdominal pain. Pelvic exam is notable for copious amounts of blood from vagin*l canal. FHR shows HR of 33. Pt has hx cocaine use with current preg. ML Dx- uterine rupture - NBSIM- crash c section - Other common presentations of uterine rupture- palpating fetal parts in the abdomen, loss of fetal station - Approach to third trimester bleeding - Painless - Placenta previa- low implantation of placenta - Vasa previa- bb’s blood vessels overlie the cervical os - Deliver baby soon, this is an obstetric emergency - Painful - Uterine rupture - Abruptio placentae- see mom with cocaine use Postpartum hemorrhage - Definition according to ACOG- Blood loss >500mL after vagin*l />1,000mL after c section - Causes- “tone, tear, tissue, thrombin, topsy turvy” - MCC- uterine atony - Etio- Uterus is over worked in rapid labor, prolonged labor, uterus infection, gave too much tocolytics, multiple gestations, polyhydramnios, macrosomia - NBSIM - uterine massage under u/s guidance - IV oxytocin to contract it down - Can also give methergin (serotonin receptor agonist, like triptans- if a lady has a history of vasospastic angina or reynaud’s phenomenon, you should not give methergin) - Can give Carboprost (Hemabate) - Dont give to person with hx of asthma - Can give dioprostone - PG2 analog - Also Keeps PDA open - Avoid it in women with profound hypotension - B lynch sutures- “like suspenders on the uterus” - Lacerations - Risk factors - precipitous delivery, difficult delivery i.e. shoulder dystocia, operative vagin*l delivery. - Tx- surgery - Retained placental tissue - More common when a succenturiate lobe of placenta is present, with an invasive placenta (placenta accreta) - Tx- d&c - DIC - Uterine inversion - Causes- weak myometrium, pulled too hard on uterus - Big risk factor- prior inversion 27

-

-

- Presentation- pulling out placenta, see a bluish bulge from the vagin* - Tx - replace uterus, give IV oxytocin to contract it back into place PPH treatments when all else fails - Ligate uterine arteries, internal iliacs - All else fails- hysterectomy Complications - Sheehan’s syndrome vs apoplexy - Apoplexy- think of it as a hemorrhagic stroke of the pituitary - Sheehan’s- ischemic pituitary stroke

Menorrhagia - PALM COEIN- Structural vs non structural causes of uterine bleeding (can cause IDA!) - structural - Polyps - Adenomyosis - Symmetric, painful, soft - Leiomyomas = fibroids - Presents as asymmetric nodularity of the uterus. Usually painless, firm. - Can be palpable on physical exam, can detect on a transvagin*l u/s - Tx- OCp to regulate the HPG axis, NSAIDS if painful - If this lady wants to get pregnant in the future, perform a myomectomy. If she is postmenopausal, can perform a hysterectomy - Before surgery, you can give continuous Leuprolide to shrink the tumorsthey are estrogen responsive - Can give tranexamic acid - helps to bleed less - Malignancy and hyperplasia - Non structural - Ccoagulopathy - Ovulatory dysfunction (PCOS) - Endometrial causes - Iatrogenic - Not yet defined - Other alphabet soup - Metrorhagia- bleeding between periods - Think of a metro bus coming more often - Polymenorrhea- bleeding occurs 35 days apart - Hypomennorhea- regularly times menses with very light flow - Menometrorrhagia- heavy bleeding at irregularly timed intervals - Life threatening hemorrhage - Type and cross, 2 large bore IVs, IVF, transfuse as needed - IV estrogen is first line - Consult OBGYN Q18 27 y/o G0P0. annual gyn exam. no abnormal pap smears. Over the past decade, she has had irregular periods. Married for past 3 years. Unprotected intercourse every other day, has not become pregnant. Physical shows male pattern hair growth around chin, no evidence of cl*toromegaly. BMI 35. BP 150/102. Requests 28

referral to dermatologist for long standing hyperpigmentation under breast and axilla since 6 months ago = PCOS - Dx PCOS is clinical. Need 2/3 criteria- 1. Polycystic ovaries on u/s 2. Evidence of hyperandrogenism (hirsut). 3. Need objective sign of anovulation/ can also see insulin resistance, LH:FSH ratio elevatednot required for sx, but supportive. - r/o other causes of hyperandrogenism - Tx - Clomiphene- an ER partial agonist.-> shut down estrogen negative feedback-> This can increase GnRH-> encourage ovulation and fertility - OCP - regulates HPG axis - Estrogen can increase synthesis of Sex hormone binding globulin (SHBG)-> binds testosterone!-> decrease testosterone levels, decreasing hirsut sx!! - Spironolactone - Helps with hyperandrogenism - Helps with hirsut- inhibits 5a reductase (testosterone-> DHT) in the skin - Metformin + LSM for insulin resistance - Longstanding PCOS complications - Metabolic syndrome antecedents (stroke, MI) - Infertility (this is the most common cause of infertility due to ovulatory factors) - Endometrial hyperplasia/cancer - If you see a 39 y/o lady with abnormal bleeding and 5 y history of PCOS- think about getting an endometrial bx!! Postmenopausal bleeding from the vagin*- think of endometrial cancer. NBSIM - endometrial biopsy. Q19 32 presents to L&D at 37 weeks gestation with consistent moderately painful uterine contractions. Pregnancy is complicated by hx diabetes. FHR normal. Offered elective c section due to macrosomia, but she insisted on having a natural birth. During second stage of labor, baby’s anterior shoulder is stuck under the pubic symphysis, the baby is successfully delivered after repeated traction on the shoulder and arm, and maternal maneuvers i.e. hip flexion. This increases risk of injury to = C5-C6 roots of the brachial plexus (erb duch*enne palsy) - Waiters tip deformity- kids hand is pronated, elbow extended, wrist is flexed - Dont confuse with klumpke’s palsy- this is C8-T1 - Shoulder dystocia- is an obstetric emergency (fetal hypoxia) - Manuevers - Apply suprapubic pressure - Mcrobert’s maneuver = flex hips - Mom on all fours - Zavanelli maneuver if all else fails- push the child back into the uterus and do a c section FHR tracings - HR - Normal- 110-160 - 160 = fetal tachycardia - Accelerations 29

-

-

Rise in HR of 15 BPM for at least 15 seconds 2 of these in a 20 min period = positive NonStress Test -> expectantly manage - Absence of this-> proceed to Biophysical Profile - 5 components- NST, amniotic fluid index (5-25), breathing, movement, tone. Each is 2 points. - Score of around 4- bad sign, thinking about delivery of some sort Variability - The squiggles- should be present Abnormal tracings- refer to vodcast - Early decel- not normal, but its benign. Gradual decrease and increase of FHR, in phase with maternal contraction. Caused by head compression. Reassuring tracing - Variable decel- is a more abrupt change of FHR. caused by umbilical cord compression. Non reassuring tracing, may want to deliver - Late decel- gradual downslope and increase of FHR that is out of phase with maternal contractions- caused by Uteroplacental insufficiency - NBSIM- increase perfusion of placenta-> mom in LLB position - Deliver soon - If you think a FHR tracing is abnormal, first determine if it is an abrupt change or not (is most likely variable). If it is gradual, ask if it is in phase with maternal contraction. In phase = early decel, not in phase = late decel - Sinusoidal wave = fetal anemia - May want to do PUBS to check for anemia, can also transfuse with PUBS1 - VEAL CHOP Variable decel

Cord compression

Early decel

Head compression

Accelerations

O for nothing

Late decels

uteroPlacental insufficiency

Q24 15 y/o primigravida complains of severe morning sickness and vagin* bleeding. Vomiting for hourse every day, has lost 15 lb since LMP 8 weeks ago. Is taking benadryl to help with the nausea. BP is 140/98, RR is 18, HR is 103. TVUS reveals central heterogenous mass in the uterus. Having a solid hyperechoic area interspersed with a multitude of cystic areas. BHcg is positive. NBSIM = suction curretage (molar pregnancy/hydatidiform mole) - solid hyperechoic area interspersed with a multitude of cystic areas = snowstorm appearance - Partial mole - 2 sperm fertilize 1 egg (triploid). Has some fetal tissue. Less Bhcg than complete moles. Could theoretically progress to choriocarcinoma, but lesser risk than complete mole. - Complete mole - 2 sperm fertilize an empty egg, or 1 sperm fertilizes an empty egg and duplicates its genetic material. diploid - No fetal tissue. Produces a lot more bhcg. Much more likely to progress to choriocarcinoma. - Moles- See size-dates discrepancy, hyperemesis gravidarum. - Hyperemesis gravidarum is due to BHcg! 30

-

Patient should be on birth control for 6 month- want to see Bhcg go to 0 to ensure no recurrence.

Psammomma bodies- can be described as laminating calcficications - Assoc with papillary thyroid cancer, meningioma (assoc with neurofibromatosis, around falx cerebri), serous cystadenoma/carcinoma Q25 2 months after a suction curretage, the patient begins to have vagin*l bleeding, SOB, and very high bhcgNBSIM = CXR. this is choriocarcinoma. Most common area of mets = lungs, then brain - Tx- methotrexate- dihydrofolate reductase inhibitor. - Side effect- severe bone marrow suppression- add leucovorin, a folinic acid analog - f/u- birth control for 1 year so that we can detect recurrence - Choriocarcinoma can present as hyperthyroidism bc BHcg and TSH share a common alpha subunit, along with FSH/LH. TSH will be low, thyroid hormone levels will be high Q26 29 y/o G2P1 female presents for first prenatal visit at 10w gestation. RhD negative. Anti immune globulin is negative. NBSIM- give RhD immune globulin at 28 w - If pt was instead having a + RhD immune globulin titer- NBSIM = check Rh- status - If dad is RhD negative, baby can not be Rh +, nothing to worry about. - If dad is RhD pos/unk - Check for fetal anemia by checking doppler u/s of middle cerebral artery of the fetusbaby is hypoxic, and tries to combat this by increasing cardiac output. if you have increased flow in the middle cerebral artery, you should be worried about fetal anemia. - Can do PUBS and transfuse Q27 35 y/o female with hx of 4 c sections presents with painless vagin*l bleeding at 32 w gestation. NBSIM- u/s first. If you did a vagin*l exam first, you are at risk of worsening the bleeding. - Placental previa- is likely in the setting of many c sections bc of the increased amounts of scar tissuethis is the #1 RF - If you detected it at 30/31 weeks- do NOT need to proceed to delivery now bc placental location can change! - Pelvic rest (stop having sex), give steroids, Mg if Pcomm. - Grape sign: Hydatidiform mole: severe hyperemesis gravidarum. Increased B-hcg → hyperemesis - Snowstorm appearance: Hydatidiform mole or molar pregnancy - Batwing: in pt with hx of CF: pulm edema - Butterfly: GBM: Severe headache over the past 6 mo. Type IV astrocytoma crossing corpus callosum. Central. necrosis and edema Tumor marker: GFAP. - Lead pipe colon: Ulcerative colitis: hint: also has a hx of Primary Sclerosing Cholangitis. Loss of haustral markings - Box heart: Ebstein’s anomaly (+mom took lithium when she was pregnant for BPD). Buzzword atrialization of right ventricle - Corkscrew esoph: diffuse esophageal spasm. Chest pain relief with nitroglycerin. Tx with nitrates or CCB. Intermittent dysphagia to solids and liquids - Egg on a string: TotGV: single s2. Newborn with cyanosis. - Honeycomb lung: IPF: 65 y/o M with 6 mo history of exercise intolerance: DLCo Dec. o Reticular opacities, ground glass opacities and honeycombing - Whirlpool spin: midgut volvulus. 3 month old with bilious vomiting. - 3 sign: COTA. EG: Pt has history of turner’s syndrome. High BP in arms and no pulse in legs. - Snowman sign: TAPVR. Pulm veins drain into RA. newborn with cyanosis. - Signet Ring sign in lung: bronchiectasis. (pt may have hx of cystic fibrosis or ciliary dyskinesia) - Rachitic Rosary: Rickets> elevated PTH low CA and low Phosphate. Vitamin D def (secondary hyperparathyroidism) - Pencil and cup deformity: psoriatic arthritis. Seronegative spondyloarthropathies. - Ladder of GI tract: SBO can also be in LBO but classic for SBO - White out lung: ARDS . ICU pt on vent. Noncardiogenic pulm edema so PCWP 30% BSA involvement → TEN ○ NBS? STOP the drug

30 yo F presents with 3 day hx of polydipsia and polyuria. Blood glucose is 650 mg/dl, Bicarb is 21, pH is 7.35. → HHNS (Hyperglycaemic hyperosmolar non-ketotic syndrome) ○ Contrast with DKA? Requires low pH & ketosis ○ Why no ketosis in pts with T2DM? They still make insulin (they have insulin resistance) which inhibits glucagon and thus prevents DKA ○ Tx? Fluids + insulin ○ Na balance? Hyperosmolar hyponatremia ■ Na correction: add 1.6 for every extra 100 of glucose ■ E.g. glucose of 900 → add 1.6*8 = 12.8 ○ K balance? ■ Serum K+ high ■ Total body K+ low ○ The patient becomes altered/comatose with rapid treatment. What happened? Cerebral edema 2/2 rapid lowering of glucose

49 yo F presents with wheezing and flushing. PE is +ve for murmurs consistent with tricuspid regurgitation and pulmonic stenosis. → carcinoid syndrome ○ Mnemonic “Be FDR” = bronchospasm + flushing + diarrhea + R-sided heart lesions ○ Mnemonic “TIPS” = tricuspid insufficiency + pulmonary stenosis ○ Why R-sided heart lesions? Lungs metabolize serotonin ○ If carcinoid is localized to the GI tract (e.g. appendix), there are no sxs because the liver metabolizes serotonin. When it’s metastatic to the liver, then the pt will start to have sxs ○ Dx? urine 5-HIAA (metabolite of serotonin) ■ If positive 5-HIAA, abdominal CT scan ● If no tumor found on CT → octreotide scintigraphy ○ Tx? Can use octreotide to reduce sxs ○ Vit deficiency? Niacin (B3) → pellagra ■ Presentation? Diarrhea, dermatitis, dementia, death ■ Pathophys? Tryptophan can be used to make serotonin and niacin. A carcinoid diverts all tryptophan to the production of serotonin, causing a niacin deficiency. Tumor of the thyroid that could present with diarrhea? Medullary thyroid cancer (calcitonin causes diarrhea)

41

61 yo M presents with exertional dyspnea. CBC is notable for a Hct of 27%. What is the next best step in management? Colonoscopy to r/u colon cancer ○ Iron studies? ■ Ferritin low ■ TIBC high ■ Transferrin sat low ○ When is transfusion indicated? ■ Hgb < 7 ■ ○ What would your diagnosis be if the patient had a similar presentation and difficulty swallowing? Plummer–Vinson syndrome ■ Triad = IDA + esophageal webs + atrophic glossitis (smooth red tongue)

29 yo F with a recent trip to India (ate local foods). Returned 2 weeks ago and initially had fevers for 1 week. Now presents with severe abdominal pain and distension. PE is notable for salmon colored circular lesions on the trunk. → Typhoid fever ○ Bug? Salmonella typhi ○ Tx? FQ

Septic arthritis ○ MC bug? Staph aureus ○ Bug in sickle cell pt? Salmonella ○ Bug in a young F with purpuric skin lesions? NG ○ NBS? Arthrocentesis ■ WBC count in joint fluid? >50k ○ Tx? Joint washout + abx ■ Tx for gonococcal arthritis? Ceftriaxone + doxy/azithro CT vs NG tx difference ○ CT: just give azithro or doxy ○ NG: ceftriaxone + azithro/doxy ■ Treat empirically for Chlamydia

● ● ● ● ● ●

Subconjunctival hemorrhage in a patient with nasty coughing episodes OR pt with post-tussive vomiting → pertussis ○ Tx? Macrolide ○ Ppx for close contacts? Macrolide ○ Lab findings? Elevated lymphocytes NBSM for pt that recently started Ramipril for the treatment of HTN who has a cough? Switch to ARB Prolonged PR interval w/o dropped beats → 1st degree ○ Tx? nothing! Progressive PR prolongation then dropped → 2nd degree Mobitz I/Wenckebach ○ “Longer longer longer drop → Wenckebach” Fixed PR prolongation + dropped beats → 2nd degree Mobitz II P-P interval constant, R-R interval constant, no relation between P waves & QRS → 3rd degree Who gets a pacemaker? ○ 2nd degree Mobitz II 42

● ●

○ 3rd degree Acute management in decompensated pt? Transcutaneous or transvenous pacing + atropine Contraindicated medications? ○ Beta blocker ○ CCB (diltiazem/verapamil) ○ Digoxin (has muscarinic agonist activity)

Reduced EF in a patient with coarse facial features and enlarging fingers. → acromegaly ○ Dx? ■ 1st step: check IGF-1 ■ 2nd step: glucose suppression test ● Normal: glucoses causes suppression of GH release ● Failure to suppress → acromegaly ■ 3rd step: MRI ○ Tx? Transsphenoidal surgery ■ Differs from prolactinoma (usually managed medically) ■ Tx prior to surgery? ● Octreotide ● dopamine agonist (bromocriptine/cabergoline) ● Pegvisomant (GH receptor antagonist) ○ MC cause of death? HF

Dysphagia to solids and liquids in a patient with thick/thin blood smears (Giemsa) revealing what appears to be motile parasites. → Achalasia 2/2 Chagas ○ Bugs? Trypanosoma cruzi ○ Dx? ■ Mnemonic “BME” ■ 1st step: Barium swallow ■ 2nd step: manometry ■ 3rd step: endoscopy ○ Imaging? bird’s beak sign ○ Tx? ■ Dihydropyridine CCB ■ Botox injection ■ Balloon dilation ● Adverse outcome? Esophageal rupture ■ Heller myotomy (cut LES muscles) ○ Manifestations of T. cruzi ■ Achalasia → megaesophagus ■ Dilated cardiomyopathy ■ Destruction of myenteric plexus → megacolon

Reduced MCV in a patient with a long history of untreated rheumatoid arthritis. → anemia of chronic disease ○ Iron studies ■ Ferritin high ■ TIBC low ■ Transferrin sat low (iron locked away in macrophages) 43

Pathophys? Hepcidin → ron sequestration by bone marrow macrophages & reduced iron export by enterocytes

25 yo M with nasal packing presents with a BP of 65/40, elevated Cr, respiratory distress, T of 104, and marginally elevated troponins. → Toxic shock syndrome ○ Pathophys? Superantigen production → ○ Bugs? Staph aureus + Strep pyogenes ○ Tx? Fluids (+/- pressors) + abx

Sequelae of CKD ○ Acid-base anomalies? Metabolic acidosis 2/2 H+ retention ■ Recall there’s a H+ pump in the alpha-intercalated cell ○ Electrolyte anomalies? ■ Hyperkalemia ● Pathophys? Kidneys not excreting K+ (normally happens via ROMK channel) ■ Hypocalcemia ● Pathophys? Low 1-alpha hydroxylase → cannot make active Vit D → can’t absorb Ca+ and phos from the gut ■ Hyperphosphatemia ● Pathophys? Hypocalcemia → 2ary hyper-PTH → BUT can’t excrete phos because the kidneys aren’t working ● Contrast with liver disease ○ Can’t make calcidiol → 2ary hyperparathyroidism → hypercalcemia & hypophosphatemia (kidneys functional) ○ Hematologic anomalies? ■ Hgb low ● Pathophys? Kidneys not producing erythropoietin ○ Manifestations of uremia ■ Pericarditis ■ Platelet dysfunction ● Tx? desmopressin ■ AMS ○ Tx ■ Hyperphosphatemia → phosphate binders (sevelamer) ■ Hypocalcemia → calcitriol ● NOT calcidiol, because they can’t convert calcidiol to calcitriol ○ How is kidney function preserved in diabetic nephropathy? ACE-I ○ Indications for dialysis ■ A = acidosis ■ E = electrolyte problems ■ I = ingestion ■ O = overload (fluid) ■ U = uremia

Rb gene mutations, Paget’s disease, and Teriparatide (PTH analog) administration increase risk of what primary bone malignancy? Osteosarcoma ○ Associated radiological features? ■ Codman’s triangle 44

Sunburst pattern

Asthma step-up treatment ○ Albuterol ○ ICS ○ LABA (e.g. salmeterol) ○ Oral steroids

Asthma + nasal polyps → Aspirin Exacerbated Respiratory Disease ○ 3 things to think about with nasal polyps? ■ AERD ■ Wegener’s = GPA ■ CF ○ Pathophys? ■ Arachidonic acid → prostaglandins (COX) + leukotrienes (LOX) ■ Aspirin blocks COX, which causes flux down LOX pathway ○ Tx? ■ Leukotriene receptor (CYSLTR1) blockers ● E.g. montelukast 45

Lox inhibitors ● E.g. zileuton ○ AE? Hepatotoxic

Regular narrow-complex tachycardia → SVT ○ Tx asymptomatic pt? ■ Acute ● Vagal maneuvers ● Adenosine ■ Chronic ● Beta blocker ● Verapamil/diltiazem ○ Tx HDUS pt? Synchronized cardioversion Indications for unsynchronized cardioversion (defibrillation) ○ V fib ○ Pulseless V tach MC EKG finding in a patient having a pulmonary embolus? Sinus tachycardia

18 yo F presents with scaly, erythematous upper extremity lesions. She has a long history of allergic rhinitis. → atopic dermatitis/eczema ○ CBC finding? Elevated eosinophils ○ Tx? Topical steroids ○ What if these lesions also had umbilicated vesicles → eczema herpeticum ■ Pathophys? HSV superinfection ■ Tx? Acyclovir

A patient presents with the classic “stones, bones, groans, and psychic overtones”. PE is notable for skin tenting. What is the FIRST step in management? Fluids! ○ Bisphosphonates ■ Indications? ● Osteoporosis ○ T-score cutoff? -2.5 ● Hypercalcemia of malignancy ● Paget’s disease ● Ppx if on oral steroids for >3 months ■ AE? ● Esophageal irritation ○ Prevention? Sit up for 30 minutes after taking pill ● Osteonecrosis of the jaw

H. Pylori infection ○ Dx? ■ Urea breath test ■ Stool antigen ■ EGD with biopsy ○ Tx? ■ Triple therapy = clarithromycin + amox + PPI ■ Quad therapy = metronidazole + bismuth + tetracycline + PPI (“MBTP”) 46

● ● ●

RF for PUD? ■ H. pylori ■ Chronic NSAID

Alpha vs Beta Thalassemias ○ Mutation in 1 alpha gene → clinically silent ○ Mutation in 2 alpha genes → alpha thalassemia trait, asymptomatic ○ Mutation in 3 alpha genes → Hb H disease ■ Hgb H = tetramers of beta ■ Presentation? At birth ● Why? There are alpha chains in HbF ○ Mutation in 4 alpha genes → Hb Barts ■ Hgb Barts = tetramers of gamma ○ Mutation in 1 beta gene → beta thalassemia minor ■ Electrophoresis findings? Elevated HbA2 ● HbA2 = alpha2-delta2 ○ Mutation in 2 beta genes → beta thalassemia ■ Electrophoresis findings? Elevated HbA2 + HbF ■ Presentation? After 6 months ● Why? Persistence of HbF until ~6 months ○ Blood smear finding in thalassemias? Target cells Classic CBC findings in hemolytic anemias ○ Low haptoglobin ○ Indirect bilirubinemia Classic “gallbladder” pathology present in patients with hemolytic anemias → pigmented gallstones These patients are at risk of aplastic crises with what bug? Parvovirus B19 Do thalassemias present as a microcytic, normocytic, or macrocytic anemia? Microcytic with normal RDW ○ Contrast with IDA (high RDW)

23 yo M who recently returned from Tucson presents with a 2 week history of fever, ankle/knee pain, chest pain, and a painful, erythematous pretibial lesion → coccidiomyocosis ○ Geographic association? Southwest US ○ Derm finding? Erythema nodosum ○ Microscopy finding? Spherules ○ Tx? itraconazole

Periorbital edema, hematuria, and HTN in a patient with a recent history of cellulitis. BUN and Cr are elevated. → Post-infectious GN ○ Pathophys? Nephritic syndrome ○ Ab? Anti-streptolysin-O and anti-DNase B ○ Type of HSR? 3 ○ Complications of… ■ Strep pharyngitis ● Rheumatic fever - abx decrease risk ● PIGN - abx have NO EFFECT on risk ■ Strep skin infection 47

Only PIGN

11:22 translocation, X-Ray imaging reveals an “onion skin like” periosteal reaction, bone biopsy with histology reveals small, round, blue cells. → Ewing’s sarcoma ○ Tx? Actinomycin D

Parapneumonic effusion (PNE) = pleural effusion in the setting of pneumonia

Uncomplicated PNE

Complicated PNE

Empyema

pH > 7.2 Glucose > 60 LDH < ⅔ ULN NO bugs

pH < 7.2 Glucose < 60 LDH > ⅔ ULN NO bugs

pH < 7.2 Glucose < 60 LDH > ⅔ ULN YES bugs

Abx

Abx + chest tube

Abx + chest tube If refractory? VATS

What is your diagnosis? V-tach ○ How would you describe the rhythm? Regular, wide-complex ○ Tx pulseless V tach → defibrillate ○ Tx V tach w/ pulse but HDUS → synchronized cardioversion ○ T V tach w/ pulse HDS → amiodarone What is the MC cause of death in the immediate period following an MI? V fib ○ Tx V fib → defibrillation = unsynchronized cardioversion

Oral mucosal ulcerations + +ve Nikolsky sign (flaccid skin blisters) in a 45 yo M. → pemphigus vulgaris ○ Pathophys? Autoantibodies against desmosomes (against desmoglein 1/3) ○ Type HSR? 2 ○ Dx? skin bx w/ immunofluorescence → intraepidermal immunofluorescence (net-like pattern) ■ Contrast with bullous pemphigoid, which will have a linear pattern ○ Tx? Oral steroids

Lab differentiation between primary and secondary hyperaldosteronism ○ Conn syndrome = primary hyperaldosteronism 48

○ ○

■ Aldosterone high ■ Renin low ■ Plasma aldosterone:renin ratio >20 Secondary hyperaldosteronism ■ E.g. young F with abdominal bruits OR resistant HTN + AV nicking → fibromuscular dysplasia ■ E.g. old man with abdominal bruits → renal artery stenosis ■ Aldosterone high ■ Renin high Dx for Conn syndrome? Plasma aldosterone:renin ratio Tx for Conn syndrome? Aldosterone antagonists (e.g. spironolactone, eplerenone)

Young F with abdominal pain relieved with defecation → IBS ○ Rome Criteria ■ Recurrent abdominal pain associated with 2 of the 3 ● Related to defecation ● Change in stool form ● Change in BM frequency ■ Are there lab abnormalities? NO ■ IBS Classification ● IBS-C → give stool softener ● IBS-D → give antidiarrheal (e.g. loperamide)

Cold vs Warm Agglutinins ○ Cold agglutinins → IgM ■ Bug? Mycoplasma ■ Tx? Cytotoxic drug (cyclosporine, azathioprine etc) ○ Warm agglutinins → IgG ■ Tx? Splenectomy ● Why? IgG opsonized things, splenic macrophages clear antibody-bound RBCs LDH, Bilirubin, and Haptoglobin levels in hemolytic anemia. ○ LDH up ○ Indirect bilirubin up ○ Haptoglobin down

32 yo Egyptian immigrant presents with a 2 week history of abdominal pain, hepatosplenomegaly, and hematuria. He swam in The Nile 2 months ago. → schistosomiasis ○ Bug? Schistosoma haematobium ○ Transmission? Direct contact with water ○ CBC finding? Elevated eosinophils ○ Associated malignancy? Squamous cell carcinoma of bladder ○ Tx? Praziquantel

Synpharyngitic glomerulonephritis → IgA nephropathy ○ Presentation? 2-6 days after URI ○ Contrast with PIGN, which occurs 2-6 weeks after URI ○ Lab findings? Low complement ○ Tx? steroids 49

○ ○

Urine microscopy findings? RBC casts Urine protein? 20) ○ R = RR > 30 ○ B = BP < 90/60 ○ Age > 65 Drugs commonly used in PNA treatment ○ Ceftriaxone ○ Levofloxacin ○ Macrolides - great for atypical PNA Pharmacological management of pulmonary arterial HTN ○ Endothelin antagonists ■ Bosentan ■ ambrisentan ○ PDE-5 inhibitors ■ Sildenafil ■ Tadalafil ○ Prostacyclin analogs ■ Iloprost ■ Epoprostenol ■ Treprostinil Causes? ○ Young female → idiopathic PAH ■ Mutation? BMPR2 55 yo F presents with a 5 week history of a rash on her forehead. PE reveals scaly macules with a sandpaper texture. → actinic keratosis ○ RF? Sun exposure ○ Tx? Topical 5-FU ○ Possible dangerous sequelae? Squamous cell carcinoma ○ Most likely disease sequelae? Resolution

57

1ary hyperparathyroidism

2ary hyperparathyroidism

3ary hyperparathyroidism

Autonomous PTH production

Low Ca++ → PTH production

PTH production despite normalized of Ca++ levels

Causes? Adenoma Parathyroid hyperplasia

Causes? CKD

PTH high Ca++ high Phos low

PTH high Ca++ low Phos high

Causes? CKD s/p transplant PTH high Ca++ high Phos low Tx? Parathyroidectomy (remove 3.5 glands) Cinacalcet (CSR modulator)

Hypercalcemia ○ Presentation? bones, stones, groans, psychic overtones ○ Tx? ■ 1st step: NS ■ Hypercalcemia of malignancy → bisphosphonates ○ EKG finding? Shortened QT

Periumbilical pain that migrates to the right lower quadrant. → appendicitis ○ PE findings? ■ McBurney’s point tenderness ■ Psoas sign ■ Obturator sign (pain with internal rotation of hip) ■ Rovsing’s sign (palpation of LLQ → pain in RLQ) ○ Dx? CT scan ■ Pregnant → US ■ Kid → US ○ Tx? Surgery

Classic drug and viral causes of aplastic anemia. ○ Drugs? ■ Carbamazepine ■ Chloramphenicol ○ Viral? Parvovirus B19 (single stranded DNA virus)

Fanconi anemia ○ Pathophys? Problems with DNA repair ○ Fanconi anemia vs Fanconi syndrome ■ Fanconi anemia → cytopenias + thumb anomalies + short stature + cafe-au-lait spots ■ Fanconi syndrome → type 2 RTA (proximal) 58

CD4 count of 94 + MRI revealing ring enhancing lesions in the cortex → toxoplasmosis ○ Tx? Pyrimethamine-sulfadiazine ■ Rescue agent for pt who becomes leukopenic with treatment? leucovorin ○ Who should get steroids? Increased ICP ■ For PCP pneumonia: ● O2 sat < 92 ● PaO2 < 70 ● A-a gradient > 35 ○ Ppx? TMP-SMX for CD4 < 100 ○ Congenital toxo ■ Hydrocephalus ■ Chorioretinitis ■ Intracranial calcifications ○ Classic methods of transmission? handling cat litter

Lupus nephritis ○ Associated autoantibody? anti-dsDNA ○ Classic “immunologic” description? “Full house” pattern ○ Tx? Steroids + cyclophosphamide

Osteoporosis ○ Screening population? women > 65 ○ Screening modality? DEXA scan ○ Dx? T-score < -2.5 ○ RF? ■ Postmenopauseal ■ Low BMI ■ Smoking ■ Alcohol ○ Preventive strategies? ■ Weight bearing exercise ■ Smoking cessation ■ Reduce alcohol consumption ○ Tx? ■ 1st line: bisphosphonates + Ca/Vit D supplementation ■ Raloxifene (SERM) ● Agonist in bone ● Antagonist in breast ○ Classic locations of osteoporotic fractures ■ Vertebral compression fracture ■ Hip fracture

Name the PNA ○ Red currant jelly sputum. → Klebsiella ○ Rust colored sputum. → Strep pneumo ○ PNA in an alcoholic. → Klebsiella ○ Post viral PNA with a cavitary CXR lesion. → Staph aureus ○ PNA in a patient that has chronically been on a ventilator. → Pseudomonas 59

○ MC cause of CAP. → Strep pneumo Pharmacological management of MRSA. ○ Vanc ○ Clinda ○ Linezolid ○ Ceftaroline (5th gen cephalosporin) ○ Tigecycline Pharmacological management of Pseudomonas. ○ Ceftazidime (only 3rd gen cephalosporin) ○ Cefepime (4th gen cephalosporin) ○ Pip-tazo ○ FQ ○ Carbapenems ○ Aztreonam ○ Aminoglycosides

JVD and exercise intolerance in a patient with a recent history of an URI. → dilated cardiomyopathy 2/2 viral myocarditis ○ MC cause? Coxsackie B ○ Drug causes? ■ Clozapine ■ Anthracyclines ● Prevention? Dexrazoxane (iron chelator) ■ Trastuzumab ○ Classic cause in a patient with recent history of travel to S. America? Chagas ■ Potential sequelae? ● Achalasia ● Dilated cardiomyopathy ● Megacolon (2/2 degeneration of myenteric plexus)

Massive skin sloughing (45% BSA) in a patient that was recently started on a gout medication? TEN ○ Dx? ■ 30% BSA → TEN ○ Tx? STOP the drug ■ IVF ■ Topical abx to prevention infection

Tetany and a prolonged QT interval in a patient with recent surgical treatment of follicular thyroid carcinoma. → hypocalcemia due to removal of parathyroids Recurrent viral infections + QT prolongation + tetany → DiGeorge syndrome ○ Pathophys? Failure of development of 3rd/4th pharyngeal pouches Trousseau and Chvostek signs. ○ Trousseau → inflation of BP cuff causes carpopedal spasm ○ Chvostek → taping on cheek causes facial muscle spasm Hypocalcemia that is refractory to repletion → consider hypomagnesemia Electrolyte/drug causes of prolonged QT intervals

● ●

● ●

60

Electrolytes? ■ Hypocalcemia ■ Hypomagnesemia ■ Hypokalemia ○ Drugs? ■ Macrolides ■ FQ ■ Haloperidol ■ Ondensatron ■ Methadone Hypoalbuminemia and Ca balance ○ Hypoalbumenia → decrease in total body Ca++, no change in ionized Ca++ ○ Drop of 1 in albumin → add 0.8 to Ca++

Abdominal pain radiating to the back → acute pancreatitis ○ Causes? ■ #1 = Gallstones ■ #2 = Alcohol ■ Hypertriglyceridemia ■ Scorpion sting ■ Handlebar injuries ○ Lab markers? ■ Lipase - most sensitive ■ Amylase ○ Physical exam signs in pancreatitis. ■ Cullen’s sign = periumbilical ecchymosis ■ Grey Turner sign = flank ecchymosis ■ ○ Tx? NPO + IVF + pain control ■ Meperidine is a good agent because it doesn’t cause sphincter of Oddi spasms ○ Management of gallstone pancreatitis ■ Dx? US then ERCP ■ Tx? DELAYED cholecystectomy ○ What if the patient becomes severely hypoxic with a CXR revealing a “white out” lung? ARDS ■ PCWP? 1 hr → RA. ○ Ab? ■ RF (IgM against IgG) ■ anti-CCP - more specific ○ HLA? DR4 ○ Pathophys? IgM constant region activates complement → inflammation → formation of pannus (hypertrophied synovium) → damage to cartilage and bone ○ Caplan syndrome = RA + pneumoconiosis ○ Felty syndrome = RA + neutropenia + splenomegaly (“RANS”) ○ Classic hand/finger findings/distribution? MCP & PIP joints of hands (DIP joints spared) ○ Imaging findings? ■ Symmetric joint space narrowing ○ Tx? Methotrexate (DMARDs) ■ If no response → TNF alpha inhibitor (e.g. infliximab) ○ Required testing prior to starting methotrexate? ■ PFTs ○ Required testing prior to starting infliximab? ■ TB ■ Hep B/Hep C

Differentiating Strep pharyngitis from Infectious Mononucleosis ○ LND distribution ■ Anterior cervical → Strep ■ Posterior cervical → Mono ○ Disease onset ■ Acute → Strep ■ Over weeks → Mono ○ Organ involvement ■ Splenomegaly → Mono Pt with sore throat takes amoxicillin and gets rash → mono ○ NOT allergic rxn! CENTOR criteria ○ C = absence of Cough ○ E = tonsillar Exudates ○ N = nodes/anterior cervical lymphadenopathy

● ●

62

○ ○

T = temp (fever) OR ■ =45 → -1 Using CENTOR score ○ 0/1 → don’t test, don’t treat ○ 2/3 → rapid antigen test ■ Positive → treat ■ Negative → throat culture ○ 4/5 → treat empirically Tx of Strep pharyngitis? ○ Amoxillcin ○ If PCN allergic → azithromycin Potential sequelae of Strep pharyngitis ○ RF - preventable with abx ○ PSGN Endocarditis ○ MC cause of endocarditis? IVDU ■ Bug? Staph aureus ■ Valve? tricuspid ○ Prosthetic valve endocarditis ■ Bug? Staph epidermidis ○ Endocarditis after dental procedure? Viridans group streptococci ■ Strep viridans, Strep mitis, Strep mutans, Strep sanguineous ○ Patient with malar rash and echo showing vegetations on both sides of the mitral valve → Libman-Sacks endocarditis ○ Presentation? Fever + night sweats + new murmur ■ Splinter hemorrhages ■ Roth spots (retinal hemorrhages) ■ Painless Janeway lesions + painful Osler nodes (immune phenomenon) ○ Dx? ■ 1st step: blood cultures ■ TEE ○ Tx? Abx that include Staph aureus coverage (e.g. vancomycin) for WEEKS ○ Bugs implicated in culture negative endocarditis ■ HACEK ● H = haemophilus ● A = actinobacillus ● C = cardiobacterium ● E = eikenella ● K = kingella ■ Coxiella burnetii ○ Blood cultures in a patient with endocarditis reveal S. Bovis (or S. Gallolyticus bacteremia). NBS? Colonoscopy ○ Who needs antibiotic prophylaxis? ■ Hx endocarditis 63

■ ■ ■

Prosthetic valve Unrepaired cyanotic congenital dz Heart transplant with valve dysfunction

Erythematous salmon colored patch with silvery scale on the elbows and knees. → psoriasis ○ Tx? Topical steroids ○ If this patient presents with joint pain (especially in the fingers)? Psoriatic arthritis ■ Imaging? Pencil-and-cup deformity ■ Tx? NSAIDs

T of 104 + tachycardia + new onset Afib in a patient with a history of Graves disease. → thyroid storm ○ Lab findings? ■ TSH low ■ T3/T4 high ○ Tx? ■ 1st step: propranolol ■ 2nd step: PTU ■ Then: ● Prednisone ● Potassium iodide (Lugul’s solution) ○ Wolff-Chaikoff effect → large amounts of iodine inhibit thyroid hormone synthesis

Biopsy revealing tennis racket shaped structures in cells of immune origin. → Langerhans cell histiocytosis ○ Electron microscopy? Birbeck granules (tennis rackets) ○ Marker? S100

Small bowel obstruction in a HIV patient with purple macules on the face, arms, and lower extremities. → Kaposi’s sarcoma ○ Bug? HHV8 ○ Tx? HAART ○ Pathophys of vascular lesions? Overexpression of VEGF

Fever + rash + eosinophiluria 10 days after a patient started an antistaphylococcal penicillin. → acute interstitial nephritis ○ Tx? STOP drug + steroids

SLE ○

SOAP BRAIN MD ■ S = serositis ■ O = oral ulcers ■ A = arthritis ■ P = photosensitivity ■ B = blood disorders (cytopenias) ■ R = renal ■ A = ANA/anti-dsDNA ■ I = immunologic ■ N = neurologic findings 64

■ M = malar rash ■ D = discoid rash ○ Type 2 vs 3 HSRs in lupus ■ Type 2 → cytopenias ■ Type 3 → all other manifestations ○ Lupus Ab? ■ ANA ■ anti-dsDNA ■ anti-Smith ○ Lupus nephritis → full house pattern on IF ○ Antiphospholipid antibody syndrome → recurrent pregnancy losses ■ Pathophys? Thrombosis of the uteroplacental arteries. ○ MC cause of death in lupus patients? ■ What I’ve read recently: CV disease ■ Per Divine: ● Treated → infection ● Untreated → renal dz ○ Also 40x risk MI ○ Endocarditis in lupus pt? Libman-Sacks endocarditis ○ Neonatal 3rd degree heart block → neonatal lupus ■ Maternal autoimmune dz? ● Sjogren’s ● SLE ■ Ab? ● anti-SSA/anti-Ro ● anti-SSB/anti-La ○ Tx? ■ Steroids ■ Cyclophosphamide ■ Hydroxychloroquine → good for skin lesions Pulmonary abscesses ○ Bugs? ■ Staph ■ Anaerobes ■ Klebsiella ○ RF? ■ Alcoholism ■ Elderly ■ Post-viral pneumonia ○ MC location of aspiration pneumonia? Superior segment of RLL Chest pain worsened by deep inspiration and relieved by sitting up in a patient with a recent MI or elevated creatinine or URI or RA/SLE. → pericarditis ○ EKG findings? Diffuse ST elevations + PR depression ○ PE finding? Friction rub (“scratchy sound on auscultation”) ○ A few days after MI → fibrinous pericarditis ○ Weeks after MI → Dressler’s 65

Tx? NSAIDS ■ Consider adding on colchicine

Cardiac tamponade ○ Beck’s triad = hypotension + JVD + muffled heart sounds ○ EKG findings? Electrical alternans ○ Type of shock? Obstructive cardiogenic (Amboss) ■ CO low ■ SVR high ■ PCWP high ○ Tx? Pericardiocentesis

Pearly lesion with telangiectasias on the ear in a farmer. → BCC ○ MC type skin cancer ○ Location? Upper lip ○ Dx? Biopsy ○ Tx? Mohs surgery

Cold intolerance in a 35 yo white F → hypothyroidism ○ MC cause? Hashimoto’s ○ Histology? lymphoid follicles w/ active germinal centers ○ Lab findings? ■ TSH high ■ T3/T4 low ○ Ab? ■ anti-TPO ■ Anti-thyroglobulin ○ HLA? DR3/DR5 ○ Tx? Levothyroxine ○ Future complication? thyroid lymphoma

Massive hematemesis in a patient with a history of chronic liver disease. → ruptured varices ○ Pathophys? L gastric vein has anastomosis with azygos veins. Increased portal pressure → backward flow from L gastric veins to azygous vein (which empties into SVC). ○ Acute tx? IVF + octreotide + ceftriaxone/cipro + EGD w/ ligation ■ Do NOT give a beta blocker for acute tx ○ Ppx? Beta blocker + spironolactone ○ Other manifestations of elevated portal pressures ■ Caput medusa ■ Internal hemorrhoids ○ Tx for cirrhotic coagulopathies? FFP ■ If uremia → give desmopressin Hemophilia A ○ Pathophys? deficiency of factor 8 ○ Inheritance? XLR ○ Coag labs? ■ Bleeding time normal ■ PTT HIGH 66

■ PT normal Hemophilia B ○ Pathophys? deficiency of factor 9 ○ Inheritance? XLR ○ Coag labs? ■ Bleeding time normal ■ PTT HIGH ■ PT normal Hemophilia C ○ Pathophys? deficiency of factor 11 ○ Inheritance? AR ○ Coag labs? ■ Bleeding time normal ■ PTT HIGH ■ PT normal Bernard Soulier Syndrome ○ Pathophys? Deficiency of GpIb ○ Coag labs? ■ Bleeding time HIGH ■ PTT normal ■ PT normal Glanzmann Thrombasthenia ○ Pathophys? Deficiency of GpIIbIIIa ○ Coag labs? ■ Bleeding time HIGH ■ PTT normal ■ PT normal Von Willebrand’s disease ○ Pathophys? Deficiency of vWF ○ Inheritance? AD ○ Coag labs? ■ Bleeding time HIGH ■ PTT HIGH ● vWF is a protecting group for factor 8 ■ PT normal ITP ○ Pathophys? Ab against GpIIbIIIa ○ Classic pt? Pt with SLE ○ Tx? ■ Observation ■ Steroids ■ IVIG ■ Splenectomy TTP ○ Pathophys? Deficiency in ADAMTS13 enzyme → cannot cleave vWF multimers → activation of platelets → thrombosis → thrombocytopenia ○ Presentation? microangiopathic hemolytic anemia + thrombocytopenia + renal failure + fever + neurologic problems 67

○ ○ HUS ○ ○

Tx? Plasma exchange transfusion

Bugs? Shigella or E. coli O157:H7 Presentation? microangiopathic hemolytic anemia + thrombocytopenia + renal failure

Platelet deficiency vs coagulation factor bleeds ○ Platelet deficiency → mucosal bleeds, petechiae, heavy menses ○ Coag factor deficiency bleeds → hemarthrosis

Why do patients with CKD have a coagulopathy? Uremia → platelet dysfunction ○ Tx? desmopressin Exercising caution with transfusion in patients with Bernard Soulier syndrome ○ Do NOT give transfusion that includes platelets ○ They can have an anaphylactic rxn to GpIb (since they don’t have GpIb)

Oropharyngeal candidiasis. ○ RF? ■ HIV ■ Chronic ICS use ■ TNF inhibitor ○ Micro finding? Germ tubes at 37C ○ Tx oral candidiasis? Nystatin swish-and-swallow ○ Tx invasive candidiasis? Amphotericin B ■ Prevention of Amphotericin B toxicity? Liposomal formulation

Pleural effusions ○ Light’s criteria (must meet all 3 to be considered transudative!) ■ LDH < 2/3 ULN ■ Pleural LDH/serum LDH < 0.6 ■ Pleural protein/serum protein < 0.5 ○ Causes of transudative effusion ■ CHF ■ Cirrhosis ■ Nephrotic syndrome ○ Causes of exudative effusion ■ Malignancy ■ Parapneumonic effusion ■ TB ○ Unique cause of both transudative & exudative effusions? PE ○ Classic PE findings? ■ Decreased breath sounds ■ Dullness to percussion ■ Decreased tactile fremitus ○ Tx? Chest tube Chylothorax = lymph in the pleural space ○ Pathophys? Obstruction of thoracic duct or injury to the thoracic duct ○ Pleural fluid findings? High TG

68

Holosystolic murmur heard best at the apex with radiation to the axilla in a patient with a recent MI. → mitral regurg 2/2 papillary muscle rupture ○ Dx? Echo ○ Why widely split S2? Aortic valve is closing earlier (LV is emptying into both aorta & LA) ○ Maneuvers that increase intensity ■ Increase preload (putting more blood in that can be regurgitated) ■ Increase afterload

Decubitus ulcers ○ RF? ■ Elderly ■ Paraplegic ■ Fecal/urinary incontinence ■ Poor nutrition ○ Staging ■ Stage 1 = non-blanchable erythema ● Tx? Repositioning q2hrs ■ Stage 2 = loss of epidermis + partial loss of dermis ● Tx? Occlusive dressing ■ Stage 3 = involves entire dermis, extending to subQ fat ● Does NOT extend past fascia ● Tx? Surgical debridement ■ Stage 4 = muscle/tendon/bose exposed ● Tx? Surgical debridement ○ ○

General tx strategies? Repositioning + good nutritional support Marjolin’s ulcer = non-healing wound that is actually squamous cell carcinoma

T1DM ○ Pathophys? Autoimmune destruction of pancreas ○ Ab? ■ anti-GAD 65 (glutamic acid decarboxylase) ■ anti-IA2 (islet tyrosine phosphatase 2) ■ Islet cell autoantibodies ■ Insulin autoantibodies ○ Dx? ■ A1c > 6.5% (twice) ■ Fasting BG >= 126 (twice) ■ Oral glucose tolerance test >= 200 (twice) ■ Sxs of DM + random glucose > 200 ○ Tx? Long-acting insulin + mealtime insulin ■ Long-acting ● Glargine ● Detemir ■ Rapid-acting ● Lispro ● Aspart 69

● Glulisine ○ 3 HY complications ■ Nephropathy ■ Retinopathy & cataracts ■ Neuropathy ○ Chronic DM care ■ A1c q3 months ■ Foot exam annually ■ Eye exam annually ■ Microalbumin:Cr ratio annually ○ Nephroprotection in DM? ACE-I GI bleed algorithm ○ 1st step: ABCs + 2 large-bore IVs + IVFs ○ 2nd step: NG lavage ■ Clear fluid → go deeper ■ Blood → UGIB → upper endoscopy ■ Bilious fluid → have ruled out UGIB → proceed to colonoscopy ● See source → intervene as needed ● See nothing → ○ CT angiography for large bleed ○ Tagged RBC scan for smaller bleed

Antiplatelet Pharmacology ○ Aspirin ■ Mechanism? Irreversibly inhibits COX-1 and COX-2 ○ Clopidogrel/ticlopidine = P2Y12 (ADP receptor) blockers ■ Mechanism? Inhibit platelet activation ○ Abciximab/eptifibatide/tirofiban = GpIIbIIIa receptor blockers ■ Mechanism? Inhibit platelet aggregation Ristocetin cofactor assay ○ Issues with adhesion step → abnormal result ○ Abnormal ristocetin cofactor assays: ■ Von Willebrand disease 70

● ●

■ Bernard Soulier disease ○ Normal ristocetin cofactor assay: ■ Glanzmann Thrombasthenia Von Willebrand disease effects on PTT? Increased ○ Pathophys? vWF is a protecting group for Factor 8. Treatment of VWD? Desmopressin ○ Mechanism? Increases release of vWF from Weibel-Palade bodies of endothelial cells

HSV1 vs HSV2. ○ Oral herpes → HSV1 ○ Genital herpes → HSV2 ○ Dx? PCR (most up-to-date) ■ Tzanck smear (outdated, not very sensitive, nonspecific) → intranuclear inclusions ○ Brain area affected by HSV encephalitis? Temporal lobes ○ CSF findings in HSV encephalitis? RBCs ○ Tx herpes encephalitis? Acyclovir ■ AE? Crystal nephropathy

Can’t see, can’t pee, can’t climb a tree. → reactive arthritis ○ HLA? B27 ○ Classic bug? Chlamydia ○ Tx? steroids ■ Need abx? Only if ongoing infection Can’t see, can’t pee, can’t hear a bee → Alport syndrome ○ Inheritance? X-linked dominant ○ Tx of NG & CT ○ NG → treat empirically for both → ceftriaxone + azithro/doxy ○ CT → azithro/doxy

Hypovolemic

Septic

Neurogenic

Cardiogenic

CO low PCWP low SVR high

CO high PCWP normal SVR low

CO low

CO low PCWP high SVR high

SVR low

Tx? norepi ●

Tx anaphylactic shock? epinephrine

Melanomas ○ ABCDE ■ A = asymmetry ■ B = irregular borders ■ C = color variation ■ D = diameter > 6 mm ■ E = evolving 71

Dx? Full-thickness biopsy ■ Excisional for small lesions ■ Punch for larger lesions Most important prognostic factor → Breslow depth

DM pharmacology ○ Lactic acidosis → metformin ○ Decreases hepatic gluconeogenesis → metformin ○ Hold before CT w/ contrast → metformin ○ Weight gain → sulfonylureas & TZDs (-glitizones) ○ Diarrhea → acarbose & migliton ■ Inhibits disaccharidases (can’t reabsorb disaccharides) ○ Recurrent UTIs → SGLT-2 inhibitors ○ Weight loss → GLP-1 agonists (e.g. liraglutide, exenatide) & DPP4 inhibitors (-gliptins) ○ Contraindicated in pt with HF → TZDs ■ PPAR-gamma receptor found in kidney → water retention ○ Contraindication in pt with MTC → GLP-1 agonists ○ Biggest risk of hypoglycemia? Sulfonylureas

RF esophageal adenocarcinoma ○ Barrett’s esophagus RF esophageal squamous cell carcinoma ○ Smoking ○ Drinking ○ Achalasia Location esophageal adenocarcinoma? Lower 1/3 Location esophageal squamous cell carcinoma? Upper 2/3 MC US? Adenocarcinoma MC worldwide? Squamous cell carcinoma Presentation? Dysphagia to solids → dysphagia to liquids Dx? EGD Staging? CT scan or esophageal US

● ● ● ● ● ● ● ●

● ● ● ●

Factor V Leiden ○ Pathophys? Resistance to protein C ○ Dx? Activated Protein C resistance assay Patient needs super large doses of heparin to record any changes in PTT → AT-III deficiency ○ Recall that heparin is a AT-III activator 35 yo with a hypercoagulable disorder that does not correct with mixing studies. → antiphospholipid antibody disorder Anaphylaxis in a patient with a long history of Hemophilia A → Ab against factor 8 that cause type 1 HSR with transfusion Hx of hemophilia, diagnosed 5 years ago. Before you would give them factor 8 concentrate and PTT would normalize. Now they’re requirizing larger doses of factor 8 to normalize PTT. → inhibitor formation (antibodies against clotting factors) Skin necrosis with Warfarin → protein C/S deficiency 72

Prothrombin G20210 mutation → overproduction of factor II

Rash in dermatomal distribution → VZV infection ○ Contraindications to VZV vaccination? ■ Pregnant woman ■ Kid < 1 year ■ Severe immunosuppression (e.g. HIV with CD4 < 200) ○ Tx? Acyclovir ■ If resistant, foscarnet ○ Tzanck smear findings? Intranuclear inclusions Shingles vaccination guidelines? Adults over 60

● ● ● ● ●

#1 cause of ESRD in the US → DM nephropathy ○ Histology? Kimmelsteil-Wilson nodules #2 cause of ESRD in the US → hypertensive nephropathy Pt with BP 240/150. How fast should you lower BP? 25% in first 24 hrs Drugs for hypertensive emergencies? ○ Nicardipine ○ Clevidipine ○ Nitroprusside ■ AE? Cyanide poisoning ● Tx? Amyl nitrate + thiosulfate OR hydroxycobalamin ○ Labelol Renal protective medications in patients with DKD or hypertensive nephropathy? ACE-I

Anemia + Cranial Nerve deficits + Thick bones + Carbonic Anhydrase 2 deficiency + Increased TRAP + Increased Alkaline Phosphatase. → osteopetrosis ○ Pathophys? Carbonic anhydrase is defective → osteoclasts cannot produce acid to resorb bone ○ Tx? IFN-gamma ■ Osteoclasts are a specialized macrophage ■ IFN-gamma is an activator of macrophages

Clinical diagnostic criteria for Chronic Bronchitis ○ Diagnostic criteria? ■ 2 years ■ 3 months/year of chronic cough ○ PFT findings ■ FEV1 low ■ FEV1/FVC ratio low ■ RV high ■ TLC high ○ Which PFT market can differentiate CB from emphysema? DLCO ■ DLCO normal → CB ■ DLCO low → emphysema ○ Tx acute exacerbation? Abx + bronchodilators + corticosteroids (“ABCs”) ○ Prevention? Stop smoking!

73

Afib ○ ○ ○ ○ ○ ○

○ ○

#1 RF? Mitral stenosis ■ #1 RF MS? Rheumatic fever MC arrhythmia in hyperthyroidism → Afib MC site of ectopic foci in Afib → pulmonary veins EKG findings? “Irregularly irregular” + no P eaves Location of emboli formation? LA appendage Who should be cardioverted back to sinus rhythm? ■ New onset (= 2 Anticoagulation options ■ Valvular cause (e.g. MS) → warfarin ■ Any other cause → warfarin or NOAC Reversal of AC ■ Warfarin → Vit K, four-factor PCC ■ Heparin → protamine sulfate ■ Dabigatran → idarucizumab

Crusty, scaly, ulcerating lesion with heaped up borders → squamous cell carcinoma ○ Classic location? Lower lip ○ Precursor lesion? Actinic keratosis ○ What if it arises in a scar or chronic wound? Marjolin ulcer

Hypothermia + hypercapnia + non pitting edema + hyponatremia + HR of 35 + hypotension in a patient with a history of papillary thyroid cancer → myxedema coma 74

○ ○

Tx? Levothyroxine + steroids Lab findings? ■ TSH high ■ T3/T4 low ■ LDL high

Acute onset “dermatologic” breakout in a patient with a recent history of weight loss and epigastric pain. → Leser–Trélat sign associated with visceral malignancy ○ Lymph node associations ■ Supraclavicular → Virchow’s node ■ Periumbilical → Sister Mary Joseph ○ What are mets to the ovaries called? Kruckenberg tumor ○ Classic bug associated with gastric cancer? H. pylori (MALToma) ○ Classic histological finding in the diffuse type of gastric cancer? Signet ring cells

RBCs without central pallor + elevated MCHC + anemia. → hereditary spherocytosis ○ Inheritance? AD ○ Pathophys? Deficiency of spectrin, ankyrin, or band 3.2 ○ Intravascular or extravascular hemolysis? Extravascular (RBCs bound by IgG, attacked by splenic macrophages) ○ Dx? ■ Osmotic fragility test ■ Eosin-5-maleimide ■ Acidified glycerol lysis test ○ Tx? Splenectomy ○ Post-splenectomy preventative care? ■ Strep pneumo ■ Hib vaccine ■ Neisseria

Septic shock ○ Hemodynamic parameters ■ CO high ■ SVR low ■ PCWP normal ■ MvO2 high ○ Tx? IVF + norepi + broad-spectrum abx (cover MRSA + Pseudomonas) ■ E.g. vanc + pip-tazo ■ E.g. vanc + ceftriaxone ○ qSOFA criteria (need 1) ■ AMS ■ RR > 22 ■ SBP < 100 ○ Acute kidney injury

Prerenal AKI

Intrarenal AKI

Postrenal AKI 75

● ● ●

● ● ●

● ●

FeNA < 1% BUN/Cr > 20 Urinary Na < 20

● ● ●

FeNA > 2% BUN/Cr < 20 Urinary Na > 40

Variable Early → like prerenal Late → like intrarenal

Cor Pulmonale ○ Pathophys? R heart failure from a pulmonary cause MC cause of right heart failure? L heart failure Papillary thyroid cancer ○ Histology? ■ Psammoma bodies ■ Orphan Annie eye nuclei Follicular thyroid cancer ○ Mode of spread? Hematogenous Medullary thyroid cancer ○ Tumor marker? Calcitonin ○ What accumulates in the thyroid? Amyloid ○ Histology? Apple green birefringence with congo red stain ○ Genetic disease associations? ■ MEN2A/B ● Mutation? RET Thyroid cancer with worst prognosis? anaplastic

Diagnostic algorithm for suspected thyroid nodule/cancer ○ TSH ■ Very low → hot nodule (unlikely cancer) ● NBS? RAIU scan ○ Single hot nodule → toxic adenoma ○ Multiple hot spots → toxic multinodular goiter ○ Diffuse uptake → Grave’s ○ No uptake → exogenous thyroid hormone ■ Normal/high → cold nodule ● NBS? Thyroid US ○ Find something → FNA (can’t r/o follicular thyroid cancer!)

Seizures after total thyroidectomy? Hypocalcemia 2/2 parathyroid gland removal

-----------------------------------------------------------------------------------------------------------------------------

Episode 31: Comprehensive Medicine Shelf Review. Session 3. ●

Epigastric pain + Jaundice + Courvoisier’s sign + Conjugated hyperbilirubinemia in a long term smoker. → pancreatic cancer ○ Courvoisier’s sign = nontender palpable gallbladder + painless jaundice ○ Dx? CT abdomen 76

○ ○ ●

Tumor markers (pancreatic)? ■ CA19-9 ■ CEA Migratory thrombophlebitis = episodes of vessel inflammation due to blood clot (thrombophlebitis) that appear in different areas throughout the body Relief of severe jaundice and pain in unresectable disease → ERCP to remove obstruction

What is the one commonly tested vitamin that should be routinely replenished in patients with hemolytic anemia? Folate ○ Why? They are constantly undergoing RBC DNA synthesis

Nephritic

Nephrotic

3.5 g protein/24 hrs Lipid casts Hypoalbuminemia Anasarca

Nephritic and nephrotic syndromes associations ○ Tram tracks → MPGN (membranoproliferative GN) ○ Subepithelial humps → PSGN ○ Kimmelstiel Wilson nodules → diabetic nephropathy ○ Subepithelial spike and dome → membranous nephropathy ○ Nephrotic Syndrome in a HIV+ patient → collapsing FSGS ○ Normal appearing glomeruli on light microscopy → minimal change ○ Apple Green Birefringence with Congo Red staining → amyloidosis

27 yo M was found down by his friend 13 hrs after they left a bar the night before. EKG is notable for peaked T waves. Creatinine is 4.5. → rhabdo ○ Pathophysiology? Rhabdo → K+ release from myocytes → hyperkalemia ○ Tx hyperkalemia? ■ 1st: calcium gluconate ■ Drive K+ inside cells ● insulin + glucose ● Albuterol ● Sodium bicarb ■ Reduce total body K+ ● Kayexalate ● Diuretic (e.g. loop or HCTZ)

Life threatening sinusitis in a patient with a HbA1C of 9%. → mucormycosis ○ Bug? Mucor or rhizopus ○ Tx? Amphotericin B

25 yo F with a history of T1DM presents with a history of recurrent abscesses in the axillary and perineal region. → hidradenitis suppurativa ○ Pathophys? Inflammation of apocrine sweat glands 77

MC cause of skin abscesses (bug)? Staph aureus ○ Tx? clinda

Total and fT4 levels in pregnancy. ○ Total T4 increases ■ Why? thyroxine-binding globulin goes up ○ Free T4 normal Best initial test in a diagnosis of hyperthyroidism? TSH Graves disease ○ Pathophys? Autoantibodies against TSH receptor RAIU results in Graves vs Toxic Adenoma vs Toxic Multinodular Goiter vs Surreptitious T4 intake. ○ Diffuse uptake → Graves ○ One spot → toxic adenoma ○ Multiple spots → toxic multinodular goiter ○ No uptake → exogenous T4 or Hash*toxicosis Differentiating between surreptitious T4 use and other hyperthyroid states. ○ Hash*toxicosis → high thyroglobulin (thyroid hormone was produced by thyroid) ○ Exogenous T4 → low thyroglobulin ○ Thyroglobulin = C-peptide of thyroid Antithyroid pharmacology ○ PTU & methimazole ■ Mechanism? Inhibit TPO

● ● ●

● ● ●

Gradations in the neoplastic potential of colonic polyps ○ Villous > tubulovillous > tubular Adenoma-Carcinoma sequence (AK-53) ○ APC → Kras → p53 Hyperpigmented macules on lip + GI tract hamartomas → Peutz-Jeghers syndrome ○ Associated malignancy? Colon, pancreatic, breast, gyn Sickle cell disease ○ Pathophys? Mutated beta-globin ○ Mutation? Glu to val ○ Vaccines? Encapsulated organisms (SHiN) ■ Why? Sickle cell pts tend to auto-infarct spleen by age 5 ○ Osteomyelitis in a SS patient → Salmonella ○ Painless hematuria in a SS patient (or SS trait) → renal papillary necrosis ■ Other non-SS causes? Chronic NSAIDS ○ Aplastic crisis in a SS patient → parvovirus B19 ○ Classic peripheral smear findings ■ Sickle cells ■ Howell-Jolly bodies ● Why? Spleen normally removes nuclear material in RBCs ○ Tx? Hydroxyurea ■ Mechanism? Inhibits ribonucleotide reductase

78

Worst headache of a patient’s life. PE is notable for palpable bilateral flank masses. → SAH in pt with ADPKD ○ Pathophys? Rupture of berry aneurysm ○ Mutation? PKD1/PKD2 ○ Associations ■ MVP ■ Berry aneurysm (circle of Willis) ● MC location? ACom ■ Liver cysts ■ Pancreatic cysts ○ Dx for SAH? Non-con head CT ■ If negative → LP (look for xanthochromia) ○ Prevention of post-SAH vasospam? nimodipine CREST syndrome

Diffuse scleroderma

C = calcinosis R = Raynaud E = esophageal dysmotility S = sclerodactyly T = telangiectasias

Ab? anti-Scl70, anti-topoisomerase

Ab? anti-centromere ● ●

Tx scleroderma renal crisis → ACE-I Nephrogenic systemic sclerosis association? Gadolinium (MRI contrast)

Sudden BP drop to 40/palpable in a patient who was recently placed on a ventilator with a PEEP of 15 cm H20. → tension pneumothorax ○ Tx? Thoracentesis (needle compression) 2nd intercostal space midclavicular line? Basic ventilator strategies ○ Hypercarbia → increase ventilation → increase RR or tidal volume ○ Improving hypoxia → increase FiO2 or increase PEEP

AV blocks ○ First degree/Mobitz 1 blocks generally require no treatment ■ Only treat if symptomatic ○ Mobitz 2/3rd degree blocks often require pacemaker placement. ○ In unstable patients with heart blocks, consider some sort of pacing.

79

HY differences between cellulitis and erysipelas ○ Cellulitis ■ Involved layers? dermis + SQ fat ■ Bugs? Staph aureus + GAS ■ Tx? Clinda ○ Erysipelas ■ Involved layers? Superficial dermis + lymphatic vessels ■ Bug? GAS ■ Presentation? More acute + more fevers ■ Appearance? Well-circ*mscribed, elevated compared to surrounding skin

G6PD deficiency ○ Pathophys? G6PD deficiency → problems with oxidative phase of the PPP → can’t produce NADPH ○ Classic triggers? ■ Stress ■ Infections ■ Fava beans 80

○ ○

■ Drugs: primaquine, dapsone Blood smear findings? ■ Heinz bodies ■ Bite cells Dx? Assay for G6PD activity AFTER hemolytic episode Inheritance? XLR

Rash after getting amoxicillin for a URI in a 19 yo M with posterior cervical lymphadenopathy. → mononucleosis ○ Blood smear findings? Atypical lymphocytes (“Downey cells”) ○ Dx? Monospot test ■ What if its looks like mono but monospot is negative? CMV ○ Associated malignancies ■ Burkitt’s lymphoma t(8,14) ■ Primary CNS lymphoma ■ Nasopharyngeal carcinoma

Heavy proteinuria + hyperlipidemia + hypoalbuminemia + EM revealing podocyte foot process effacement + normal glomeruli on LM. → minimal change disease ○ Malignancy associations? Hematologic malignancies

Dry eyes + Dry mouth in a 40 yo F who delivered a child with congenital heart block. → Sjogren’s disease ○ Ab? ■ anti-SSA/anti-Ro ■ anti-SSB/anti-La ○ Bx findings? Lymphocytic infiltrate ○ Tx? Artificial tears + pilocarpine (muscarinic agonist)

Panacinar emphysema + elevated LFTs in a 35 yo M. → alpha-1 antitrypsin deficiency ○ Pathophys? ■ Misfolded alpha-1 AT accumulates in hepatic cells. ■ Alpha-1 AT is a protease inhibitor, so lungs are more susceptible to proteases ○ Inheritance? Autosomal codominant ■ PiZZ is worst Liver bx finding? PAS-positive globules Tx? Pooled alpha-1 antitrypsin Prevention? NO SMOKING!

● ● ● ●

Systolic crescendo-decrescendo murmur heard best at the right 2nd interspace with radiation to the carotids. → aortic stenosis ○ Pathophys? Calcification of the valve leaflets ○ Early onset? Bicuspid aortic valve ○ Prognostic indicators by severity? “ASH” (pt will become ashes) ■ Angina ■ Syncope ■ HF sxs - worst (death within 1 year) ○ Maneuvers that changes murmur intensity ■ Increase preload → murmur increases ■ Increase afterload → murmur decreases 81

○ ○ ○

Dx? Echo Tx? Replace valve Associated anemia? Shearing RBCs ■ Lab findings? ● LDH increased ● Haptoglobin decreased ● Indirect hyperbilirubinemia Heyde’s syndrome = gastrointestinal bleeding from angiodysplasia in the setting of AS ■ Pathophy hypothesis? Calcified valve → cleavage of vWF multimers → issues with clotting

Acne vulgaris ○ Pathophys? Inflammation of sebaceous glands ○ Bug? Propionibacterium acnes ○ Stepwise management ■ 1st: topical retinoids ■ 2nd: topical abx ■ 3rd: oral abx ■ 4th: isotretinoid (Vit A derivative) ● Pregnancy test before starting ● 2 forms of birth control ● AE? ○ Teratogen ○ Hepatotoxic ○ Skin dryness ○ Adjunct considerations in treatment? OCPs in woman ○ Treatment of acne in patients with PCOS? OCPs or spironolactone

Congenital adrenal hyperplasia ○ 21-hydroxylase → virilization only ■ Female - virilization ■ Male - precocious puberty ○ 11-hydroxylase → HTN & virilization ○ 17-hydroxylase → HTN only ■ Hypokalemia ■ Only zona glomerulosa is working ○ Dx of 21-hydroxylase deficiency? Elevated 17-hydroxyprogesterone ○ Cosyntropin stimulation test → no rise in cortisol ○ Tx? Replace glucocorticoids & mineralocorticoids* (*if needed) ■ Don’t forget about stress dosing of steroids!

Increased direct bilirubin + Increased urine bilirubin + decreased urine urobilinogen + elevated alkaline phosphatase → biliary obstruction ○ Increased urine bilirubin because D-bili is water soluble ○ Urobilinogen is decreased because GI flora convert D-bili to urobilinogen Increased indirect bilirubin + No urine bilirubin + increased urine urobilinogen. → hemolysis ○ Urine bilirubin is NOT increased because I-bili in not water soluble ○ Urine urobilinogen increased because you are sending more bilirubin to the GI tract

82

Common causes of direct and indirect hyperbilirubinemia. ○ Direct ■ Biliary atresia ■ Primary sclerosing cholangitis ● Pathophys? destruction of intrahepatic and extrahepatic bile ducts ● Association? Ulcerative colitis ● Marker? p-ANCA ● Imaging finding? Beads on a string ■ Primary biliary cholangitis ● Pathophys? Destruction of intrahepatic bile ducts ● Ab? Anti-mitochondrial ○ Indirect ■ Gilbert syndrome ■ Crigler-Najjar syndrome ■ Hemolysis

High output heart failure in a patient with hemolytic anemia ○ Pathophys? Anemia → low O2 carrying capacity → have to increase CO to meet tissue O2 demand Classic bone cause of high output heart failure? Paget’s disease ○ Pathophys? Hypervascular Classic vascular cause of high output heart failure? AV fistula ○ E.g. from trauma ○ PE finding? Pulsatile mass

● ●

Lyme disease. ○ Bug? Borrelia burgdorferi 83

○ ○ ○

○ ○

■ Shape? spirochete Vector? Ixodes tick ■ Other things carried? Babesia & Anaplasma Classic skin presentation in early disease? Bullseye rash Classic presentations in later stage disease ■ Arthritis ■ Bell’s palsy ■ Heart block ■ Meningitis Dx? ELISA ■ Confirmatory test? Western blot Tx? Doxy for > 8 yo ■ Pregnant → amoxicillin ■ Kid 100 ● Stool osmotic gap = 290 - 2(Na + K) ● Large gap means that unmeasured non-electrolytes account for most of stool osmolality ○ Inflammatory diarrhea ■ Causes? ● UC ● Crohn’s ● Shigella 85

● ● ● ●

EHEC Salmonella Campylobacter Entamoeba histolytica

Reed Sternberg cells on lymph node biopsy → Hogkin’s lymphoma ○ Immune markers? CD15 & CD30 ○ What are the 4 types of this malignancy? ■ Lymphocyte predominant - best prognosis ■ Nodular sclerosing - MC type ■ Mixed cellularity ■ Lymphocyte depleted - worst prognosis ○ What does an increased number of RS cells correlate with? Worse prognosis ○ Associated bug? EBV ○ Dx? LN biopsy ○ Staging? ■ 1 LN → stage 1 ■ >1 LN on same side of diaphragm → stage 2 ■ >1 LN on opposites side of diaphragm → stage 3

Rheumatic fever ○ Diagnostic criteria ■ J = joints ■ O = heart ■ N = subcutaneous nodules ■ E = erythema marginatum ■ S = Sydeham’s chorea ○ Prevention? Abx for Strep pharyngitis ■ Abx do NOT prevent PSGN ○ Affect heart valve? “MAT” ■ MC = Mitral ■ 2nd MC = Aortic ■ 3rd MC = Tricuspid ○ Tx? penicillin/amoxicillin

Membranous nephropathy ○ Associated bug? Hep B/C ○ Associated malignancy? hematologic ○ EM finding? Subepithelial spike-and-dome ○ Sudden onset severe flank pain in a patient with a history of membranous nephropathy → renal vein thrombosis ■ Pathophys? Loss of AT-III in urine → hypercoagulable state Venous thrombosis sites ○ P. vera → hepatic vein thrombosis ○ Pt with pancreatitis → splenic vein thrombosis → gastric varices ○ Nephrotic syndrome → renal vein thrombosis

86

Dermatomyositis

Polymyositis

Kids CD4-mediated Perimysial/perifascicular inflammation

Adults CD8-mediated damage Endomysial inflammation

● ○ ○

○ ○

○ ●

Presentation? Proximal muscle weakness Skin findings associated with dermatomyositis ■ Heliotrope rash ■ Gottron’s papules ■ Shawl sign Associated malignancies? Lung Dx? ■ 1st step: check CK ■ If CK elevated → MRI muscle ■ If MRI equivocal → muscle biopsy & EMG Ab? Not involved in diagnostic pathway ■ anti-Jo ■ anti-Mi 2 ■ anti-SRP Tx? High-dose steroids

Sarcoidosis. ○ Demographic? African American female ○ CXR findings? Bilateral hilar lymphadenopathy ○ Associations ■ Eye → anterior uveitis ■ Skin → erythema nodosum ○ Pattern of lung disease? Restrictive ○ Lab findings? ■ Elevated ACE ■ Hypercalcemia ● Why? Macrophages produce 1-alpha-hydroxylase, which release in high calcitriol ○ Histology? Non-caseating granulomas 87

○ ○

TB skin testing → false negative Tx? Most people don’t require tx ■ Steroids if they need tx

Systolic vs diastolic heart failure ○ Systolic HF ■ Reduced EF ■ Eccentric hypertrophy ■ Volume overload ○ Diastolic HF ■ Normal/high EF ■ Concentric hypertrophy ■ Pressure overload Heart failure ○ Signs from history? Dyspnea, orthopnea, PND ○ Dx? echo ○ Tx acute CHF exacerbation? LMNOP ■ L = Lasix (furosemide) ■ M = morphine ● Can dilates coronary vessels ■ N = nitrates ■ O = O2 ■ P = positioning ○ Drugs shown to improve survival in HF patients? ■ Beta blockers: carvedilol, bisoprolol, ER metoprolol ■ Spironolactone ■ ACE-I/ARB ■ BiDil (isosorbide dinitrate + hydralazine) ○ HY Inotropes ■ Digoxin (inhibits Na-K ATPase) ■ Milrinone (PDE-3 inhibitor) Targetoid skin lesions in a patient with Mycoplasma Pneumoniae (or HSV) → erythema multiforme ○ Classic drug association? ■ Beta-lactams ■ Sulfonamides ■ Phenytoin Ectopic ACTH

Cushing’s disease (anterior pituitary overproducing ACTH)

Iatrogenic Cushing’s syndrome

CRH low ACTH high Cortisol high

CRH low ACTH high Cortisol high

CRH low ACTH low Cortisol high

Determining the cause of hypercortisolism with excessive ACTH secretion when the high dose dexamethasone suppression test is equivocal? Inferior petrosal sinus sampling ○ Elevated inferior petrosal ACTH → likely pituitary tumor 88

Tx of Cushing’s syndrome? Ketoconazole ○ Mechanism? Inhibits several enzymes in the steroid synthesis pathway

Fever + RUQ pain radiating to shoulder → acute cholecystitis ○ Dx? US ■ If negative but high suspicion? HIDA scan ○ Tx? Antibiotics + cholecystectomy ■ Antibiotics ● cipro + metronidazole ● MAG = metronidazole + amoxicillin + gentamicin

Cholecystitis without stones in a very sick pt → acalculous cholecystitis ○ RF? ■ Prolonged ICU stay ■ Prolonged TPN ● GB isn’t contracting a lot ○ Tx? Percutaneous cholecystostomy ■ Do NOT do surgery!!

Non-Hodgkin’s lymphoma ○ Follicular lymphoma ■ t(14, 18) ■ Pathophys? Overproduce BCL2 → reduced lymphocyte apoptosis ■ Biopsy findings? Lymphoid follicles ○ Burkitt’s Lymphoma ■ t(8,14) ■ Pathophys? C-myc overamplication ■ Biopsy findings? Starry sky pattern ■ Endemic presentation → jaw mass ■ Sporadic presentation → abdominal mass ○ Mantle cell lymphoma ■ t(11,14) ■ Pathophys? Overexpression of cyclin D1 → aberrant progression from G1 to S phase

TB ○

TB induration guidelines ■ > 5 mm ● Immunosuppressed ● HIV+ ● TB exposure ■ > 10 mm ● Incarcerated ● Healthcare worker ■ > 15 mm ● Everyone Dx? TST or IGRA ■ Positive skin test → CXR 89

○ ○ ○

○ ●

● ●

● ●

● Nothing on CXR → latent TB Tx active TB? ■ RIPE for 2 months ■ RI for 4 months Tx latent TB? INH for 9 months Vitamin supplementation? B6 TB drug adverse effects ■ Red secretions + hepatotoxic → rifampin ■ Nephrotoxic + hepatotoxic → isoniazid ● Use “INH” as mnemonic ■ Sideroblastic anemia → isoniazid ■ Drug-induced Lupus → isoniazid ● Ab? Anti-histone ■ Ethambutol → color blindness Where does TB love to reactivate? Upper lobes

Bartter’s Syndrome ○ Location? Thick ascending limb of loop of Henle ○ Pathophys? Na-K-2Cl transporter defective (like taking a loop diuretic) ○ Inheritance? AR ○ Lab findings? ■ Hypocalcemia Gitelman’s Syndrome ○ Location? DCT ○ Inheritance? AR ○ Pathophys? Na-Cl symporter is dysfunctional (like taking a thiazide diuretic) ○ Lab findings? ■ Hypercalcemia ○ Association? ■ CPPD Diastolic blowing murmur heard best at the left sternal border in the setting of a widened pulse pressure → aortic regurg ○ PE findings? ■ Waterhamer pulse ■ Head bobbing Wide pulse pressure in a kid → PDA Next step in the management of the following scenarios ○ 2+ systolic murmur → nothing ○ 3+ systolic murmur → echo ○ symptomatic 1+ systolic murmur → echo ○ diastolic murmurs → echo General diagnostic strategy for cutaneous fungal infections → KOH prep Tinea Versicolor ○ Bug? Malassezia furfur ○ Microscopy? Spaghetti & meatballs 90

○ Tx? Selenium sulfide topical 3 common causes of dermatophyte infection ○ Trichophyton ○ Microsporum ○ Epidermophyton Differences between the treatment strategies employed in tinea capitis vs tinea unguium vs tinea corporis ○ Tinea corporis → topicals ○ Tinea capitis / tinea unguium → oral (terbinafine or griseofulvin) Painful red lesion under skin folds → intertrigo ○ Bug? Candida

Beads on a string appearance with ERCP in a 30 yo M with a past history of chronic bloody diarrhea. → PSC in UC pt ○ Pathophys? Destruction or intrahepatic & extrahepatic bile ducts ○ Marker? p-ANCA ○ Tx? Liver transplant

HY antineoplastic associations ○ Pulmonary fibrosis → methotrexate, bleomycin, busulfan ■ Which works in the G2 phase? bleomycin ■ Other drugs causing pulmonary fibrosis? Nitrofurantoin, amiodarone ○ Methotrexate toxicity rescue agent → leucovorin ○ Inhibits thymidylate synthase → 5-FU ■ Toxicity worsened by leucovorin ○ Toxicity with allopurinol/febuxostat→ 6-MP/Azathioprine ■ Why? Metabolized by xanthine oxidase ○ Antineoplastic for pediatric cancers → Actinomycin D ○ Irreversible dilated cardiomyopathy → Doxo/Daunorubicin ■ Prevention? Dexrazoxane (iron chelator) ○ Hemorrhagic cystitis→ cyclophosphamide ■ Metabolic? Acrolein ■ Prevention? Mesna ○ Antineoplastic for brain cancer → nitrosoureas ○ Peripheral neuropathy → Vinca Alkaloids/Taxanes ○ Nephrotoxicity + ototoxicity → Cisplatin ■ Prevention of nephrotoxicity? Amifostine ○ Strongest association with n/v → Cisplatin ○ Inhibits ribonucleotide reductase → hydroxyurea ■ Indication? Sickle cell (increased HbF)

● ●

Most common HIV treatment regimen? 2 NRTIs + protease inhibitor/integrase inhibitor HY HIV drug side effects ○ Ritonavir → fat redistribution & CYP inhibition ○ Indinavir → nephrolithiasis ○ Other protease inhibitors → metabolic syndrome ○ Stavudine & didanosine → pancreatitis 91

○ ○ ○

AZT → bone marrow suppression Abacavir → life-threatening HSR ■ Check which HLA? HLA B57 NNRTIs like Efavirenz → teratogen & vivid dreams

55 yo M + hats don’t fit + mild deafness + high output heart failure → Paget’s disease ○ Lab findings? Elevated alk phos ○ Tx? Bisphosphates ○ Dx? Bone scan ○ Associated malignancy? Osteosarcoma ■ Genetic mutation that increases risk? Rb ■ Osteoporosis drug that increases risk? Teriparatide

50 yo F with a history of proximal shoulder pain + jaw pain with swallowing presents with a severe unilateral headache and eye pain. → temporal arteritis ○ NBS? Steroids ○ Dx? Temporal artery biopsy ■ AFTER giving steroids! ○ Classic demographic? Female > 50 ○ Lab findings? Elevated ESR

Intense pruritus between the finger webs and toes in a patient with poor grooming and hygiene. → scabies ○ Tx? Permethrin ■ Lindane can cause seizures

Type 1 diabetic with abdominal pain + ketones in urine → DKA ○ Presentation? ■ “DKA can cause Death Fast” ■ D = Delirium/altered mental status/psychosis ■ K = Kussmaul respirations (rapid and deep breathing) ■ A = Abdominal pain/n/v ■ D = Dehydration ■ F= Fruity breath ○ Pathophys? Absolute insulin deficiency → can’t inhibit glucagon ○ Lab findings? ■ AG metabolic acidosis → low pH ■ Ketones in urine ○ Contrast K and Na balance ■ Low total body K+ (even if hyperkalemic) ■ Hyperosmolar hyponatremia ○ Tx? IVF + regular insulin infusion + potassium repletion ○ When do you stop an insulin infusion? Anion gap has closed ○ Discussion of the different kinds of insulin ■ Long acting = glargine + detemir ■ Intermediate acting = NPH ■ Rapid acting = regular ■ Ultra rapid acting = lispro + aspart + glulisine 92

Risk of reducing glucose too quickly? Cerebral edema

Pruritus and RUQ pain in a 45 yo F with conjugated hyperbilirubinemia. → PBC ○ Pathophys? Destruction of intrahepatic bile ducts ○ Ab? Anti-mitochondrial ○ Tx? Ursodiol (improves survival) ■ Symptomatic tx of pruritus? Benadryl ○ Curative tx? Liver transplant

“Moonshiner” with abdominal pain, cognitive deficits per family, and wrist drop → lead poisoning ○ Anemia associated with this disease? Sideroblastic ■ Pathophys? ALAD and ferrochelastase inhibition ○ Blood smear findings? ■ Ringed sideroblasts ■ Basophilic stippling ○ Dx? Lead level ○ Tx? ■ Lead level 45-69 → Succimer = DSMA ■ Lead level > 70 → EDTA or dimercaprol ○ Other RF? ■ Lives in old house ■ Drinks from foreign pottery

Presentation of Shigella and EHEC diarrhea → bloody diarrhea ○ HUS triad = microangiopathic hemolytic anemia + thrombocytopenia + renal failure ○ Tx? Supportive ■ Do NOT give abx as this increases mortality!

Flank pain, palpable flank mass, hematuria, and polycythemia in a long term smoker → RCC ○ Associated hematologic phenomenon? Polycythemia 2/2 epo production ○ Potential testicular phenomenon? R-sided varicocele Carpal Tunnel Syndrome ○ Affected nerve? Median nerve ○ Distribution? Radial 3.5 digits ○ RF? ■ Pregnancy ■ Hypothyroidism ■ RA ○ Pathophys? Compression of median nerve by flexor retinaculum ○ Dx testing prior to surgery? Nerve conduction study ○ Tx? ■ 1st step = wrist splint ■ 2nd step = corticosteroid injection ■ 3rd step = surgery ○ Provocative exam maneuvers ■ Tinnel’s sign = tapping on the wrist causes sxs ■ Phalen’s sign = wrist flexion for 1 min causes sxs

93

Pulmonary embolism. ○ Virchow’s triad = stasis + hypercoaguability + endothelial injury ○ MC EKG anomaly? Sinus tachycardia ○ Classic EKG anomaly? S1Q3T3 ○ ABG findings ■ pH high (respiratory alkalosis 2/2 hyperventilation) ○ Dx? ■ Low risk pt → d-dimer ■ High risk pt → CTA ■ Pregnant pt → V/Q scan ○ Gold standard testing? Pulmonary angiography ○ Tx? ■ Stable → heparin ■ HDUS → tPA ■ HDUS with recent brain surgery → embolectomy ○ Anticoagulation review ■ Heparin ● Mechanism? Activates AT-III → inhibiting factor X and II ● Reversal? Protamine sulfate ■ Warfarin ● Mechanism? Inhibits Vit K epoxide reductase ● Reversal? Vit K + four-factor PCC ○ FFP if PCC is not an answer ■ Drop in platelets 5 days after starting heparin → STOP ALL heparin ● Start direct thrombin inhibitor (e.g. dabigatran, argatroban) ● Dx? anti-PF4 (platelet factor 4) ○ Confirmatory test? Serotonin release assay Pt with a big fracture that develops petechiae + respiratory distress → fat embolism syndrome

Diagnosis based on EKG above? A-flutter ○ Classic EKG description? “Sawtooth” pattern ○ Acute Tx? ■ Stable → vagal maneuvers then adenosine ■ HDUS → synchronized cardioversion 94

Long-term Tx? ■ Beta blocker ■ ND-CCB

4 types of hypersensitivity reactions ○ Type 1 = IgE-mediated ■ Pathophys? Antigen → IgE cross-linking → mast cell degranulation ○ Type 2 = antibody mediated ■ Pathophys? Ab binds Ag → complement activation ■ Example? ● Grave’s disease ● Autoimmune hemolytic anemia ○ Type 3 = immune complex mediated ■ Example? ● PSGN ● Serum sickness ○ Type 4 = delayed ■ Example? ● TB skin testing

Low urine osmolality + high serum osmolality in a patient with polyuria and polydipsia who recently started taking Li. → nephrogenic DI ○ Tx? ENaC blockers (e.g. amiloride, triamterene) ○ Other drug cause of nephrogenic DI? Demeclocycline ○ How is this disease differentiated from psychogenic polydipsia? ■ Psychogenic polydipsia has low serum osmolality

CD10/19/20+ + History of Down’s Syndrome + Positivity for Terminal Deoxynucleotidyl Transferase + May present as an anterior mediastinal mass in a teenager (CD3/7+) → ALL ○ Good prognosis translocation? t(12,21) ○ Bad prognosis translocation? t(9,22)

Watery or bloody diarrhea that lasts for a few days after consuming poultry/eggs or having exposure to turtles. → Salmonella enteritidis High fevers in a 7 yo in a developing country + “Rose” spots on the abdomen. → Salmonella typhi Salmonella antibiotic coverage strategies? FQ or macrolide

● ● ●

75 yo F presents with a 2 week history of neck pain and pain in her shoulders and hips bilaterally. ESR is markedly elevated. → polymyalgia rheumatica ○ How is this differentiated from fibromyalgia? Fibromyalgia has no lab abnormalities ○ Tx PMR? Low-dose steroids ○ Tx fibromyalgia? ■ TCAs ■ Duloxetine

Recurrent pneumonia in the same anatomic distribution in a long term smoker. → lung cancer ○ RF? #1 is smoking 95

○ ○ ○ ○

○ ○ ○

Dx central cancer? Endobronchial ultrasound Dx peripheral cancer? VATS Which lung cancer is NOT amenable to surgery? Small cell Type by location ■ Central → small cell, squamous cell ■ Peripheral → adenocarcinoma, large cell Edema of the face/arms + dilated veins on chest wall → SVC syndrome Ptosis + miosis + anhidrosis → Pancoast tumor causing Horner’s syndrome. ■ Pathophys? Tumor involvement of the sympathetic chain Testing before organ resection? FEV1 ■ Need to have enough pulmonary reserve to survive resection

Diagnosis based on EKG above? Multifocal atrial tachycardia ○ Classic EKG findings? irregular rhythm with rate >100 bpm and at least 3 P wave morphologies ○ Classic “disease” and “electrolyte” associations ■ COPD ■ Hypokalemia ■ Hypomagnesemia ○ Tx? ND-CCB ○ Should propranolol be used in the treatment of this arrhythmia? NO (pt has reactive airway disease) ○ Should a patient with this arrhythmia be treated with electrical cardioversion or amiodarone? NO

Skin lesion in the perineal region that shows up as a shiny papule with central umbilication in a patient with HIV. → molloscum contangiousum ○ Bug? Poxvirus ○ Tx? Salicylic acid ○ Transmission? contact

Serum hypoosmolarity and urine hyperosmolarity in a patient on Carbamazepine. → SIADH ○ Common causes? ■ Drugs ● SSRIs ● Cyclophosphamide ● Chlorpropamide (1st gen sulfonylureas) ■ Small cell lung cancer ○ Tx? Fluid restriction 96

○ ○

● ● ● ●

If resistant to fluid restriction? ● Demeclocycline ● ADH receptor antagonists (-vaptans) When should hypertonic saline be administered? Seizures + Na < 120 What happens when hyponatremia is corrected too quickly? Osmotic demyelination syndrome = central pontine myelinolysis

Elevated serum markers in liver disease ○ hepatic pattern: AST/ALT ○ cholestatic pattern: alk phos Which is more specific for liver disease (AST or ALT)? ALT What is the classic AST/ALT ratio in alcoholic liver disease? >2:1 In addition to alkaline phosphatase, what is a unique marker for obstructive liver lesions? GGT What is one HY serum marker that is somewhat unique to alcoholism? GGT

Hemophilia A and B ○ Pathophys? Deficiency of factor 8 or 9 ○ Inheritance? XLR ○ Coagulation labs? ■ Normal bleeding time ■ Elevated PTT ■ Normal PT ○ Mixing studies? ■ PTT usually corrects with mixing study ■ If it doesn’t correct, inhibitor present ○ Tx? Factor concentrates ○ Contrast with lab findings in VWD ■ VWD will have elevated platelet time ○ Pattern of bleeding with platelet vs clotting factor deficiency. ■ Platelet issue → epistaxis, bleeding gums, heavy periods ■ Factor deficiency → hemarthrosis, muscle hematomas

Rice water stools leading to circulatory collapse in a peace corps volunteer visiting some African or Asian country. ○ Bug? cholera ○ Diarrheal mechanism? Secretory diarrhea ○ Tx? Oral rehydration solution ■ Mechanism? Na-glucose symporter in GI tract In general (rough approximation), what is the most accurate diagnostic test in gastroenteritis? Stool culture

Renal mass in a patient with a history of cardiac rhabdomyomas and periventricular tubers. → angiomyolipoma in pt with tuberous sclerosis ○ Mutated proteins? Hamartin & tuberin

Chondrocalcinosis observed on a knee XR in a patient with a genetic history of a C282Y mutation in the HFE gene. → CPPD in pt with hemochromatosis ○ Classic joint aspirate findings? Rhomboid-shaped, positively birefringent 97

○ ○

● ● ● ●

■ Contrast with gout → needle-shaped, negatively birefringent What is the renal disorder that has a strong association with this disease? Giletman syndrome Tx? ■ 1st NSAIDs ■ 2nd steroids ■ 3rd colchicine

Fevers, dyspnea, and joint pain in a HIV+ patient that recently went on a cave expedition in Missouri. → Histoplasmosis ○ Bug? Histoplasma capsulatum ○ CXR findings? Interstitial infiltrates ○ Tx? ■ Milder → itraconazole ■ Disseminated → amphotericin B ○ Ppx? Itraconazole when CD4 < 150 Other prophylactic strategies in HIV+ patients. ○ Cocci → CD4 < 250 with itraconazole ○ PCP → CD4 < 20 with TMP-SMX, pentamidine, or atovaquone ○ Histo → CD4 < 150 with itraconazole ○ Toxo → CD < 100 with TMP-SMX ○ MAC → CD5 < 50 with azithro

Diagnosis based on EKK above? V-fib ○ NBS? Defibrillation + high quality chest compressions ○ Meds? Epi + amiodarone MC cause of death in the immediate period surrounding an MI? V fib Should a person in PEA or asystole receive defibrillation? NO Other rhythm that we should shock? Pulseless V tach 19 yo F presents with a large number of small, red macular lesions on the back arranged in a “Christmas tree” distribution. She had a similar but singular large lesion 1 week ago that cleared within a few days. → pityriasis rosea 50 yo M presents with erectile dysfunction, gynecomastia, low libido, and loss of peripheral vision. → pituitary adenoma, like producing prolactin ○ Tx? Dopamine agonists (bromocriptine or cabergoline) 98

■ Surgery is NOT first-line Other causes of hyperprolactinemia ○ Anti-psychotics (especially risperidone) ○ Hypothyroidism ■ Pathophys? Lack of negative feedback → high TRH → TRH is a releasing factor for prolactin Ascites ○ Exam findings? ■ Fluid ○ SAAG gradient = serum albumin - ascites albumin ■ SAAG < 1.1 g/dL causes ● malignancy ■ SAAG > 1.1 g/dL causes ● Portal hypertension ● Nephrotic syndrome ○ Tx? Paracentesis ■ Renal protection for large-volume para? Albumin Ascites with >250 neutrophils? SBP ○ NBS? Give abx (Ceftriaxone or FQ) ○ MC bug? E. coli ○ Ppx? FQ

Tetanus ○ Pathophys? Tetanus toxic prevents release of GABA and glycine from Renshaw cells → spastic paralysis ○ Contrast with botulism, which will cause flaccid paralysis ○ Should a booster vaccination dose be given if the patient’s last booster was administered 12 years ago? YES - booster q10 years ○ Acute management of the tetanus? ■ NO abx ■ Tetanus immune globulin ■ Tetanus vaccine

Recurrent pyelonephritis and nephrolithiasis in a patient with a low posterior hairline, breast underdevelopment, and a history of aortic coarctation. → Turner syndrome ○ Associations ■ Horseshoe kidney - stuck under IMA ■ Bicuspid aortic valve

Pleural thickening on chest CT, recurrent hemorrhagic pleural effusions, weight loss, and severe dyspnea in a patient that spent 30 years working at a shipyard. → mesothelioma ○ Does smoking increase the risk of this malignancy? NO ○ Most common lung malignancy in this population? Bronchogenic carcinoma ■ Does smoking increase the risk of this common malignancy? YES

Broad categorization and presentation of The Acute Coronary Syndromes. ○ Unstable angina ○ STEMI ○ NSTEMI 99

HPV ○

○ ○ ○ ●

Types ■ plantar warts → HPV 1,4 ■ genital warts → HPV 6,11 ■ cervical cancer → HPV 16, 18, 30s Transmission? Direct contact Tx? Liquid nitrogen Gardasil covers? 6, 11, 16, 18

What is true of total thyroid hormone and free thyroid hormone levels in the setting of OCP use? ○ Total → increase ○ Free → normal Quick overview of thyroid physiology ○ Import iodine ○ Oxidize I- to I2 ○ Organification (iodide to tyrosine → MIT & DIT) ○ Coupling of MIT & DIT → T3 and T4 How can factitious hyperthyroidism be distinguished from the transient hyperthyroidism of Hashimoto’s thyroiditis (based on labs)? Thyroglobulin ○ High → Hash*toxicosis ○ Low → factitious hyperthyroidism

Spontaneous bacterial peritonitis. ○ Bugs? ■ #1 = E. coli ■ Klebsiella ■ Strep pneumo ○ Dx? >250 PMNs on paracentesis ○ Tx? Ceftriaxone OR cipro ○ Ppx? FQ + albumin 100

Microangiopathic hemolytic anemia + fever + thrombocytopenia + elevated creatinine + neurologic deficits. → TTP (thrombotic thrombocytopenic purpura) ○ Pathophys? ADAMTS13 deficiency ○ Tx? Plasma exchange

Foul smelling diarrhea 1 week after treatment for a bacterial skin infection. → C. diff ○ Dx? Stool toxin assay ○ Tx? Oral vancomycin ■ Fidaxomicin ■ Fecal transplant ○ Prevention? Hand washing ○ A patient with the history described above deteriorates rapidly with severe abdominal pain and distension → toxic megacolon ■ NBS? surgery

Can’t see + Can’t pee + Can’t hear a high C. → Alport syndrome ○ Pathophys? Defect in type 4 collagen ○ Mutated gene? COL4A5 Small cell lung cancer ○ Paraneoplastic syndrome? ■ SIADH ■ LEMS ■ ACTH ○ Tx? Chemo ■ NOT surgical candidates Squamous cell lung cancer ○ Paraneoplastic syndrome? ■ PTHrP What are the 2 peripherally presenting lung malignancies? ○ Adenocarcinoma ○ Large cell MC lung cancer in smokers? Squamous cell Most likely lung ca in a 60 yo F with no history of smoking → adenocarcinoma Pt with hx smoking starts to have joint pain & finger clubbing → hypertrophic osteoarthropathy (paraneoplastic syndrome associated with lung cancer)

● ● ●

CP with exercise → stable angina ○ Dx? Stress test ○ Management? ■ Nitrates - decrease preload & myocardial O2 demand

Kayser-Fleischer rings in the cornea + Parkinsonian symptoms + Liver dysfunction + Decreased serum ceruloplasmin. → Wilson’s disease ○ Inheritance? AR ○ Chromosome? 13 ○ Mutated gene? ATP7B 101

Tx? Penicillamine or triamterene

Thrombocytopenia after heparin → HIT ○ Dx? anti-PF4 antibodies ○ Confirmatory test? Serotonin release assay ○ Tx? STOP heparin + give direct thrombin inhibitor ○ Is HIT a pro or an antithrombotic state? Prothrombotic

Osteomyelitis ○ MC cause of osteomyelitis → Staph aureus ○ Osteomyelitis in a sickle cell patient → Salmonella ○ Osteomyelitis with recent history of a cat or dog bite → Pasteurella ○ Dx? MRI ○ Tx? Prolonged tx, often requires abx ■ How to determine abx sensitivity? Bone culture Sinusitis (e.g. nasal polyps, saddle nose deformity) + hemoptysis + hematuria → Granulomatosis with polyangiitis (Wegener’s) ○ Marker? c-ANCA ○ Tx? Steroids + cyclophosphamide

Nerve Lesions ○ Surgical neck fracture of the humerus OR anterior shoulder dislocation → axillary nerve ■ Muscle? deltoid ○ Humeral midshaft fracture with wrist drop and loss of dorsal hand sensation → radial nerve ○ Medial humeral epicondylar fracture with failed finger spread + failed MCP joint flexion and IP joint extension for digits 4-5 → ulnar nerve ■ Muscles? Dorsal interossei + lumbricals

Stress test ○ Who should get a pharmacological stress test? Can’t exercise ○ Who should not get stress tests that involve EKG? Hx abnormal EKG ○ 2 pharmacological stress test methods ■ Dobutamine ■ Dipyridamole (coronary steal principle) ■ Regadenoson ○ Gold standard for CAD diagnosis → coronary angiogram (cardiac cath) ○ Principle behind stress echocardiograms? Looking for wall motion anomalies ○ Drugs to be held before stress testing? Beta blocker or ND-CCB

Hypocalcemia + Elevated PTH + Hyperphosphatemia + Short Stature + Short 4th and 5th metacarpals + Mental retardation. → Albright hereditary osteodystrophy (pseudohypoparathyroidism) ○ Pathophys? symptoms of hypoparathyroidism despite normal or elevated PTH levels due to end-organ resistance to PTH ○ Gene? GNAS1 ○ Tx? Vit D + calcium

Hepatitis virus matching game 102

○ ○ ○ ○ ○ ○

Fecal oral transmission → Hep A & Hep E Depends on Hep B for transmission → Hep D Virulent in pregnancy → Hep E Highest risk of a chronic carrier state → Hep C Transmitted by blood → Hep B & Hep C & Hep D Associated with chronic disease → Hep B & Hep C

Hep B serologies ○ General rules ■ Core Ab → exposure to virus (“scar”) ■ Surface Ag → currently infected ■ Surface Ab → has immunity (vaccine-mediated or natural) ○ Scenarios: ■ Immunized: +surface Ab ■ Prior infection (recovered): +surface Ab, +core Ab IgG ■ Acute infection: +surface Ag, +core Ab IgM, +eAg (high infectivity) ■ Chronic infection: +surface Ag, +core Ab IgG ■ Window period: +core Ab ● Surface Ag is disappearing and surface Ab is appearing ○ Definition of chronic infection? Hep B surface Ag for > 6 months

Recent viral infection or history of lupus + Isolated Thrombocytopenia. → ITP ○ Pathophys? Autoantibodies to GpIIbIIIa ○ Tx? ■ 1st line Steroids + IVIG ■ 2nd line spelectony ○ Should platelet transfusion ever be considered in these patients? NO ○ Drug that “replicates” the pathophysiology of the diagnosis? ■ Abciximab, eptifibatide, tirofiban ○ Deficiency that replicates the pathophysiology of the diagnosis? Glanzmann thrombasthenia ■ Ristocetin cofactor assay? normal

19 yo sexually active F with migratory arthritis + purpuric lesions on the lower extremities bilaterally. → disseminated gonococcal infection ○ Tx? Ceftriaxone + azithro/doxy

Hyponatremia ○ Hyperosmolar hyponatremia ■ Pathophys? Some other osmole present ■ Causes? ● Hyperglycemia ○ Iso-osmolar hyponatremia ■ Causes? (lab error) ● Hypertriglyceridemia ● Hyperproteinemia ○ Hypoosmolar hyponatremia ■ Hypervolemic 103

○ ○ ○

Causes? ○ CHF ○ Cirrhosis ○ Nephrotic syndrome ■ Euvolemic ● SIADH ● Hyporthyroidism ● Psychogenic polydipsia ● Tea & toast/beer potomania ○ Pathophys? Low solute intake ■ Hypovolemic ● Intrarenal (urine Na > 10) ○ Diuretics ○ Barter’s/Gitelman’s ● Extrarenal ( ○ Dehydration 2/2 vomiting/diarrhea ○ Burns Lab differentiation between intrarenal and extrarenal causes of hypovolemic hypoosmolar hyponatremia? ■ Urine Na > 10 → intrarenal ■ Urine Na < 10 → extrarenal Distinguishing between SIADH and psychogenic polydipsia ■ Dilute urine (Uosm < 100) → psychogenic polydipsia ■ Concentrated urine (Uosm > 100) → SIADH Risk of rapid Na+ correction? osmotic demyelination syndrome Indications for hypertonic saline? Seizures AND Na < 120 Correcting Na+ for hyperglycemia? Add 1.6 for every 100 glucose over 100 ■ Correction factor = 1.6 x (glucose - 100)/100

Bamboo spine on XR + Stiffness relieved with exercise + HLA-B27 positive + 25 yo guy + Sacroiliitis + Anterior Uveitis. → ankylosing spondylitis 104

Tx? NSAIDs ■ 2nd line TNF inhibitor or methotrexate

Central vs Obstructive Sleep Apnea ○ Central pathophys? Decreased respiratory drive ○ OSA pathophys? Blockage of upper airway ○ OSA RF? ■ Obesity ■ Large tonsils ○ Dx? Polysomnography ○ Tx? ■ OSA → CPAP + weight loss ■ CSA → acetazolamide ○ Surgical tx? uvulopalatopharyngoplasty ○ Complications/consequences ■ Pulmonary HTN ■ Hypoxia → Elevated Epo → Polycythemia Obesity hypoventilation syndrome ○ Lab findings? ■ Elevated CO2 on ABG

Athlete collapses suddenly on the field. → HOCM ○ Inheritance? AD ○ Mutations? Sarcomere proteins ■ beta myosin heavy chain ■ myosin binding protein C ○ Classic murmur presentation? Systolic crescendo-decrescendo murmur heard at LSB ■ What other murmur is heard here? AR ○ Maneuvers that increase and decrease murmur intensity ■ Anything that increases blood in LV → decrease intensity ● Increase preload ● Increase afterload ○ Tx? ■ Medical? Beta blocker to increase filling in diastole ■ Surgical? Septal myectomy ○ Associated GAA repeat AR disorder? Friedrich’s ataxia

A 35 yo business man returns to the US from a trip to Mexico. He has felt unwell for the past 2 weeks with a primary presentation of flu like symptoms. Scleral icterus is observed on exam. AST/ALT is approximately 5K each. IgM antibodies specific to a single stranded picornavirus are detected in his serum. → Hep A ○ Tx? Supportive care

Heavy menstrual bleeding + Normal PT + Elevated PTT + Increased bleeding time → Von Willebrand disease ○ Inheritance? AD ○ Pathophys? vWF deficiency ○ Why does PPT increase? vWF is a protecting group for factor 8 ○ Ristocetin cofactor assay? Abnormal 105

Tx? Desmopressin ■ Mechanism? Increases release of vWF for Weibel-Palade bodies

25 yo F who went hiking in Oklahoma presents with a 6 day history of high fevers, headache, and myalgias. Has a rash that started on the extremities and spread inwards. Now has involvement of the palms and soles. CBC notable for low platelets. → Rocky mountain spotted fever ○ Bug? Rickettsia rickettsii ○ Tx? Doxy ■ Kids < 8 → doxy ■ Pregnancy → chloramphenicol ○ Disease with rash on palms/soles ■ Kawasaki ■ Coxsackie B ■ Rickettsia ■ Syphilis

Acute onset exquisite pain/tenderness at the first MTP. → gout ○ Joint aspiration findings? Needle-shaped negatively bifrefringement crystals ○ Acute Tx? ■ 1st NSAID ● NOT aspirin (decreases excretion of uric acid) ■ 2nd Steroid ■ 3rd Colchicine ■ Do not start allopurinol, but continue it if they’re already on it ○ Chronic tx? ■ Xanthine oxidase inhibitors ● Allopurinol ● Febuxostat ■ Probenecid (increases uric acid excretion) ■ Converts uric acid to water-soluble allantoin ● Rasburicase ● Pegloticase ○ Tumor lysis syndrome ■ Prevention? Allopurinol ○ Drugs that increase azathioprine toxicity? Xanthine oxidase inhibitors

23 yo M smoker presents with a 6 month of history of mild dyspnea and productive cough. CT reveals cystonodular opacities in the upper and middle lung zones. A lung biopsy of one of the lesions is consistent with tennis shaped intracellular organelles. → Langerhans histiocytosis ○ EM findings? Birbeck granules

Diastolic dysfunction in a patient with a history of sarcoidosis, amyloidosis, or hemochromatosis. → restrictive cardiomyopathy ○ Pathophys? Infiltration of cardiac muscle with various substances ○ Dx? echo ■ Cardiac MRI ■ Endomyocardial biopsy 106

A high school student shadowing at a local hospital is mistakenly stuck by a needle from a patient who is known to have chronic Hep B infection. Testing 2 weeks ago revealed that the student was negative for all Hep B markers. ○ NBS? Give Hep B vaccine + immune globulin ○ If pt has been vaccinated? Don’t need to do anything ○ Tx chronic Hep B infection? BELT ■ B = Hep B ■ E = emtricitabine ■ L = lamivudine ■ T = tenofovir

Septic patient begins to bleed from IV Venipuncture sites. CBC is notable for elevations in PT and PTT. Fibrinogen levels and platelet count are low. A blood smear is positive for schistocytes. → DIC ○ Associated malignancy? APML ■ Histology? Aeur rods ■ Translocation? t(15,17)

21 yo presents with a 1 week history of fevers, fatigue, myalgias, and hemoglobinuria. She recently went hiking on a trail in Long Island, NY. Blood smear stained with Giemsa reveals “Maltese cross” shaped organisms within RBCs. → Babesia ○ Bug? Babesia microti ○ Vector? Ixodes tick ○ Tx? Atovaquone

Differentiating between muscle strain, lumbar spinal stenosis, degenerative disk disease, and cauda equina syndrome (unique features of each) ○ Paraspinal tenderness → muscle strain ○ Pain better when leaning over → spinal stenosis ○ Sciatica + positive SLR → degenerative disk disease ○ Loss of bowel/bladder fxn + saddle anesthesia + progressive neurologic deficits → cauda equina 65 yo M with no past history of smoking presents with a 6 mo history of dyspnea on exertion and fatigue. Lung auscultation reveals fine crackles. DLCO is markedly decreased. → idiopathic pulmonary fibrosis ○ Key PE finding? Fine crackles ○ Dx? high-CT chest ○ Imaging finding? honeycombing

Kind of “dysfunction” associated with dilated cardiomyopathy. → systolic ○ Common causes of DCM ■ Coxsackie B ■ Chagas ■ Anthracyclines ■ Trastuzumab ■ Alcohol (wet beriberi) ■ Postpartum cardiomyopathy ○ Extra heart sound associated with DCM? S3 ○ What is the bug that causes DCM, achalasia, and megacolonosis? T. cruzii ○ Dx? Echo 107

Tx? “BAD” ■ B = beta blocker ■ A = ACE-I ■ D = diuretics

Palpable purpura on the skin + Hypocomplementemia + Monoclonal IgM expansion in the serum + History of Hep C infection. → mixed cryoglobulinemia ○ Dx? PCR ○ Risk chronic hepatitis? Hep C > Hep B ○ Tx Hep C? ■ ledipasvir/sofosbuvir (Harvoni) ■ Pegylated interferon-alpha ● Screen for what? depression ■ Ribavirin ○ MC cause of hepatocellular carcinoma in US? Hep C ○ Derm manifestation? Porphyria cutanea tarda ■ Pathophys? UROD

Patient is rushed to the ED by ambulance from a restaurant after complaining of worsening respiratory difficulty and developing a diffuse rash. BP is 60/palpable. → anaphylaxis ○ NBS? IM Epinephrine ○ Type HSR? 1 ○ Pathophys? IgE crosslinking → mast cell degranulation ○ Mediating chemokine? histamine

Prerenal AKI ○ Pathophys? Hypovolemia → hypoperfusion of kidney → decrease GFR ○ Lab findings? ■ FeNa < 1% ■ Urinary Na < 20

● ● ●

Tx hypovolemic hypernatremia? NS until euvolemic then correct free water deficit Tx hypovolemic hyponatremia? NS Tx hypervolemic/euvolemic hyponatremia → fluid restriction

● ●

Drug of choice in the management of hypercalcemia of malignancy? bisphosphonates Bone met ○ Lytic ■ Multiple myeloma ■ Lung ■ Thyroid ■ Kidney/RCC ■ Breast ○ Blastic ■ #1 prostate ■ Breast ○ Dx? Bone scan ○ Presentation? Point tenderness over spinous process 108

Bilateral patchy infiltrates in a patient with a history of chronic treatment for symptomatic VTach. → amiodarone toxicity ○ Common drug causes of pulmonary fibrosis ■ Bleomycin ■ Busulfan ■ MTX ■ Nitrofurantoin ■ Amiodarone

Classic bug cause of hypersensitivity pneumonitis in a patient that works in a barn. → thermophilic actinomyces

Fevers + respiratory difficulty + negative sputum cultures + bilateral micronodular consolidations on CXR in a patient with a long history of rheumatoid arthritis. → bronchiolitis obliterans organizing pneumonia ○ Tx? steroids

● ●

What is the most important modifiable risk factor for atherosclerosis related heart disease? smoking Vitamin deficiencies associated with hyperhom*ocysteinemia ○ B12 ○ B9/Folate ○ B6 The 3 MEN syndromes ○ Inheritance? AD ○ Gene ■ MEN1 → MEN (menin protein) ■ MEN2 → RET ○ MEN 1 = “para pan pit” ■ Pituitary adenomas ■ Parathyroid hyperplasia ■ Pancreatic neuroendocrine tumors ● MC? Gastrinoma → Z-E syndrome ○ MEN2A ■ Parathyroid hyperplasia ■ MTC ■ Pheo ○ MEN2B ■ Marfanoid habitus ● Ddx? Marfan’s, hyperhom*ocysteinemia ■ Mucosal neuromas ■ MTC ■ Pheo ○ Screening for family members? ■ MEN1 → no ■ MEN2 → YES ○ Prevention of MTC? Thyroidectomy ■ 100% rick of MTC in MEN2 ○ Tumor marker MTC? Calcitonin

109

● ●

● ● ●

Esophageal varices ○ Ppx? Beta-blocker + spironolactone Tx of hepatic encephalopathy ○ Lactulose ■ Mechanism? ● Prevents NH3 reabsorption in the gut ● Lactulose metabolized by GI flora to lactic acid. Acidic environment favors NH4+, which is trapped in the colon and pooped out. ○ Rifaximin ■ Mechanism? Kills bugs that make ammonia What procedure increases the risk of hepatic encephalopathy? TIPS Spider angiomata association? Hyperestrogenism Pruritus after hot shower → polycythemia vera ○ Mutation? JAK2 ○ Lab findings? ■ EPO low ■ Hct high ■ O2 saturation normal ○ Differentiating PV from paraneoplastic EPO secretion ■ PV → low EPO ■ Paraneoplastic → high EPO ○ Complications ■ Budd Chiari syndrome = hepatic vein thrombosis ■ Gout ○ Tx? Phlebotomy until Hct < 45% + aspirin ■ For high-risk pts (age > 60, previous thrombosis): hydroxyurea

-----------------------------------------------------------------------------------------------------------------------------

Episode 32: Comprehensive Medicine Shelf Review Session 4 ●

Fever and absolute neutrophil count < 1500 in a patient on chronic treatment for Graves disease. → agranulocytosis 2/2 PTU or methimazole ○ Tx? STOP the drug ○ Abx for neutropenic fever? Broad-spectrum abx ■ Pip-tazo ■ Carbapenem ○ Common drug causes? ■ PTU/methimazole ■ Clozapine ■ Carbamazepine ■ Chloramphenicol

Common causes of hypokalemia? ○ Anything that makes you volume down → RAAS activation ○ Anything that causes afferent arteriole hypoperfusion 110

● ●

● ● ●

● ●

● ● ● ●

RTAs associated with hypokalemia ○ Type 1 = distal (can’t excrete H+) ○ Type 2 = proximal (can’t reabsorb bicarb) Type 4 RTA is hypoaldosteronism → can’t secrete K+ → hyperkalemia Hypokalemia/metabolic alkalosis and unrelenting HTN→ Conn’s syndrome ○ Pathophys? Adrenal adenoma that secretes aldosterone ■ HTN due to reabsorption of Na+/water ■ Metabolic alkalosis because H+-ATPase is upregulated by aldosterone ○ Consumption of what substance can mimic this? licorice ■ Substance? Glycyrrhizic acid ■ Mechanism? Inhibits 11-betaHSD-2 which converts cortisol to cortisone. Cortisol has mild agonist activity and mineralocorticoid receptors. Excess cortisol acts like aldosterone → hypokalemia & metabolic acidosis Causes of hypokalemia and a metabolic alkalosis? Most diuretics Diuretic that causes hypokalemia and metabolic acidosis? Acetazolamide Genetic renal causes of hypokalemia? ○ Barter’s ○ Gitelman’s ○ Liddle’s ■ Pathophys? Activating mutation in ENaC ■ Inheritance? AD EKG findings in hypokalemia ○ Flat T waves + U waves Electrolyte causes of QT prolongation ○ Hypokalemia ○ Hypocalcemia ○ Hypomagnesemia Tx Torsades de Pointe? IV Mag What if you’re repleting K+ and it’s not really working? Likely hypomagnesemia How to replete K+? Oral preferred 45 yo M lifting boxes presents with sudden onset radicular pain. Straight leg raise is +ve. The achilles reflex cannot be elicited on exam. → herniated disk ○ Most likely involved nerve root? S1 ○ Level of herniation? L5-S1 ○ Pathophys? Herniation of nucleus pulposus → nerve root compression ○ Tx? PT + NSAIDs ○ What is not recommended? Bedrest or surgery HY nerve roots ○ C5 = triceps ○ C6 = brachioradialis ○ C7 = triceps ○ L4 = patellar ○ S1 = Achilles Pulmonary arterial HTN ○ Idiopathic PAH classic patient demographic? Young female ○ Gene mutation? BMPR2 111

○ ○ ○ ○ ○

● ● ● ● ● ● ●

MC cause of PAH in US? COPD ■ Pathophys? Hypoxic vasoconstriction Dx? R heart cath PA pressure cutoff? PA pressure > 25 mmHg Classic auscultatory finding? Loud S2 Pharmacological management of idiopathic PAH ■ Endothelin antagonists (e.g. bosentan, ambrisentan, atrasentan) ■ PDE-5 inhibitors (e.g.sildenafil) ■ Prostacyclin analogs (e.g. iloprostol, epoprostenol)

Dyslipidemia screening guidelines ○ High-risk individuals starting at age 20 ○ Males starting at age 35 ○ Females starting at age 45 4 general groups of individuals who should be placed on statins ○ LDL > 190 ○ Existing ASCVD ■ MI, stroke, PAD ○ Age 40-75 with ASCVD risk > 7.5% ○ Age 40-75 with DM Lipid lowering pharmacology associations ○ Flushing → niacin ■ Tx? NSAIDs (or just wait it out) ○ Best drug for LDL lowering → statins ■ Mechanism? HMG CoA reductase ○ Best drug for lowering TGs → fibrates ○ Best drug for raising HDL → niacin ○ Hepatotoxic drugs → statins & fibrates (especially when given together) ■ Check LFTs when starting pt on statin ■ Only check again if pt has sxs ○ Diarrhea → bile acid resins (e.g. cholestyramine, colesevelam) ○ Inhibits reabsorption of cholesterol in the GI tract → ezetimibe Liver abscess + bloody diarrhea in an African immigrant. → Entamoeba histolytica ○ Tx? Metronidazole ■ Well-circ*mscribed liver lesion → inject iodoquinol Tx of hydatid cysts (Echinococcus)? Albendazole Bug that causes cholangiocarcinoma? Clonorchis sinensis Painless chancre + hard base + indurated margins → syphilis Painless ulcer with a beefy red base and irregular borders → granuloma inguinale ○ Bug? Klebsiella granulomatis Painful purulent ulcer with soft, ragged edges → Haemophilus ducreyi Painful ulcer with surrounding vesicular lesions → genital herpes Stages of syphilis ○ Primary 112

○ ○

○ ○

○ ○ ○

■ Presentation? Painless chancre Secondary ■ Presentation? Rash on palms/soles Tertiary ■ Presentation? ● Argyll-Robertson pupil ● Aortic regurg ● Tabes dorsalis ○ Test for posterior column dysfxn? Romberg Tx? Penicillin If penicillin-allergic? Doxy or macrolide ■ If pregnant? Desensitize ■ If neurosyphilis? Desensitize Screening test? RPR or VDRL ■ Cause of false-positive? Antiphospholipid syndrome Confirmatory test? FTA-Abs Classic presentation of primary, secondary, and tertiary syphilis

Hyperkalemia ○ Common causes? ■ Potassium-sparing diuretics ● Spironolactone/eplerenone ● Triamterene/amiloride ● ACE-I/ARBs ■ Addison’s ■ Tumor lysis syndrome ● Renal complication? Uric acid stones ● Prevention? Allopurinol or ■ Rhabdo ■ Common causes of hyperkalemia ○ Tx? ■ 1st step = calcium gluconate ■ Reduce serum K+ ● Insulin + glucose ● Albuterol ● Sodium bicarb ○ Alkalosis draws H+ out of cell, K+ goes in ■ Reduce total body K+ ● Diuretics ● Kayexalate ○ AE? Bowel necrosis ○ Pt with serum potassium of 8 but normal EKG → hemolyzed sample ○ EKG findings? Peaked T waves → wide QRS → sinu wave

● ●

Wrist drop after a humeral fracture. → radial nerve injury Tenderness over the anatomical snuffbox. → scaphoid fracture ○ Complication? Avascular necrosis due to retrograde blood supply Loss of lateral arm sensation and abductive ability. → axillary nerve injury

113

Causes? ■ Anterior shoulder dislocation ■ Surgical neck fracture

COPD ○ Pathophys? Imbalance of proteases vs antiproteases ■ E.g. alpha-1 antitrypsin is an antiprotease ○ PFT findings? ■ FEV1 decreased ■ FEV1/FVC ratio < 70% ■ Lung volumes high ○ Differentiating b/w chronic bronchitis and emphysema based on PFTs ■ Normal DLCO → chronic bronchitis ■ Reduced DLCO → emphysema ○ ABG findings? respiratory acidosis w/ compensatory metabolic alkalosis ■ Elevated CO2 ■ Increased bicarb ○ CBC findings? Polycythemia ■ Pathophys? Hypoxia → EPO production ○ Interventions shown to reduce mortality in COPD ■ Smoking cessation ■ Home O2 ○ Indications for home O2 ■ O2 Sat < 88% at rest ■ PaO2 < 55 ■ Pulmonary HTN ■ Polycythemia ○ Treatment of a COPD exacerbation ■ A = antibiotics (azithro or levofloxacin) ■ B = bronchodilators ■ C = corticosteroids ○ When should you use Non-Invasive Positive Pressure Ventilation (NPPV)? hypercarbia ○ Stepwise chronic management of COPD ■ SABA/SAMA/SABA-SAMA ■ LAMA (e.g. tiotropium) ■ LAMA-LABA ● Alternative: LABA-ICS ■ Roflumilast (PDE-4 inhibitor) ○ Why don’t you want to get O2 to 99% in COPD pts? ■ Pathophys? Pts with COPD have a hypoxic respiratory drive (rather than a hypercarbic respiratory drive because they are chronic CO2 retainers)

HTN ○ ○

Dx? BP > 130/80 on TWO occasions ■ Most accurate is 24 hr ambulatory monitoring Lifestyle modifications (from most to least effective) ■ Weight loss ■ DASH diet 114

■ Exercise ■ Na+ restriction Special antihypertensive indications ■ DM → ACE-I ■ CKD → ACE-I ■ HF or hx MI → ACE-I ■ Hx kidney stones → thiazides ■ BPH → non-selective alpha-1 blocker (e.g. prazosin, not tamsulosin) ■ African American with no other conditions → thiazides ■ Africna American w/ DM, HF, of hx MI → ACE-I ■ Pregnant → nifedipine Contraindications to antihypertensives ■ C1 esterase inhibitor deficiency → NO ACE-I ■ Bilateral renal artery stenosis → NO ACE-I Antihypertensive meds in pregnancy ■ Hydralazine ■ Methyldopa ■ Labetalol ■ Nifedipine

Fever + abdominal pain + shoulder pain + pleural effusion on CXR + recent history of small bowel surgery. → subphrenic abscess ○ Pathophys? Referred pain from diaphragm to shoulder

Reversal of anticoagulation ○ Heparin → protamine ○ Warfarin → Vit K & four-factor PCC ○ Dabigatran → idarucizumab Should LMWH be used in the treatment of HIT? NO! ○ Tx HIT = direct thrombin inhibitor (e.g. dabigatran) Should dabigatran or bivalirudin be used in the treatment of valvular Afib? NO! ○ Only warfarin for valvular Afib

● ●

Recurrent seizures in a patient who recently returned from a trip to Venezuela and consumed pork → neurocysticercosis ○ Tx neurocysticercosis? Albendazole + corticosteroids ○ Tx Taenia solium GI infection? Praziquantel

Hypercalcemia ○ MC cause? Primary hyperparathyroidism (parathyroid adenoma) ○ Other causes? ■ Thiazides ■ Malignancy ○ Signs of hypercalcemia? “Stones, bones, groans, psychic overtones” ○ EKG findings? Shortened QT ○ First step in management of symptomatic hypercalcemia? Fluids

115

Differentiating b/w primary hyperparathyroidism and FHH ○ Urinary Ca+ high → primary hyperpara ○ Urinary Ca+ low → FHH Tx? ○ Hypercalcemia of malignancy → bisphosphonates ○ Criteria for parathyroidectomy for primary hyperpara? ■ Symptomatic ■ Age < 50 ■ Calcium > 1 mg/dL over ULN ■ Renal dysfunction 1ary hyperpara

Hypercalcemia of malignancy

Renal dz (2ary hyperpara)

3ary hyperpara

Liver dz

Familial hypocalciuric hypercalcemia

Milk alkali syndrome

PTH high Ca+ high Phos low

PTH low (suppressed) Ca+ high Phos low

PTH high Ca+ low Phos high

PTH high Ca+ high Phos low

PTH high Ca+ low Phos low

Ca+ high UCa+ low Phos low PTH low

Ca+ high PTH low

pH = 7.25, pCO2 = 65, HCO3- = 29. → respiratory acidosis w/ compensatory metabolic alkalosis ○ Drug overdose? Opioids & benzos ○ Chronic disease? COPD

● ●

HTN in a patient with Turner’s syndrome. → coarctation HTN in the setting of hypokalemia, metabolic alkalosis, and an elevated plasma aldosterone to renin ratio. → Conn’s syndrome ○ Tx? Surgery ■ Spironolactone prior to surgery Episodic HTN in a patient that is accompanied by headaches. → pheo HTN in a 36 yo sexually active F. → OCPs HTN in a patient on long term treatment for osteoarthritis with NSAIDS. → renal dysfunction (NSAIDs → afferent arteriole constriction → RAAS activation) HTN in a 25 yo F with an abdominal bruit heard on exam → fibromuscular dysplasia HTN in 70 yo M with an abdominal bruit heard on exam + fundoscopic finding of AV nicking → renal artery stenosis

● ● ● ● ●

70 yo F with a creatinine of 3 + bone pain + lytic lesions seen on XR + Ca of 12.9 mg/dL + Hb of 8.1. → multiple myeloma ○ CRAB mnemonic ■ C = hypercalcemia ■ R = renal failure ■ A = anemia ■ B = bone pain ○ Dx? SPEP/UPEP ○ Blood smear findings? Rouleaux formation ○ Bortezomib mechanism? Proteasome inhibitor ■ Garbage proteins build up and kill the MM cells 116

How is this disorder differentiated from MGUS? ■ 10% plasma cells on bone marrow bx → MM ■ Presentation MGUS? MGUS will NOT have CRAB sxs ■ Prognosis MGUS? 1% progress to MM every year How is this disorder differentiated from Waldenstrom Gammaglobulinemia? ■ Waldenstrom’s is an IgM monoclonal gammopathy ■ Complications? Hyperviscosity syndrome

Cyclical fevers + anemia + headaches + hepatosplenomegaly in a patient that recently returned from an African country. ○ Bug? Plasmodium species ○ Vector? Anopheles (female mosquitoes) ○ Dx? Thick & thin blood smear ■ Stain? Giemsa ○ Species with hypnozoite forms? P. vivax & P. ovale ○ Tx of hypnozoite forms? Primaquine ○ Species that causes the most severe disease? Falciparum ○ Tx? ■ Chloroquine - high resistance ■ Mefloquine ■ Primaquine ● Causes hemolysis in G6PD deficiency ■ Atovaquone ■ Artemether/lumefantrine

Prolonged QT and carpopedal spasms with manual BP checks 24 hrs after a thyroidectomy for papillary thyroid cancer. → hypocalcemia 2/2 parathyroid removal Same presentation in a patient that recently got 9 units of blood for severe hemorrhage→ EDTA chelation of Ca+ + Hypocalcemia ○ EKG finding? QT prolongation ○ Mg interaction? If pt is hypomagnesemic, they won’t respond to calcium repletion Hypocalcemia + elevated blood phosphate → CKD Hypocalcemia + low blood phosphate → liver disease or Vit D deficiency Alkalosis & calcium balance? Alkalosis causes hypocalcemia ○ Mechanism? Alkalosis → negative charges on albumin → binds Ca++ Tx? Calcium + Vit D

● ●

● ● ● ● ●

● ●

Next best step in the management of a patient presenting with hypercapnia and RR of 8 in the setting of multiple rib fractures. → pain control (e.g. nerve block) ○ Should rib fractures be casted? NO pH = 7.52, pCO2 = 31, HCO3- = 24 → respiratory alkalosis ○ Causes? Hyperventilation Acid base anomaly associated with aspirin overdose? Respiratory alkalosis + metabolic acidosis ○ Hyperventilation → respiratory alkalosis ○ Salicylic acid → AG metabolic acidosis 117

Hyponatremia Discussion ● Hyperosmolar ○ Causes? Hyperglycemia (e.g. DKA or HHNS) ○ Pathophys? Glucose is an osmole and draws water into vascular space → decreased sodium concentration ○ Na+ correction for hyperglycemia? add 1.6*(BG - 100)/100 ● Iso-osmolar ○ Causes? Hyperlipidemia & hyperproteinemia ○ Why? Lab artifact ● Hypoosmolar ○ Hypervolemic ■ Pathophys? Decreased intravascular volume → hypoperfusion of afferent arteriole → aldosterone & ADH release* → increase in Na+ and increase in water, but gain more water than Na+ ● *Non-osmotic release of ADH (due to hypovolemia) ■ Causes? ● Cirrhosis ● Nephrotic ● CHF ■ Tx? Restrict fluids ○ Euvolemic ■ Pathophys? Pt is gaining just water ● Why doesn’t pt get hypovolemic? Water intake → serum osmolality down → water redistributes into cells ■ Cause? ● SIADH ○ Serum osm? Low ○ Urine osm? High (>100) ● Hypothyroidism ● Psychogenic polydipsia ○ Serum osm? Low ○ Urine osm? Low ( 10 ● Vomiting/diarrhea (loss of electrolyte-rich fluid) ○ Urinary Na+? 20 → Pseudomonas ○ Dx? Sweat chloride testing ■ Serum trypsinogen (low due to obstruction of pancreatic ducts by thick secretions) ○ Tx? ■ N-acetylcysteine (breaks disulfide bonds → mucolytic) ■ Dornase alfa ■ Nebulized tobramycin ■ Chest physiotherapy ■ Vit ADEK supplementation ■ Nutritional support ○ Associations ■ Nasal polyps ■ Rectal prolapse ■ Congenital absence of the vas deferens Severe chest pain at night in a 25 yo F smoker with a history of migraines. → Prinzmetal angina = variant angina ○ Pathophys? Vasospasm of the coronary vessels ○ Dx? Provocative testing w/ ergotamine in the cath lab ■ If pt has ST elevations & elevated troponins → cath lab ○ Tx? CCB (per UTD, diltiazem or amlodipine) ○ Contraindicated drugs ■ Sumatriptan ■ Ergots ■ Nonselective beta blockers (e.g. propranolol) ● Can exacerbate vasospasm

Nasty anal pruritus in a child. → enterobius vermicularis (pinworm) ○ Dx? Scotch tape test ○ Tx? Albendazole or pyrantel pamoate

65 yo F presents with signs of pancytopenia. Bone marrow biopsy reveals > 20% blasts. Cells are positive for myeloperoxidase. Auer rods are found on biopsy. → APML ○ Translocation? t(15,17) ○ Tx? ATRA ○ Serious complication? DIC

Protozoal cause of bloody diarrhea and liver abscesses. → entamoeba histolytica ○ Tx? metronidazole ■ If abscess? Inject iodoquinol What is the bug that increases the risk of cholangiocarcinoma? Clonorchis sinensis

Hypoxemia with a normal A-a gradient 120

● ● ●

○ Hypoventilation ○ High altitude Hypoxemia with a elevated A-a gradient ○ Shunt ○ V/Q mismatch ○ Diffusion issue (e.g. pulmonary fibrosis) EKG leads and MI correlations. ○ II, III, avF → inferior MI / RCA ○ I, aVL, V5, V6 → lateral MI / LCx ○ V1-V4 → anterior MI / LAD What should be avoided in a patient with a right sided infarct? nitrates ○ These pts are preload dependent Most sensitive serum marker for MI? troponin Diagnosing a reinfarction? CK-MD (short half-life)

75 yo M with recurrent infections. CBC is notable for a lymphocyte count of 109k. PE reveals anterior and posterior cervical lymphadenopathy with a blood smear revealing a preponderance of smudge cells. These cells are CD5/19/20/21 +ve. → CLL ○ Prevention of tumor lysis syndrome with tx? ■ Allopurinol ■ Rasburicase/pegloticase

Watery diarrhea after consuming undercooked oysters. → vibrio parahaemolyticus or vulnificus ○ Population at risk of severe infection? Liver disease pts (for vulnificus) ○ Other presentation of V. vulnificus? Severe cellulitis, even necrotizing fasciitis ○ Tx? Doxycycline

Patient on eclampsia prophylaxis is found by a med student to have ⅖ patellar tendon reflexes. The resident elicited 5/5 reflexes a few hours ago. → magnesium toxicity ○ Sequence of events with rising Mg? ■ Hyporeflexia → respiratory depression → arrhythmia ○ Tx? Calcium gluconate ○ PTH release based on mag levels ■ High → low PTH (acts on CSR) ■ Low → high PTH ● Recall the hypomag-hypocalcemia association ■ Very low → low PTH ○ Hypomagnesemia and electrolytes refractory to repletion? ■ Calcium ■ Potassium

Cardiogenic vs noncardiogenic pulmonary edema. ○ Cardiogeic ■ PCWP > 18 mmHg ■ Tx? Furosemide + inotrope (e..g digoxin, dobutamine, milrinone) ○ Noncardiogenic 121

■ ■ ●

● ●

PCWP < 18 mmHg Classic cause? ARDS

Pt with MI. Nearest PCI center is 30 mins away → transport ○ > 2 hrs away from PCI-capable center → give tPA ○ Contraindications to tPA? Recent brain bleed Treatment of left main stenosis or 3 vessel disease → CABG Acute MI management ○ 1st thing? Aspirin ○ Morphine ○ Nitrates ○ O2 if hypoxic ○ Heparin ○ Statin

Brush border enzyme deficiency in an Asian immigrant that presents with abdominal pain and bloating after consuming cheese. → lactose intolerance ○ Pathophys? lactase deficiency ○ Dx? Hydrogen breath test

Abdominal discomfort + pancytopenia + bone marrow biopsy revealing a dry tap + cells that stain +ve for Tartrate Resistant Acid Phosphatase + lymphoid cells with fine cytoplasmic projections on histology. → Hairy cell leukemia

Watery diarrhea and severe vomiting for a 3 day period after consuming fried rice at a Korean restaurant. → Bacillus cereus

Patient with a history of HBV presents with asymmetric neurological deficits + weight loss + severe abdominal pain after meals + ANCA negative + segmental transmural inflammation detected on renal angiography. → polyarteritis nodosa ○ What organ doesn’t it affect? Lungs ○ Dx? Sural nerve biopsy

Primary spontaneous PTX vs tension PTX. ○ Primary spontaneous PTX ■ Classic population ● Tall, skinny male ● Emphysema ○ Tension PTX ■ Classic population ● Trauma pt ● Ventilated pt (barotrauma) ● Central line placement ■ Auscultation? ● Decreased breath sounds ● Hyperresonant to percussion ○ Tx stable pt → give O2 & observe 122

Emphysema ○ RF ■ centriacinar → smoking ■ Panacinar → alpha-1 antitrypsin deficiency ○

Tx unstable pt → needle thoracostomy then place chest tube ■ Where? 2nd intercostal space midclavicular line ■ Bottom or top of intercostal space? Top (intercostal nerve/vein/artery run along bottom of ribs)

Imaging findings? ■ Hyperinflation → flattened diaphragms ■ Bullae PFT testing? ■ FEV1/FVC < 70% ■ FEV1 decreased ■ Lung volumes increased ■ DLCO ● Decreased → emphysema ● Normal → chronic bronchitis Gold staging ■ GOLD 1 = FEV1 > 80% ■ GOLD 2 = FEV1 50-80% ■ GOLD 3 = FEV1 30-50% ■ GOLD 4 = FEV1 < 30%

Key difference b/w asthma and emphysema management ○ Asthma: start ICS before LABA 123

○ COPD: start LAMA/LABA before ICS Improving survival ○ Smoking cessation ○ Home O2 Cor Pulmonale = R-heart failure 2/2 pulmonary arterial hypertension ○ PAH = PA pressure > 25 mmHg

Pneumoconiosis ○ PFT findings? Restrictive pattern ■ FEV1/FVC normal to high ○ Asbestosis ■ Lobes affected? Lower lobes ● Every other cause affects lower lobes ● Mnemonic: “roofs affect lower lobes, ground affects upper” ■ Imaging? Pleural plaques ■ Histology? Ferruginous bodies ■ MC malignancy? Bronchogenic carcinoma ■ Thoracentesis presentation of mesothelioma? Hemorrhagic pleural effusion ○ Caplan syndrome? RA + pneumoconiosis ○ Pneumoconiosis with increased TB risk? silicosis

Interstitial infiltrates on CXR + Sputum sample revealing broad based budding yeasts on histology + hemoptysis/productive cough/shortness of breath + Indiana resident + Gray skin lesions. → Blastomycosis ○ Tx? Itraconazole ■ If disseminated? Amphotericin B

● ● ● ● ●

Hospitalized patient with fever and diarrhea. → C. diff Endocarditis in a patient with prosthetic valves. → Staph epidermidis Fever in an ICU patient with a long history of alcohol abuse. → Klebsiella aspiration pneumonia Common causes of drug fever. → abx & heparin Classic LP/imaging findings in HSV encephalitis. ○ Imaging? Temporal lobe enhancement ○ LP? RBCs Drugs that cover MRSA ○ Vanc ○ Daptomycin ○ Clinda ○ Linezolid ○ Ceftaroline ○ Doxy ○ TMP-SMX Drugs that cover Pseudomonas ○ Ceftazidime ○ Cefepime ○ Pip-tazo ○ FQ ○ Carbapenems ○ Aztreonam

124

○ ● ●

● ●

● ● ●

Aminoglycosides

Antibiotic coverage of Neisserial infection? ceftriaxone Coverage of most abdominal infections? ○ Cipro + metro ○ Metro + amoxicillin + gentamicin Simple UTI management ○ Nitrofurantoin ○ TMP-SMX ○ Cipro Pyelonephritis treatment? ceftriaxone Community acquired pneumonia? ○ Ceftriaxone ○ Azithromycin ○ Levofloxacin RMSF? Doxy ○ In pregnancy? Chloramphenicol Syphilis management? Penicillin Flu coverage? 5.5 → type 1 (can’t acidify urine) ● Urinary pH < 5.5 → type 2 ○ Winters Formula? Expected pCO2 = 1.5*bicarb + 8 +/- 2 ■ Use? In metabolic acidosis, to determine if there is there is appropriate respiratory compensatory ■ Example: bicarb = 16, pCO2 = 20 ● Expected pCO2 = 28-32 ● Conclusion? Metabolic acidosis + respiratory alkalosis

Common causes of a metabolic alkalosis ○ Drugs ■ Diuretics ○ Disease ■ Barter’s/Gitelman’s ■ Conn syndrome ○ Volume depletion

127

Smoker OR Egyptian with history of chronic schistosomiasis OR patient with a history of nephrotic syndrome well controlled with cyclophosphamide presenting with hematuria → bladder cancer ○ Dx? cystoscopy ○ HY causes of hemorrhagic cystitis. ■ Cyclophosphamide ■ Adenovirus ■ Schistosoma haematobium

Indications for dialysis ○ A = acidosis ○ E = electrolyte imbalance ○ I = ingestion ○ O = fluid overload ○ U = uremia Complications of uremia ○ Pericarditis ○ Platelet dysfxn ■ Tx? desmopressin First use syndrome = anaphylaxis + CP + back pain during first dialysis session ○ Pathophys? Anaphylactic rxn to membrane or materials used to sterilize dialysis machines Amyloidosis in patients on chronic dialysis (involved protein) → β2 microglobulin Abdominal pain, distension, and high fevers in a patient that undergoes regular peritoneal dialysis → SBP

● ● ● ●

Flank pain + CVA tenderness + High fevers + Urgency/Frequency/Dysuria. ○ Dx? UA w/ culture ○ Casts? WBC ○ Tx? Ceftriaxone or cipro ○ Who should get outpatient treatment vs IV antibiotics ■ Vomiting / can’t po → admit ○ MC cause pyelo? E. coli

FSGS ○ Classic demographic? ■ African Americans ■ HIV+ ○ Variant in HIV pt? Collapsing variant ○ Electron microscopy findings? Effacement of foot processes ○ Tx? Steroids + ACE-I +/- cyclophosphamide

Goodpasture’s syndrome ○ Presentation? Hemoptysis + hematuria ○ Ab? anti-GBM Post Strep glomerulonephritis ○ Presentation? Cola-colored urine + Strep infection 2-6 weeks beforehand ○ EM findings? Subepithelial humps Lupus nephritis ○ Histology? Wire loop appearance IgA nephropathy

● ●

128

○ Presentation? Hematuria 2-6 days after URI Henoch Schonlein purpura ○ Presentation? Joint pain + abdominal pain + palpable purpura on buttocks ○ Systemic manifestation of IgA nephropathy Granulomatosis with polyangiitis (Wegener’s) ○ Triad = sinusitis + hemoptysis + hematuria ○ Marker? c-ANCA EGPA (Churg Strauss) ○ Presentation? asthma + eosinophilia + granulomas on biopsy ○ Marker? p-ANCA Microscopic polyangiitis ○ Marker? p-ANCA

Beck’s triad of cardiac tamponade = JVD + muffled heart sounds + hypotension ○ Tx? pericardiocentesis

AVRT ○ Orthodromic ■ Pathway? Down the AV node (& up the accessory pathway) ■ QRS? Narrow ■ Chronic tx? Beta-blocker or ND-CCB ■ Acute tx if HDUS? Synchronized cardioversion ■ Acute tx if stable? adenosine ○ Antidromic ■ Pathway? Down the accessory pathway (& up the AV node) ■ QRS? Wide ■ Classic cause? WPW ■ Tx? Procainamide ■ Contraindicated drugs? Beta-blocker or ND-CCB

Prolonged QT interval ○ Drugs ■ Ondansetron ■ Haloperidol ■ Ziprasidone ■ Quinidine/disopyramide/procainamide ■ Amiodarone/sotalol ■ FQ ■ Macrolides ○ Electrolytes ■ Hypokalemia ■ Hypocalcemia ■ Hypomagnesemia ○ Risk of QT prolongation? Torsade de pointes ■ Tx TdP? IV Mg

Cardiovascular parameter changes with physiologic manipulation ○ Milrinone (“inodilator”) 129

○ ○ ○

■ CO up ■ SBP up ■ SVR down ■ DBP down ■ Pulse pressure widens Norepi Hydralazine Nitrate

Recent knife wound to the arm + PE revealing a warm, pulsating, palpable arm mass + Echocardiography revealing an EF of 75%. → high output HF 2/2 AV fistula

Japanese female with abdominal pain + > 10 mmHg difference in systolic BPs in the arms + 6 mo history of low grade fevers + CBC revealing marked elevations in ESR and CRP. → Takayasu arteritis

Pericardial knock on auscultation + JVP increase with Reduced EDV on echocardiography. → constrictive pericarditis ○ JVP increase with inspiration? Kussmaul’s sign ○ Constrictive pericarditis vs. cardiac tamponade. ■ Blunted Y descent on JVP waveform → cardiac ■ Steep Y descent on JVP waveform → pericarditis

● ● ● ● ● ● ● ●

Most common cause of death after an MI? V-fib Ventricular free wall rupture physiology? Tamponade Interventricular septal rupture murmur? Harsh holosystolic Papillary muscle rupture presentation? Acute mitral regurg Peri-infarction pericarditis tx? aspirin Pericarditis weeks after MI → Dressler’s syndrome ○ Pathophys? autoimmune Post-MI medications? Aspirin + statin + ACE-I + beta-blocker

inspiration +

tamponade constrictive

physiology murmur

Colon cancer screening guidelines ○ Start when? 50 ○ Modalities? ■ Colonoscopy q10 years ■ Flex sig q5 years ■ FOBT annually ● If positive, NBS? colonoscopy ○ First degree relative diagnosed with colon cancer at 52 → start at age 40 ○ First degree relative diagnosed with colon cancer at 48 → start at age 38 ○ Screen in pts with first degree relativ with colon cancer? ■ Age 40 OR 10 years before dx (whichever is sooner!) ○ When to start screening after UC diagnosis? 8 years Multiple colon polyps in a 22 yo → FAP 130

● ● ●

○ What if they also have a brain tumor → Turcot syndrome ○ What if they also have bone & soft tissue tumors → Gardner's syndrome ○ Mutation? APC Early colon cancer arising from normal mucosa → HNPCC/Lynch syndrome ○ Mutation? MSH/MLH Hyperpigmented macules on lips + GI tract hamartomas → Peutz Jeghers Syndrome. Tx for GERD? PPIs ○ AE? Aspiration pneumonia + osteoporosis + B12 deficiency ○ Cimetidine AE? gynecomastia Tx for H. pylori ○ Triple therapy → clarithromycin + amox + PPI ○ Quad therapy → metro + bismuth + tetracycline + PPI (MBTP)

35 yo F with a 15 year history of OCP use presenting with RUQ pain. US reveals a well circ*mscribed hepatic mass. → hepatic adenoma

60 yo M presents with a 7 week history of bilateral knee pain and chronic diarrhea. Endoscopy with biopsy of the intestinal mucosa reveals PAS+ macrophages and villous atrophy. A PE is notable a murmur that was not present at the last visit. → Whipple’s disease ○ Bug? Tropheryma whipplei ○ Tx? Ceftrixone/penicillin G for 2 weeks then TMP-SMX for 1 year PAS+ in liver → alpha-1 antitrypsin PAS+ in GI tract → Whipple’s disease

● ● ●

60 yo M presents with a long history of halitosis and difficulty swallowing. A palpable mass is felt in the neck with swallowing. → Zencker diverticulum ○ Dx? Barium swallow ■ Do NOT do upper endoscopy (you could perforate the esophageal)

Difficulty swallowing + chest pain radiating to the jaw relieved with nitrates → diffuse esophageal spasm ○ Dx? Manometry ○ Tx? CCBs or nitrates

47 yo F with a history of “relentless DM” presents with a 1 week history of a necrotic, erythematous skin lesion → glucagonoma ○ Associated syndrome? MEN1 ○ Tx? Surgery or octreotide

Diabetic gastroparesis ○ Pathophys? Damage to enteric nervous system ○ Tx? ■ Metoclopramide ● AE? EPS ■ Erythromycin (motilin receptor agonist) Severe abdominal pain and distension in a 60 yo M septic patient. There is no obstructing lesion identified on abdominal CT. → Ogilvie syndrome (colonic pseudo-obstruction)

131

Tx? Neostigmine ■ AE? bradycardia

Elevated creatinine and oliguria in a patient with a history of ESLD. → hepatorenal syndrome ○ Pathophys hypothesis? Nitric oxide produced → lower SVR → hypoperfuse the kidney ○ Urine labs? Looks like prerenal AKI ■ FeNa < 1% ■ BUN/Cr > 20 ○ Distinguishing this presentation from prerenal azotemia? Doesn’t improve with fluids ○ Tx? Liver transplant

Hypoxia with standing/walking that improves with recumbency in a patient with a history of ESLD → Platypneaorthodeoxia syndrome ○ Pathognomonic for what? Hepatopulmonary syndrome

Diarrhea associations ○ Cruise ship gastroenteritis → norovirus ○ AIDS patient → cryptosporidium parvum ○ 2 hrs after eating potato salad → Staph aureus ○ Daycare → rotavirus ○ oyster consumer → vibrio parahaemolyticus ○ consuming fried rice → Bacillus cereus ○ pork consumption → Yersinia enterocolitica ○ bloody diarrhea from eggs and poultry → Salmonella ○ common cause of bloody diarrhea in the US → Campylobacter

MEN1 syndrome + jejunal ulcers + unrelenting GERD. → Zollinger-Ellison syndrome ○ Dx? Gastrin level ■ If high but not crazy high → secretin stimulation test ● Increase in gastrin w/ secretin → Z-E ○ Tx? ■ Medical? PPIs ■ Surgical? resection

Associated autoantibodies in Type 1 and 2 autoimmune hepatitis. ○ Type 1 → anti-SM ○ Type 2 → anti-LKM (anti-liver kidney microsomal)

UC vs Crohn’s ○ UC ■ Blood diarrhea ■ Continuous lesions ■ PSC ○ Crohn’s ■ Skip lesions ■ Non-caseating granulomas 132

■ ■ ■

Oxalate kidney stones Pyoderma gangrenosum Uveitis

Hemochromatosis associated joint disease? CPPD.

Pancreatic cancer dx? CT scan

Budd Chiari syndrome = thrombosis of hepatic vein ○ Classic populations? ■ OCP users ■ Polycythemia vera ■ PNH

Acetaminophen OD tx? N-acetylcysteine ○ Toxic metabolite? NAPQI

133

134

-------------------------------------------------------------------------------------------------------------------------------

Episode 36: Ophthalmology 1. Central scotomas, straight lines look wavy = age related macular degeneration ● ● ●

● ●

2 types - wet (exudative/neovascular) and dry (atrophic) Wet: more progressive, but also more treatment options offered ○ Tx: VEGF inhibitors (Ranibizumab, Bevacizumab) or laser Dry: drusen on fundoscopy; more common than wet ○ Tx: Vitamins C, E, beta-carotene, zinc slow progression ○ Note: (careful in smoking pt as increased mortality rate from lung cancer with Vit E and beta carotene) MCC of vision loss in developing world = macular degeneration Other risk factors = smoking, aspirin (chronic use) 135

● ●

Buzz word = neovascularization Lose central vision (macula) (whereas in glaucoma lose peripheral vision)

2. Punched in eye + floaters and bright streaks of light = retinal detachment ● ● ●

Risk factors = trauma, cataract surgery ○ Note: others metabolic disorders, myopia, degenerative diseases, vascular disorders Treatment strategies = laser photocoagulation (buzzword to remember) Ophtho emergency

3. Newborn with R eye white reflex = retinoblastoma ●

Associated with = Osteosarcoma ○ Sunburst (codman’s triangle) on Xray

4. Amblyopia vs strabismus ● Amblyopia = cortical blindness -> eye normal but brain doesn’t recognize is ○ MCC = strabismus ○ Others: anything that obstructs light - cataracts, glaucoma ● Strabismus = misalignment of eye ○ Treat if persists past 3 months with patching of unaffected eye - makes the brain work the affected eye to align itself; eyepatch or “paralytic” drops with atropine ○ Corneal light reflex is uncentered in affected eye (not symmetric) ○ Writers note: red reflex will be brighter in affected eye

5. Asian male with sudden onset severe eye pain + nausea and vomiting following nasal decongestant use = acute angle closure glaucoma ● ● ●

● ● ● ●

Alpha one agonist - phenylephrine will cause mydriasis Treatment = laser iridotomy (ophtho emergency), clonidine, muscarinic agonist (pilocarpine), mannitol or acetazolamide, latanoprost, beta-blockers Risk factors = African American, diabetes, steroid use (can decrease outflow at anterior chambers) ○ Closed (aCute) think Asians, Open (chrOnic) think African Americans MCRF = increase in intraocular pressure *Keep in mind Dx: Tonometry Fundoscopic exam = increased optic cup:disc ratio Ciliary epithelium makes aqueous humor made by ciliary epithelium (driven by sympathetic NS via beta2 receptors) → post chamber of eye → ant chamber ni→ drain via canal of Schlemm/ trabecular meshwork (M3 receptors) & some (~25%) through the uveoscleral outflow tract (controlled by prostaglandins) Treatment targets the different points in the pathway: ○ Aqueous humor production ■ Beta blockers (Timolol): decrease aqueous humor synthesis (Beta2) ■ Alpha2 agonist (briminodone)- (Gi coupled inhibit adenylate cyclase) inhibit presynaptic release of NE ● NOT in closed angle glaucoma - can precipitate closed (acute) ■ Carbonic anhydrase inhibitor (acetazolamide) - decrease the bicarb at the ciliary epithelium will decrease the Na+ and H20 transport and thus dec synthesis of aqueous humor 136

Uveoscleral outflow (note: his mechanism is a little off here so I’m adding the correct mechanism for PG agonists): increase outflow through the uveoscleral tract by increasing permeability with prostaglandin agonists ■ SE: Permanent discoloration of iris Trabecular outflow - increased drainage of meshwork/canal with increase episcleral vasculature with M3 agonists (Carbachol, Pilocarpine)

6. 2 days of severe R eye pain + blurry vision + sinusitis taking Benadryl + pain with eye movement and 20/200 vision and difficulty moving EOM = orbital cellulitis ●

● ● ●

Dx: clinical +/- ocular CT scan ○ Differentiate from preseptal cellulitis by involvement of EOM and pain with movement in orbital cellulitis - orbital cellulitis is preseptal cellulitis + other worsening symptoms Path: polymicrobial infection Tx: clindamycin + penicillin family Big risk factor = sinusitis

7. female with severe L eye pain, can barely see, worse after hot shower = optic neuritis due to multiple sclerosis ● ● ●

Diagnostic imaging = MRI Tx = IV steroid (po steroid increases risk of recurrence of optic neuritis) *Very high yield info* Most likely sequelae = resolve over time

8. 76 yo F difficulty seeing while driving at night and reading road signs = cataracts ● ● ● ● ●

Path = opacification of the lens Metabolic RF = diabetes (aldose reductase converts glucose to sorbitol, lens lacks sorbitol dehydrogenase to convert it to fructose so sorbitol accumulates in lens) Drug associated RF = steroids Infectious RF = congenital syphilis, congenital CMV, congenital toxo Newborn with a bilateral presentation = classic galactosemia (Galactose-1-phosphate uridyltransferase deficiency or GALT) - galactitol accumulates in lens and opacifies it ○ WNote: red reflex = dark, dull, white ○ Most common cause of death in newborns with galactosemia = e. coli sepsis**HY

9A. ○ ○

MCC conjunctivitis in first 24 hours of life = chemical conjunctivitis from silver nitrate (not used much anymore) - 5 day old newborn with “super” purulent conjunctivitis ■ Bug = gonorrhea - within 1st week of life - most dangerous for blindness in US ■ Tx = IV cefotaxime (3rd gen cephalosporin) Ceftriaxone causes kernicterus and cholestasis in newborns ■ Prophylaxis = topical macrolide (erythromycin) ● § DOES NOT PPX AGAINST CHLAMYDIA - 12 day old newborn with “watery” discharge conjunctivitis and eyelid swelling ■ Bug = chlamydia (serovars A-C) - appears 1-2 weeks after birth - leading cause of preventable blindness worldwide ■ Tx = PO (oral!) erythromycin 137

■ ■ ■

Use an oral treatment to eliminate eye infection and possible nasopharyngeal infection otherwise will develop PNA at 1-3 months ● Topical erythromycin does NOT ppx against chlamydia though remember Prophylaxis relationship to presentation 2 = topical erythromycin does NOT ppx against chlamydia conjunctivitis Pneumonia presentation = newborn with “staccato cough” “down the line” nonbilious vomiting after tx (pear shaped mass) = pyloric stenosis from erythromycin

9B. Most likely Conjunctivitis ●

Glued eyes in am (unilateral) + water discharge + rhinorrhea = viral ○ Tx = warm/cold compress ○ o HY bug = adenovirus “pharyngoconjunctivitis” ■ Remember adenovirus causes gastroenteritis and hemorrhagic cystitis ○ MCC conjunctivitis = viral Glued eyes in AM (unilateral then becomes bilateral) + purulent discharge + no rhinorrhea = bacterial ○ MCC bacterial conjunctivitis = s. aureus ○ Tx: antibiotic eye drop (erythromycin except contact-wearers use fluoroquinolone drops to cover for possible pseudomonas) - Hx of asthma + BL itchy red eye = allergic conjunctivitis ○ o Tx: antihistamine eye drops (Olopatadine, Azelastine)

10. Eye pain + worsened in light in 23yo male with hx of chronic LBP and morning stiffness = ankylosing spondylitis = anterior uveitis ● ● ●

Pulmonary disease association = sarcoid Pediatric rheumatology association = oligoarticular JRA ○ If given JRA pt and asked next best step - slit lamp If determine caused by HSV - give acyclovir +/- topical steroid vs. cause is autoimmune phenomenon give topical steroid

11. 70 yo F with difficulty reading books. Has to move books way from her eyes to make out the words = presbyopia ●

Path = lens loses elasticity with age so can’t accommodate (just like skin loses elasticity)

12. Differentiate central retinal artery occlusion from central retinal venous occlusion - HY* ●

CRAO = acute, painless, monocular vision loss ○ Amaurosis fugax – curtain over eye ■ Note: assoc with Temporal arteritis, carotid a stenosis; presents as transient loss of vision for several minutes and then eventually present with persistent vision loss (aka CRAO) ○ Dx: ocular u/s + fundoscopic exam = thin retinal vessels, fundal pallor (swelling of the retina)* helps distinguish from venous occlusion ○ Tx: ocular massage and high flow oxygen while en route to hospital; once at hospital give TPA CRVO = rapid, painless loss of vision of varied severity ○ Dx: fundoscopic exam = optic DISC swelling (vs retinal swelling in CRAO)thick retinal vessels, blood and thunder retina, cotton wool spots, hard exudates ■ Fluorescein angiography not u/s if suspect venous occlusion 138

○ Tx: VEGF-I, photocoagulation Don’t forget about these both presenting with cherry red spot on fundoscopy: ○ Tay-sachs (hexosaminidase A deficiency) losing milestones and impaired startle, no HSM ○ Niemann-pick (sphingomyelinase deficiency) - similar to Tay Sachs but with hepatosplenomegaly Diabetic Retinopathy - presents very similar to CRVO **note similar descriptions ■ Nonproliferative: dot and blot hemorrhages, hard exudates, retinal edema, microaneurysm ■ Preproliferative: cotton wool spots ■ Proliferative: neovascularization (buzzword), macular edema ○ Do fundoscopic exam - repeat every year; to make definitive diagnosis do fluorescein angiography to confirm diagnosis (r/o CRVO) ○ Tx: same as CRVO - VEGF-Inhibitor, laser photocoagulation

13. 19 yo F with pain and a “foreign body sensation” in the eye + wears contact lenses = corneal abrasion ● ●

Dx = fluorescein slit lamp exam - can’t see with naked eye Tx = REMOVE contact, topical broad spec antibiotic coverage with pseudomonas coverage (fluoroquinolone) topical NSAID drop for eye, eye patch (he says you can but avoid patching it)

Cross checked: Yes -------------------------------------------------------------------------------------------------------------------------------

Episode 37: Risk Factors #1 RF = number one risk factor MCC = most common cause MCCOD = most common cause of death

1. 2. 3. 4. 5 6.

#1 RF for suicide is prior suicide attempt #1 RF for AFib → Mitral stenosis. Most common arrhythmia in hyperthyroidism → A fib. #1 RF for Mitral stenosis → Rheumatic fever #1 RF for AAA development→ smoking Screen AAA → men, 65-75, smoker. If >5.5 cm → surgery. Or >0.5cm/6 mos or 1 cm/yr growth 7. Greatest predisposing risk factor to rupture of a AAA → diameter of aneurysm 8. Most important modifiable RF for CAD → smoking 9. #1 RF for stroke and aortic dissection → HTN 10. Biggest RF in iron deficiency anemia pt with h/o H.Pylori→ PUD 11. #1 RF for esophageal adenoCa → barrett’s esophagus which is caused by GERD 12. MCC CAP → Strep. Pneumo 13. MCC UTI→ E.coli 14. Common RF for osteoporosis → Low BMI. 139

15. Controlling blood glucose only decreases microvascular risks associated with retinopathy, neuro, and nephropathy, not macrovascular like CAD and stroke. 16. #1 RF for SCC → cumulative sun exposure 17. #1 RF for ovarian cancer → FHx of OCa. 18. #1 RF for prostate,breast Ca → Age 19. MCCOD in pt with kidney transplant → CV disease 20. MCCOD in acromegaly → heart failure 21. MCCOD (most common cause of death) in CKD → CVDisease./ arrhythmia followed by infection 22. #1 RF for OSA → obesity 23. #1 modifiable RF for knee OA → obesity 24. MCCOD in ADPKD → CVDisease NOT SAH. 25. MCCOD in SLE → ischemic heart disease 26. #1 RF for pancreatic Ca → smoking 27. Prior asbestosis exposure → Bronchogenic carcinoma 27.5) Mesothelioma has no relation to smoking 28. Silicosis increase the risk for TB 29. MCC of Acute pancreatitis in America →Gallstones https://www.pancreapedia.org/reviews/introduction-to-pancreatic-disease-acute-pancreatitis 30. MCC Chronic pancreatitis → alcoholism 31. Papillary thyroid cancer #1 RF → prior chest/neck radiation 32. BCa Screening → every year from 40. USPSTF →every 2 years start at 50 until age 75, 33. Cervical Ca screen → every 3 years 21-30 regardless of sexual activity. >30 → HPV + pap every 5 years. 34. Hyperlipidemia screen → every 5 year >35 men, >45 women 35. Colon Ca → CScopy every 10 years 50-75. In UC 8 years after dx and every 1-2 years after. FHx colon cancer screen 10 year prior to family members Dx age. FAP yearly starting at 10-15 yo 36. All pregnant women 1st prenatal visit → screen syphilis, asymptomatic bacteriuria, HIV 37. Rh immunoglobulin → 28-32 wks + within 72 hours of delivery. Determine dose → kleihauer-betke test 38. Melanoma most likely determination of spread→ breslow depth 39. #1 prevention of Hepato cellular carcinoma → Hep B vaccination 40. #1 RF for ED → CVD 41. MCCOD in cervical cancer → Renal failure esp ureters 42. Most important prognostic factor in cancer→ stage of tumor (TNM staging) 43. Lung CA → low dose CT, men and women, 55-80 who have >30pp smoking hx, who currently smoke + less than 15 yr since quitting. 44. #1 RF for endometritis → C-section 45. #1 RF for preeclampsia → prev hx of Preeclampsia/ nulliparity 46. #1 RF for uterine inversion → Prior inversion 47. #1 RF for chorioamnionitis → prolonged rupture of membranes 48. #1 RF for placenta previa → prior C-section 140

49. 50. 51. 52. 53. 54. 55. 56. 57.

HY RF to preterm labor → bacterial vaginosis #1 RF for endometrial cancer → unopposed estrogen> endometrial hyperplasia #1 RF for ectopic pregnancy→ hx of ectopic pregnancy and smoking (affects cilia) #1 RF for Cervical incompetence→ cervical conization procedure or LEEP #1 RF for cervical cancer → HPV #1 RF for fetal macrosomia → gestational diabetes or preexisting diabetes #1 RF for shoulder dystocia→ fetal macrosomia MCC of placental abruption → trauma/cocaine RF of urinary incontinence→ age and multiple vagin*l deliveries (stress incontinence), Diabetes→ overflow incontinence, MS→ urge incontinence 58. RF for Asherman’s syndrome→ history of uterine curettage 59. #1 RF for bladder cancer -> smoking Cross checked? Yes

----------------------------------------------------------------------------------------------------------------------------

Episode 41: Antibiotic Guide 1. Newborn presenting with nuchal rigidity. Bug? Treatment? a. First 20 days of life – Group B strep most common b. Remember CVA – C = 3rd gen cephalosporin - Cefotaxime (GBS/S.pneumo/Neisseria) + Vancomycin (MRSA) + Ampicillin (Listeria coverage) i. May see alternate option of ampicillin + gentamicin ii. Do NOT use ceftriaxone in neonates --> biliary sludge 2. 65 yo male presenting with nuchal rigidity, fever, and headache. Treatment? a. >50yo – use same CVA treatment b. CVA – 3rd gen cephalosporin - Ceftriaxone (Neisseria/S.pneumo) + Vancomycin (MRSA) + Ampicillin (Listeria) i. Listeria nearly 100% fatal if not treated 3. 33 yo F with no relevant PMH presenting with nuchal rigidity, headache, and fever. Treatment? a. Ceftriaxone and Vancomycin b. SIDE NOTE, how is hospital associated bacterial meningitis/post neurosurgical procedure bacterial meningitis managed? i. cover Pseudomonas and MRSA ii. change ceftriaxone to: ceftazidime (only 3rd gen ceph covering Pseudo) or cefepime (4th gen ceph) 4. Patient presents with headache, nuchal rigidity, and fever. LP reveals gram positive diplococci and a neutrophilic predominance. In addition to the standard antibiotic therapy, how can patient morbidity and mortality be reduced? a. S. pneumo – give standard antibiotics PLUS add IV dexamethasone i. Give minutes to hours prior to antibiotics ii. Decreases M&M and sequelae like hearing loss 5. HIV+ patient presents with a severe headache and nuchal rigidity. Papilledema is detected on a fundoscopic exam. a. What will a lumbar puncture most likely reveal? ↑ WBC but lymphocytes (not neutrophils), low glucose, 141

high protein --> Cryptococcal meningitis i. Significantly elevated opening pressure is a clue for cryptococcus because occludes CSF flow; often presents with signs of ↑ ICP (papilledema) but absent meningeal signs ii. How is this bug tested for? 1. CSF antigen test (highly sensitive and specific) 2. Latex agglutination assay or India ink stain iii. How is this bug treated? 1. Amphotericin B – binds ergosterol in membranes in fungal cells and makes holes 2. Liposomal form of Amp B ↓ nephrotoxicity 3. PLUS Flucytosine – inhibits thymidylate synthase iv. How is this bug treated after the fact? 1. Maintenance fluconazole for until symptoms resolve and CD4>100 for >1yr 2. Coverage of bacterial meningitis in a HIV+ patient. 3. CAV – Ceftriaxone, Vanc, Ampicillin (like a 50+yo) 6. HIV prophylaxis for the following scenarios a. CD4 < 250 living in Arizona i. Coccidioidomycosis if living in high risk area ppx with Itraconazole b. CD4 < 200 i. Pneumocystis jirovecii – ppx with TMP-SMX 1. Alternate options: aerosolized pentamidine, Dapsone (more HY), also atovaquone c. CD4 < 150 living in Kentucky i. Histoplasmosis – ppx with itraconazole d. CD4 < 100 i. Toxoplasmosis gondii – ppx TMP-SMX (which should be covered already at 1yr and until CD4>100 for 6months (vs 1yr in Cryptococcal meningitis maintenance tx) 7. Patient on chemotherapy develops a fever. WBC count is 2000. What are the 2 bugs you’re trying to cover? What is the empiric treatment strategy for this kind of fever? a. Pseudomonas + MRSA i. Ceftazidime 3rd gen(Cefepime 4th gen) + Vancomycin 8. HIV+ patient presents with seizures. MRI reveals multiple ring enhancing lesions in the brain. a. Diagnosis? Treatment? When should this patient get steroids? i. Toxoplasmosis ii. Tx: pyrimethamine (DHF reductase inhibitor) + sulfadiazine (DHF synthetase inhibitor) iii. Prophylaxis is trimethoprim-sulfamethoxazole iv. Add steroids if signs of elevated ICP v. If patient has seizure from neurocysticercosis – do NOT give anti-helminths because can have severe inflammatory response 1. Give phenytoin/antiepileptics 9. HIV+ patient presents with headache and nuchal rigidity. LP is positive for CSF with 500 RBCs. MRI reveals hyperintense lesions in the temporal lobe. Bug? Treatment? a. HSV meningitis – red cells on CSF and temporal region b. Tx: IV acyclovir or foscarnet i. If patient has SUDDEN severe headache and nuchal rigidity and see tons of red cells 142

think more about subarachnoid hemorrhage 10. HIV+ patient presents with headache and nuchal rigidity. LP is +ve for CSF with lymphocytic pleocytosis. Brain imaging reveals enhancement at the base of the brain (or in “basilar cisterns”). Bug? Treatment? a. Tuberculosis b. Tx: RIPE i. Rifampin – RNA polymerase inhibitor ii. Isoniazid – rx-induced lupus; ADD B6 to prevent peripheral neuropathy iii. Pyrazinamide iv. Ethambutol – ocular/color blindness 11. Patient presents with a tender lesion on the left lower extremity. PE reveals a well circ*mscribed erythematous lesion on the calf. The patient’s vital signs are wnl. Diagnosis? Empiric treatment? a. Cellulitis – Staph aureus b. Tx: Clindamycin, TMP-SMX, Cephalexin c. DDX: i. Erysipelas – more red, tender, elevated, and well-circ*mscribed – caused by Strep pyogenes ii. Folliculitis – hair follicle infection – caused by Staph aureus 1. Hot tub – pseudomonas infection 12. Empiric management of animal/human bite wounds. a. Amoxicillin + Clavulanic acid (same for human bites) 13. Empiric management of community acquired PNA. a. Tx: Macrolide (Azithromycin) or Doxycycline – remember MD b. Empiric management of hospital acquired PNA. i. Tx: Macrolide (Azithromycin) or Doxycycline – remember MD ii. PLUS Ceftriaxone – if PCN allergic or not an option can use monotherapy – moxifloxacin or levofloxacin iii. What if stem mentions the presence of a cavitary infiltrate? 1. S. aureus – MRSA involvement 2. Tx: Clindamycin, Vancomycin, Linezolid 14. (All from Boston) → Lyme Disease a. Bullseye rash in a 7 yo M. Treatment? i. 4.5, clue cells=epithelial cells coated in the bug; most common of the 3 – makes sense because normal vagin*l flora b. Tx=Metronidazole 22. Female with a frothy green, foul smelling vagin*l discharge (bug, pH, tx)? i. Trichom*onas vagin*lis – strawberry cervix, pH>4.5 ii. Tx=Metronidazole for patient + partner b. Cottage cheese discharge (bug, pH, tx)? i. Candida vaginitis – grows in germ tukasbes 35 or PaO2 250) ○ Tx? ■ Acetazolamide ■ Serial LPs

41 yo M with 1 year of forgetfulness + behavioral changes. His father had similar presentation before dying at 47 y.o. → Huntington’s disease ○ Pathophys? Atrophy of caudate (part of striatum) ○ Trinucleotide repeats? CAG ○ Chromosome? 4 ○ Inheritance? AD ■ With anticipation! ○ Tx? Anti-dopaminergic drugs 151

■ ■

Haloperidol Tetrabenazine (VMAT inhibitors)

Worst headache of patient’s life + bilateral flank masses. → SAH in pt with ADPKD ○ Pathophys? Rupture of berry aneurysm in circle of Willis ○ MC aneurysm location? ACom aneurysm (anterior communicating artery) ○ Associate heart murmur? MVP ○ NBSIM? CT head non-contrast (because blood will show up as hyperdense) ■ If CT is negative → do LP (looking for xanthochromia) ○ Tx? Lower pt’s BP to 140/90 ○ What drug is given to prevent superimposed ischemia? nimodipine

Cluster headache ○ Sxs? Unilateral tearing/conjunctival injection/rhinorrhea, occurs for several days ○ Tx? 100% O2 and triptans

Tension headache ○ Sxs? Bilateral + high-stress environment + worse at end of day ○ Tx? NSAIDs

Migraine headache ○ Sxs? Unilateral + pulsatile + photophobia & phonophobia +/- aura ○ Abortive tx? ■ Sumatriptan (5-HT receptor agonist) ● Contraindications? variant angina aka Prinzmetal angina, CAD, CREST syndrome ○ Can trigger serotonin syndrome ● Ergotamine ○ PPx? ■ Propranolol ■ Topiramate ■ TCA ● Contraindications? Elderly pt’s, glaucoma pt’s ○ Avoid with glaucoma (anticholinergic mydriasis) ■ CCB’s

Analgesic rebound headaches ○ Sxs: use of analgesics at least 8-10x in last month; headache returns once analgesic effects wear off ○ Tx? Wean off analgesics

Headache + mental status changes in patient using fireplace to warm house in winter → CO poisoning ○ Diagnostic? Carboxyhemoglobin levels ■ Note: Pulse O2 cannot distinguish between oxyhemoglobin and carboxyhemoglobin ○ Tx? High-flow O2 / hyperbaric O2 ○ Imaging? Hyperintense lesions in globus pallidus on MRI 152

35 yo smoker + right-sided headache + occurs everyday at same time for past month + PEx notable for rhinorrhea & pupillary miosis on right → cluster headache

----------------------------------------------------------------------------------------------------------------------------

Episode 46: Neurology Clerkship Shelf Review Part 3 ●

A child is not doing well in school. His teacher often observes him staring into space with no awareness of his surroundings. → absence seizures ○ EEG pattern? 3 Hz spike and wave ○ Tx? Ethosuximide (T-type Ca++ channel blocker)

55 yo F presents with a 6 mo history of feeling like the room is spinning around her that lasts for about 3 days at a time. She has also had difficulty hearing at home. She also complains of a “ringing sound” in her ear. → Menierre’s disease ○ Triad: vertigo + tinnitus + sensorineural hearing loss ○ Pathophys? Endolymphatic hydrops, poor reabsorption of endolymph in ear → high pressures ○ Tx? ■ Dietary Na+ reduction ■ For vertiginous sxs: ● scopolamine (anticholinergic) meclizine (antihistamine, anticholinergic) ● diphenhydramine (antihistamine, anticholinergic) ■ Diuretics ○ Definitive tx? Gentamicin injection to ablate CN8 ■ Causes of permanent sensorineural hearing loss Vertigo triggered by changes in position, NO hearing loss → BPPV ○ Pathophys? otoconia/otoliths in semicircular canals ○ Dx? Dix-Hallpike maneuver ○ Tx? Epley maneuver Recent viral URI + constant non-positional vertigo, lasts for days to weeks before spontaneous resolution, NO hearing loss → Vestibular neuritis Recent viral URI + constant vertigo + hearing loss → Labrynthitis

● ● ●

Hit in the head with a bat, passed out for < 30s, finished out the game, now presents with severe headache and somnolence. → epidural hematoma ○ Pathophys? Fracture of frontal bone → tearing of the middle meningeal artery ○ Dx? Non-con head CT → lens-shaped hematoma ■ Do NOT perform an LP because increased ICP will cause herniation ○ Tx? ■ Neurosurgical evacuation ■ Reduce ICP ● Elevate head of bed ● Hyperventilate pt (CO2 down → cerebral vasoconstriction) ● Mannitol ■ What is the quickest means of reducing ICP acutely? Hyperventilation 153

● ●

● ●

Old person on warfarin with multiple falls, acting out-of-it recently OR alcoholic → subdural hematoma ○ Pathophys? Brains shrink with aging or alcoholism, stretches the bridging veins, causing greater susceptibility to shearing ○ Dx? Non-con head CT → crest-shaped Brain bleed in shaken baby → subdural hematoma Super tense muscles and T of 105 after intubation. → malignant hyperthermia ○ Pathophys? Mutation in the ryanodine receptor → increased release of Ca++ from the sarcoplasmic reticulum → hypercontractile state that generates heat ○ Inheritance? AD ○ Tx? Dantrolene (ryanodine receptor antagonist) ○ Electrolyte disturbance? hyperkalemia → peaked T waves → wide QRS → sinusoidal pattern ■ Tx? ● 1st calcium gluconate ● Then insulin/glucose, albuterol, or sodium bicarb ● Kayexalate (helps excrete K+) ● Furosemide (loop diuretic with hypokalemia as side effect) ○ Why elevated Cr? ■ Rhabdomyolysis → myoglobin release → damages kidney ■ Tx? IVF If same presentation after starting fluphenazine (1st gen high-potency antipsychotic) → neuroleptic malignant syndrome ○ Tx? ■ Dantrolene ■ Dopamine agonist (e.g. bromocriptine/cabergoline, amantadine) Hx of depression + Tx for S. Aureus bacteremia (or migraines) presents with high fevers, diarrhea, clonus → serotonin syndrome ○ Drugs that can trigger ■ Any antidepressant ■ Linezolid ■ Ondensatron ■ Triptans ■ MAO-B ■ MDMA/ecstasy ○ Tx? ■ Cyproheptadine (antihistamine with powerful serotonin receptor blocking activity) ■ Benzo Tremors 6 hrs after the successful completion of a AAA repair in a business executive. → alcohol withdrawal ○ Ppx? Long-acting benzo (e.g.chlordiazepoxide, diazepam) Tremors after starting Indapamide (thiazide-like diuretic) in a patient with a history of “episodes” where they spend tons of money/jump into severe depression. → lithium toxicity in pt with bipolar disorder ○ Pathophys? Diuretics can raise lithium levels → lithium toxicity, which manifests as tremors ○ Causes of lithium toxicity? anything that causes RAS activation (aldosterone acts on principal cell to increase Na+ reabsorption → Li+ can allow be reabsorbed through this channel)

154

Child presents with bulging fontanelles and somnolence. What is the most likely region of stenosis responsible for the presenting hydrocephalus? Cerebral aqueduct of Sylvius

70 yo F that is forgetful. She staggers into the exam room. She has a history of recurrent perianal sores from urinary incontinence. → “wet wobbly wacky” → normal pressure hydrocephalus ○ Dx? Non-con head CT with enlarged ventricles ■ LP → normal opening pressure ○ Tx? VP shunt

Tuberous sclerosis ○ Inheritance? AD ○ Mutated genes? TCS1 & TCS2 (tumor suppressor genes) ○ Mutated proteins? Hamartin & tuberin ○ Brain findings? Subependymal tumors (usually calcified) ○ Renal findings? Angiomyolipoma ○ Cardiac findings? Rhabdomyoma ○ Skin findings ■ Hypopigmented = ash leaf spots ■ Hyperpigmented = Shagreen patch ○ Kind of infantile seizure? West syndrome = infantile spasms ■ EEG findings? hypsarrhythmia ■ Tx? ACTH or vigabatrin

Mom has a history of inconsistent condom use. She delivers a stillborn fetus with no brain. → anencephaly 2/2 Zika ○ Pathophys? Anterior neuropore fails to close ■ Polyhydramnios because no swallowing center Dimple or tuft on hair in lumbosacral area → Spina bifida occulta ○ Pathophys? Failure of the spinous processes to fuse around the spinal cord ○ Usually causes no problems Meningocele ○ Pathophys? Failure of fusion of the posterior neural arch → herniation of meningeal tissue and CSF ○ Associated deficiency in mom? folate ○ Tx? Surgery ○ Good prognosis after surgery Myelomeningocele ○ Pathophys? Failure of neural tube to close → herniation of meninges and spinal cord tissue through defect ○ Associated deficiency in mom? folate ○ Tx? Surgery ○ Worse prognosis ○ Associated dz? Chiari 2 malformation Type 1 Chiari malformation ○ Pathophys? Cerebellar tonsils herniate through foramen magnum ○ Associated dz? Syringomyelia (“Syr1ngomyelia”) Type 2 Chiari malformation ○ Pathophys? Cerebellar tonsils herniate through foramen magnum ○ Associated dz? Lumbosacral myelomeningocele Serum marker for NTD/spinal dysraphism? Elevated maternal serum AFP 155

○ ○

Also elevated ACh esterase What if msAFP is decreased? Down syndrome

Subacute combined degeneration of spinal cord ○ Vitamin deficiency? B12 ○ Loss of dorsal columns → loss of vibration, touch, proprioception ○ Loss of lateral corticospinal tract → UMN signs ○ Differentiate folate & B12 deficiency with serum markers ■ Megaloblastic anemia in both ■ hom*ocysteine high in both ■ MMA high in ONLY B12 deficiency

Cafe au lait spots + brown “pigments” in the axilla + “tuber like” skin growths + episodic headache and severe HTN. → NF-1 ○ Eye malignancy? Optic nerve gliomas ○ Iris finding? Lisch nodules ○ Brain malignancy? Meningioma ○ Neuroendocrine malignancy? Pheo ○ Ear malignancy? Bilateral acoustic neuromas/vestibular Schwannomas in NF-2 ■ Location? Cerebellopontine angle ○ Inheritance? AD ○ Gene? ■ NF-1 → neurofibromin ■ NF-2 → merlin ○ Chromosome ■ NF-1 → 17 ■ NF-2 → 22

----------------------------------------------------------------------------------------------------------------------------

Episode 47: Neurology Clerkship Shelf Review Part 4 ●

Student in a lab is unknowingly touching a lighted bunsen burner in the chemistry lab until his hand is pulled away by the TA. PE is notable for loss of pain and temperature sensation in the UEs bilaterally and mild hand muscle weakness. → syringomyelia ○ Pathophys? Obstruction in the central canal of the spinal cord → cystic expansion of central canal caudally → compression of anterior while commissure → knocks of spinothalamic tract ○ Tracts involved? ■ Spinothalamic → loss of pain + temperature bilaterally ■ If left untreated, will expand and affect the ventral horn → motor sxs (UE first because these tracts are most medium) ○ Dx? MRI

Nuchal rigidity in a recent immigrant presenting with hemoptysis and high fevers. There is marked “enhancement” at the base of the brain. → TB meningitis ○ Dx? ■ LP 156

○ ○

● ●

● ●

● ● ●

CSF findings? ■ Lymphocytic predominance ■ Protein HIGH ■ Glucose LOW ■ High OP Tx? RIPE regimen + Vit B6 Differentiating meningitis from encephalitis ■ Encephalitis → more neuro deficits, may not have nuchal rigidity ■ Meningitis → nuchal rigidity

Most common neurologic sequelae of meningitis in kids → hearing loss Top 3 causes of meningitis in neonates ○ GBS ○ E. coli ○ Listeria Empiric tx for neonatal meningitis? Cefoxamine + vanc + ampicillin ○ Ceftriaxone can cause intrahepatic cholestasis in neonates CSF studies in bacterial, viral, and fungal meningitis. ○ Bacterial ■ Neutrophilic predominance ■ Protein HIGH ■ Glucose LOW ○ Fungal ■ Lymphocytic predominance ■ Protein HIGH ■ Glucose LOW ○ Viral ■ Lymphocytic predominance ■ Protein high ■ Glucose low/normal ○ Patient with symmetric ascending paralysis after a recent bloody diarrheal infection → GBS after Campylobacter infection ■ CSF finding? Albuminocytologic dissociation ● Usually WBC & protein go up together ● In this case, high protein but new WBCs ○ 30 yo F with an afferent pupillary defect and other neuro findings → MS ■ CSF finding? Oligoclonal bands ■ Dx? MRI ○ CSF with a ton of RBCs ■ HSV encephalitis ■ Xanthochromia 2/2 SAH General treatment co*cktail for bacterial meningitis. ○ Ceftriaxone + vancomycin + steroids (CVS) What should be added in the elderly, immunocompromised, or super young (infants/neonates)? ampicillin CSF studies with normal glucose, slightly elevated protein, and a lymphocytic pleocytosis → viral meningitis 157

Classic imaging and CSF findings in HSV meningitis OR encephalitis → enhancement of temporal lobes

25 yo M presents with a T of 103, severe headache, and a 2 day history of profound LE muscle weakness. He was recently treated for Otitis Media. → brain abscess ○ Triad: fever + HA + neurologic deficit ■ No nuchal rigidity ○ Dx? MRI ○ Tx? Drain abscess + steroids to decrease swelling + abx ○ What is the bug that is classically associated with meningitis with MRI enhancement at “the base of the brain”? TB

35 yo zoologist is brought to the ED by ambulance after becoming unresponsive at home. He studies cave dwelling animals. Over the past 2 weeks he has resisted attempts by his parents to give him water or get him to the shower. → rabies ○ Prevention? Rabies immunoglobulin + vaccine (at two different sites) ○ Pathognomonic histologic finding? Negri bodies

Bilateral acoustic neuromas → NF-2 ○ Inheritance? AD ○ Gene? Merlin ○ Chromosome? 22 ○ Involved cranial nerve? CN8 ○ Classic brainstem location? Cerebellopontine angle ○ Dx? MRI

Port wine stain + glaucoma + seizures + ID + tram track CT calcifications → Sturge-Weber syndrome ○ Inheritance? Non-heritable! Sporadic! ○ Gene? GNAQ activating mutation

Woman with trouble sleeping due to weird sensations in legs → restless legs syndrome ○ Associated dz? IDA ○ Tx? ■ pramipexole/ropinirole ■ primidone

Pt that has paresthesias over palmar aspect of hand (+ thenar atrophy → carpal tunnel syndrome ○ Associations ■ RA ■ Pregnancy ■ Hypothyroidism ○ PE maneuvers ■ Tinnel’s sign (tapping over carpal tunnel) ■ Phalen sign (flexed wrists) ○ Tx? ■ Wrist splint ■ NSAID ■ Inject steroids 158

■ Carpal tunnel release ○ What should be done to confirm the dx before pursuing surgery? Nerve conduction study! ----------------------------------------------------------------------------------------------------------------------------

Episode 48: Neurology Clerkship Shelf Review Part 5 ●

32 yo Asian M presents with severe R eye pain. PE reveals a “rock hard” unreactive pupil. → acute-angle glaucoma ○ Dx? Tonometry (eye pressures) ○ Tx? ■ Usually requires surgery: laser iridotomy ■ Medical management ● Mannitol (lows IOP by drawing water out of the vitreous humor of the eye and into the intravascular space) ● Acetazolamide (decreases aqueous humor production) ● Timolol (decreases aqueous humor production) ● Pilocarpine (muscarinic agonist, constricts pupil)

A 3 yo F is brought to the ED by her dad. She has been using the potty continuously for the last 3 hrs. PE is notable for pupillary miosis and profuse sweating. She played in the farm this morning. → organophosphate poisoning ○ Pathophys? Organophosphates inhibit ACh esterase → ACh levels rise → parasympathetic overactivation ○ Tx? Atropine (blocks AChR) + pralidoxime (regenerated AChE)

Flaccid paralysis after consuming home canned goods. → botulism ○ Pathophys? Botulinum toxin cleaves SNARE proteins → prevents release of ACh from presynaptic neuron ○ Tx? ■ Botulinum immune globulin ■ Intubate for respiratory support ○ Contrast to tetanus, which will have a spastic paralysis ■ Pathophys? tetanus toxin prevents the release of glycine & GABA (inhibitory NTs) from Renschaw cells → spastic paralysis Difficulty getting out of a chair in a patient with a 45 pack year smoking history that improves with muscle use. → LEMS ○ Pathophys? Ab to the presynaptic voltage-gated Ca++ channel ○ Increment in muscle contraction with repetitive nerve stimulation ○ Associated malignancy? Small cell lung cancer Difficulty swallowing and droopy eyelids in a 35 yo F with an anterior mediastinal mass → myasthenia gravis in s/o thymoma ○ Pathophys? Ab against nicotinic AChR ○ Decrement in muscle contraction with repetitive nerve stimulation ○ Dx? anti-AChR antibodies ■ Not Tensilon test anymore ○ Tx? ■ AChE inhibitors (e.g. pyridostigmine)

159

Resection of thymoma ○ In addition to tx with AChE what is NBSM? Chest CT (look for thymoma) ○ How is this condition differentiated from ChAT deficiency? ■ ChAT = choline acetyltransferase ■ ChAT deficiency sxs will not improve with AChE therapy Anticholinergic toxidrome = “hot as a desert, dry as a bone, red as a beet, blind as a bad, mad as a hatter” ○ Classic meds: ■ Diphenhydramine ■ TCAs ■ 1st gen low potency antipsychotics (e.g. chlorpromazine) ■

Tongue fasciculations in a 6 mo. → SMA (spinal muscular atrophy) ○ Inheritance? AR ○ Pathophys? Pure LMN disease ○ Mutated gene? SMN1 ○ Chromosome? 5 ○ Viruses that affect anterior horn cells? ■ Polio ■ West nile Tongue fasciculations in a 65 yo M with asymmetric weakness. → ALS ○ UMN + LMN problems ○ Pathophys? Destruction of the UMNs (corticospinal/corticobulbar tract), LMNs (anterior horn of spinal cord), and cranial nerves ○ Genetic mutation in familial cases? SOD1, C9orf72 ○ What is spared in ALS? Sensation, bowel/bladder function, extraocular muscles ○ EMG findings? Chronic denervation, fibrillation potentials ○ Tx that improves survival? ■ Riluzole (NMDA receptor antagonist, reduce glutamate excitotoxicity) ■ Bipap ○ Tx for spasticity? ■ Baclofen (GABA receptor agonist) ■ Tizanidine (alpha-2 agonist) ■ Botox injection

Dermatomyositis

Polymyositis

Kids CD4+ T-cells Perifascicular damage

Adults CD8+ T-cells (poly so it’s more) Endomysial damage

Dermatomyositis/polymyositis ○ Ab? Anti-Jo1, anti-Mi-2, anti-SRP 160

○ ○

Dx? ■ 1st muscle MRI ■ 2nd muscle biopsy Lab findings? Elevated creatinine kinase Associated malignancy? Lung cancer (especially small cell)

Dysarthria + truncal, gait, and limb ataxia in a patient that is subsequently found to have lung cancer → paraneoplastic cerebellar degeneration ○ Associated malignancies? ■ Lung ■ Breast ○ Ab? Anti-Hu, anti-Yo ○ Pathophys? Tumors express cerebellar proteins, immune system tries to attack tumor but also mounts response against cerebellum

Confusion, ophthalmoplegia, and ataxia in an alcoholic. → Wernicke’s syndrome ○ What if there’s also forgetfulness & confabulation → Korsakoff’s syndrome ○ Tx? IV thiamine (Vit B1) Pharmacological management of hepatic encephalopathy ○ Lactulose (convert ammonia to ammonium, which you poop out) ○ Rifaximin (a lot of ammonia comes from bacteria in GI tract)

Simple partial seizures = focal seizure w/o loss of awareness ○ Is there a loss of consciousness? NO ○ 4 types: motor, sensory, autonomic, psychic ■ Rigidity or jerking of extremity → motor ■ Hissing sound or can’t see or smell burnt rubber → sensory ■ Sweating, mydriasis, rising sensation in abdomen → autonomic ○ Auras that localize to temporal lobe ■ Smelling burnt rubber ■ Deja vu ■ Rising sensation in abdomen

Required imaging before recommending sports in a 12 yo with a history of Trisomy 21. Why? High risk of atlanto-axial instability/subluxation ○ Dx? Lateral neck XR ○ Another patient population at risk? ■ RA ■ Ankylosing spondylitis

25 yo M goes into respiratory failure. He recently recovered from a 7 day episode of bloody diarrhea. PE is notable for pronounced areflexia in the LE bilaterally. His parents report that he had muscle weakness that “started in the legs and progressed upward”. He initially complained of leg tingling and numbness before his other sxs started. → GBS ○ Alternative term? AIDP (acute inflammatory demyelinating polyneuropathy) 161

Pathophys? Peripheral demyelinating disease ■ Cells affected? Schwann cells ○ Classically associated bug? Campylobacter jejuni ○ CSF findings? Albuminocytologic dissociation ○ Tx? ■ IVIG ■ Plasmapheresis GBS-like presentation + nystagmus + ataxia? Miller-Fisher syndrome ○

● ●

Pt that has syncope or neuro deficit when they use their arms → subclavian steal syndrome ○ Pathophys? Proximal subclavian stenosis → low pressure system distal to stenosis → reversal of blood flow in vertebral arteries → “stealing” blood from basilar → hypoperfusion to brainstem

162

WEBER & RINNE TESTS ● Weber test compares bone conduction between ears ○ Normally, it should be the same in both ○ If Weber lateralized to the R ear, there are TWO possibilities: ■ Conductive hearing loss in R ear ■ Sensorineural hearing loss in L ear ● Rinne test compares air conduction to bone conduction for each ear (single ear test) ○ Normally, air conduction is better than bone conduction ○ If bone conduction is better than air conduction, it implies conductive hearing loss

Meningitis in a HIV+ patient → cryptococcus neoformans ○ How to ID? ■ India ink stain ■ Latex agglutination test ○ Tx? 163

■ ■

Acutely: Amphotericin B + 5-flucytosine Chronically: fluconazole

Multiple ring enhancing lesions on MR imaging in a HIV+ patient → toxoplasmosis or primary CNS lymphoma ○ Toxoplasmosis ■ Tx? Pyrimethamine & sulfadiazine ■ Ppx for CD4 < 100? TMP-SMX ○ Primary CNS lymphoma - usually single lesion ■ Infectious association? EBV

MMSE is 20/30 in a patient with a CD4 count of 25 and a high viral load. He last saw his PCP after an initial dx 21 yrs ago → HIV-associated dementia ○ Poor prognosis ○ Tx? ■ HAART ■ Alzheimer’s drugs (e.g. rivastigmine, galantamine, donepezil)

HIV+ patient with neuro deficits and multiple areas of demyelination on MR imaging → progressive multifocal leukoencephalopathy ○ Infectious agent? JC virus ○ Cause of drug-induced PML? Natalizumab ■ Indication? MS ■ Mechanism? Alpha-4 integrin inhibitor

----------------------------------------------------------------------------------------------------------------------------

Episode 49: Neurology Clerkship Shelf Review Part 6 ●

Multiple neuro deficits in a kid after getting the VZV vaccine or after an URI. → ADEM (acute disseminated encephalomyelitis) ○ Pathophys? Inflammatory demyelinating lesions in brain & spinal cord ○ Dx? MRI ○ Prognosis? Full recovery

Multiple Sclerosis ○ Classic presentations? Female in her 30s with disparate neuro deficits ■ Uhthoff's phenomenon = sxs worse in heat ○ CN2 pathology? Optic neuritis ■ Presentation? Unilateral eye pain + decreased visual acuity (e.g. 20/200) + afferent pupillary defect ○ Associated Vit deficiency? Vit D ■ Higher prevalence in places farther from the equation ○ Dx? MRI brain & spinal cord ■ Multiple demyelinating lesions separated in space & time ○ LP findings? Oligoclonal bands ○ Exam findings? ■ APD 164

○ ○

■ Lhermitte's sign = electric shock with neck flexion Tx acute exacerbation? Very high-dose corticosteroids Tx chronic/DMARDs? ■ Fingolimod ■ Natalizumab ● Mechanism ● AE? JC virus reactivation → PML ■ Glatiramer ■ Rituximab ■ Interferon-beta Tx urge & overflow incontinence? ■ Urge → oxybutynin, trospium, darifenacin/solifenacin, tolterodine ● Detrusor muscles are hyperactive, so quiet them down with an antimuscarinic ■ Overflow → bethanechol, neostigmine ● Alternative? Intermittent self-cath ● Destrusor hypotonia, so activate detrusor with muscarinic agonists or AChEinhibitors ● High PVRs Tx of spasticity? ■ Baclofen (GABA-B receptor agonist) ■ Tizanidine (alpha-2 agonist → decreases norepi release) ■ Dantrolene (CCB) ■ Benzos (GABA-A receptor agonist) ■ Botulinum toxin for localized spasticity

Pt sees blood or has emotional stressor then passes out → vasovagal syncope ○ Dx? Tilt table test ○ Tx? Midodrine (alpha-1 agonist)

Most common cause of death in patients with Factor 8/9 deficiencies? Hemorrhagic stroke ○ Pathophys? Defect of secondary hemostasis ○ Inheritance? X-linked recessive

Stepwise diagnostic testing in stroke management. ○ 1st dx step? Non-con head CT ■ Why? Differentiate b/t hemorrhagic & ischemic stroke ○ If NCCT is negative? Diffusion-weighted MRI ○ Additional diagnostics? ■ Carotid artery duplex US of internal carotid arteries ■ Echo to look for LA appendage thrombus or PFO ○ Initial tx for ischemic stroke? ■ Aspirin ■ If aspirin isn’t an answer choice, choose another antiplatelet agent ● Clopidogrel ● Dipyridamole ■ Do NOT give anticoagulant unless A-fib ○ If high suspicion for SAH but NCCT negative? LP → look for xanthochromia 165

■ ●

Tx for SAH? Lower BP + nimodipine

Paralysis of ipsilateral upper and lower facial muscles, dry mouth, loss of lacrimation → LMN CN7 lesion ○ Pathway? ■ CNs (except CN2) are LMNs ■ UMNs comprise the corticobulbar tract (connect cerebral cortex to CNs) ● Decussate prior to synapsing with CNs ■ CN7 also carries parasympathetic fibers ○ Why ipsilateral? Decussation happens upstream to CN7 Paralysis of lower facial muscles, sparing of upper face→ contralateral corticobulbar tract lesion ○ Why contralateral? corticobulbar tract decussates BEFORE synapsing on CN7 nucleus ○ Why upper face sparing? Bilateral corticobulbar pathways that synapse on the CN7 nucleus (so you have to knock out both corticobulbar pathways to affect the upper face)

Loss of facial sensation, jaw deviation to the ipsilateral side, impaired corneal reflex. → CN5 (trigeminal) ○ Pathways? ■ Facial sensation is carried by CN5 ■ Corneal reflex ● Afferent = CN5 ● Efferent = CN7 ■ CN5 provides motor innervation to muscles of mastication (e.g. masseter, pterygoids, temporalis) Anosmia → CN1 ○ Associated syndrome? Kallman’s syndrome ■ Presentation? Anosmia + hypogonadotropic hypogonadism 166

● ●

■ Pathophys? Failure of GnRH neurons & olfactory neurons to migrate Mydriasis with the eye deviated down and out. → CN3 (oculomotor) ○ Pathways? ■ CN3 innervates all muscles of eye except superior oblique (CN4 trochlear) & lateral rectus (CN6 abducens) ■ CN3 also carries parasympathetic fibers to go to pupillary constrictors ○ Why down & out? ■ Lateral rectus → abduction ■ Superior oblique → depression ○ Causes of CN3 lesions? ■ PCom aneurysm ■ Uncal herniation ○ For compression lesions, mydriasis often happens before eye deviation ■ Why? Parasympathetic fibers run on the outside of CN3 (so more susceptible to compression) ○ For ischemic lesions (e.g. due to DM), eye deviation will happen first

Bitemporal hemianopsia (“tunnel vision”), afferent pupillary defect. → optic chiasm compression ○ Causes? ■ Pituitary adenoma ■ Craniopharyngioma ■ Contralateral lower facial weakness with forehead sparing and loss of taste sensation with the anterior ⅔ of the tongue. → UMN CN7 lesion ○ Pathways? ■ General sensation anterior ⅔ tongue → CN5 ■ Taste anterior ⅔ tongue → CN7 Horizontal diplopia with failed abduction on lateral conjugate gaze. → CN6 lesion Vertical diplopia with the jaw tilted towards the side of the lesion + difficulty going down stairs. → CN4 (trochlear) lesion 167

● ●

Pathway? ■ Trochlear emerges dorsally & crosses to opposite side ■ It’s the only CN that decussates Note that the head tilt if TOWARDS the side of the lesion

Sensorineural hearing loss, vertigo, and abnormal caloric test results. → CN8 (vestibulocochlear nerve) ○ Caloric test results → COWS mnemonic (cold opposite, warm same) Deviation of the protruded tongue to the ipsilateral side. → CN12 ○ Mnemonic “Lick your wounds” ○ Pathway? Innervates all intrinsic muscles of the tongue except palatoglossus (innervates by CN10) Weakness in shoulder shrug + problems turning the head to the opposite side. → CN11 (spinal accessory nerve)

168

● ● ● ●

Loss of taste sensation in the posterior ⅓ of the tongue + dysphagia + absent gag reflex. → CN9 or 10 ○ Pathways? ■ Taste posterior ⅓ of tongue → CN9 ■ General sensation posterior ⅓ of tongue → CN9 ■ Taste extreme posterior tongue → CN10 ■ General sensation extreme posterior tongue → CN10 ■ Gag reflex ● Afferent limb → CN9 ● Efferent limb → CN10 Innervates the superior oblique muscle → CN4 (trochlear) Innervates the lateral rectus → CN6 (abducens) ○ Location? medially Anosmia + hypogonadotropic hypogonadism → Kallmann syndrome Differentiating between an afferent and efferent pupillary defect ○ Assume R-sided lesion ○ APD ■ Shine light into R eye → nothing ■ Shine light into L eye → bilateral constriction ○ EPD ■ Shine light into R eye → only L constriction ■ Shine light into L eye → only L constriction ■ Eye with the defect doesn’t constrict regardless Most common pediatric brain tumor → pilocytic astrocytoma ○ Location? Posterior fossa 169

Hemorrhagic lesion in the cerebellum with a path specimen revealing some kind of rosette (perivascular, Homer Wright) → medulloblastoma ○ 2nd MC pediatric brain tumor ○ Location? Cerebellum ○ Presentation? Ataxia + elevated ICP ○ Associated syndrome? Turcot syndrome (colon cancer + brain tumors) Brain tumor that drains “motor oil fluid” and is calcified/could present with tunnel vision → craniopharyngioma ○ Presentation? Bitemporal hemianopsia 2/2 optic chiasm compression ○ Embryologic origin? Rathke’s pouch ○ Location? Sella turcica ○ Imaging? Calcified Kid with abdominal mass that crosses midline with calcifications on imaging → neuroblastoma ○ Alternate location? Posterior mediastinum ○ Presentation? Opsoclonus myoclonus syndrome (“dancing eyes, dancing feet”)

Neck pain radiating to the arm with specific dermatomes affected and multiple peripheral nerves involved → cervical radiculopathy ○ Pathophys? Problem with the nerve ROOT ■ Versus myelopathy, which is a problem with the spinal cord

Complete paralysis of the face, arms, and legs with no sensory losses and contralateral “clumsiness” → internal capsule lesion (subcortical lesion) ○ Pathway? Motor fibers from the cortex condense and form the posterior limb of the internal capsule ○ Artery involved? Lenticulostriate arteries

Inability to calculate + eyes looking towards the side of the lesion + facial paralysis contralateral to the side of the lesion + UMN signs → cortical stroke ○ Acalcula → dominant parietal lobe lesion ■ Contrast with nondominant parietal lobe (often R) lesion → L-sided hemineglect ○ Eyes looking toward side of the lesion → frontal eye field lesion ■ R front eye field → L PPRF→ controls L abducens and R oculomotor ■ R frontal eye field lesion → no communication to L PPRF → R PPRF unopposed → R deviation

Complete sensory loss on the left + severe pain on the right → lesion of the thalamus (subcortical lesion)

Ataxia, past pointing, impaired rapid alternating movements (dysdiadochokinesia), and intention tremor → ipsilateral cerebellar lesion

Bowel/bladder dysfunction + UMN and LMN findings + sensory level → spinal cord compression ○ E.g. cauda equina syndrome ○ UMN findings below level of lesion ○ LMN findings at level of lesion

Trouble swallowing + problems with eye adduction on conjugate gaze + sensory loss on the left face + sensory loss on the “right body” → lateral medullary syndrome (Wallenberg syndrome) ○ Pathophys? 170

■ Sensory loss L face → L spinal trigeminal nucleus ■ Sensory loss R body → L spinothalamic tract ■ Trouble swallowing → nucleus ambiguus lesion (supplies CN9/10) Affected arteries? PICA

Hemiballismus and bradykinesia → contralateral subthalamic nucleus lesion

Stocking/glove distribution of sensory loss + LMN findings → peripheral nerve lesion

Trouble rising from the seated position + Elevated CK → polymyositis/dermatomyositis ○ Pathophys? Problem at the level of the muscle

Fatigable muscle weakness → neuromuscular junction issue (e.g. MG)

4 signs of a basilar skull fracture. ○ Bruising behind mastoid (“Battle's sign”) ○ Raccoon eyes ○ CSF rhinorrhea ○ CSF otorrhea

peripheral neuropathy vs. radiculopathy ○ One nerve involved, stocking-glove distribution → peripheral neuropathy ○ Multiple nerves involved, dermatomal distribution → radiculopathy

Weakness in shoulder abduction/deltoid paralysis. → axillary nerve lesion ○ Cause? ■ Anterior shoulder dislocation ■ Surgical neck fracture Weakness in foot eversion and dorsiflexion → peroneal nerve lesion ○ DroPED → peroneal nerve lesion/eversion/dorsiflexion Weakness in foot inversion and plantarflexion → tibial nerve lesion ○ TIP → tibial/inversion/plantarflexion Weakness in wrist extension. → radial nerve lesion ○ Cause? Midshalf fracture of humerus ○ Reflex supplied by radial nerve? triceps Problems with thumb abduction with sensory losses on the ventrolateral 3.5 digits. → medial nerve compression in CTS ○ Pathophys? Compression of medial nerve by flexor retinaculum ○ Risk factors? ■ Pregnancy ■ RA ■ Hypothyroidism Problems with hip flexion → femoral nerve ○ F for Femoral and Flexion Problems with hip adduction. → obturator nerve ○ Also cannot internally rotation ○ Lots of AIR between thighs (AIR = adduction/internal rotation)

● ● ●

● ●

171

Weakness with finger spread and problems with flexion at the MCPs and extension at the IP joints → ulnar nerve lesion ○ Pathophys? Ulnar nerve supplies the dorsal interossei, whose function is finger aBduction (spreading) + MCP flexion + IP joint extension Nerve roots controlling the following reflexes: ○ Biceps → C5/6 ○ Triceps → C7/8 (radial nerve) ○ Knee extension/patellar tendon reflex → L3/4 ○ plantarflexion/ankle jerk reflex → S1/2 Nerve roots associated with the following levels: ○ Nipple line → T4 ○ Xiphoid process → T7 ○ Umbilicus → T10

----------------------------------------------------------------------------------------------------------------------------

Episode 58: Neurology Clerkship Shelf Review Part 7 ●

Discussion of the pathway/information carried by the 3 HY spinal cord tracts: ○ DCMLS ■ Fine touch, vibration, proprioception ■ 1st order: ascends in the dorsal spinal cord → fasciculus gracilis/cuneatus → ascend ipsilaterally to caudal medulla → SYNAPSE ■ 2nd order: nucleus gracilis/cuneatus → decussates in medulla → ascend contralaterally as medial lemniscus → thalamus → SYNAPSE ■ 3rd order: thalamus → postcentral gyrus ○ Spinothalamic tract ■ Pain & temperature ■ 1st order: rises 1-2 levels → SYNAPSE ■ 2nd order: decussates via anterior white commissure → ascends contralaterally through lateral spinal cord & lateral brainstem → thalamus → SYNAPSE ■ 3rd: thalamus → postcentral gyrus ○ Corticospinal tract ■ Motor pathway ■ 1st order: precentral gyrus → travel medially in brainstem → pyramidal decussation (medulla) → ventral spinal cord → SYNAPSE ■ 2nd order: anterior horn motor neurons (LMN) → skeletal muscle Romberg test = close eyes while standing with feet together ○ Test of dorsal column function (proprioceptive ability) ○ Need ⅔ things to know where you are in space ■ Vision ■ Vestibular system ■ Dorsal columns ○ Examples where Romberg will be positive: ■ 3ary syphilis with tabes dorsalis Rapid alternating movement ○ Dysdiadochokinesia = inability to perform rapid alternating movements ○ Test for cerebellar ataxia 172

Homunculus ○ Most medial: leg ○ In between: arm ○ Lateral: face Truncal ataxia is indicative of what kind of lesion? Cerebellar vermis

CIRCLE OF WILLIS & INFARCTION SYNDROMES ● Anterior spinal artery ○ Supplies medial medulla ○ Infarction → tongue deviation (CN12), contralateral hemiparesis (CST) ○ AAA repair: can infarct artery of Adamkowitz, which is precursor artery to ASA → anterior spinal cord syndrome ■ Lose everything in cord except for dorsal columns ● PICA ○ Supplies lateral medulla ○ Infarction → uvular deviation, swallowing issues, loss of gag reflex (CN9/10) ● AICA ○ Supplies lateral pons ○ Infarction → facial drop (CN7), CN8 dysfunction, Horner’s syndrome (hypothalamic-sympathetic tract), pain & temp (spinothalamic tract) ● Basilar ○ Supplies medial pons 173

● ●

● ●

○ Infarction → “locked in syndrome” Posterior cerebral artery ○ Supplies the midbrain & primary visual cortex (occipital lobe) ○ Infarction → cortical blindness (loss of vision everywhere except macula) ■ Macula has dual blood supply from PCA & MCA CN3 courses between the superior cerebellar artery & posterior cerebral artery PCom runs along CN3 ○ PCom aneurysm → compression of CN3 → parasympathetic fibers lie on the outside → loss of efferent limb of pupillary light reflex

Most common location for aneurysms? ACom Anterior cerebral artery ○ Supply medial cerebral cortex ○ Infarction → LE paralysis

Headache that is worse with sitting upright and better with lying down after a lumbar puncture → post-LP headache (“spinal headache”) ○ Pathophys? CSF leak after LP → lower intracranial/intraspinal pressures → loss of cushion ○ Tx? ■ Usually self-limited ■ Can do a blood patch ■ Caffeine for pain CN3 LESIONS ● Isolated mydriasis with sparing of other CN3 functions. Down and out pupil without mydriasis → CN3 compression ○ Pathophys? Parasympathetic fibers are on the outside, so are most affected by compression ○ What aneurysm could cause this? PCom ● Down and out pupil without mydriasis ○ Pathophys? Ischemia of CN3 → infarction of deep vasculature that supplies motor fibers, but outer parasympathetic fibers are supplied by vessels on the surface of the nerve ○ Associated condition? DM ● Down and out pupil + Mydriasis + weakness of other extraocular muscles + sensory loss over the forehead → CN3/4/6 + V1 affected → cavernous sinus thrombosis ○ Pathophys? CN3/4/5 + V1 + V2 run through the cavernous sinus ● Down and out pupil + Mydriasis + Contralateral motor paralysis → uncal herniation ○ Pathophys? Uncal herniation → CN3 compression + brainstem compression (contralateral motor deficits bc it occurs before decussation in the medulla) CN7 LESIONS ● Recall: cranial nerves are the LMNs for the skeletal muscle in the head & neck 174

● ●

Derived from neural crest (except CN2, which is an outgrowth of the diencephalon so derived from neural tube) ○ Corticobulbar tract = UMN for cranial nerves ○ UMN cranial nerve lesion → contralateral deficit ○ LMN cranial nerve lesion → ipsilateral deficit ■ Go straight from nucleus to skeletal muscles ■ Exception: trochlear nerve (also decussates) ● UMN trochlear nerve lesion → ipsi deficit ● LMN trochlear nerve lesion → contra deficit UMN CN7 lesion → contralateral facial droop with forehead sparing ○ Pathophys? Cortical infarct ○ Why forehead sparing? Bilateral UMN innervation for forehead ○ Associated deficit? Hemiparesis on same side as facial droop LMN CN7 lesion → ipsilateral facial droop Bell’s palsy ○ Presentation? Can’t close mouth / can’t close eye on one side ○ Pathophys? LMN CN7 lesion ○ Causes? ■ Lyme disease ● Bug? Borrelia berdoferi ● Vector? Ixodes tick ■ HSV ■ MS ■ Sarcoid ○ Tx? ■ Eye drops for affected eye ■ Steroids ■ Acyclovir if presumed 2/2 HSV

Most susceptible CN to elevated ICP? CN6 (abducens)

55 yo F presents with a severe headache that appears to be concentrated just over her right eye. She has a history of jaw pain when chewing nuts and proximal shoulder pain that was relieved with low dose prednisone → temporal arteritis ○ NBSM? High-dose steroids BEFORE biopsy ○ Dx? Temporal artery biopsy ■ If one side is negative, biopsy the other side! ○ Lab findings? ■ High ESR/CRP

39 yo F with a PMH of MS presents with severe left eye pain and decreased vision. The right and left eye do not constrict when light is shone in the left eye. → APD 2/2 optic neuritis ○ Pupillary light reflex? CN2 afferents → pretectal nucleus → CN3 efferents ○ Dx? Slit-lamp exam ○ Tx? steroids

175

65 yo M with a past history of DM and CAD presents with a 5 day hx of decreased vision/floaters in his right eye. Funduscopic exam reveals retinal venous engorgement and diffuse hemorrhage → retinal detachment ○ Dx? US eye or ophthalmoscopy ○ Tx? Retinal reattachment procedure Same patient presents with the sudden loss of vision in his R eye. Funduscopic exam is notable for optic disc pallor and a cherry red spot on the macula. → central retinal artery occlusion (CRAO) ○ Dx? ■ Usually a clinical dx, but can use fluorescein angiography if needed ■ Carotid US (usually embolic from carotid plaque) ○ Tx? ■ Ocular massage ■ intra-arterial tPA Transient loss of vision in the same patient with a return to baseline after 10 mins. He says that it felt like a “curtain coming down” → amaurosis fugax ○ Pathophys? TIA of the eye ○ Future complication? stroke ○ Dx? ■ Carotid US Ischemic strokes ○ Time window for TPA administration? Within 4.5 hrs ○ Medical therapy for most strokes? Antiplatelet agent ■ Aspirin ■ Clopidogrel ○ What is the only scenario that is amenable to a consideration of heparin/warfarin administration in a stroke setting? A-fib

A = optic nerve → loss of vision in one eye B = optic chiasm → heteronymous hemianopsia C = optic tract → hom*onymous hemianopsia D = Meyer’s loop (lower fibers) → “pie in the sky”/superior quadrantanopia 176

E = visual cortex → hom*onymous hemianopsia with macular sparing

● ●

Underlying mass in a child that presents with bitemporal hemianopsia? Craniopharyngioma ○ Imaging? Calcified suprasellar mass ○ Derived from? Rathke’s pouch Underlying mass in an adult that presents with bitemporal hemianopsia? Pituitary adenoma In a cerebral cortex stroke, how would the eyes deviate in conjugate gaze (vs a brainstem stroke)? Toward the side of the lesion ○ “Into the hole, away from the fire” ■ Same side in cortical strokes ■ Opposite side in seizures ○ R frontal eye field → decussates → L PPRF → controls L CN6 & R CN3 (connected by MLF) ○ Pathophys (R lesion)? Loss of R frontal eye field → L CN6 & R CN3 not working → R CN6 & L CN3 unopposed → R gaze ○ What side would have hemiparesis? Contralateral In a brainstem stroke, how would the eyes deviate in conjugate gaze? Away from the side of the lesion ○ Pathophys (R lesion)? Loss R PPRF → R CN6 & L CN3 not working → L CN6 & R CN3 unopposed → L gaze ○ What side would have hemiparesis? Contralateral ■ Lesion is before decussation of corticospinal tract in medulla A patient with a history of MS presents with a R abduction nystagmus and impaired L eye adduction when asked to look to the right. → INO ○ Pathophys? MLF lesion ipsilateral to the eye that can’t adduct ■ L MLF lesion → signal doesn’t get to L CN3 (can’t adduct L eye)

Ptosis + miosis + anhidrosis → Horner’s syndrome ○ Locations: ■ Sympathetic trunk (Pancoast tumor) ■ Lateral brainstem stroke ○ Contract with cluster HA, which will have ptosis + miosis but NO anhidrosis

19 yo M visited a chiropractor for some neck massages 2 hrs ago. He was brought to the ED by ambulance after complaining of right sided neck pain. PE is notable for right sided pupillary ptosis and miosis. There are no left sided findings. → carotid artery dissection 2/2 neck trauma ○ Dx? CT/MR angiography

177

178

179

180

----------------------------------------------------------------------------------------------------------------------------

Episode 59: Neurology Clerkship Shelf Review Part ●

Paresthesias in pt being treated for TB? INH toxicity ○ Prevention? Vit B6 (pyridoxine)

181

Pt with hx of poorly treated Crohn’s disease now with paresthesias + loss of proprioception + lower extremity hyperreflexia → B12 deficiency ○ Pathophys: ■ Crohn’s destroys the terminal ileum, where B12-IF is absorbed ■ B12 deficiency → subacute combined degeneration of spinal cord ● Destroy dorsal columns → loss of proprioception, vibration, and fine touch ● Destroy lateral corticospinal tract → UMN signs (spasticity, hyperreflexia) ■ Dx? Macrocytic anemia on CBC + high MMA + high hom*ocysteine ■ Schilling’s test: inject B12 then give radiolabeled oral B12, everything is saturated so oral B12 should go to urine → if not, it indicates pernicious anemia or terminal ileum disease. ● Then give radiolabeled oral B12 + IF. ● If you see B12 in the urine now, the problem is pernicious anemia. ● If not, it’s probably a terminal ileum absorption issue ○ D-xylose test will be abnormal ■ Tx? Supplementation

Ataxia and hemolytic anemia in a pt with hx abetalipoproteinemia. MRIb shows cerebellar atrophy. → Vit E deficiency ○ Pathophys? deficiency in microsomal triglyceride transfer protein which is necessary for creating beta-lipoproteins (ApoB48 & ApoB100), which transport fats, cholesterol, and fat-soluble vitamins from intestines to bloodstream ○ Peripheral blood smear finding? Acanthocytosis

Mamillary body infarcts in a chronic alcoholic. → Wernecke’s syndrome ○ Pathophys? Thiamine (B1) deficiency

Dermatitis and dementia in a patient with a long history of carcinoid syndrome. → pellagra → 2/2 niacin deficiency ○ Pathophys? All of the tryptophan is shunted towards serotonin production, so niacin cannot be produced ○ Dx of carcinoid? Urine 5-HIAA (serotonin metabolite) ○ Other causes of pellagra? ■ Hartnup disease ● Pathophys? Defect in neutral amino acid transporter

Child with chronic abdominal pain and foot drop who over the past 6 mo has been performing poorly in school. → lead poisoning ○ Peripheral blood smear findings? ringed-sideroblast ○ Dx? Blood lead levels ■ Confirmatory test? Venous lead level ○ Tx? Lead chelator ■ EDTA ■ succimer

Weird lines on the fingers + garlic breath. → arsenic poisoning ○ Derm findings? Mees lines (horizontal white lines) on nails

182

● ● ●

Child with declining grades and many staring episodes → absence seizures ○ Seizure type? Generalized seizure ○ EEG findings? 3 Hz spike-and-wave ○ Tx? Ethosuximide (T-type CCB) Seizures heralded by olfactory hallucinations and a rising sensation in the abdomen → temporal lobe epilepsy ○ Seizure type? Auras = simple partial seizure Seizures associated with sensory/motor problems → parietal/frontal lobe seizures Focal post-seizure paralysis and weakness → Todd’s paralysis ○ Will go away on is own in hours Focal seizures = partial seizures (activity localized to 1 hemisphere) ○ Focal seizure w/o loss of awareness = simple partial seizures ■ Examples: ● Focal muscle rigidity ● Bizarre sensations (e.g hearing hissing sound) ● Autonomic sxs ○ Focal seizure with loss of awareness = complex partial seizures ■ Can be a person who appears awake but is not aware of surroundings ■ Presentation? Automatisms (e.g. chewing, smacking lips) Generalized seizures (activity in both hemispheres) ○ Absence ○ Generalized tonic-clonic ○ Atonic seizures (loss of muscle tone → pt drops to ground) ■ Also consider cataplexy in narcolepsy ○ Myoclonic seizures (jerking movements)

Brain death = loss of all brain function, including the brainstem ○ Loss of vestibulo-ocular reflex (perform caloric testing) ○ Loss of pupillary light reflex ○ Apnea test → no spontaneous respiration after CO2 is allowed to rise ■ Hypercarbia normally stimulates respiratory center in medulla

Reflex syncope ○ Syncope with sight of blood, emotional event, pain → vasovagal syncope ■ Dx? Tilt table test ○ Syncope after tightening tie → carotid sinus hypersensitivity ○ Pt trying to pee and they pass out → micturition syncope Syncope w/o prodrome in pt with ASCVD risk factors → cardiogenic syncope

● ●

Algorithm for the management of a patient in status epilepticus. ○ 1st step: benzo (e.g. lorazepam) ○ 2nd step: IV phenytoin/fosphenytoin ○ 3rd step: sedate with phenobarbital

AED with the strongest risk of neural tube defects → valproic acid ○ What if the patient is already pregnant and couldn’t be controlled on other agents but is now well-controlled on valproic acid? Continue valproic acid 183

Why? Seizures are very dangerous to the fetus

Involuntary flailing movement of 1 arm → hemiballismus ○ Pathophys? Lesion in contralateral subthalamic nucleus

Involuntary movements of the jaw with no other neuro deficits relieved with “gentle stroking” (geste antagoniste) → focal dystonia ○ Tx? Botulinum toxin injection ■ Mechanism? Cleaves SNARE proteins → no vesicle fusion → no ACh release

Fever, headache, stiff neck → meningitis ○ NBS? Head CT, especially if signs of increased ICP on exam Sudden onset severe headache and neck stiffness → SAH ○ NBS? Non-con head CT Fever, headache, and FNDs → brain abscess ○ Dx? Brain MRI ○ Tx? Broad-spectrum abx + drainage by neurosurg Recent history of otitis media with FNDs and papilledema on fundoscopic exam → brain abscess IVDU with back pain + FNDs → spinal epidural abscess ○ Dx? MRI spine ○ Tx? Broad-spectrum abx + drainage by neurosurg

● ●

● ●

Ataxia, urinary incontinence, and dementia → normal pressure hydrocephalus ○ “Wacky, wet, and wobbly” ○ Tx? Decrease ICP with VP shunt Elderly patient with a MMSE of 20/30 with no past neurological history who took Benadryl last night → deliremia ○ Pathophys? Diphenhydramine has powerful anticholinergic activity ○ Anticholinergic drugs? ■ Drugs for urge incontinence (e.g. oxybutynin, trospium, darifenacin/solifenacin, tolterodine) MMSE 19/30 + difficulty speaking + inability to perform ADLs → Alzheimer’s disease ○ What do you need before formal AD dx? Brain MRI ○ Apo association? ■ ApoE4 → higher risk ■ ApoE2 → protective ○ Associated genetic dz? Down syndrome ■ Pathophys? Amyloid precursor protein is on chromosome 21 ■ Presentation? AD in 40s ○ Pathology? ■ Senile plaques (extracellular deposits of amyloid) ■ Neurofibrillary tangles (intracellular aggregations of hyperphosphorylated tau protein) ○ Neuroanatomical association? Basal nucleus of Meynert (produces ACh) ○ Enzyme association? ChAT = choline acetyltransferase ○ Tx? ■ Three select AChE-inhibitors 184

● Galantamine ● Donepezil ● Rivastigmine Memantine (NMDA receptor antagonist)

Parkinsonian features + visual hallucinations + syncopal episodes → Lewy body dementia

Dementia + Choreiform movements → Huntington’s disease ○ Inheritance? AD ○ Repeat? CAG ○ Imaging? Atrophy of the caudate ○ Pathophys? Think of it as a high-dopamine disorders ○ Tx? Dopamine antagonist ■ Haloperidol ■ Tetrabenazine (VMAT inhibitor)

Mild dementia + difficulty speaking + inappropriate behavior + Knife’s edge appearance on brain imaging → Pick’s disease = frontotemporal dementia

“Stepwise” deterioration in cognitive function w/neuro deficits → vascular dementia ○ Presentation? 5 years ago pt started to be forgetful. 2 years ago this pt xyz. ○ RF? ■ HLD ■ CAD ■ Hx stroke ○ Tx? Cholinesterase inhibitors used in Alzheimer’s Rapidly progressive dementia in a patient that got a corneal transplant a year ago with myoclonus → CreutzfeldtJakob disease ○ CSF findings? Elevated 14-3-3 protein Mild dementia + Urinary incontinence + gait problems. → normal pressure hydrocephalus

● ● ●

Inability to calculate + left to right disorientation. → dominant parietal lobe lesion (Gerstmann's syndrome) ○ Which side in most people? Left Neglect of one side of the body. → nondominant parietal lobe lesion ○ Which side in most people? Right HY AED side effects ○ Valproate ■ Highest risk of NTD ■ hepatotoxic ○ Carbamazepine ■ Agranulocytosis ■ SIADH ■ Teratogen ○ Phenytoin ■ Gingival hyperplasia ● Other drug? cyclosporin 185

■ ■ ■ ■ ■

Arrhythmias (class 1b antiarrhythmic) Diplopia/visual issues SJS Drug-induced lupus ● Ab? anti-histone Fetal hydantoin syndrome (microcephaly + flat nasal bridge + cleft lip/palate)

15 yo with jerky hand movements in the morning and occasional generalized tonic clonic seizures. → juvenile myoclonic epilepsy

Recurrent seizures + contralateral hom*onymous hemianopia + problems understanding speech (or can talk but speech is not understandable) + recurrent nosebleeds + Positive FOBT → Hereditary hemorrhagic telangiectasia (Osler-Weber-Rendu disease) ○ Presentation? Lip telangiectasias ○ Pathophys? AVMs in multiple organs (e.g. brain, GI tract) ■ In this example, AVM in the temporal lobe ○ Imaging? Can have dystrophic calcifications ○ Inheritance? AD ○ Contrast with Peutz-Jeghers ■ Hyperpigmented macules on lips ■ multiple nonmalignant hamartomatous polyps in GI tract ■ Increased risk of colon, pancreatic, breast, gyn cancers

Sudden onset of redness in the eye + “bulging eyes” + “humming sound” on auscultation of the skull + visual acuity 20/100 in the affected eye → carotid cavernous fistula ○ Pathophys? Essentially an AVM ○ PE finding? Bruit on auscultation of the skull ○ Dx? MRA ○ Tx? surgery

12 yo M that stands from a seated position by moving his hands over his legs → duch*enne muscular dystrophy ○ Inheritance? X-linked recessive ○ Mutated protein? dystrophy ○ Pathophys? Defective cytoskeleton proteins ○ Prognosis? Death in teens to early 20s ○ Contrast with Becker muscular dystrophy ■ Inheritance? X-linked recessive ■ Pathophys? Mutated dystrophin still has some function ■ Prognosis? Live to 50s+

Kid that is hypotonic at birth + pediatrician has trouble releasing grip from his mom during a 3 mo well child check OR 25 yo M with “severe balding” → myotonic dystrophy ○ Inheritance? AD ○ Repeat? CTG ○ Mutated gene? DMPK

18 yo M presents with a chief complaint of facial weakness and UE weakness that have progressively worsened over the past 18 months → facial-scapulo-humeral dystrophy 186

○ ○

Presentation? Usually in teens Inheritance? AD ■ Affects boys and girls!

Polymyositis/Dermatomyositis ○ Presentation? Symmetric proximal muscle weakness ○ Pathophys? Inflammatory myopathies ○ Ab? anti-Jo1, anti-Mi2, anti-SRP ○ Derm findings in dermatomyositis? ■ Gottron's papules ■ Heliotrope rash ■ Shawl sign ○ Associated malignancy? Usually lung cancer or visceral malignancy ○ Dx? CK level ■ If elevated → MRI of muscle (no longer muscle bx) ○ Tx? Steroids ○ Which one can show up in kids? dermatomyositis

Inclusion body myositis ○ Presentation? Asymmetric distal arm + proximal leg muscle weakness

Toxidromes ○ Diarrhea + rhinorrhea + sweating + miosis → cholinergic ■ Causes? ● Nerve gases (e.g. sarin) ● Organophosphate poisoning ■ Tx? Atropine + pralidoxime ○ Mydriasis + constipation + flushing + hot skin + tachycardia → anticholinergic ■ Causes? ● Jimson weed ○ Pt with psych hx with arrhythmia or wide QRS → TCAs ■ Tx? Sodium bicarb ○ Hypertension (possibly MI) + mydriasis + nasal septum perforation → cocaine ■ Tx? Benzos + alpha-1 blocker (e.g. phentolamine) ■ What drug is contraindicated? Beta blocker ○ AMS + rash near nose/mouth + nothing on Utox + type 1 RTA → Glue/Toluene ■ Pathophys? Glue sniffing causes hyperadrenergic response, which gives you the high. But this can also cause arrhythmias (often V-fib). ■ Electrolyte imbalance? Hypokalemia due to type 1 RTA ● No H+ excretion and K+ reabsorption in alpha-intercalated cells ○ Miosis + respiratory depression (RR = 6) → opioid overdose ■ Tx? naloxone ■ There is no tolerance to which effects of opioids? ● Miosis ● Constipation ○ Normal pupils + respiratory depression → benzodiazepines (could also be barbiturate) ■ Tx? Flumazenil for benzos or Z-drugs 187

○ ○ ○ ○ ○

Flashbacks + visual hallucinations + synesthesias → LSD Pt that is acting like the hulk + nystagmus → PCP Bad oral hygiene + sympathetic activation → Methamphetamine Hyperthermia + hyponatremia + affectionate behavior → Ecstasy/MDMA Someone who took drugs now with parkinsonism → MPTP-mediated destruction of substantia nigra

● ●

Symmetric descending flaccid paralysis w/o sensory deficits → botulism Ascending paralysis w/o sensory deficits → GBS ○ CSF findings? Albuminocytologic dissociation

Seizing patient with small cell lung cancer with a Na of 115 → hyponatremia 2/2 SIADH ○ NBS? Hypertonic saline ■ Only indication on NBMEs is seizing pt with Na+ < 120

2 HY electrolyte imbalances that could cause seizures in the infant of a diabetic mother? ○ Hypoglycemia ■ Pathophys? Hyperplasia of pancreatic islet cells → hypersecretion of insulin → hypoglycemia after delivery ○ Hypocalcemia ■ Associated syndrome? DiGeorge

----------------------------------------------------------------------------------------------------------------------------

Episode 65: Ventilator Physiology for the USMLEs Episode 65 notes were graciously provided by Divine Intervention from an anonymous contributor.

4 parameters to adjust to deal with ventilator issues 1. 2. 3. 4.

Respiratory rate Tidal volume PEEP FiO2

Problem with oxygen ● ●

Pathophys: problems with oxygenation! Tx: adjust PEEP, FiO2 o Increased PEEP ● Keep alveoli open --> promotes gas exchange ▪ i.e. ARDS: PCWP 21%

Problem with CO2 ● ●

Pathophys: problems with ventilation! Tx: adjust RR, tidal volume o Increased RR --> breath faster --> blow off CO2 o Increased tidal volume --> blow off CO2 o ^opposite for low CO2 Asthma, CO2 tension rising: NBS intubation o Usually, asthma pts hyperventilate --> decreased CO2 188

● ● ●

● ●

PE --> hyperventilation --> hypocapnia COPD --> high CO2 tension "chronic CO2 retainers" --> compensatory metabolic alkalosis Pt travels to higher elevation --> oxygen tension supporting less weight (less gravity) --> atmosphere pressure goes down i.e. 760mmhg --> 750mmHg --> less O2 inhaled (FiO2 stays the same, but less absolute pressure) --> less O2 delivered to alveoli --> body compensation via hyperventilation --> blow off CO2 --> hypocapnic = respiratory alkalosis --> kidneys try to respond by getting rid of HCO3- (can help this along w acetazolamide) Pt overdoses on aspirin --> increased respiratory rate --> 1. respiratory alkalosis 2. metabolic acidosis 100% O2 not improving O2 status: shunt o Shunt physiology: perfusion without ventilation ● i.e. PFO: blood bypassing lungs --> not getting oxygenated

Cross checked? YES ----------------------------------------------------------------------------------------------------------------------------

Episode 94: Rapid Review, Series 1, Peds 1.Hemangioma + thrombocytopenia = Kasabach-Merritt Syndrome 2.Gallbladder hydrops classic association with Kawasaki disease 3.Kid is tachycardic + hypokalemic (increased activity of Na/K pump) + treated for asthma → albuterol 4.Triad: nonthrombocytopenic purpura + arthritis + abdominal pain = Henoch Schoenlein purpura -

HSP = Risk factor for intussusception Manifestation of IgA nephropathy

5.Drug of choice in tx of lyme disease in kid 10, ankylosing spondylitis like picture = polyarticular JRA 8.JRA + spiking fevers + salmon colored rash = still's disease (systemic JRA) 9.MCC osteomyelitis in kid with Sickle cell = salmonella 10.MCC osteomyelitis in neonate = group B strep 11.Infection in kid from 0-28 days old = group B strep until proven otherwise 12.Kids w/ recent URI or otitis media followed by hip pain with little elevated ESR and CRP with full ROM, no leukocytosis =transient synovitis tx with NSAIDs 13.Kid okay during day, normal PE, thigh hurts at night = growing pains 14.Palpable clunk on neonate exam = developmental dysplasia of hip 15.Hip pain < 9 years old = legg-calve-perthes disease 16. Kid >9 years old + obese + hip pain = slipped capital femoral epiphysis ***alphabetical order by age see previous 3*** D L S 17. Hyperfunctioning athlete + pain on tibial tubercle = Osgood-schlatter -

Buzz word = Traction apophysitis?

18. Joint pain + many bleeding episodes + elevated PTT = hemophilia a/b 19. Prodrome URI + inspiratory stridor + seal bark cough = croup aka ”laryngotracheobronchitis” -

Tx: racemic epi, steroids Cause = parainfluenza virus 189

-

Example of Subglottic laryngitis

20. Toxic kid + drooling + dysphagia + tripoding = epiglottitis -

Cause = h. flu in unvaccinated Next step in management = intubate Xray = classic thumbprint sign Example of supra-glottic laryngitis

In general: Inspiratory stridor if upper airway problem In general: expiratory wheezing if lower airway problem 21. Neonate (first 28 days of life) + wheezing + vent = bronchopulmonary dysplasia 22. MCC pneumo in child 21 years old = pseudomonas c. DF508 mutation is the MC d. Male infertility (congenital absence of vas deferens), rectal prolapse, nasal polyps i. Tx nasal polyps with inhaled corticosteroids e. Tx of PNA = coverage of Pseudomonas with tobramycin i. Pancreatic enzyme replacement ii. Dornase alpha (breaks phosphodiesterase in mucous) iii. High calorie diet 2. Calcified mass in abdominal CT scan or in posterior mediastinum = neuroblastoma a. Crosses the midline vs Wilms tumor which is not calcified and does not cross midline b. Opsoclonus myoclonus syndrome 3. Clustered seizures in child, contraction of neck and arms, EEG shows hypsarrhythmia, hypopigmented macules on skin 190

a. Tuberous sclerosis, the seizures are West syndrome (infantile spasms) b. West syndrome tx = ACTH, also vigabatrin (GABAergic agent) c. Intellectual disability, nodules in ventricular system (subependymal nodule), subependymal giant cell astrocytoma, cardiac rhabdomyoma, renal angiomyolipoma 4. Generalized tonic clonic seizures in morning (teenager) = Juvenile myoclonic epilepsy a. Autosomal dominant, good prognosis 5. Kid w/ many kinds of seizures, bad cognitive dysfunction, EEG shows generalized spikes and slow waves = Lennox Gastaut syndrome 6. Kid not doing well in school, stares into space, no postictal state = Absence seizures a. EEG = 3 hertz spike and slow waves b. Tx = ethosuximide, T type Ca channel blocker 7. Diet to decrease recurrence of seizures = ketogenic diet 8. High fever, seizure for 15 min, recurrence within 24 hour span, focal seizure, no fever i. Get imaging or EEG b. Typical = generalized tonic clonic (NOT focal) i. No imaging, no EEG 10. Cafe au lait spots (hyperpigmented), chr 17 mutation = NF1 a. Gliomas, lisch nodules on iris b. Pheochromocytoma c. Autosomal dominant inheritance 11. Triad: vestibular schwannomas, chr 22 defect, AD inheritance = NF2 12. Alzheimer in 30s, increased risk of ALL, epicanthal folds = down syndrome (tri 21) a. Meconium ileus from hirschsprung disease b. Maternal nondisjunction potential cause of DS c. Atlantoaxial instability i. Before starting sports, lateral neck x-ray 1. Same thing for pts with ankylosing spondylitis and rheumatoid arthritis prior to surgery d. Increased risk of ALL 13. 6 month old kid, losing motor milestones, dies before age of 2, autosomal recessive disease = spinal muscular atrophy (Werdnig-Hoffman) a. Chr 5 (SMN1 gene) b. Hypotonic infant with fasciculations i. Fasciculations in adult = ALS 14. Defect in chr 15, happy puppet syndrome in girls = angelman syndrome in girls and prader willi in boys 15. Kid with large ears, large testicl*s, low IQ = fragile X a. CGG repeats b. X-linked dominant 16. UMN disease in a child secondary to initial insult to brain = cerebral palsy 17. Triad: MAHA + low platelet + renal failure = HUS a. Shigella, E.coli b. No steroids or antibiotics in tx 18. Kid with HTN with preceding upper respiratory infection = postinfectious glomerulonephritis 191

a. Proteinuria = 2+ or less b. UA shows dysmorphic RBCs c. Low complement levels on labs, ASO/Anti DNAse B titers are high 19. Low C3 + nephritic syndrome = C3 nephritic factor seen in type MPGN 20. IgA nephropathy and Henoch Scholein Purpura a. Avoid rotavirus vaccine d/t intussusception 21. Membranous nephropathy associated the most with renal vein thrombosis 22. Minimal change disease associated with hematologic malignancies (leukemias, lymphomas) a. Light microscopy normal b. Electron microscopy would show effacement of foot processes 23. Kid with BP > 95th percentile = HTN in kid 24. Kid with secondary HTN, different BP in arms vs legs, delayed femoral pulse = coarctation of the aorta a. Associated with Turner (also bicuspid aortic valve → aortic dissection, early onset aortic stenosis) 25. Kid has secondary hypertension with flank mass that crosses the midline, increased catecholamine levels = neuroblastoma 26. Kid with secondary hypertension, episodic hypertension with headaches, increased metanephrines in urine = pheochromocytomas a. Flank mass that is not calcified on imaging, and does not cross midline b. Before surgery give alpha blocker (phentolamine, phenoxybenzamine) 27. Kid with hypertension, mom was bathing baby and felt a hard flank mass, kid has no iris = Wilms tumor a. WAGR b. Aniridia, gu anomalies, mental retardation c. Tx dactinomycin 28. Denys-Drash = genital anomalies, Wilms tumor 29. RUQ mass in a kid, large tongue, one side of body is larger = Beckwith Wiedeman syndrome a. Hepatoblastoma (RUQ mass) b. Can present as a newborn w/ seizures d/t increased insulin producing cells → hypoglycemia → seizures 30. Babies of diabetic moms are hyperinsulinemic in utero → after delivery, still hyperinsulinemia → hypoglycemia 31. Hypocalcemic seizures in infants of diabetic mothers a. 2 causes of seizures in infants of diabetic moms: hypoglycemia or hypocalcemia b. Hypocalcemia also seen in DiGeorge syndrome pts 32. Kid has urine problems, visual problems, hearing problems = Alport syndrome a. Collagen IV mutation b. Inherited in AD fashion (COL4A5 mutation). However, X-linked is MC CROSS CHECKED? Yes

---------------------------------------------------------------------------------------------------------------------------

Episode 97: “Most Important ” MI = most important MIPF = most important prognostic factor #1 RF = most important risk factor 1. MIPF in retinal detachment → time to surgical intervention to fix 2. MIPF of Breast Ca. → 1. number of axillary LN “spread”. 2. Size of tumor 192

3. MI predictor of survival in aortic coarctation → Age of defect repair 4. #1 RF that predisposes to rupture of AAA → diameter of aneurysm 5. #1 RF for development of SCC of skin → cumulative exposure to sun 6. MIPF of melanoma → depth of invasion. “Breslow depth” 7. MIPF in mesenteric ischemia → bowel infarction 8. MI measure in prevention of HCC → Hep B vaccine 9. #1 RF of development of ovarian Ca → FHx of Ovarian Ca. 10. MIPF of cervical cancer → 1. stage at which it’s diagnosed. 2. Involvement of pelvic/paraaortic LN 11. #1 RF of prostate ca → Age 12. #1 RF of bladder ca → smoking 13. MIPF in determining outcome of treatment in colon ca → Stage at diagnosis 14. MI test needed in dx of IE → Blood culture 15. MI complication of peritoneal dialysis → peritonitis 16. MI goal in mgmt. of COPD → smoking cessation 17. #1 RF for OSA development → obesity 18. MI disease associated with central sleep apnea → heart failure 19. MI non-pharmacologic intervention in mgmt. of osteoarthritis → exercise 20. MI modifiable RF in development of osteoarthritis → obesity 21. 2 #1 RF for development of C. diff colitis → hospitalization and Abx 22. MI opportunistic pathogen in transplant pt → CMV 23. #1 RF for development of cholangiocarcinoma → primary sclerosing cholangitis 24. 2 #1 RF in development of HCC → Cirrhosis + Hep B infx 25. #1 RF in dev of lung disease related to asbestos → cumulative exposure to asbestos fibers. 26. MI therapeutic step in mgmt. of acute pancreatitis → IV fluid resuscitation 27. MI physical exam finding in dx of AS → pulsus parvus et tardus (delayed upstroke in carotid pulse) 28. MIPF in pt with CAD → Left ventricular function 29. Most important factor in preventing lung injury when using ventilator → mechanical ventilation with lung protective strategies. I.E low TV. 30. MI oncologic association in long term celiac disease → small bowel lymphoma (enteric associated T-cell lymphoma) 31. MC disease of the retina in elderly → macular degeneration 32. #1 RF in development of pressure ulcers → immobility 33. MI physical exam finding in dx of RAS → presence of abdominal bruit 34. MIPF in preserving neurologic function in spinal cord compression → early diagnosis 35. MI factor in determining TB treatment success → adhering to treatment regimen. Association of infectious agents with neoplasms 1. Cervical/anal/head and neck/vocal cord cancers → HPV 2. Kaposi Sarcoma → HHV-8 3. Bacillary angiomatosis → Bartonella henselae 193

4. Hodgkin’s Lymphoma → EBV 5. Burkitt’s Lymphoma → EBV 6. Lymphomas after transplant → EBV 7. MALToma → H pylori 8. HCC → Hep B + C CROSS CHECKED? YES -------------------------------------------------------------------------------------------------------------------------------

Episode 100: The Clutch Micro 1. Bloody dysentery and liver abscess→ Entamoeba histolytica 2. Foul-smelling diarrhea and recently drank contaminated water; presents in Pts with CVID or IgA deficiency. → Giardia Lamblia → do stool ova and parasites 3. Diarrhea in AIDS patient with Acid-fast oocysts in stool→ Cryptosporidium Parvo 4. vagin*l infection with green discharge and pH >4.5 → Trichom*onas vagin*lis give PO Metro both partners 5. Recently came back from a developing country with thrombocytopenia, anemia, fevers every 48hrs→ P. Vivax and Ovale what if every 72 hours→ P. Malariae 6. Which Plasmodium with dominant stage in liver→ P. Vivax and Ovale 7. Patient plays with cats at home with ring-enhancing brain lesions→ T. Gondii 8. Indirect Hyperbilirubinemia with a Hgb of 8 and recently treated for malaria what dz you thinking→ G6PD deficiency 9. Diarrhea after consuming pork→ T. Solium 10. Cysts in brain from pork→ neurocysticercosis; keep back in mind consuming eggs associated with this 11. Child that itches a lot especially his buttocks→ Enterobius Vermicularis remember scotch tape test 12. Microcytic anemia and colonoscopy shows worms attaching to intestines→ Necator Americanus or Ancylostama duodenale 13. Recently came back from Egypt and recent swimming expedition→ Schistosoma Haematobium 14. Eggs with terminal spine and came back from egypt→ Schistosoma haematobium 15. Liver abscess and consumes snails→ Schistosoma Mansoni or Japanicum 16. Pt immunocompromised and diffuse markings in CXR→ PCP and silver stain +; If there A-a gradient > 35 or PaO2 60 → S. Pneumo 40. Meningitis in unimmunized infant→ S. Pneumo as well 41. Meningitis in neonate→ GBS 42. Neonate with encephalitis and mom works in pet stores→ Toxoplasma 43. Bloody diarrhea recently consumed chicken→ C. Jejuni; don't forget association with Reiters 44. Bug that has hyaluronic capsule→ Strep. Pyogenes (more for Step 1 he said) 45. Ataxic, loss sensation, sexually active female→ Tabes Dorsalis tertiary syph. 46. Pt with encephalitis weeks after undercooked pork→ T. Solium 47. HIV pt CT scan shows ring-enhancing lesions→ T. Gondii ppx with TMP-SMX; seen in CD4 count V. parahaemolyticus Diarrhea after oysters and elevated AST and ALT especially in those with Liver dz→ V. Vulnificus Water diarrhea after swimming in fresh water or aquarium→ Aeromonas; A aquarium A Aeromonas Diarrhea with massive amounts of e- abnormalities→ V. Cholera Korean BBQ with watery diarrhea espec. Fried rice→ B. Cereus Diarrhea after two hours having potato salad→ S. Aureus also vomiting C. Perfringens also may cause diarrhea Swims in freshwater then dies after→ Naeglerii flower Fever and new-onset murmur in IV drug abuser -->S. aureus Fever and recent dental procedure→ S. Viridans Fever and heart murmur in prosthetic valve→ S. epidermidis!!!!!!!! Fever and heart murmur after sore throat→ S. Pyogenes If It comes from UTI → enterococcus S. Pyogenes IE attacks left side of heart vs S. aureus on right side of the heart You cannot prevent PSGN w/ penicillin Sterile vegetations on each side of the valve→ Libman-sack Endocarditis assoc. with SLE

CROSS CHECKED? No -------------------------------------------------------------------------------------------------------------------------------

Episode 102: The “Clutch” cancer podcast (IN PROGRESS) 1. Kid presenting with ataxia and necrotic mass in middle cerebellum(around vermis) on imagining medulloblastoma 1. If you saw an answer w juvenile pilocytic astrocytoma choose this first because they have a similar presentation 2. pilocytic is the most common brain tumor in kids 1. Arises in posterior fossa in the cerebellum 2. Contain eosinophilic fibers= rosenthal fibers 3. Person with brain mass attached by a dural tail (aka parasagittal mass )or they say mass is stuck to meninges meningioma 1. Histo finding= psammoma body 4. Bilateral hearing loss + hx of genetic disease nf2 (chromosome 22) 1. Ptn: bilat acoustic neuromas or schwannomas at the cerebello pontine angles 5. Versus in nf1 chromosome 17 1. Café au lait spots + skin neurofibromas + optic nerve gliomas 2. Also acoustic neuroma , but acoustic neuroma is more highly assoc with nf2 6. Farmer + had multiple rough lesions on forehead + sun exposure. Multiple lesions all over face, forearms actinic keratosis 1. #1RF Sun exposure 198

7.

8. 9. 10.

11.

12. 13. 14.

15.

16.

2. usually self resolves 3. High risk of progression to squamous cell carcinoma Skin findings for melanoma? ABCDE 1. Asymmetry, borders, color variation, diameter >6mm, evolving character and appearance 2. Melanoma prognostic factor is breslow depth 3. Can arise under nails = acral lentiginous. Mc in african americans Lesion on face, pearly gray papules, telangiectasia above the lip basal cell cancer of the skin squamous cell cancer of skin 1. Marjolin ulcers (non healing) Bone pain, fevers, + xray with sunburst osteosarcoma 1. Also see: codman triangle 2. RFs: Retinoblastoma, Pagets, Teriparatide (PTH analog) 2 yo + htn, flank mass. Ct abd mass that crosses midline with calcifications neuroblastoma 1. Posterior mediastinal mass (neurogenic origin) 2. histology = homer wright pseudorosettes 3. N-myc oncogene 4. Opisthoclonus myoclonus association 1. “weird hand movements” and “weird eye movements” Child + htn, but no calcified and no crossing midline Wilms tumor 1. Wagr—wilms, aniridia, gu anomalies and retardation Child + choreiform arm movements PMH sore throat Rheumatic fever. Called PANDAS 1. Pediatric autoimmune neurocyte disorder assoc with strep (group A) Episodic HA + HTN, encephalopathy that is periodic with HIGH BP pheochromocytoma 1. Chromaffin cells of adrenal medulla 2. Nicotinic Ach receptor 3. Pheo check urinary and serum metanephrine 4. Before surgery must block alpha before blocking beta to prevent unopposed alpha 1 stimulation 1. Similar to cocaine OD (give benzo, or phenoxybenzamine or phentolamine or a-b blocker) Pheo + Genetic association= MEN2 1. 2 A and 2B 1. 2a==medullary thyroid cancer pheochromocytoma, and primary hyperparathyroidism 1. Hyper parathyroid due to parathyroid hyperplasia 2. 2b—medullary thyroid cancer, pheochromocytoma, and mucosal neuromas + marfanoid habitus 2. Pheo also associated with NF1 tumor marker for Medullary thyroid cancer = calcitonin 1. Stain Congo red 199

2. If you see amyloid around the thyroid gland—think medullary thyroid cancer. The calcitonin deposits as amyloid 3. It arises from c cells surrounding the thyroid 17. Hx of men 2 b or 2 a resect the thyroid. (progresses to medullary thyroid cancer) 18. Neck mass, bx done papillary thyroid cancer 1. Classic rf Neck / Chest radiation 2. Histo psammoma bodies 3. Metastasis via lymphatics 19. Follicular thyroid cancer Mets through blood 20. Many of the thyroid cancers present as cold nodules 1. Check TSH first (TSH will be high for cold nodule) 2. Then do an us of thyroid with biopsy 21. Kid + seizures + eeg is chaotic disorganized pattern tuberous sclerosis 1. AD, subependymal hamartomas 2. Seizure = west syndrome = classic infantile spasms. Give acth 1. Or pick vigabatrin (gaba analog) 3. Assoc w: 1. Cardiac rhabdomyoma, renal angiomyolipomas (fat + blood vessels + muscle) 2. Sega brain tumor = subependymal giant cell astrocytoma 4. Chaotic EEG = hypsarrhythmia 22. Post menopausal female + visual problem/tunnel vision +galactorrhea; or young female with infertility prolactinoma 1. Tx: bromocriptine or cabergoline (dopamine aka prolactin inhibiting factor) 2. Do not go straight for transsphenoidal resection most prolactinomas resolve with dopamine 3. bitemporal heteronymous hemianopsia 23. Pituitary adenoma assoc with men1 24. MC pituitary adenoma = prolactinoma 25. Child + visual field problems + calcified mass on imagine craniopharyngioma 1. Rathke's pouch (motor oil fluid histo) 26. Lady + weight loss + high fever. One breast is large +tender/edematous inflammatory carcinoma 1. Infiltrating ductal carcinoma invaded the dermal lymphatics 27. mastitis is unilateral and assoc with recent delivery of baby 1. MCC= Staph aureus nafcillin or dicloxacillin 28. Eczematoid change of nipple = Pagets of breast 1. Assoc w/ infiltrating ductal carcinoma 29. 25 yo F + mobile firm lump in breast Fibroadenoma 1. Can do ultrasound of breast for imaging because they are less than 30 2. It grows with menstrual cycle, is benign 3. Do a physical again in a few weeks and see if it has resolved by then 30. Lady at 10 weeks gestation; size >> date. US= hypo echoic structures molar pregnancy 200

1. 2.

Snow storm appearance; grape like structure Either complete mole or incomplete mole 1. Incomplete =Triploid, Contains fetal parts and less likely to progress to choriocarcinoma 2. Complete mole = diploid; very likely to become choriocarcinoma 3. Tumor marker = b-hcg 31. Lady w recent hx of molar pregnancy and now has sob. Next step in dx? Chest imaging (xray) 1. Mc location of chorio met is the lung 2. Management doc—methotrexate inhibits dihydrofolate reductase but is also a dmard 32. Mother + used a drug in the 80s + now has vagin*l cancer DES exposure = Clear cell adenocarcinoma 33. Female + abd fullness and early satiety. Imaging ascites and “omental caking” Ovarian Cancer 1. Ca-125 tumor marker 34. How Ovarian Cancers can present: 1. Ovarian cancer + sob + blunting of costophrenic angles on the right - Meigs syndrome 2. Ovarian cancer + myoclonus, new onset Afib, lid lag, struma ovarii 1. contains functioning thyroid tissue 3. Teratomas—contain multiple tissue types 1. Especially in younger people. Show up in anterior mediastinum 4. Psammoma bodies on histo serous cystadenocarcinoma 5. Ovarian mass + histo = tumor cells surrounding blood vessel Endodermal sinus tumor = yolk sac tumor 1. afp is the tumor marker 2. Schiller duval bodies 35. Painless mass in testicl* ML seminoma 1. shows up in anterior mediastinum 36. Early satiety + severe epigastric pain, stomach is not moving + ovarian mass histo signet ring cells krukenberg tumor 1. (gastric carcinoma that has metastasized to ovaries) 37. Pt working in textiles, or prior tx w/ cyclophosphamide. Now has hematuria +weight loss? Next best step? Cystoscopy with biopsy 1. Bladder cancer 2. Textiles—exposure to dye (aniline dye) 3. Cyclophosphamide can cause hemorrhagic cystitis 1. Break down product =acrolein 2. Prevent this with mesna, which binds the it 3. Irrigate the bladder to decrease the amount of contact time with the bladder 4. Another rf is schistosoma haematobium 1. Dx: cystoscopy with biopsy 5. #1RF for bladder cancer smoking 38. Hemorrhagic cystitis is also caused by adenovirus 201

39. 40.

41. 42.

43. 44.

1. Adeno causes pharyngoconjunctivitis Cisplatin nephrotoxic 1. Prevent this with amifostine High Hgb + flank mass + smoker + right sided varicocele RCC (renal cell cancer) 1. EPO production is its paraneoplastic syndrome 2. Mets to bone lytic lesions (like lung and thyroid cancer) 3. R sided varicocele Blastic lesions in bone is caused by prostate and breast 50-60s, jaundice bad weight loss, epigastric pain pancreatic cancer of head 1. Dx with ct of abdomen 2. invades sma 3. To relieve sx (jaundice +itching ) ERCP to place a biliary stent 4. Marker= ca-199 LFT anomalies in pancreatic cancer high alk phos, and direct hyperbilirubinemia Colon cancer mets to Liver

CROSS CHECKED? No Still in progress

-------------------------------------------------------------------------------------------------------------------------------

Episode 102: The “Clutch” cancer podcast (Version 2) ●

● ●

● ●

Kid with ataxia + necrotic mass in cerebellum on imaging → pilocytic astrocytoma or medulloblastoma ○ MC brain tumor in kiddos? pilocytic astrocytoma ○ Location? Middle cerebellum/posterior fossa ○ pilocytic astrocytoma ■ Histology? Rosenthal fibers (eosinophilic fibers) ○ Medulloblastoma ■ Histology? Small blue cells ■ Drop mets to spine Brain mass with dural tail OR parasagittal mass → meningioma ○ Histology? Psammoma bodies Bilateral hearing loss + hx genetic disease → NF-2 w/ bilateral acoustic neuromas ○ Location? Cerebellopontine angle ○ Chromosome? 22 Cafe-au-lait spots + neurofibromas + optic gliomas → NF-1 Farmer w/ multiple rough lesions on forehead → actinic keratosis ○ RF? Sun exposure ○ Most likely outcome? resolution ○ Precursor to what? Squamous cell carcinoma Characteristics of melanoma 202

● ●

● ● ●

● ●

○ A = asymmetric ○ B = irregular borders ○ C = multiple colors ○ D = diameter > 6 mm ○ E = evolution #1 prognostic factor = Breslow depth Melanoma in usual location? acral lentiginous melanoma ○ Palms/soles or under nails (subungual) ○ Demographic? African American “Pearly gray papule” with telangiectasias, above the lip → basal cell skin cancer Non-healing ulcer, associated with burn wounds → Marjolin ulcer (squamous cell carcinoma) Kid with bone pain + fevers + XR with sunburst pattern → osteosarcoma ○ RF? ■ Retinoblastoma ■ Paget’s disease ■ Teriparatide (PTH analog given in pulsatile manner) ○ Imaging? Codman’s triangle & sunburst pattern 2 yo with HTN + flank mass + mass crossing midline w/ calcifications → neuroblastoma ○ Location? Abdomen or posterior mediastinum ○ Histology? Homer-Wright pseudorosettes ○ Oncogene? N-myc ○ Opsoclonus-myoclonus syndrome = weird have movements & weird eye movements ○ Contrast to Wilms tumor: flank mass that does NOT cross midline, NOT calcified ■ Association? WAGR (Wilms tumor, aniridia, GUI anomalies, mental retardation) Episodic HA + HTN → pheochromocytoma ○ Derived from? Chromaffin cells of adrenal medulla ■ Have nicotinic ACh receptors (adrenal medulla is modified postganglionic sympathetic neuron) ■ Dx? Urine metanephrines ■ Premedication prior to surgery? Alpha block → beta block → surg ● Same concept as don’t give beta blocker in cocaine overdose ○ Association? ■ MEN2A: primary hyperparathyroidism (parathyroid hyperplasia) + medullary thyroid cancer + pheo ■ MEN2B: medullary thyroid cancer + pheo + mucosal neuromas + Marfanoid body habitus ■ Von Hippel-Lindau syndrome (VHL) ■ NF-1 MC cause of hyperparathyroidism overall ? Parathyroid adenoma Marfanoid body habitus differential? ○ Marfan’s - aortic dissection, lens dislocation upward, MVP, pectus excavatum ■ Gene? Fibrillin ■ Chromosome? 15 ■ Inheritance? AD ○ Hyperhom*ocysteinemia ■ Enzymes? MTHF reductase or CBS deficiency ○ MEN2B 203

THYROID CANCER ● Medullary thyroid cancer ○ Tumor marker? Calcitonin ○ Stain? Congo red → apple green birefringence due to amyloid (deposits of calcitonin) ○ Derived from? C-cells ○ Association? MEN2A or MEN2B ■ Prevention? Prophylactic thyroidectomy ● Papillary thyroid cancer ○ #1 RF? head/neck/chest radiation ■ E.g. radiation for Hodgkin’s lymphoma as teenager ○ Histology? Psammoma bodies ○ Spread? Lymphatic system ● Follicular thyroid cancer ○ Spread? Hematogenous ● Nodule in thyroid with normal/high TSH → hypofunctioning → likely malignant ○ NBS? US + biopsy ● Nodule in thyroid with low TSH → hyperfunctioning → likely non-malignant ○ NBS? RAIU scan ○ RAIU w/ single focus → toxic adenoma ○ RAIU w/ multiple areas of uptake → toxic multinodular goiter ○ RAIU w/ diffuse uptake → Grave’s ● Kid tubular adenoma ○ Genetic syndromes ■ FAP ● Inheritance? AD ● Mutation? APC ● Prevention? Colectomy by 20 ● If brain mass → Turcot syndrome ● If soft tissue tumors → Gardner syndrome ■ HNPPC = Lynch Syndrome ○ Adenoma-carcinoma sequence ■ APC → Kras → p53

Systolic murmur at LLSB + diarrhea + episodes of stridor + RLQ abdominal mass + rash → carcinoid tumor ○ Location? Often appendix ■ If mediastinum, will be posterior mediastinum ○ Dx? Urine 5-HIAA ○ Pathophys of rash? Carcinoid tumors produce serotonin, so this diverts tryptophan from niacin pathways, causing pellagra ■ 4 D’s of pellagra: dementia, diarrhea, dermatitis, death ○ Why only R-sided lung lesions? Lungs breakdown carcinoid products ■ “TIPS” → tricuspid insufficiency & pulmonic stenosis ○ Why no sxs until liver mets? Liver breaks down carcinoid products

Pt with longstanding GERD + mass in distal esophagus → esophageal adenocarcinoma ○ #1 RF = Barrett’s esophagus (squamous cells → intestinal epithelium = non-ciliated columnar epithelium with goblet cells) ○ Contrast with RF for squamous cell carcinoma ■ Zenker’s diverticulum ■ Achalasia ■ Ingestion Pt with dysphagia + weight loss? EGD w/ biopsy

207

52 yo with dysphagia, no other alarm sxs → EGD ○ 50+ is an indication for EGD ○ Progressive dysphagia suggests growing mass

Longtime smoker with proximal muscle weakness, improves with repetition → LEMS 2/2 small cell lung cancer ○ Pathophys? Ab against presynaptic voltage-gated Ca++ channels, improved with use because there is more recruitment of Ca++ ○ EMG? Incremental response to repetitive nerve stimulation ○ Contrast with MG, where weakness worsens with use ○ Small cell paraneoplastic syndrome ■ LEMS ■ ACTH ● Does NOT suppress with high-dose dex ■ SIADH → euvolemic hyponatremia ● Tx? Fluid restrict ○ Squamous cell paraneoplastic ■ PTHrP → hypercalcemia ● Tx? Bisphosphonate for hypercalcemia of malginancy ● If acutely ill, NBS? Hydrate! ○ Squamous & small cell are CENTRAL ○ Adenocarcinoma is PERIPHERAL ■ Female or non-smoker → consider adenocarcinoma ○ Lung cancer pt + facial fullness → SVC syndrome ■ Pathophys? Not draining H&N veins ○ Unilateral eye drop + constricted pupil → Horner’s 2/2 Pancoast tumor ■ Partial Horner’s associated with? Cluster HA ● Tx? 100% O2 ○ Joint pain + finger clubbing in smoker → hypertrophic osteoarthropathy (associated with lung cancer) ■ NBS? CXR ■ Electrolyte association? hypoglycemia

Tumor associated with MG? Thymoma ○ Location? Anterior mediastinum

Multiple new SKs → sign of Leser-Trelat ○ Associated malignancy? pancreatic/gastric cancer ○ NBS? Abdominal CT ○ If succussion splash on exam? Gastric cancer ■ Pathophys? Gastric outlet obstruction

Velvety lesions in axillary → acanthosis nigricans ○ MC cause? Insulin resistance (obesity & DM) ○ Associated malignancy? Gastric cancer

Mesothelioma ○ RF? ■ Asbestos exposure → shipyard worker 208

○ ○ ○ ○

Smoking does NOT increase risk of mesothelioma Imaging? Pleural thickening on CXR Histology? Psammoma bodies MC asbestos-related malignancy? Bronchogenic carcinoma, NOT mesothelioma

IVDU/prostitution + purple lesions on skin → Kaposi’s sarcoma ○ Associated virus? HHV-8

Immigrant or African kid with jaw mass → Burkitt’s lymphoma ○ Translocation? t(8,14) ○ Gene? C-myc ○ Histology? Starry sky

Pt with hematologic malignancy, now hypotensive & bleeding from tumor sites → APML ○ Pathophys? Auer rods trigger DIC ○ Translocation? t(15,17) ○ Histology? Auer rods ○ Tx? ATRA

Kid < 8 yo with weight loss & fevers → ALL ○ Association? Down syndrome

Old person with pneumonia + WBC 80k → CLL

Middle age person with fevers + weight loss + early satiety → CML ○ Translocation? t(9,22) ○ Protein? BCR-ABL ○ Tx? Imatinib (tyrosine kinase inhibitor)

Hodgkin’s lymphoma ○ Epi? Bimodal distribution ■ teens-20s ■ 50s-60s ○ Histology? Reed-Sternberg cell ○ Renal association? Minimal change disease ○ MC type? Nodular sclerosing ○ Type with best prognosis? Lymphocyte predominant ○ Future cancer? Papillary thyroid cancer due to chest radiation

Nephrotic-malignancy associations ○ Hematologic malignancy → minimal change disease ○ Solid malignancy → membranous nephropathy ■ Ab? Phospholipase A2 receptor

Cervical cancer ○ RF? HPV ○ CIN 1 → CIN 2 → CIN 3 209

MC cause of death? Renal failure 2/2 tumor invasion of ureters ■ Imaging? Hydronephrosis

Genetic diseases associated with malignancy ○ Peutz Jeghers → hamartoma + hyperpigmented macules on lips + pancreatic ca ○ XP → skin cancer ■ Pathophys? Mutation in DNA repair genes ○ VHL → hemangioblastoma + bilateral RCC + pancreatic cysts/cancer + pheo ■ High Hct 2/2 hemangioblastoma producing Epo ○ Tuberous sclerosis → rhabdomyoma

Brain tumor that crosses corpus callosum → glioblastoma multiforme ○ Tumor marker? GFAP ○ Imaging? Vasogenic edema with central necrosis

Autoimmune-associated malignancy ○ Sjogren’s → salivary gland lymphoma ○ Celiac → lymphoma of GI tract (EATL) ○ Hashimoto’s → thyroid lymphoma

Post-menopausal pt with vagin*l bleeding → endometrial cancer ○ endometrial biopsy ○ MC cause? Atrophic vaginitis ○ SERM that increases risk? Tamoxifen (agonist in uterus) ○ SERM that doesn’t increase risk? Raloxifene Murmur that changes with weight change + fevers → rhabdomyoma ○ Location? Left atrium

ID-associated malignancy ● H. pylori → MALT lymphoma ○ Stain? Silver strain ○ Tx? ■ Triple therapy = clarithromycin + amox + PPI ■ Quadruple therapy = bismuth + metro + tetracycline + PPI ● Clonorchis sinensis → biliary tract ● Schistosoma haematobium → bladder cancer ● EBV → nasopharyngeal carcinoma & Burkitt’s lymphoma ● HPV → cancers of lower reproductive tract ○ Strains? 16, 18, 30s

----------------------------------------------------------------------------------------------------------------------------

Episode 104: ACLS, Arrhythmias, and HY Cardiac Pharm Episode 104 notes were graciously provided by Divine Intervention from an anonymous contributor. Code blue! 210

1. Who is running the code? 2. Identify yourself if running the code 3. Assign tasks to people a. Electricity/defib/cardiovert b. Recorder c. Timer d. Pharmacy e. Pill-pusher Patient codes in front of you! 1. Tap the patient: do they respond? 2. Feel for a pulse i.e. carotids 3. Start CPR (30 compressions 120/min:2 breaths) 4. Call for help 5. Try to identify arrythmia on monitor a. Vfib or pulseless Vtach i. Defibrillate = unsynchronized cardioversion 200 joules q2m 1. Pulse and rhythm check prior to defibrillation ii. Epinephrine 1mg iii. 2m iv. Amiodarone 300mg IV v. 2m vi. Epinephrine 1mg vii. 2m viii. Amiodarone 150mg IV ix. 2m x. Epinephrine 1mg xi. 2m xii. Amiodarone 150mg IV xiii. 2m xiv. Lidocaine 1mg/kg xv. "every 2 minutes, do 3 things: pulse/rhythm check + shock + drug" b. No pulse = pulseless electrical activity or flat-line = asystole i. Epinephrine 1mg q4m 1. "all roads lead to epi" ii. Continue CPR 30:2 iii. DDx 1. 5Ts: tension pneumo, thromboses, toxins, trauma, tamponade 2. 5Hs: hypo/hyperkalemia, hypothermia, hypoglycemia, H+/acidosis, hypovolemia ● Wide-complex tachyarrythmia: assume in Vtach 1. Pulse? i. No: ACLS algorithm ^^^ ii. Yes: stable or unstable (hypotensive, altered, symptomatic)? 1. Stable: amiodarone 2. Unstable: synchronized/direct current/direct current countershock cardioversion 100 joules a. You don’t want to defibrillate them while they're repolarizing ● Narrow-complex tachyarrythmia: assume in SVT 1. Vagal maneuvers, cold water, massage carotids 2. Adenosine 6 --> 12 --> 12mg; ~trying to slow their heart down~ 3. IV beta blocker or CCB (non-dihydro: verapamil, diltiazem) 4. Synchronized cardioversion 211

Afib (irregularly irregular w/o p-waves) 1. Stable? i. Yes 1. Rate control: beta blocker, non-dihydro CCB 2. Rhythm control: amiodarone (class III anti-arrhythmic) ii. No 1. Synchronized cardioversion! Atrial flutter (sawtooth) 1. Stable? i. Yes 1. Rate control: beta blocker, non-dihydro CCB 2. Rhythm control: amiodarone (class III anti-arrythmic) ii. No 1. Synchronized cardioversion! Wolf-Parkinson-White (delta wave + short PR interval) ● Do not block AV node (then they'll keep running through bundle of kent!) ● Give Procainamide Multifocal atrial tachycardia (3+ p-wave morphologies) ● Treat underlying disorder ● BB, CCB, stop smoking Bradyarrythmia (HR in 30s) ● Atropine (muscarinic antagonist --> speed up conduction through AV node) ● Pacing Drugs ● Anti-arrhythmics ● Sodium channel blockers, prolong QRS ▪ Class IA ● Disopyramide ● Quinidine ● Procainamide ● Ae: drug-induced lupus ● ~phenytoin~ ▪ Class 1B ● Lidocaine ● Mexiletine ● Tocainide ▪ Class IC ● Flecainide ● Propafenone ● Class II beta blockers ▪ Metoprolol ▪ Labetalol ● Class III ▪ Amiodarone: K+ channel blocker ▪ Ae: hypotension, hypo/hyperthyroidism (Wolf-Chaikoff), blue discoloration of skin, pulmonary fibrosis (decreased DLCO) ● Class IV non-dihydro CCB; slow conduction through AV node; @phase4 ▪ Verapamil ▪ Diltiazem ● Digoxin = positive inotrope, musc agonist 212

▪ ▪ ▪ ●

--| Na/K ATPase pump --> increased intracellular Na --| Na/Ca exchanger --> increased Ca --> strength of contraction Does not improve survival! Ae: digoxin-toxicity (esp when hypokalemic), hyperkalemia (e.g. on diuretic) ● Tx: anti-dig Fab fragment

Beta blockers ● Do not give in cocaine overdose (instead give phentolamine, carvedilol/labetalol) ● Improve survival in heart failure : metoprolol, bisoprolol, carvedilol ● In addition to: ▪ Spironolactone --| aldosterone receptor ● Also for PCOS, liver disease ● Ae: gynecomastia ▪ BiDil = hydralazine + isosorbide ▪ ACEi ● c/I bilateral renal artery stenosis ● Esp useful for diabetics! ● --| ATII preferential constriction of efferent arteriole and intraglomerular filtration injury ● --| CCB side effects: ● peripheral edema (dilation of pre-capillary arterioles --> increased hydrostatic pressure) ● ACEi dilate post-capillary venules --| peripheral edema ● Reflex tachy Nitroprusside --| hypertensive emergency ● Ae: cyanide toxicity Hydralazine: vasodilator ● Ae: drug-induced lupus, tachycardia Cardiogenic shock? 1. Dobutamine = beta1 agonist --> HR, stroke volume 2. Milrinone = inodilator/PDE inhibitor --> increase cAMP (@cardiac muscle: contraction; @smooth muscle: blood vessels relaxation and decreased SVR) i. !!! So cool

● ● ●

-------------------------------------------------------------------------------------------------------------------------------

Episode 111: The “Clutch” Pharmacology Podcast Round 1 ●

Nitrates ○ Indication? angina ○ AE? ■ HA (due to vasodilation) Statins ○ Mechanism? Inhibit HMG CoA-reductase ○ AE? ■ Hepatotoxic ■ Myotoxic ○ Interactions? ■ Fibrates Niacin ○ Raises HDL the best 213

● ●

● ●

Cholestyramine ○ Mechanism? Prevent bile acid reabsorption Fibrates ○ Lower TGs ○ AE? ■ Hepatotoxic ■ Myotoxic Digoxin ○ Mechanism? Inhibits Na/K-ATPase so acts as positive inotrope + muscarinic agonist activity ○ Indications? ■ Afib ■ CHF - doesn’t improve survival ○ AE? ■ Hyperkalemia ○ What prediposes to digoxin toxicity? Hypokalemia ○ Sign of dig toxicity? PVCs, vision problems, abdominal pain Procainamide ○ Class Ia antiarrhythmic ○ Indication? WPW ○ AE? Drug-induced lupus Amiodarone ○ Class III antiarrhythmic ○ Mechanism? K+ channel blocker ○ AE? ■ Hypothyroidism/hyperthyroidism ■ Skin discoloration ■ Pulmonary fibrosis Beta blockers ○ A-M → selective ○ N-Z → nonselective ■ Contraindication? Asthma ○ Indications? ■ Performance anxiety ■ Afib Nondihydropyridine CCBs (e.g. verapamil, diltiazem) ○ Mechanism? Negative inotropes + slow conduction through AV node ○ Indication? ■ Afib ■ Prinzmetal angina CCB indicated for pt with recent SAH? Nimodipine ○ Prevents post-SAH vasospasm and resultant ischemic stroke DIhydropyridine CCBs ○ AE? Peripheral edema (dilation of precapillary arterioles → increase hydrostatic pressure in capillaries) ■ How to decrease this? Dilate post-cap venules with ACE-I Nitroprusside ○ Indication? Hypertensive emergency ○ AE? Cyanide poisoning Bisphosphonates 214

Indication? ■ Osteoporosis ■ Hypercalcemia of malignancy ○ AE? ■ Pill esophagitis (drink water, be upright for 30 mins after taking) ■ Osteonecrosis of jaw Teriparatide ○ Mechanism? PTH analog ■ Continuous PTH → suppressed bone production ■ Pulsatile PTH → increases bone formation ○ AE? ■ Increased risk of osteosarcoma Uses of Mg ○ Prevent seizures in pt with pre-eclampsia ○ Severe asthma attack Dopamine agonist (e.g. bromocriptine/cabergoline) ○ Mechanism? Dopamine inhibits the release of prolactin (via tuberoinfundibular pathway) ○ Indication? prolactinoma Typical antipsychotics? ○ High-potency: Haloperidol, fluphenazine, trifluoperazine ○ Low-potency: chlorpromazine, thioridazine ○ AE? ■ QT prolongation ■ Hyperprolactinemia ● Dopamine = “prolactin-inhibiting factor” ■ EPS ● Acute dystonia ○ Tx? Benztropine, diphenhydramine ● Akinesthesia ○ Tx? Beta blocker ● Parkinsonian ● Tardive dyskinesia Metoclopramide ○ Mechanism? Dopamine antagonist ○ Indication? Diabetic gastroparesis ○ AE? ■ EPS (e.g. drug-induced parkinsonism) Atypical antipsychotics ○ Risperidone → hyperprolactinemia ○ Ziprasidone → QT prolongation ○ Quetiapine → cataracts ■ Special indication? Psychosis in the s/o of treatment for Parkinson’s ○ Olanzapine → metabolic syndrome ■ Indication aside from schizophrenia? OCD ● O = olanzapine ● C = clomipramine ● D = antidepressants (SSRIs) 215

Clozapine → agranulocytosis, myocarditis ■ Decreases risk of suicide ○ Aripiprazole → partial dopamine receptor agonist Tricyclic antidepressants ○ AE? anti-HAM ■ Anti-histamine → sedation ■ Anti-alpha-1 → orthostatic hypotension ■ Antimuscarinic → urinary retention, constipation ○ Tx for toxicity? Sodium bicarb ○ Special indications? ■ OCD (clomipramine) ■ Nocturnal enuresis (imipramine) ● Other tx options? Desmopressin = ADH analog MAO-Is ○ Isocarboxazid, phenelzine, tranylcypromine ○ MAO-B inhibitors: rasagiline, selegiline ■ Indication? Parkinson’s ○ AE? Hypertensive crisis with tyramine consumption Serotonin syndrome ○ Presentation? High fevers + myoclonus ○ NBS? Supportive care + benzos ○ If not responding? cyproheptadine (anti-histamine with serotonin receptor-blocking activity) SSRIs ○ Indications? ■ 1st line for depression ■ 1st line for GAD ■ 1st line for PSTD ○ AE? ■ Sexual dysfunction ■ Weight gain SNRIs (venlafaxine, duloxetine) ○ Indications? Neuropathic pain ○ AE? HTN NDRIs (bupropion) ○ Benefits? No seuxal side effects, no weight gain, helps with smoking cessation ○ Contraindications? Anorexia & bulemia ■ Why? Lowers seizure threshold Lithium ○ Decreases risk of suicide ○ AE? ■ Hypothyroidism ■ Nephrogenic DI ■ Tremors - sign of toxicity -Vaptans (e.g. conivaptan, tolvaptan) ○ Mechanism? ADH receptor antagonists ○ Indications? SIADH Darbepoetin 216

○ Mechanism? EPO analog ○ Indications? Anemia associated with CKD Cinacalcet ○ Mechanism? CSR modulator → decreased PTH release ○ Indications? Loop diuretics ○ Site of action? Na-K-2Cl transporter in the thick ascending limb of the loop of Henle ○ AE? ■ Hypokalemia ■ Hypocalcemia & hypercalciuria ■ Hypomagnesemia ○ Contraindication? Hx nephrolithiasis Thiazide diuretics ○ Site of action? Na-Cl symporter in the DCT ○ Strongest association with ○ AE? hyper-GLUC ■ Hyperglycemia ■ Hyperlipidemia ■ Hyperuricemia ■ Hypercalcemia Potassium-sparing diuretics ○ ENaC blockers (amiloride, triamterene) ■ Special indication? Nephrogenic DI associated with lithium ○ Aldosterone antagonists ■ Spironolactone ● AE? Gynecomastia (also blocks androgen receptors) ● Indications? ○ Conn’s syndrome ○ Hirsutism of PCOS (inhibits 5-alpha reductase in the skin) ○ Improves survival in HF ■ Eplerenone ○ Type of RTA? 4 Drugs that improve survival in HF ○ ACE-I ○ Beta blockers: bisoprolol, carvedilol (alpha-beta blocker), ER metoprolol ○ Spironolactone ○ Isosorbide dinitrate-hydralazine in African Americans Acetazolamide ○ Mechanism? Carbonic anhydrase inhibitor ○ Electrolyte effects? Hypokalemia + metabolic acidosis ○ Indications? ■ Central sleep apnea (metabolic acidosis → compensatory resp alkalosis → hyperventilation → increases respiratory drive) ■ Idiopathic intracranial HTN ■ Glaucoma ■ Altitude sickness ○ Type of RTA? 2 Mannitol 217

○ ○

Mechanism? Non-reabsorbable sugar Indications? ■ Elevated ICP ■ Diuretic Ethosuxamide ○ Mechanism? T-type CCB ○ Indications? Absence seizures Carbamazepine ○ Indications? ■ Seizures (narrow spectrum) ■ Trigeminal neuralgia ○ AE? Agranulocytosis Phenytoin ○ AE? ■ NTD in fetus ■ Drug-induced lupus Valproic acid ○ AE? ■ Most teratogenic antiepileptic ■ Hepatotoxic Hyperthyroidism drugs ○ PTU ■ Better in pregnancy (1st trimester) ■ Can inhibit peripheral T4 → T3 conversion ■ Useful in the tx of thyroid storm ○ Methimazole ○ AE? Agranulocytosis Tx of thyroid storm ○ 1st: propranolol ○ 2nd: PTU ○ Then steroids, Lugol’s solution, etc. Abx that cover MRSA ○ Vancomycin ■ AE? ● Red man syndrome ○ Tx? Slow infusion, give antihistamine or NSAID before infusion ● Nephrotoxic ● Ototoxic ○ Daptomycin ■ Where can’t it be used? Lungs (inhibited by surfactant) AE? Myotoxicity ○ Linezolid ■ Mechanism? 50S inhibitor ■ AE? Serotonin syndrome ○ Rifampin ■ AE? Orange bodily fluids ○ Tigecycline 218

Abx that cover Pseudomonas ○ Ceftazidime (only 3rd gen that covers) ○ Cefepime (4th gen) ○ Aztreonam ■ Little cross-reactivity with other penicillins ○ Aminoglycosides ■ AE? ● Nephrotoxicity → ATN ● Ototoxicity ○ FQ ■ AE? ● QT prolongation ● Achilles’ tendon rupture ■ Contraindications? ● Pregnancy ● Children ○ Pip-tazo ○ Colistin/polymyxin E ■ Last-resort for serious GNR infections ■ VERY nephrotoxic

TMP-SMX ○ Mechanism: ■ TMP → inhibits dihydrofolate reductase ■ SMX → inhibits dihydropteroate synthetase ○ Tx for resultant bone marrow suppression? Leucovorin ○ Ppx for PCP with CD4 < 200 ○ Ppx for toxo with CD4 < 100 Pyrimethamine-sulfadiazine ○ Pyrimethamine → inhibits dihydrofolate reductase ○ -Sulfadiazine → inhibits dihydropteroate synthetase ○ Indications? Tx of toxo Macrolides ○ Indications? ■ Diabetic gastroparesis (macrolides are motilin receptor agonists) ■ Atypical pneumonia ● Mycoplasma ● Chlamydia ● Legionella ■ Triple therapy for H. pylori (azithromycin) ■ Ppx against MAC in AIDS pt with CD4 < 50 (azithromycin) ■ Pertussis tx & ppx for close contacts ○ AE? ■ Diarrhea ■ QT prolongation ■ Erythromycin → pyloric stenosis in NB Ceftriaxone (3rd gen cephalosporin)

219

Indications? ■ Meningitis ■ Ppx of close contacts of pts with meningitis ● Rifampin is the preferred agent ○ AE? ■ Intrahepatic cholestasis in neonates ● Use cefotaxime (“baby ceftriaxone”) Penicillins ○ Indications? ■ Syphilis ● Jarisch-Herxheimer reaction Aminopenicillins (e.g. PO amoxicillin, IV ampicillin) ○ Indications? ■ Ampicillin for Listeria infection ● #3 cause of serious infections in the neonate Acyclovir ○ Indications? ■ Herpes ■ ZVZ ○ AE? Crystalline nephropathy ○ If resistant give what? Foscarnet Ganciclovir ○ Indication? CMV ○ Mechanism of mutation? UL97 kinase mutation ○ If resistant give what? Foscarnet Alpha-1 receptor blockers ○ Phenoxybenzamine (irreversible) ○ Phentolamine (reversible) ○ Indications? ■ Pheo (alpha block before beta block) Alpha-beta blockers ○ Carvedilol ○ Labetalol Alpha-1 agonist ○ Phenylephrine ■ Indications? ● Allergic rhinitis ● Hypotension ■ AE? Nasal septum perforation Alpha-2 agonists (e.g. clonidine, guanfacine) ○ Indications? ■ Opioid withdrawal ■ Resistant HTN ■ Tourette’s ■ ADHD Beta blockers ○ Indications? ■ Improve survival in HF (bisoprolol, carvedilol, metoprolol) 220

● ● ●

○ ○ ○ PPIs ○ ○

■ Thyroid storm (propranolol) ■ Glaucoma (timolol) ■ Performance anxiety Contraindications? Cocaine intoxication (unopposed alpha → profound HTN) Tx for toxicity? Glucagon RAAS response? Decreased (beta-1 receptors on afferent arteriole)

Mechanism? Irreversible inhibit Na-H antiporter on parietal cells → decrease acid in lumen of stomach Indications? ■ GERD ■ Barrett’s esophagus ■ Triple/quad therapy in H. pylori ● Hold PPI before testing to avoid false neg ○ AE? ■ CYP inhibition ■ Osteoporosis ■ Aspiration pneumonia H2 receptor antagonists (e.g. cimetidine) ○ Indications? ■ GERD ■ PUD ○ AE? ■ CYP inhibition ■ Gynecomastia (cimetidine) GnRH analogs (e.g. leuprolide) ○ Indication? ■ Shrink fibroid prior to surgery ■ Fertility tx (pulsatile) ■ Metastatic testicular cancer (continuous) ■ Metastatic prostate cancer (continuous) ● Alternatives? Androgen-receptor blockers (e.g. flutamide, bicalutamide) Taxofimen ○ Mechanism? SERM ■ Antagonist in breast ■ Agonist in bone & uterus ○ AE? Increased risk of endometrial cancer Raloxifene ○ Mechanism? SERM ■ Antagonist in breast ■ Agonist in bone ■ Doesn’t act in uterus Pegvisomant ○ Mechanism? Growth hormone receptor antagonist ○ Indications? ■ Acromegaly Dexamethasone → glucocorticoid Hydrocortisone → glucocorticoid & mineralocorticoid Fludracortisone → mineralocorticoid 221

● ●

● ●

PDE inhibitors (e.g. sildenafil, tadalafil) ○ Indications? ■ Erectile dysfunction ■ Pulmonary arterial HTN ○ Contraindications? ■ Pts on other vasodilators Aspirin ○ Mechanism? Irreversible inhibitor of COX-1 and COX-2 ○ Indications? ■ Chest pain 1st drug ○ AE? ■ PUD ● Prevention? Misoprostol (prostaglandin) P2Y12 receptor blockers (e.g. clopidogrel, prasugrel, ticagrelor) ○ Indications? ■ Post-stroke ■ Post-MI Dipyridamole ○ Mechanism? Increases cAMP → relaxes SM & increased cardiac contractility ○ Indications? ■ Stress test (uses coronary steal principle) Cilostazol ○ Mechanism? Increases cAMP → relaxes SM ○ Indications? ■ Symptomatic relief of PAD GpIIbIIIa receptor antagonists (e.g. abciximab, tirofiban, eptifibatide) ○ Mechanism? Prevents platelet aggregation Heparin ○ Mechanism? Activates AT-III ○ Reversal agent? Protamine sulfate (for unfractionated heparin) ○ AE? HIT = heparin-induced thrombocytopenia ■ NBS? Give direct thrombin inhibitor (e.g. dabigatran, argatroban, bivalirudin) ● Reversal of dabigatran? Idarucizumab Factor X inhibitors (e.g. apixaban, rivaroxaban) Warfarin ○ Mechanism? Inhibitors VKOR → decreases gamma-carboxylation of factors 2,7, 9, 10, protein C & S ○ Bridge with heparin (avoid transient hypercoagulability) ○ Reversal agents? ■ Vit K ■ Four-factor PCC Nitrofurantoin ○ Indication? Cystitis in females ○ AE? Pulmonary fibrosis Drugs associated with pulmonary fibrosis ○ Busulfan ○ Bleomycin (G2 phase) ○ Amiodarone ○ Methotrexate 222

■ ■ ■ ■

● ● ●

Mechanism? Dihydrofolate reductase inhibitor Rescue agent? Leucovorin AE? Hepatotoxic Indications? ● Autoimmune dz (e.g. RA) ● Choriocarcinoma ● Ectopic pregnancy Cyclophosphamide ○ Mechanism? Alkylating agent ○ AE? ■ Hemorrhagic cystitis ● Prevention? mesna ■ Bladder cancer Cisplatin ○ AE? ■ Ototoxicity ■ Nephrotoxic (causes ATN) ● Prevention? Amifostine Vinc alkaloids (e.g. vincristine, vinblastine) ○ Mechanism? Prevent polymerization of microtubes ○ AE? Peripheral neuropathy Taxanes ○ Mechanism? Prevent depolymerization of microtubules TNF inhibitors (e.g. adalimumab, etanercept) ○ Check for latent TB before starting ATRA ○ Indication? APML ■ t(15,17) ■ DIC risk Imatinib ○ Indication? CML ■ t(9,22) ■ BCR-ABL fusion protein Eculizumab ○ Mechanism? Monoclonal Ab against C5 ○ Indication? PNH ○ Vaccine against? Neisseria meningitidis Adenosine ○ Indication? ■ Break SVT ■ Regadenoson for cardiac stress tests ○ What antagonizes its effects? ■ Caffeine ■ Theophylline Octreotide ○ Indication? ■ Variceal bleed ● Give spironolactone + propranolol to reduce recurrence risk 223

● ●

● ●

Insulin ○ Ultra rapid-acting: lispro, aspart, glulisine ○ Rapid-acting: regular ○ Intermediate-acting: NPH ○ Long-acting: detemir & glargine Metformin ○ Weight neutral ○ Stop metformin before getting contrast. If contrast causes renal injury, they can have a lifethreatening lactic acidosis Sulfonylureas ○ Mechanism? K+ channel blockers. Close ATP-dependent K+ channel which leads to insulin release ○ Weight gain (2/2 increased insulin release) ○ 2nd gen: glyburide, glipizide, glimepiride PPAR-gamma activation (e.g. pioglitazone, rosiglitazone) ○ Contraindication? CHF ○ Weight gain Alpha-glucosidase inhibitors (e.g. acarbose, miglitol) ○ AE? Diarrhea GLP-1 analogs (e.g. exenatide, liraglutide) ○ Contraindication? MTC ○ AE? Pancreatitis ○ Which can be prescribed as a weight loss drug? liraglutide DDP4-inhibitors (e.g. sitagliptin) ○ Contraindication? MTC SGLT-2 inhibitors (e.g. canagliflozin, dapagliflozin) ○ Contraindication? Renal failure ○ AE? UTIs

-------------------------------------------------------------------------------------------------------------------------------

Episode 112: The “Clutch” Pharmacology Podcast Round 2 ● ●

Alpha-1 agonists (e.g. phenylephrine, ephedrine) ○ Indication? Hypotension in the s/o anesthesia Alpha-1 antagonists ○ Phenoxybenzamine → irreversible ○ Phentolamine → reversible ○ Indication? pheo Other alpha-1 antagonists ○ Prazosin, doxazosin ■ Indication? Short-term tx of BPH ● Long-term BPH tx? 5-alpha-reductase inhibitors (e.g. finasteride) ■ AE? Orthostatic hypotension ○ Tamsulosin (selective alpha-1a blocker) ■ NO orthostatic hypotension Alpha-methyldopa 224

● ●

Indication? HTN in pregnancy ■ Other meds for HTN In pregnancy ● Hydralazine ○ AE? Reflex tachycardia & drug-induced lupus ● Labetalol ● Nifedipine ○ AE? Drug-induced lupus Thyroid storm ○ 1st: propranolol ○ 2nd: PTU ■ AE? Agranulocytosis Beta-2 agonists ○ SABA → albuterol ■ Indications: ● Asthma ● Hyperkalemia ○ LABA → salmeterol, formoterol ■ Never use LABA before ICS in asthma! ○ Terbutaline/ritodrine ■ Indication? Tocolytics Asthma ladder ○ SABA ○ low-dose ICS ○ LABA or medium-dose ICS or leukotriene inhibitor ○ PO steroids Epinephrine ○ Indication? ■ Anaphylaxis ■ ACLS Norepinephrine ○ Indication? Septic shock Inotropes ○ Indication? Cardiogenic shock ○ Dobutamine (beta-1 agonist) ○ Milrinone (PDE inhibitor) Muscarinic agonists ○ Bethanechol/carbachol ■ Indications? ● Overflow incontinence ● Ogilvie syndrome (acute colonic pseudo-obstruction) ○ Pilocarpine ■ Indication? ● Glaucoma ● Sjogren’s Acetylcholinesterase inhibitors ○ Edrophonium ■ Very short acting ■ Indication? Diagnostic testing for MG (“Tensilon test”) 225

Neostigmine ■ Indication? Ileus ○ Pyridostigmine ■ Indication? MG ○ Physostigmine ■ Indication? Atropine overdose ■ Mechanism? ACh levels rise and can outcompete atropine (muscarinic antagonist) ○ Donepezil, galantamine, rivastigmine ■ Indication? Alzheimer’s Organophosphates ○ Mechanism? irreversible AChE inhibitors → cholinergic toxidrome ○ Sweating, diarrhea, miosis, bradycardia ○ Tx? ■ Atropine ■ Pralidoxime (regenerates ACh) Methacholine ○ Mechanism? Muscarinic receptor agonist → triggers bronchospasm ○ Indication? Provocative testing for asthma Scopolamine ○ Mechanism? Anticholinergic ○ Indication? Motion sickness Tx of urge incontinence/“overactive bladder”? ○ Muscarinic antagonists ■ “On The Darn Toilet” ● Oxybutynin, Trospium, Darifenacin/solifenacin, Tolterodine Ipratropium/tiotropium ○ Mechanism? Muscarinic antagonist ○ Ipratropium → short-acting ○ Tiotropium → long-acting ○ Indication? COPD ■ SABA vs SAMA → choose SAMA Midodrine ○ Mechanism? Alpha-1 agonist → vasoconstrictor ○ Indication? Orthostatic hypotension Haloperidol (high-potency typical antipsychotic) ○ AE? ■ QT prolongation ■ Hyperprolactinemia ■ EPS ■ NMS Isoniazid ○ Indication? TB tx ○ AE? ■ B6 depletion ● Sideroblastic anemia (cofactor for ALAS) ● Seizures (cofactor for glutamate decarboxylase) ■ Drug-induced lupus 226

● ●

● ●

■ Hepatotoxic Levodopa/carbidopa ○ Levodopa = dopamine precursor ○ Carbidopa = dopa decarboxylase inhibitor, prevents peripheral breakdown of levodopa ○ Indication? Parkinson’s Entacapone/tolcapone ○ Mechanism? COMT inhibitors → prevent breakdown of dopamine ○ Indication? Parkinson’s Selegiline/rasagiline ○ Mechanism? MOA-B inhibitors ○ Indication? Parkinson’s ○ AE? Tyramine hypertensive crisis Amantadine ○ Indication? Parkinson’s Pramipexole/ropinirole ○ Mechanism? Dopamine receptor agonists ○ Indication? ■ Restless leg syndrome ● Associated with IDA ● Other option? Primidone (barbiturate) ■ Parkinson’s Bromocriptine/cabergoline ○ Mechanism? Dopamine agonists ○ Indication? Prolactinoma ■ Choose medical tx before transphenoidal resection Halothane ○ Inhaled anesthetic ○ AE? Malignant hyperthermia ■ Tx? Dantrolene (CCB) Succinylcholine ○ Mechanism? Depolarizing neuromuscular blocker ○ AE? Malignant hyperthermia ■ Tx? Dantrolene (CCB) Cyproheptadine ○ Mechanism? Anti-histamine w/ serotonin receptor block activity ○ Indication? Serotonin syndrome ■ Try benzo first Benztropine/trihexyphenidyl ○ Mechanism? Muscarinic receptor antagonists ○ Indication? EPS (e.g. acute dystonia, parkinsonism) Memantine ○ Mechanism? NMDA receptor antagonist (combats glutamate excitotoxicity) ○ Indication? ALS Ketamine ○ Dissociative anesthetic, NMDA receptor blocking acting Riluzole ○ Mechanism? NMDA receptor antagonist (combats glutamate excitotoxicity) ○ Indication? ALS 227

● ●

● ●

Ethosuximide ○ Mechanism? T-type CCB ○ Indication? Absence seizures Phenytoin ○ AE? ■ Teratogen ● But if woman is already pregnant & well-controlled, continue it ■ Drug-induced lupus Carbamazepine ○ Indication? Trigeminal neuralgia ○ AE? ■ Teratogen ■ Agranulocytosis Valproic acid ○ Teratogen (causes NTD) ○ Hepatotoxic Topiramate ○ AE? ■ Mental dullness ■ Nephrolithiasis ○ Indications ■ Epilepsy ■ Migraines Lamotrigine ○ AE? SJS Benzodiazepines ○ Mechanism? Increase frequency of GABA receptor (Cl- channel) opening ○ Indication? ■ Cocaine intoxication ■ Seizures ○ Benzos okay for liver dysfunction? LOT ■ L = lorazepam ■ O = oxazepam ■ T = temazepam ○ Benzo for alcohol withdrawal? Chlordiazepoxide (very long-acting) ○ Reversal agent? Flumazenil Barbiturates ○ Mechanism? Increase duration of GABA receptor (Cl- channel) opening Gabapentin ○ Dose adjust in renal disease ○ Indication? Neuropathic pain Propofol ○ Indications? ■ Induction & maintenance anesthesia ■ Sedation of mechanically ventilated pts ■ Procedural sedation ○ AE? ■ Respiratory depression 228

● ● ● ●

● ● ● ● ●

■ Hypotension ■ Propofol infusion syndrome = bradycardia + lactic acidosis ○ Contraindications? Familial hypertriglyceridemia ■ Propofil is suspended in lipid Baclofen ○ Indication? Spasticity (e.g in cerebral palsy pt) Naloxone ○ Indication? Opioid overdose Naltrexone ○ Indication? Alcohol or opioid dependence Triptans ○ Mechanism? Serotonin receptor agonists ○ Indication? Acute tx of migraines ○ Contraindications? Vasospastic disease ■ Prinzmetal angina ■ Raynaud’s ○ AE? Serotonin syndrome Tx of cluster HA → 100% O2 tPA ○ Reversal agent? Aminocaproic acid Drug to give after SAH? Nimodipine ○ Mechanism? Prevents post-SAH vasospasm VEGF inhibitors (e.g. bevacizumab) ○ Indication? Wet age-related macular degeneration Latanoprost ○ Mechanism? Prostaglandin analogs ○ Indication? Glaucoma Acetazolamide ○ Mechanism? Carbonic anhydrase inhibitor ○ Indications? ■ Central sleep apnea ■ Idiopathic intracranial hypertension ■ Elevated ICP ■ Glaucoma ■ Diuretic → metabolic acidosis + hypokalemia Endogenous opioids ○ Beta-endorphin → mu ○ Dynorphin → kappa ○ Enkephalin → delta MAO-Is (isocarboxazid, phenelzine, tranylcypromine) ○ 2 week taper when switching from SSRI to MAO-I ○ AE? Tyramine hypertensive crisis TCAs ○ anti-HAM side effects ■ Sedation ■ Orthostatic hypotension ■ Urinary retention ○ Imipramine 229

○ ●

■ Indication? Nocturnal enuresis Clomipramine ■ Indication? OCD

SSRIs ○ Which has the longest half-life? Fluoxetine ■ Least associated with SSRI discontinuation syndrome SNRIs (e.g. venlafaxine, duloxetine) ○ Which used to treat neuropathic pain? duloxetine ○ AE? hypertension Trazodone ○ AE? ■ Priapism (due to alpha-1 blockade) ● Tx? Phenylephrine injection into penis Mirtazapine ○ Mechanism? Alpha-2 antagonist → ultimately increases release of norepi ○ Indication? Depression in pts with anorexia or insomnia ○ AE? ■ Weight gain ■ Sedation Bupropion ○ Mechanism? NDRI ○ Indication? ■ Smoking cessation ■ Depression ■ Weight loss ○ AE? Lowers seizure threshold ○ Contraindication? ■ Anorexia/bulimia Clozapine ○ Reduces risk of suicide ○ AE? ■ hypersalivation ■ Myocarditis ■ Agranulocytosis ● STOP drug if their white count falls Atypical antipsychotics ○ Risperidone → hyperprolactinemia ○ ZIprasidone → QT prolongation ○ Quetiapine → cataracts ■ Special indication? Psychosis in s/o Parkinson’s ○ Olanzapine → metabolic syndrome ○ Aripiprazole → partial dopamine agonist Lithium ○ Sign of toxicity? tremors ○ AE? ■ Tremors ■ Nephrogenic DI 230

● ●

● ●

■ Hypothyroidism Z-drugs (e.g. zolpidem, zaleplon, eszopiclone) ○ Mechanism? GABA agonist ○ Indication? Insomnia ○ AE? Addictive potential ○ Reversal agent? Flumazenil Ramelteon ○ Mechanism? Melatonin receptor agonist ○ Indication? Insomnia ○ NO addictive potential Suvorexant ○ Mechanism? Orexin antagonist ■ Recall that pts with narcolepsy have low CSF orexin ○ Indication? Insomnia Buspirone ○ Mechanism? Partial agonist at serotonin receptors ○ Indication? Anxiety Drugs of abuse ○ PCP → very agitated ○ LSD → flashbacks ○ Marjuana → hunger + conjunctival injection If a pt is getting an adenosine-based stress test, caffeine or theophylline (methylxanthines) will reverse the effects of adenosine Varenicline ○ Mechanism? Nicotinic receptor partial agonist ○ Indication? Smoking cessation Methylphenidate ○ Mechanism? Stimulant ○ Indication? ADHD ○ AE? ■ Insomnia ■ Weight loss Atomoxetine ○ Non-stimulant ○ Indication/ ADHD Tx for alcohol withdrawal? Benzos Aspirin ○ Mechanism? Irreversible inhibitor of COX-1 and COX-2 ○ Indications? ■ MI ■ Post-stent ■ Stroke ○ Signs of toxicity? Respiratory alkalosis + metabolic acidosis Roflumilast ○ Mechanism? PDE-4 inhibitor ○ Severe COPD Sildenafil ○ Mechanism? PDE-5 inhibitor 231

● ●

● ●

Indication? ■ Pulmonary arterial hypertension ■ Erectile dysfxn Desmopressin ○ Mechanism? ■ Increases release of wVF from Weibel-Palade bodies ■ ADH analog → water reabsorption in the collecting duct ○ Indication? ■ Von Willebrand disease ■ Central DI ■ Nocturnal enuresis ● Caution: it can cause hyponatremia → seizures Heparin ○ Mechanism? Activates AT-III to inhibit factors X and II ○ Indication? Anticoagulant ○ AE? HIT ○ Reversal? Protamine LMWH ○ Mechanism? Activates AT-III to inhibit factors X ○ Indication? Anticoagulant Factor X inhibitors (e.g. rivaroxaban, apixaban) Direct thrombin inhibitors (e.g. dabigatran, argatroban, bivalirudin) ○ Indication? HIT ○ Reversal? For dabigatran → idarucizumab Warfarin ○ Reversal? Four-factor PCC (prothrombin tPA (e.g. alteplase, tenecteplase) ○ Indication? Ischemic stroke within 3.5 hrs symptom onset ○ Reversal? ■ Aminocaproic acid ■ Tranexamic acid ● Other indication? Postpartum hemorrhage Eculizumab ○ Mechanism? C5 inhibitor ○ Indication? Paroxysmal nocturnal hemoglobinuria ■ Pathophys? Defective GPI anchors ○ AE? Induced terminal complement deficiency ○ Ppx? Vaccinate against Neisseria meningitidis Hydroxyurea ○ Mechanism? Inhibits ribonucleotide reductase → increased Hgb F ○ Indication? Sickle cell disease Azathioprine & 6-mercaptopurine ○ Indication? Chemo ○ Metabolized by xanthine oxidase, so co-administration with xanthine oxidase inhibitors (e.g allopurinol) can cause toxicity Methotrexate ○ Mechanism? Inhibits dihydrofolate reductase ○ Indications? 232

● ●

■ Autoimmune disorders ■ Molar pregnancy ■ Choriocarcinoma ○ AE? ■ Hepatotoxicity ■ Bone marrow suppression → rescue with leucovorin Drugs that cause pulmonary fibrosis ○ Bleomycin (works in the G2 phase of the cell cycle) ○ Busulfan ○ Amiodarone ○ Methotrexate 5-fluorouracil ○ Mechanism? Inhibits thymidylate synthase ○ Indication? Chemo ○ Leucovorin WORSENS toxicity ○ Flucytosine is metabolized into 5-FU Cyclophosphamide ○ Mechanism? Alkylating agent ○ AE? Hemorrhagic cystis ■ Prevention? mesna Doxorubicin/daunorubicin (anthracyclines) ○ Indication? Breast cancer ○ AE? Irreversible dilated cardiomyopathy ■ Prevention? Dexrazoxane (iron chelator) Trastuzumab ○ Mechanism? Monoclonal Ab against HER2 tyrosine kinase receptor ○ Indication? HER2+ Breast cancer ○ AE? Reversible dilated cardiomyopathy actinomycin D ○ Indications? ■ Ewing sarcoma ■ Childhood cancer Vincristine/vinblastine ○ Mechanism? Inhibit microtubule polymerization ○ Indication? chemo ○ AE? ■ Bone marrow suppression (vinblastine blasts the bone marrow) ■ Peripheral neuropathy Taxanes (e.g. pacl*taxel) ○ Mechanism? Inhibit microtubule depolymerization Platinum drugs (e.g. cisplatin) ○ Mechanism? Alkylating agents ○ AE? ■ Nephrotic ● Prevention? Amifostine ■ Ototoxic 4 drugs that are nephrotoxic + ototoxic ○ Cisplatin 233

● ●

● ●

○ Vancomycin ○ Aminoglycosides ○ Loop diuretics Topoisomerase inhibitors ○ Irinotecan/topotecan → inhibit topo-1 ○ Etoposide → inhibits topo-2 Imatinib ○ Mechanism? Tyrosine kinase inhibitor ○ Indication? CML Rituximab ○ Mechanism? Monoclonal Ab against CD20 ○ Indication? B cell cancers Tamoxifen ○ SERM ■ Antagonist in breast ■ Agonist in bone ■ Agonist in uterus Raloxifene ○ SERM ■ Antagonist in breast ■ Agonist in bone ■ No effect on uterus What med is used for breast cancer chemoprophylaxis in ○ premenopausal women? tamoxifen ○ postmenopausal woman? Aromatase inhibitors (e.g. letrozole, anastrozole, exemestane) ■ AE? Osteoporosis 5-alpha reductase inhibitors (e.g. finasteride, dutasteride) ○ Mechanism? Prevent formation of DHT ○ Indication? Long-term tx of BPH Epo analogs (e.g. darbepoetin, erythropoietin) ○ Indication? Anemia in CKD Filgrastim/sargramostim ○ Mechanism? G-CSF analogs ○ Indication? Profound bone marrow suppression in chemo pts Ondansetron ○ Mechanism? Serotonin receptor antagonist ○ Indication? Chemo-induced vomiting Aprepitant ○ Mechanism? NK-1 receptor antagonists ○ Indication? Chemo-induced vomiting Metoclopramide ○ Mechanism? Dopamine antagonist ○ Inidcation? Diabetic gastroparesis ○ AE? EPS ATRA ○ Indication? APML t(15,17) Bortezomib ○ Mechanism? Proteasome inhibitor 234

● ●

○ Indication? Multiple myeloma SGLT-2 inhibitors (e.g. canagliflozin) ○ Mechanism? Prevent renal reabsorption of glucose ○ Indication? DM ○ AE? UTIs ○ Contraindication? Renal insufficiency Conivaptan/tolvaptan ○ Mechanism? ADH receptor antagonists ○ Indication? SIADH ○ Very expensive! Mannitol ○ Mechanism? Non-absorbable sugar ○ Indications ■ Elevated ICP ○ Contraindication ■ CHF - mannitol increases vascular oncotic pressure → increased intravascular volume Loop diuretics ○ AE? ■ Nephrotoxicity & ototoxicity ● Especially ethacrynic acid ■ Hypocalcemia & hypercalciuria ■ Hypomagnesemia ■ Hypokalemia ○ Contraindication? ■ Hx nephrolithiasis (dump Ca++ in the urine) ■ Sulfa allergy (all except ethacrynic acid) Thiazides (HCTZ, chlorthalidone, indapamide, metolazone) ○ AE? ■ Strongest association with hyponatremia ■ Hyper-GLUC (glucose, lipids, uric acid, calcium) Probenecid ○ Mechanism? Uricosuric agent (increases excretion of uric acid) ○ Restricted use because pt must be a uric acid underexcretion ○ Contraindication? Nephrolithiasis from uric acid stones Aldosterone receptor antagonists (e.g. spironolactone, eplerenone) ○ Indications? ■ CHF → prolongs survival ■ Conn’s syndrome ■ Hirsutism in PCOS ■ Chronic tx of elevated portal pressures & esophageal varices ○ AE? ■ Gynecomastia (only spironolactone) ENaC channel blockers (e.g. amiloride, triamterene) ○ Indications? ■ Nephrogenic DI 2/2 lithium Aliskren ○ Mechanism? Direct renin inhibitor ACE-Is 235

○ ○ ● ●

● ● ● ●

Indication? ■ CHF → improves survival ■ Nephroprotection in DM ● Dilate efferent arteriole → reduce intraglomerular HTN ■ Peripheral edema associated with dihydropyridine CCBs ● Mechanism? ACE-Is dilate post-capillary venules ■ Scleroderma renal crisis Contraindications? ■ Bilateral renal artery stenosis ■ C1 esterase deficiency Don’t start as first-line antihypertensive in African American pt unless that pt is diabetic AE? ■ Dry cough

ARBs ○ Good for pts with hx dry cough from taking ACE-I Ivabradine ○ Mechanism? Inhibits funny current channels ○ Indications? Symptomatic treatment of angina Class 1a antiarrhythmics (procainamide, quinidine, disopyramide) ○ Procainamide → use for WPW, AE drug-induced lupus ○ Quinidine → AE cichinism ○ Increase AP duration, wider QRS & increased risk Torsades Class 1b antiarrhythmics (e.g. lidocaine, mexiletine, tocainide) ○ Shorten AP duration Class 1c antiarrhythmics (e.g. flecainide, propafenone) Class 2 antiarrhythmics = beta blockers Class 3 antiarrhythmics = K+ channel blockers (e.g. amiodarone, sotalol) ○ Sotalol also has beta-blocking activity ■ Prolongs PR interval Class 4 antiarrhythmics = verapamil & diltiazem ○ Indications? ■ Afib rate control ■ Variant angina ■ Preventative tx for cluster HAs Dihydropyridine CCBs ○ Clevidipine & nicardipine → hypertensive emergencies ○ Nifedipine → safe in pregnancy ○ Nimodipine → prevents post-SAH vasospasm Digoxin ○ Mechanism? Inhibits Na-K ATPase ■ Positive inotrope ■ Muscarinic antagonist ○ Hypokalemia predisposes to toxicity ○ AE? Hyperkalemia ○ Tx of toxicity? Anti-dig Fab Magnesium ○ Indications? 236

■ ■ ●

● ●

● ● ●

Severe asthma exacerbations Eclamptic seizures

Atropine ○ Indications? ■ Organophosphate toxicity ■ Heart block ■ Bradyarrhythmias in ACLS Hydralazine ○ Mechanism? Arterial vasodilator → decreases afterload → increases CO ○ AE? ■ Reflex tachycardia ■ Peripheral edema Nitroglycerin ○ Mechanism? Venodilator ○ Indication? angina ○ Contraindications? Pt taking PDE-inhibitor Nitroprusside ○ Indication? Hypertensive emergency ○ AE? Cyanide poisoning ■ Tx? Amyl nitrate (oxidizing agent) + sodium thiosulfate Minoxidil ○ Indication? Balding in men Diazoxide ○ Mechanism? Opens K+ channel → prevents insulin release ○ Indication? insulinoma Nesiritide ○ Mechanism? BNP analog ○ Indication? Severe CHF Neprilysin inhibitors (e.g. sacubitril) ○ Mechanism? Neprilysin breaks down BNP, so neprilysin inhibitors prevent BNP breakdown ○ Indication? CHF PCSK9 inhibitors (e.g. alirocumab, evolocumab) ○ Mechanism? PCSK9 breaks down LDL receptors. PCSK9 inhibition → less breakdown of LDL receptors → can clear more LDL from serum Antistaphylococcal penicillins (e.g. nafcillin, dicloxacillin, oxacillin) ○ Cover MSSA ○ Indication? Mastitis ○ AE? Acute interstitial nephritis (fever + rash + eosinophils in blood or urine) Cefazolin (1st gen cephalosporins) ○ Indication? Surgical ppx Cefotetan/cefoxitin/cefuroxime (2nd gen cephalosporins) Ceftriaxone (3rd gen cephalosporin) ○ Indication? Neisserial infections ○ Does NOT cover MRSA or Pseudomonas ○ AE? Intrahepatic cholestasis in neonates → give cefotaxime ○ 3rd cephalosporin that covers Pseudomonas? Ceftazidime Cefepime (4th gen cephalosporin) ○ Covers Pseudomonas 237

● ●

● ●

Ceftaroline (5h gen cephalosporin) ○ Covers MRSA Carbapenems (e.g. meropenem, imipenem, doripenem) ○ Covers Pseudomonas, anaerobes, and much more ○ Ertapenem doesn’t cover Pseudomonas Aztreonam (monobactam) ○ Covers Pseudomonas Vancomycin ○ Mechanism? Binds to D-ala-D-ala, so transpeptidase cannot bind and create cross linkages ○ Resistance mechanism? Change D-ala-D-ala to D-ala-D-lac ○ Indications? ■ Covers Gram-positives including MRSA ■ Oral vanc → 1st line for C. diff ○ Does NOT cover Gram-negatives ○ AE? ■ Red man syndrome ● Prevention? Slow infusion rate or give NSAID beforehand ■ Nephrotoxic + ototoxic Daptomycin ○ Mechanism? Disrupts cell membrane ○ Indications? ■ Covers Gram-positives including MRSA ○ AE? NMS Aminoglycosides ○ Mechanism? Bactericidal 30s inhibitor ○ Indications? ■ Covers Gram-negatives ■ Inhaled tobramycin → prevent Pseudomonas in CF pt ○ Does NOT cover anaerobes (because it requires O2 to gain entry to cell) ○ AE? ■ Nephrotoxic + ototoxic ■ ATN → muddy brown cases Tetracyclines ○ Mechanism? Bacteriostatic 30s inhibitor ○ Indications? ■ Zoonotic infections ● Lyme disease ○ Avoid doxy in kids < 8 yo ● RMSF ○ Give doxy to kids, give chloramphenicol in pregnancy ■ Cholera ■ Acne ■ Demeclocycline (causes nephrogenic DI) → SIADH ■ Tigecycline → covers MRSA ○ AE? ■ Photosensitivity ● “SAT for a photo” ○ S = Sulfonamides 238

○ A = amiodarone ○ T = tetracyclines Tooth dislocation

■ Linezolid ○ Mechanism? Bactericidal 50s inhibitor ■ Prevents formation of the initiation complex for protein synthesis ○ Indication? ■ Covers MRSA ■ Covers VRE ○ AE? Serotonin syndrome (weak MAO-I activity)

-------------------------------------------------------------------------------------------------------------------------------

Episode 118: Confusing Breast Pathologies ● ● ● ● ●

Mastitis ○ Unilateral erythema, tenderness, fever, recent postpartum Abscess ○ Unilateral painful, fluctuant mass, recent postpartum, fever Engorgement ○ Bilateral engorgement in recent postpartum F w/fever Galactocele ○ Unilateral could present as tender, fluctuant mass, in breast feeding F, NO FEVER Inflammatory Breast Cancer ○ Unilateral process in non-postpartum older F (50s +), peau d’orange

CROSS CHECKED? Yes

----------------------------------------------------------------------------------------------------------------------------

Episode 119: Rapid Review Series 3 (IM) Episode 119 notes were graciously provided by Divine Intervention from an anonymous contributor. ●

Hypotensive, temp103, crackles at LLB, productive cough for 2-3 days: pneumonia sepsis o Tx: cover MRSA, pseudomonas ● MRSA: vancomycin (--| D-ALA), linezolid (--| 50S), daptomycin (--| gram+ by poking holes in cell membrane), ceftaroline (5th gen ceph), TMP-SMX, clindamycin (--| 50S), doxycycline ● Pseudomonas: ceftazidime (3rd gen ceph), cefepime (4th gen ceph), aztreonam (monobactam), fluoroquinolones, aminoglycosides (--| bactericidal 50S, *do not cover anaerobes), pip-tazo, carbapenems 2mos fatigue, hyponatremic, hyperkalemic, elevated eosinophils, 80/50: Addison's disease o RF: other autoimmune disease o Path: ● autoimmune destruction of adrenal cortex ▪ Zona glomerulosa: mineralocorticoids i.e. aldosterone ● Not affected by secondary adrenal insufficiency (failure of hypothalamus) ● Aldosterone is controlled by RAAS 239

Zona fasciculata: cortisol ● --| eosinophils ● DDx of eosinophilia ▪ Zona reticularis: sex steroids ● Alpha intercalated cell pumps out protons into urine; aldosterone helps make this happen through excretion of K+. If no aldosterone --> non-anion gap metabolic acidosis (Type IV RTA) o Dx: ACTH/cosyntropin --> cortisol levels will not rise o Tx: fludrocortisone Dialysis a month ago, chest pain, difficulty breathing, bilateral/extremity edema, crackles: high-output heart failure o DDx: AV fistula (arteries-veins w/o capillary resistance), Paget's disease (bone marrow becomes hypervascular), AV malformation, severe anemia (CO increases to increase O2 delivery) Blood glucose 900, HCO3 15, Na 125: DKA o Vs. HHNS (not acidotic) ● Pathophys: insulin prevents release of glucagon --> decreased lipolysis, production of ketone bodies Skin hyperpigmentation, Hgb 15, blood glucose 300: hereditary hemochromatosis o Pathophys: C282Y mutation --> increased reabsorption of iron from gut ● "bronze diabetes" d/t iron-Fenten (free radical) reaction that leads to fibrosis of pancreas o Skin: @skin, gonads, heart (1. restrictive 2. dilated cardiomyopathy), calcium-pyrophosphate-deposition disease/CPPD/pseudogout ● CPPD ddx: hereditary hemochromatosis, Gitelman's syndrome (loss of ions @ tubules) ▪ Positive birefringence, rhomboid crystals, chondrocalcinosis on imaging o Dx: ferritin o Tx: phlebotomy ● "menstrual cycles are monthly phlebotomy" --> delayed onset in women Pneumococcal pneumonia, 118 Na: SIADH o Pathophys: any lung or brain pathology! ● DDx: small-cell lung cancer (ACTH, SIADH, Lambert-Eaton) o Sx: euvolemic hyponatremia ● Urine is hyperosmolar o Tx: fluid restriction Vs. diabetes insipidus: not observing effects of ADH in body --> not reabsorbing water --> hyperosmolarity, hypernatremia, low urine osmolarity ● Dx: water deprivation, ADH analog (desmopressin) ● urine osmolarity shoots up: ADH deficiency @ supraoptic nucleus of hypothalamus = central DI ● Urine osmolarity does not change: nephrogenic DI ● RF: lithium (also suicide preventative), demeclocycline Vs. Psychogenic polydipsia: serum osmolarity goes down, urine osmolarity goes down (everything working fine)

Paraneoplastic phenomena Etiology

Pathophys

High hematocrit

polycythemia vera, hepatocellular carcinoma, renal cell carcinoma

Moon facies, hypercortisolism

small cell lung cancer

Ectopic production that does not suppress with high dose dexamethasone

Smoker, pain in bones, finger

lung cancer

hypertrophic pulmonary 240

clubbing

osteoarthropathy

Dermatomyositis, polymyositis

lung cancer

Sudden-onset acanthosis nigricans/Leser Trelat sign

GI cancer

High Ca (tx fluids, biphosphonates i.e zoledronic acid)

multiple myeloma, squamouscell lung cancer, mets, sarcoidosis

multiple myeloma (IL-1 = osteoclastactivating factor), squamous-cell lung cancer (PTHrP), lytic mets (i.e. from thyroid/renal cancer), blastic mets (i.e. from prostate/breast cancer), sarcoidosis (non-caseating granulomas produce 1alphahydroxylase calcidiol --> calcitriol --> reabsorption of Ca/P in gut)

----------------------------------------------------------------------------------------------------------------------------

Episode 120: Rapid Review Series 4 (IM/Peds) Episode 120 notes were graciously provided by Divine Intervention from an anonymous contributor. ●

80Y forgetful, impaired IADL, MMSE~20/30: Alzheimer's Disease ● Pathophys: low Ach (produced at basal nucleus of Meynert via acetylcholinesterase) ● RF: Down syndrome (presenilin gene) ● Tx: ● AChE inhibitors = donepezil, rivastigmine, galantamine Kiddo w/ crying-->cyanosis, reverts back to normal after a while, heart = harsh holosystolic @ LLSB: Tetralogy of Fallot ● Pathophys: ● aorticopulmonary septum usually tries to meet up w muscular interventricular septum ● If aorticopulmonary septum descends to the right of muscular interventricular septum --> you're not dividing the heart too well ● R side of heart: too little space now! ● Sx: ● Pulmonic stenosis ● Right ventricular hypertrophy ● R-L shunt (cyanotic) ● Cry/squat --> compressing blood vessels --> increase SVR --> blood will prefer to go to pulmonic route --> L-R shunt ● Overriding aorta ● VSD

RF

Sx

Tx 241

ASD

Wide-thick split of S2

Aortic stenosis

Bicuspid +/Turner's syndrome, old age

Syncopal episodes, angina, Replace valve; high mortality delayed carotid, systolic once sx develop ("ASH" 1. murmur @ RUSB angina 2. syncope 3. heart failure)

Coarctation of the aorta

Hypertension in UE, hypotension in LE

Aortic regurgitation

Bounding pulses, diastolic murmur @ LLSB

PDA

Continuous machine-like murmur

● ●

Turner's syndrome ● Sx: Hypergonadotropic hypogonadism, horseshoe kidney, bicuspid valve, coarctation of the aorta Wide pulse pressure? ● Adult: aortic regurgitation ● Kiddo: patent ductus arteriosus Marfan's syndrome ● Sx: mitral valve prolapse ("myxomatous degeneration of the valve), aortic dissection ("cystic medial necrosis") ● Aortic problems also a/w Ehlers Danlos, ankylosing spondylitis, syphilis ("vaso vasorum of the arch") Immigrant, diastolic murmur-opening snap: mitral stenosis ● RF: untreated GAS-rheumatic fever ● C/b Afib

RF ● ● ● ●

AAA: smoking Stroke, aortic dissection: HTN Afib: mitral stenosis

Carcinoid syndrome ● Sx: ● 1. none if localized to bowel ● 2. mets to liver release serotonin ● 3. R-sided heart lesions ("TIPS" tricuspid insufficiency, pulmonic stenosis) ● Does not affect lungs bc lungs can metabolize serotonin ● Dx: elevated 5-HIAA ● *Depression: low levels of serotonin, NE, dopamine --> low levels of serotonin metabolites (5-HIAA)

-->

5-hydroxytryptophan serotonin (5-HT) ● Not making niacin/B3 bc all diverted to serotonin! ● Sx: diarrhea, dermatitis, dementia, death Pt over last 3mos feeling very tired, skin hyperpigmentation, CBC looks terrible -- high eosinophil, sodium 123, potassium 6.5: addison's disease/primary adrenal insufficiency ● Pathophys: autoimmune destruction of adrenal cortex ● Sx: Adrenal cortex

Hormone missing

Sx 242

Zona glomerulosa

Aldosterone @ ENAC

Hyponatremia, hyperkalemia, RTA 4/ non-anion gap metabolic acidosis (H+ pump)

Zona fasciculata

Cortisol

Eosinophilia (lack of inhibition)

Zona reticulata

● ●

-->

Increased ACTH bc lack of adrenal response: pro-opiomelanocortin/POMC --> ACTH melanocytestimulating hormone --> hyperpigmentation ● POMC = precursor to b-endorphin + ACTH + melanocytes ● ACTH --| Tuberous sclerosis: cortisol levels do not rise Tx: replace hormones (fludrocortisone, glucocorticoid)

Eosinophilia ● ● ● ● ● ● ● ●

Drugs Neoplasm Addison's disease Acute interstitial nephritis Allergies Asthma Collagen vascular disease (lupus, scleroderma) Parasites

Cross checked? YES ----------------------------------------------------------------------------------------------------------------------------

Episode 121: Rapid Review Series 5 (IM/Peds) Episode 121 notes were graciously provided by Divine Intervention from an anonymous contributor. ●

Chronic alcoholic, AMS, low-grade fever, vague abd pain w/ distension: spontaneous bacterial peritonitis o NBS: paracentesis (>250 WBC, bugs) o Tx: ceftriaxone, fluoroquinolone (gram+ & gram- coverage) o Ppx: fluoroquinolone o RF: ascites, peritoneal dialysis, VP shunt (h/o hydrocephalus) Infection few days ago, got a drug, now 101.5 fever w/ small rash on face and trunk, eosinophilia: acute interstitial nephritis o Sx: fever + rash + eosinophils (@blood or urine) ● DDx eosinophilia ● Drugs i.e. abx ● Neoplasms i.e. hematologic malignancies ● Asthma ● Addison's disease (hyperkalemia, skin hyperpig, hypotension, etc) ● AIN ● Collagen/vascular disease i.e. scleroderma 243

o

Parasites Dx: urine with elevated eosinophils

Urinalysis

Dx

Et

Eosinophils

AIN

Drug

WBC casts

Pyelonephritis

pigmented muddy brown casts

acute tubular necrosis

intrarenal acute injury

reddish urine, but w/o RBC on microscopy

Rhabdomyolysis/Myoglobinuri a

RF: alcoholics, older person fell down and couldn't get up, recent crush injury, combination of two myotoxic drugs (i.e. statins, fibrates, daptomycin)

RBC casts

nephritic syndrome

lupus, IgA, PSGN

fatty casts

nephrotic syndrome (>3.5g/day) minimal change disease, hematologic malig, FSGS 2/2 HIV/AA, membranous nephropathy 2/2 solid malignancy, diabetic nephropathy @ Kimmelstein nodules

Hyaline casts

Dehydration

Afferent arteriole underperfused --> RAAS --> Na absorbed in DCT, ADH release @ principal cell of nephron --> increased urine concentration

Back pain o

+ lost sensation around buttocks, infertility: Leriche syndrome ● Et: severe PAD @ distal aorta + iliac arteries ● RF: smoking o + lifting heavy boxes, sudden-onset severe back pain, paravertebral muscle tenderness: muscle strain ● Tx: NSAIDs, activity modification o + osteoporosis, positive straight leg raise: herniated disk ● Et: nucleus pulposus herniation --> impingement on nerve roots ● Tx: rest + gradual return to activity for 6wks o + sudden-onset, severe abdominal pain, BP 100/50: ruptured AAA ● Dx: "draped aorta sign" = calcifications in front of vertebral bodies ● RF: smoking 6 mos kiddo with loss of motor milestones, fasciculations: spinomuscular atrophy o Vs. adult with fasciculations: ALS o Autosomal recessive @ chr5 SMN1 gene Kiddo with multiple bouts of pneumococcal sepsis, giardia, family members with multiple severe infections: Bruton's agammaglobulinemia o Et: X-linked mutation @ Bruton's tyrosine kinase (B-cell maturation) ● "humoral immunity defect/antibody defect" ● Vs. T cell @ PCP, viral, fungal infections o Sx: show up after 6mos because mom's Ab protects until then o Dx: low levels of every Ig 244

o

Tx: monthly IVIG Kiddo with recurrent infections, petechiae on skin, bleeds when mom tries to brush teeth, eczema: Wiskott Aldrich syndrome o Et: X-linked immunodeficiency o Sx: thrombocytopenia, eczema, recurrent infections Kiddo with recurrent abscesses with bacteria: CGD o Et: mutation in NADPH oxidase (oxidative burst) o Sx: infection with catalase positive organisms (i.e. Staph, Serratia) Kiddo with immunodeficiency, seizures, recurrent parainfluenza, prolonged QT interval: diGeorge syndrome o Et: failed development of 3rd/4th pouches (where T-cells reach puberty) ● T-cell deficiency --> infections ● Hypocalcemia --> prolonged QT, seizures ● DDx: infants of diabetic moms ● Sx: hypoglycemia 2/2 hyperinsulinemia developed in utero, VSD, HOCM Pediatric seizure, on desmopressin for nocturnal enuresis: hyponatremic seizure 2/2 ADH-analog over-secretion o DDx volume depleted kiddo: replenish with isotonic saline

Cross checked? YES ----------------------------------------------------------------------------------------------------------------------------

Episode 123: Ethics I ●

32 yo M s/p drug overdose in the ICU on a ventilator. No brainstem reflexes can be elicited. He is homeless and estranged from family. Girlfriend says that the pt wouldn’t want this. Family wants everything done. NBS? ○ Withdraw care ○ Principle: If you have an accurate, recent representation of a pt’s wishes, you should strongly consider following these wishes.

Pt is septic with hallucinations and waxing-and-waning levels of consciousness. They don’t want you to place IV to administer fluids and antibiotics. They say “I don’t want any of those medications. Don’t give me any of those poisons.” ○ NBS? Administer abx and fluids against pt’s wishes ○ Principle: If the patient isn’t “with it” mentally, then their wishes don’t count. Instead, do what the “rational person” would decide to do.

4 week old infant was brought to ED with 2-3 days of bilious vomiting. He has a hypokalemic hypochloremic metabolic acidosis. Upper GI series shows malrotation with volvulus. You recommend immediate surgical intervention. Parents do not want surgery. NBS? ○ Proceed with surgery against the parents’ wishes ○ Principle: If something is life-threatening to a kid, perform the medically indicated intervention regardless of parents’ wishes ○ Another example: child of Jehovah’s witnesses that needs blood transfusion

Adult with clear mentation has a life-threatening condition. You explain that they will die without treatment. They reiterate that they don’t want the intervention. NBS? ○ Do NOT give treatment 245

Principle: adults with clear mentation have autonomy

Man with hx Alzheimer’s dementia is hospitalized. He has no written directives. Over the past several weeks, he’s been jaundiced and has lost 15 lbs. Imaging shows metastatic pancreatic cancer. A study states that similar pts there is no survival with xyz intervention. Family member wants xyz intervention to be done. NBS? ○ Comfort care measures ○ Do NOT do something that is not medically indicated even if the family wants it ○ Principle: if it’s not medically indicated, don’t do it

14 yo kid with a medical condition requiring surgery. The child is developmentally normal and clearly mentating. The child does not want the surgery. Parents want the surgery. NBS? ○ Proceed with the surgery ○ Principle: If pt is < 18 yo, their don’t matter (see exceptions below). ■ Exception: mental health ■ Exception: reproductive health (e.g. decisions regarding continuing pregnancy & abortion) ■ Exception: child married before age 18 ■ Exception: emancipated minor

Pt comes in with diffuse lymphadenopathy & chronic diarrhea. He reports inconsistent condom use with multiple partners. He tests positive for HIV. He doesn’t want to tell his partner. NBS? ○ Inform the health department. The health department will perform partner notification. ○ Do NOT promise the patient that you won’t inform authorities or that partners won’t be told. ○ Similar to Tarasoff case (duty to warn when the pt expresses intention to harm another person)

Pt says that he’s going to kill his wife. You call the wife. Option 1 = tell the wife to get away and go somewhere safe. Option 2 = tell the wife all the details of how he is going to attempt to kill her. ○ Option 1 is best. Only give enough information so the other person can get out of the unsafe situation.

Pt that is depressed. Pt says “I don’t think I can take this anymore” and “I can’t continue like this much longer.” NBS? ○ Further screening for suicide OR hospitalization ○ Trick answer will be “give antidepressants.” They will take too long to go into effect and the person may already be dead.

Pt expresses suicidal ideation. NBS? Option 1 = ask the pt how they feel about hospitalization. Option 2 = more forceful approach, where you tell the pt that they will be involuntarily admitted. ○ Option 1 is best. Try to convince the pt to voluntarily be admitted before pursuing involuntary admission.

“Mental status exam is negative for evidence of suicidality” → you do NOT need to bring this pt into the hospital

---------------------------------------------------------------------------------------------------------------------------246

Episode 125: Rapid Review Series 6 (IM) Episode 125 notes were graciously provided by Divine Intervention from an anonymous contributor. ● Young adult, AMS for last 3d, close acquaintances think they're "weird recently", obtunded, not an alcoholic, CSF with RBC: HSV encephalitis o Sx: encephalitis/meningitis in newborn or young adult o Dx: @ temporal lobe o DDx ● Subarachnoid hemorrhage: 2-3h duration ● Bloody CSF tap o C/b increased ICP --> uncal herniation (transtentorial herniation) --| CN3 --> "down and out" eye w/ blown pupil (intact CN4,6), --| midbrain/contralateral cerebral peduncle --> ipsilateral hemiparesis ● TSA agent, IVDU (#risky behavior), 2wks losing weight, night sweats, hemoptysis, CXR hilar adenopathy + cavitary lesion: TB o RF: travel, immunocompromised o Dx: serial acid-fast stains, TB skin test ● Induration ● >15mm: TB for all! ● >10mm: healthcare worker ● >5mm: HIV pt ● f/u positive results with CXR --> latent vs active TB ● *negative skin test does not rule out TB! (i.e. sarcoid, HIV) ● Enhancement @ base of brain ● Elevated lymphocytes in CSF o Tx: ● Latent: isoniazid 9mo + B6/pyridoxine ● Ae: drug-induced lupus, slow vs fast acetylators ● B6 deficiency --| glutamine decarboxylase --> increased glutamate --> seizures ● B6 deficiency --| ALAS (heme pathway) --> sideroblastic anemia ● Active: RIPE 2mo + RI 4mo ● Ae ● Rifampin: hepatotoxicity, orange secretions ● Also used to treat leprosy (or dapsone, clofazimine) ● Also used for Neisseria meningitidis ppx (or cipro, ceftriaxone) ● Isoniazid: B6 deficiency, neurotoxicity, hepatotoxicity; "INH" ● Pyrazinamide: n/a ● Ethambutol: color blindness o C/b Potts disease (@vertebral bodies) ● Painful sores under axilla, chronic: hidradenitis suppurativa o Et: inflamed apocrine glands ● Vs. sebaceous glands @ acne o Tx: surgery ● Obtunded, alcoholic/elderly, elevated Cr: rhabdomyolysis o RF: crush injury, intense exercise o Dx: blood on dipstick w/o RBC on microscopy ● Hypocalcemia --> prolonged QT ● Hyperkalemia --> 1. peak t-waves 2. wide QRS 3. sine wave 4. asystole ● Tx: calcium gluconate, insulin + glucose, albuterol, sodium bicarbonate, diuresis w/ furosemide, kayexalate o C/b acute tubular necrosis o Tx: fluids 247

Drug overdose, wide QRS: TCA

Hypersensitivity rxn

Mechanism

Type 1

IgE (@basophils, mast cells) crosslinking --> release of cytokines

Ex ● ●

Type 2

antibodies against fixed antigen

● ● ●

Allergy history, episodic wheezing: asthma ABPA (fungus ball in lung) RBC: autoimmune hemolytic anemia Gp2b3a: ITP Follicular cells of thyroid gland/TSH receptor: Graves disease

Type 3

antigen-antibody complex "two people partner up together and wreak havoc"

● ●

PSGN Hypersensitivity pneumonitis ● RF: close to warm spring, birds, farmer ● A/w thermophilic actinomyces ● Sx: ~ fibrotic lung disease, episodic lung sx

Type 4

delayed-type; T-cells/macrophages

● ● ●

Contact dermatitis TB skin test Poison ivy

Hot tub? Hot topic! o Hypersensitivity pneumonitis o Hot tub lung d/t mycobacterium avium complex ● Sx: female with low level infection for 3wks ● Not always immunocompromised! ● Tx: macrolide ● Ae: prolonged QT, erythromycin (diarrhea, pyloric stenosis in newborn) o Hot tub folliculitis d/t pseudomonas

----------------------------------------------------------------------------------------------------------------------------

Episode 126: Rapid Review Series 7 (IM) Episode 126 notes were graciously provided by Divine Intervention from an anonymous contributor. ●

Newborn, leukocoria: retinoblastoma (Rb gene mutation) o C/b osteosarcoma ● RF: Paget's disease (hypervascularized bone), teriparatide (pulsatile PTH analog) ● Dx: Codman's triangle, sunburst pattern ● C/b mets to lung ● (Just like choriocarcinoma) 248

● ●

60M, back pain on walks, better with leaning forward: spinal stenosis o MCC location: cervical spine 23F severe joint pain starting at L elbow, now in R knee, red vesicular lesions on skin, vagin*l discharge: gonococcal arthritis o NBS: ceftriaxone + doxy/azithro o Dx: no gram stain (@chlamydia, only seen with giemsa stain) o DDx migratory arthritis ● Lyme disease ● Rheumatic fever 89M, AMS, 103F, BP 90/65, HR 107bpm, RR 32, WBC 95k, Hct15% (Hb = 15/3 = 5), Plts 17k, fullness/tenderness of palpation in upper quadrants, recurrent bacterial infections: chronic lymphocytic leukemia o "isolated leukocytosis" ● Infections still occur d/t immature proliferated WBC o Dx: CD5+, "smudge cell" 45M falling asleep a lot at work, BP 145/85, HR 87 bpm, RR 16, Hb 16, BMI 32: obstructive sleep apnea o Dx: ● chronic respiratory acidosis = chronic CO2 retainers ● High HCO3 d/t kidney compensation ● reactive polycythemia d/t chronic hypoxia (high EPO, Hct/Hb) ● ~ COPD labs o Tx: CPAP, weight loss, uvulopalatopharyngoplasty (cut off some tissue in throat) o Vs. central sleep apnea (decreased respiratory drive) ● Tx: acetazolamide --|CA --> waste HCO3 in urine via type 2 RTA --> metabolic acidosis stimulates respiratory alkalosis Elevated Hct, normal/low EPO: polycythemia vera o RF: JAK2 mutation o Sx: "ruddy" appearance, aquagenic pruritus isolated, elevated EPO o DDx ● HCC ● VHL: hemangioblastoma, renal cell carcinoma 62M weight loss, cachectic, temporal wasting, quit smoking 6mo ago (74pack-year history), CXR spiculated cavitary lesion in LUL that shows dysplastic cells that are undifferentiated: squamous cell lung cancer o Assoc w: hypercalcemia via PTHrP (stones, bones, groans, psych, short QT), exudative effusions, Pancoast tumor (--| sympathetic chain --> ipsilateral Horner's), diffuse joint pain (hypertrophic pulmonary osteoarthropathy) o Tx: surgery ● *do PFTs (esp FEV1) before taking them to surgery! o Vs ● Skin hyperpig, hyponatremia, Cushing-oid, LEMS: small cell lung cancer ● Tx: metastatic @ dx! ● Assoc w ● SIADH: euvolemic low serum osm, high urine osm ● ACTH: will not suppress w cortisol d/t exogenous production ● LEMS: prox muscle weakness d/t Ab to presynaptic Ca channels; improves w use ● Dx: incremental (better & better) response with nerve stimulation ● @arms, legs (vs. bulbar sx @ MG)

------------------------------------------------------------------------------------------------------------------------------249

Episode 127: Rapid Review, Series 8 OB/GYN ●

● ● ●

● ● ●

Pt w/CD4 150 cc of vagin*l itching. KOH show pseudohyphae → Candida ○ Risk factors for candida: DM, IC, chronic steroid use, abx use, smoking ○ vagin*l pH is 10% plasma cells 65M with pancytopenia, peripheral smear shows tear-dropped shape RBC ("dacrocytes"): primary myelofibrosis

o ●

80F 6mo history of recurrent infections, WBC 87k, Hgb low, recurrent pneumococcal infections: CLL o Dx: smudge cell

Heme malignancy algorithm ●

LL lie at extremes of age o ALL: very young ● RF: Down syndrome ● *sx onset + decline can be sudden o CLL: older (70-80Y) ● *classically presents with recurrent infections d/t haphazard B-cell production Middle age o Auer rods: Acute promyelocytic leukemia ● Sx: DIC d/t auer rods spilling into circulation ● Pathophys: 15:17 translocation ● Tx: all-trans-retinoic acid ATRA (vitA derivative) o Bcr-Abl fusion protein, philadelphia chromosome, myelocyte proliferation: chronic myeloid leukemia ● Pathophys: 9:22 translocation (philadelphia) ● Tx: imatinib (tyrosine kinase inhibitor)

● ●

78M, RBC stuck like coins ("rouleaux"): multiple myeloma 78M, pancytopenia, dry tap on bone aspiration: 1. Primary myelofibrosis 2. Essential thrombocythemia (a/w JAK2 mutation) 3. Aplastic anemia 70F, 6mos recurrent infections, WBC 47k, many cells in different stages of aspiration that are MPO+, reduced ALP activity: chronic myeloid leukemia o Vs leukemoid reaction: good WBC, elevated ALP (WBC marker) 55M, s/p heme malignancy treatment, DIC: acute promyelocytic leukemia o DIC = low Plt, high fibrin degradation product, high PT/PTT o Tx: ATRA 251

● ●

● ●

● ●

5F 6wks of weight loss, daily fevers, low Hgb/Plt, high WBC, cells +TdT: ALL 50M, peripheral smear shows B-cells with bi-lobed nucleus: Hodgkin's lymphoma o Dx: "Reed-Sternberg cells" CD15+ CD30+ o Epi: @ young ppl + 40s-60s H/o CML, high fevers, weight loss, diffuse LAD for 3wks: progression of CML --> AML = blast crisis 45F with 6mo history of intense pruritus, conjugated hyperbili, intrahepatic bile ducts dilated: primary biliary cholangitis o Dx: anti-mitochondrial Ab o Tx: UDSA/ursodiol Liver transplant (diphenhydramine for itching) 45M with h/o UC, 6mo history of pruritus, dilation of intra + extra hepatic bile ducts: primary sclerosing cholangitis o Dx: p-ANCA (anti-MPO Ab) ● Also seen in Churg-Strauss, microscopic polyangiitis, PSC o Tx: Liver transplant Newborn with conjugated hyperbili o DDx ● Biliary atresia ● Choledochal cyst, Caroli's disease 25M, recurrent hemoptysis, sinusitis, occasional hematuria: Wegener's/granulomatosis with polyangiitis o Pathophys: antigen-Ab complexes = T3HS o Sx: rapidly progressive glomerulonephritis o Vs. Goodpasture (no sinusitis) = T2HS o Tx: steroids, cyclophosphamide 25M, new onset asthma, dysmorphic erythrocytes (RBC casts): Churg-Strauss/EGPA o Dx: eosinophilia, p-ANCA 35F, intermittent discoloration of fingers, diffuse skin thinking: scleroderma o Limited CREST: anti-centromere Ab, calcinosis, Raynauds, esophageal dysmotility, sclerodactyly, telangiectasias o Diffuse: anti-Scl70, anti-topoisomerase o C/b scleroderma renal crisis |-- ACEi o C/I vasospastic drugs i.e. sumatriptan (5HT agonist triggers vasoconstriction) ● Also c/I for Prinzmetal angina 35F, labs nl, PEx nl, up-to-date for vaccines, benign 2/6 diastolic murmur with opening snap at apex: MS o TTE required for ● Systolic >= 3/6 ● Diastolic ● Symptomatic o Vs ● Influenza vaccine: required in fall ● V/Q scan: r/o PE Stress test o c/I pre-existing EKG abnormality (i.e. RBBB), cannot exercise o Alternative: pharmacological stress test via speeding up heart ● i.e. dobutamine (beta agonist), adenosine analog (ae: bronchoconstriction. c/I recent caffeine, theophylline, asthma)

252

-------------------------------------------------------------------------------------------------------------------------------

Ep 129: (Targeted Pulmonary Review) Pulmonary Function Tests ● Pt long term smoker presents with cavitary lesions on CXR and CT and presents with hypercalcemia -> SC cancer of the lung (Paraneoplastic fashion) o Why hypercalcemia? Parathyroid related peptide o NBS: Consider PFT’s (FEV1) ▪

Why? ● Understand lung function before and after. Plan is to resect part of the lung ● Want to make sure pt has enough FEV1 to undergo surgery. ● Fev1 500cc? probably not going to live ●

% is controlled by the good lung

o What’s a reasonable FEV1 compatible with life? ▪

Approx. 1 Liter

● Restrictive vs obstructive o NBS: Pulmonary Function test ● Pt. has rheumatoid arthritis. About to start on methotrexate (dihydrofolate reductase inhibitors). o NBS: Pulmonary Function Test ▪

Why? ● Methotrexate can be toxic to lungs

● Pt. started on methotrexate or amiodarone or busulfan or bleomycin? o NBS: PFT’s ▪

Why? ● Figure out lung function before starting therapy

● Pt. with bloody diarrhea and SOB o Think? ▪

Neuromuscular problem (i.e., Guillain Barré – ascending muscle paralysis)

o If it involves the airway? ▪

Intubate!

o What is used to monitor progression of Guillain Barré? ▪

Spirometry

Lung Volume 253

● Restrictive vs Obstructive ● Restrictive o Pt. with fibrosis lungs, scarred down? ▪

Pulmonary fibrosis ● Why? o Lungs are scarred down. Like shrink wrap. ● What is their lung volume? o Decreased

● Obstructive o Pt. with big lung volume, on imaging on CXR flattened diaphragm or hyperinflated lung. Tend to have air trapping. o Volume overload ▪

Chronically exposed to increased volume ● What does the body do in response? o Dilate

Chronic increased volume on lungs causes their lung to dilate over time. Expire systole (lung contract to release air) of lung. Air trapping = systolic lung function hard to get air out.

● Different Air o Total lung capacity (Sit down and take in all the air you can) o Forced vital capacity (all the air you try to blow out forcibly) o Residual volume = TLC – FVC ● Forced air volumes. o Measuring how much happens in 1, 2 or 3 seconds o How much air is blown out in one second? ▪

FEV1 ● Most people? o 80 – 90% of air ● Obstructive lung disease o Systolic function (ejecting air) is deficient o Barely blow anything out in one second ▪

“Ejection fraction of lung sucks”

o FEV1/FVC ratio goes DOWN ● Restrictive Disease 254

o Lungs are shrink-wrapped. Can’t retain as much oxygen as normal. Can’t accommodate as much volume o 80% of income given to you in a year. I will give you 80k of 100k I make. If I only make 100 dollars, I’ll give you 80 dollars. Same thing happens with restrictive. We have less. o FEV1/FVC is normal or mildly increased. Getting little in only getting a little out. ▪

FEF 25 – 75% ● FEF = forced expiratory flow ● Surrogate to FEV1/FVC for OBSTRUCTIVE disease ● If FEF is down = dealing with obstructive diseases

Percent predicted ● Population measure you want to remember: 70-80% ● If you are less than 70% in FEV1 means probably have some lung pathology ● If 120% = higher than normal lung volumes think obstructive

DLCO ● Gas exchange test for the lungs. How good of a job does lung perform with gas exchange? ● Oxygen traverse’s alveolar membrane and wall of pulmonary capillaries and then to blood vessels ● Pulmonary fibrosis is like having more stuff in the way (adding distance with fibrotic tissue) so diffusion is less efficient. Therefore, there is a decreased DLCO! ● Apart from pulmonary fibrosis, what other conditions cause decreased DLCO? o Emphysema (Even though its obstructive) ▪

Why? ● Destruction of proteases ● No membranes (no alveoli) no surface area for diffusion to happen.

● DLCO of pt. with defect with ab on Type IV collagen (Goodpasture)? o Goodpasture. ▪

DLCO is high ● Blood in lung? Essentially cut out middleman. No blood in vessels. Already at lung. As oxygen comes from the atmosphere it bathes blood directly. Therefore, increased DLCO 255

DLCO of pt with sinusitis, kidney problem (C-ANCA glomerulonephritis), hemoptysis? o Wegener ▪

DLCO is high ● Blood in lung? Essentially cut out middleman. No blood in vessels. Already at lung. As oxygen comes from the atmosphere it bathes blood directly. Therefore, increased DLCO

● DLCO of pt with pulmonary hemorrhage? o DLCO is high. ● DLCO for pt with extrapulmonary cause of restrictive lung disease? o DLCO is normal ● Example of an obstructive disease with potential normal DLCO o Chronic bronchitis ● BUT, remember that obstructive diseases with low DLCO o Emphysema (Assoc. with reduced surface area) ▪

Pt. with asthma with equivocal spirometry? ● NBS: o Methacholine challenge ▪

Muscarinic receptor agonist.

Causes bronchoconstriction

● Ergotamine challenge o Prinzmetal angina or variant angina ▪

Promote vasoconstriction

Look at Slides (~37 mins in)

256

Attached Slides Normal flow volume loop

Restrictive disease

Obstructive disease

257

Fixed obstruction

Oxygen delivery equation = Cardiac Output x Hemoglobin x O2 Sat x 1.34 + (0.0031 x Pa02) 258

PAo2 = partial pressure of oxygen at Alveoli Pao2 = partial pressure of oxygen at pulmonary capillary / blood vessels What happens to patients with Hb 4? ○ Oxygen delivery will drop (Cause Hb dropped) ○ How to compensate? ■ Increase Cardiac Output (to meet oxygen delivery demands ■ Complications? ● High output heart failure ○ Patient hypotensive ■ Give fluid to expand and increase preload ■ Cardiac output goes up. Increase oxygen concentration in liquid part not really improving oxygen content that much (0.0031 factor) Pt has CO poisoning so your SaO2 is not great. CO takes the spot of Oxygen in Hb (which normally has 4 spots for oxygen) -patient becomes hypoxemic A-a gradient. What does it mean? ● Spread between PAO2 and PaO2. ● PAO2 = partial pressure of oxygen in the alveoli ○ PAO2 = 150 - (PaCO2 * 1.25) ○ Normal PAO2 is ~100. (PaCO2 is normally ~ 40) ● PaO2 = partial pressure of oxygen in the blood vessels O2 comes into alveoli and then goes to the bloodstream. As time goes on you kind of lose little amounts of things (like oxygen). Let's say a patient receives 100 and then at blood vessels has 97 it means pt lost 3 of oxygen. Usual A-a gradient = 10 or less. (Don't lose a lot of O2 between Alveoli and artery.) If spread between A-a gradient high? That means something in the lungs that's preventing the blood from being oxygenated properly. ●

Pulmonary embolism ○ blood not flowing through part of the lung. ○ That lowers PaO2 making the gradient wider.

Right to Left shunt PFO, VSD, or ASD with Eisenmenger physiology. ○ Intracardiac shunts (blood is not seeing oxygen form lung) ○ Blood bypasses the lung. Blood visibly not touching the lung. ○ PaO2 goes down, making the gradient wider.

Pt has a big A-a gradient with a wide cardiac shunt. ○ Physiologic - in utero ■ Between pulmonary artery and aorta (Ductus arteriosus) 259

Hereditary hemorrhagic telangiectasia (Osler Weber Rendu) ■ Arteriovenous malformations ● Pulmonary artery and vein connected ○ Blood not passing through capillaries not good gas exchange ● Hypoxemia on a widened A-a gradient

Hypoxemia with normal A-a gradient ○ Think extrapulmonary causes ■ Overdose on opioids. RR down not receiving enough oxygen in the first place ● PAO2 is not getting enough oxygen! ○ PaO2 will not get enough oxygen! ■ High elevations ● Higher elevations = less oxygen content in the atmosphere

-------------------------------------------------------------------------------------------------------------------------------

Episode 130 : Rapid Review Series 10 (Peds) ●

2 yo kid has rhinorrhea for 2 days, mild fever, barky cough ○ Dx: croup ○ Patho: parainfluenza virus ○ Tx: supportive care ○ Symptoms ■ Subglottic problem ■ Steeple sign on CXR ■ Upper airway problem- stridor, less wheezing ● If stridor is mentioned think of croup or epiglottitis ● If wheezing (Lower airway) think of asthma and RSV pneumo (most common in >28 days old) ● Kid in 1st month of life pneumo= GBS ● Kid presents with SUDDEN onset high fevers, grunting, drooling, tripod position ○ Dx: epiglottitis ○ Patho: staph aureus, strep pneumo or H.flu ○ ** not only kids can get epiglottitis, may see it in adults! ● Kid with really bad bouts of cough with cyanosis and vomiting ○ Dx: bordetella pertussis ○ *bacterial infection (lymphocytosis instead of neutrophilia) and WBCs are very high! ○ Vaccine preventable ○ Tx: macrolide (erythromycin * may prolong QT interval) ■ PPX for close contact give erythromycin ● New born that has bilious vomiting ○ Differentials ( if they give you an abdominal xray ask: do i see a double or triple bubble?) ○ 260

Malrotation w/volvulus Dx: upper GI series

Duodenal Atresia

Jejunal atresia

Double bubble on x ray

Triple bubble on x ray

pathophys: Failure to recanalization -Associated w/ down syndrome

Pathophys: associated with vascular problem

Kid that is 10 percentile of weight, bone development problem, very tiny , FTT and floating stool ○ Dx: celiac disease ○ Pathophys: problem with microvilli ○ Check for anti-gliadin or anti-endomysial antibodies and anti-TGA ○ Tx: avoid gluten ○ Bone problems because cant absorb ADEK vitamins→ secondary hyperparathyroidism(low calcium, high PTH and low PO4)→ rickets or osteomalacia ○ Vitamin K deficiency- recurrent bleeds ○ Vitamin E deficiency: Acanthocytosis on blood smear beca ○ Vitamin A deficiency: night blindness ○ Derm: dermatitis Herpetiformis Pt has a h/o celiac and has lost a ton of weight over the last few months, abdominal distension ○ Dx: malignant degeneration ○ Causes an enteric associated lymphoma (EATL) Kid has intermittent abdominal pain and between episodes they feel ok ○ Dx: Intussusception- Telescoping of the bowel ○ Presentation ■ recent URI or GI infection ■ history of Meckel's diverticulum → no Rotavirus vaccine because can predispose intussusception ■ h/o IgA nephropathy- also avoid rotavirus ○ Tx: air or contrast enema - both diagnostic and therapeutic Kid has edema everywhere and had URI 4 days ago and has 4+ proteinuria ○ Dx: Minimal change disease ■ Effacement of the foot processes ■ Pee more that 3.5g of protein ● oncotic pressure is very low= edema everywhere! ● ascites may become infected→ SBP (parencetesis >250 neutrophils and treat with 3rd gen cephalosporin) ● DVT susceptible and pee out antithrombin III so cant inhibit factor 10 and 2 and become hypercoagulable! 261

● ●

● ●

● ●

● Budd chiari: hepatic vein thrombosis (usually in setting of Polycythemia vera) ● renal vein thrombosis( usually in setting of pancreatitis) ■ MCC of nephropathy in kids! ■ Associations: URI, hematologic malignancies ○ Tx: glucocorticoids Renal Vein thrombosis usually associated in the setting of Membranous Nephropathy URI 4-6 weeks ago with dysmorphic erythrocytes/ RBC casts ○ Nephritic syndrome: ■ Post strep Glomerulonephritis - URI 2-6 weeks ago ● Had a skin rash and then hematuria ● Labs: AntiASO titers or Anti DNAse B antibodies (skin infection present) ■ IgA nephropathy : URI 2-6 days ago Kid has recurrent skin abscess and recurrent candidal or aspergillus infections ○ Dx: CGD ■ NADPH oxidase deficiency Kid has anaphylactic response to transfusions, recurrent URI and Giardial diarrhea ○ Dx: IgA deficiency Kid has recurrent URI and giardial infections in a BOY after 6 mo of age ○ Dx: bruton's agammaglobulinemia ■ Problem with B cell maturation (BTK gene) ■ GI infection is almost always giardia! ○ Tx: Monthly IVIG Kid has seizures, EKG = prolong QT interval and PCP infection ○ Digeorge syndrome ■ 3rd and 4th pouch dont form ■ Recurrent viral and fungal infections ■ Hypocalcemia→ prolong QT interval ■ Tracheomalacia Seizure in infant of diabetic mother= hypoglycemia Boy has recurrent infections, CBC= platelet 40000 and augmented lesions on skin ○ Dx: Wiskott aldridge syndrome ■ Associated with thrombocytopenia and eczema Male associated immunodeficiencies - X linked recessive ○ Wiskott aldridge ○ CGD ○ Bruton's Agammaglobulinemia

Cross Checked: YES -------------------------------------------------------------------------------------------------------------------------------

Episode 131: Rapid Review Series 11 (OB, Psych, IM, Neuro) Episode 131 notes were graciously provided by Divine Intervention from an anonymous contributor. ●

25F with multiple nose bleeds, Plt 10k, WBC nl, Hgb nl: ITP 262

o o o

Pathophys: Ab-Gp2b3a RF: lupus Tx: 1. Mild: observation 2. Severe: steroids, splenectomy ● *prior to splenectomy, vaccinate against SHiN Newborn, has not pooped for a wk: o DDx ● Hirschsprung's disease = aganglionic distal colon ● Meconium ileus 2/2 CF ● T. cruzi ● Big heart ● Big colon ● Big esophagus o Supposed to poop within the first 48h!

Septic arthritis

osteomyelitis

pathophys

Infection within a joint

Infection within a bone

sx

Tenderness over joint

Tenderness over bone

dx

Arthrocentesis with high WBC

MRI, Triple phase bone scan

tx

Joint washout, abx

Abx

● ●

● ● ●

Postpartum, looking for means of contraception, what to avoid? o Estrogen-based contraceptives! ● Other c/i: ● VTE ● Stroke ● MI ● Breast ca (ER/PR responsive) ● *also avoid progestin-containing OCP ● Hepatic adenoma ● >35 + smoker ● HTN Copper IUD best means of emergency contraception; c/I heavy menstrual bleeding Protect against STIs? Condoms or abstinence o Spermicides do not offer protection! o RF: IUD, diaphragm ● c/b toxic shock syndrome Injectable progestin analogs q3mo: c/I rapid return to fertility Progestin IUD Mirena --| adenomyosis Basal body temperature measurements LH surge Progestin makes temperature rise = the egg is around! Have intercourse! Kiddo w difficulty hearing, cataracts, deaf uncles, RBC casts in urine: Alport syndrome Sx: boy who "can't see, can't pee, can't hear a high C" Pathophys: COL4A5 mutation @ Type4 collagen; X-linked Genital warts 263

Pathophys: HPV 6, 11 NBS: screen for STIs, Pap smear if >21Y Pap smear Cytology: q3y HPV + cytology: q5y *pap smear every year if immunodeficient HPV vaccine 11-26Y + pap smear --> colposcopy Atypical glandular cells? Endometrial biopsy! MCC death d/t cervical cancer: invasion of ureters 75M with impaired IADL: Alzheimer's dementia Pathophys: low Ach (@basal nucleus of Meynert), decreased choline acetyltransferase (CHAT) ● CHAT deficiency: no Ach! - edrophonium test, similar sx to MG ● Vs. MG: + edrophonium test RF: age, FMHx, Down syndrome Squamous bladder cancer RF: 1. Smoking 2. Anilin dyes 3. Cyclophosphamide 4. Schistosoma haematobium o Vs glandular d/t failure of urachus involution 35F morning stiffness, pain in fingers: rheumatoid arthritis o Dx: anti-RF (IgM-IgG), anti-CCP o Tx: methotrexate, NSAIDs ● *PFTs before starting methotrexate dt ae: pulmonary fibrosis, hepatotoxicity ● Rescue bone marrow with leucovorin

Cerebrospinal fluid Sx

Dx

CSF

Tx

Dementia for 3 weeks, myoclonus, death

CJD

elevated 14-3-3 protein

Bloody diarrhea two weeks ago, symmetric ascending paralysis, shortness of breath

Guillain-Barre syndrome/acute inflammatory demyelinating polyneuropathy

Albumin-cytologic dissociation

o

plasmapheresis o > IVIG Plasmapheresis o also used in tx of TTP

MS

oligoclonal bands

Can also dx with MRI

Narcolepsy

low hypocretin

SAH

Xanthochromia

HSV

RBC

@temporal lobes

High opening pressure (>250)

Morning headache, Pseudotumor papilledema cerebri

o

RF: URI, GI illness NBS: spirometry

264

Bacterial meningitis

High opening pressure, lots of neutrophils, low glucose

Fungal meningitis

High opening pressure, lots of lymphocytes, low glucose

Episodic palpitations, HA, hypertension: pheochromocytoma o NBS: check metanephrine levels (HVA, VMA) o Dx: MIBG scan, posterior mediastinum o Pathophys: chromaffin cells of adrenal medulla (~ sympathetic neuron) o RF: MEN2 (MCC sx is MTC), NF1 o Tx: 1. Alpha blockade i.e. phentolamine, phenoxybenzamine 2. Beta blockade 3. Surgery ● Profound hypertensive crisis during surgery d/t spilling catecholamines? ● NBS: phentolamine, nitroprusside Anti-vaxxer, coughing paroxysms: pertussis o Tx: macrolide ● Ae: prolong QT interval, diarrhea "Weird cluster of sx": rock-hard thyroid gland, hypothyroid, recurrent epigastric pain radiating to back, no biliary tract dz, no EtOH: IgG4 related disease o = ● Reidel's thyroiditis ● Interstitial lung disease ● Autoimmune pancreatitis ● PSC ● Recurrent cholecystitis ● Prostatitis ● Urinary retention 2/2 retroperitoneal fibrosis

-------------------------------------------------------------------------------------------------------------------------------

Episode 132: Weird topics of Social Sciences ● ● ● ● ● ● ●

Px lung CA, Terminal in Hospice care, cachectic: progestin analogs, cannabinoids, steroids. ○ These drugs do not increase longevity Px terminal CA, super depressed, hopeless feel guilty in hospice Tx: Methylphenidate. ○ NO SSRI. Px terminal CA, bad shortness of breath: you give opioids. ○ Give bowel regimen, as opioids cause constipation Px in Qx, nausea: Ondansetron (prolong Qt) Px terminal brain CA, feeling nausea with ICP: steroids, prednisone Px has neuropathic pain, burning tingling, neuropathy: TCAs, amitriptyline, Duloxetine, Pregabalin/gabapentin Cancer pain hx of depression, and after Tx they have fever, myoclonus, hyperreflexia: serotonin sx and tramadol ○ Tramadol is usually the wrong answer for CA pain 265

● ● ● ● ● ● ● ● ● ● ● ●

Person with severe pain with mets to liver, kidney failure: Phentanyl, very safe in px with kidney disease. ○ Do not use morphine in liver injury or kidney failure ○ Another important with liver failure - lower the dose (?) of hydrocodone, oxycodone Cancer pain, Px in tears, complaining of more pain with already opioids: give him MORE opioids: Increase the dose. The person is at the end of life, the best management is giving more opioids. Cancer pain drug to avoid: meperidine → seizures You smell EtOH in another physician : go to the people in charge; report it. Conduct a person in severe illness go to palliative care: improve outcomes in patients, patients fits in this care because benefits of this treatment vs hospice ( 7 drinks/week or >3 drinks/sitting → at risk drinking ■ Male > 14 drinks/week or >6 drinks/sitting → at risk drinking If a pt is withdrawing from EtOH → do not give antipsychotic → increase risk of seizure ○ Best treated with benzodiazepines ○ Treat EtOH chronic with thiamine before glucose etc. Addicted to opioids: give buprenorphine and naloxone to help with opioid dependence ○ If opioid withdrawal → clonidine Pt trying quit smoking: ○ Nicotine replacement therapy ○ Bupropion - if comorbid depression ○ Varenicline - works on nicotine receptors (partial agonist) - most effective ○ Smoking cessation is almost always the correct answer, never E-cigarettes Screening guideline for AAA: abd US between 65-75 if they ever smoked ○ Only applies to men Osteoporosis, screens women, >65, T < 2.5 → bisphosphonate If 9.3% risk of osteoporosis HPV ○ Age 21 - 65 screen every 3 years ○ Age 30 - 65 can do co-testing for every 5 267

● ● ● ● ●

○ Immunodeficient: screen every year with pap ○ If she had hysterectomy for benign reason → can stop pap ○ If pt had surgery for endometrial cancer/hyperplasia → must do pap of vagin*l cuff Colon cancer: start at 50, every 10 years if colonoscopy ○ FOBT every year ○ FIT every year ○ Flex sig = every 5 ○ Flex sig + FIT q1yr = flex sig every 10 years ○ If FHx, screen 10 years earlier ○ Family member at 53 dx w/Colon Ca when do you start? ■ Screen at 10 years earlier or at 40 ← whichever one comes earlier Lung Ca: if pt has 30pack history, quit 40, you can screen for breast Ca, mammography q2yr (NBME exam specific) STI screening, if they sexually active ○ If 14 drinks/week or >4 drinks/one sitting ○ THEN employ formal screen ■ CAGE ● Cut Down ● Annoyed ● Guilt ● Eye opened ■ AUDIT-C is better than CAGE ○ Clues that suggest alcoholism on NBMEs: ■ Megaloblastic anemia (MCV > 100) ■ High GGT ■ AST/ALT > 2:1 ○ Signs/sxs of alcohol withdrawal → give benzons ■ E.g. alcoholic hallucinosis, DTs ○ Remember thiamine before glucose if alcoholic comes to ED ■ This avoids precipitating Wernicke-Korsakoff syndrome ○ If person is delirious because they are withdrawing from EtOH ■ Don’t give antipsychotic (anti-dopaminergic agent) because this can cause seizures ■ Tx? Benzos

Pt addicted to opioids ○ Tx? Suboxone (buprenorphine/naloxone) Pt going through opiate withdrawal ○ Tx? Clonidine

Pt that is trying to quit smoking. What tx would you consider? ○ Nicotine replacement therapy ○ Bupropion (NDRI) ■ Contraindications: eating disorder ○ Varenicline (nicotinic receptor partial agonist) ○ Most effective? Combination > varenicline > bupropion > NRT ○ Never choose e-cigarette on NBME

Screening for AAA? ○ US for men age 65-76 if they’ve ever smoked or have a family hx

Screening for osteoporosis? ○ Women > 65 yo ○ Women < 65 yo with risk factor (e.g. anorexia, prolonged steroid use) ○ FRAX > 9.3 ○ T < -2.5 → bisphosphonate 272

Screening for cervical cancer ○ Age age 70 → NOT recommended

Screening for breast cancer ○ USPSTF: start at age 50, do it every 2 years ○ ACS: start at age 40, do it every year ○ Vaccines ○ Healthcare workers should get Hep B ○ Influenza every year in the fall ○ 1 dose Tdap + Td booster every 10 years ■ Tdap in every pregnancy at 27-36 weeks ○ Varicella can’t be given to infants < 1 year old ○ HPV: ■ Girls age 9-26 ■ Boys 11-21 ■ MSM 11-26 ○ Meningococcal vaccine

273

○ ○ ○

■ Teens going to college ■ Military recruits Asplenic or functionally asplenic → vaccinate against encapsulated organisms ■ Strep pneumo ■ H. flu ■ Neisseria meningitidis Pt on eculizumab (monoclonal Ab against C5) or terminal complement deficiency → vaccinate against Neisseria meningitidis Can pt with egg allergy can influenza vaccine? YES Live attenuated vaccines ■ Varicella ■ Zostavax Herpes zoster ■ MMR ■ Yellow fever ■ Intranasal influenza ■ Oral polio Who shouldn’t get live attenuated vaccines? ■ Babies < 1 year ■ Pregnant women ■ HIV with low CD4 (65 yo → just PPSV23 OR PCV13 then after 1 year PPSV23 ■ dx with CK-MB +PCI, rising Cr, "blue-toe syndrome", mat-like rash "livedo reticularis" on LE? Cholesterol embolization +few days, chest pain (worse in supine position), scratchy sound in systole + diastole? Post-infarction pericarditis o Widespread ST elevation, PR depression +weeks, chest pain, scratchy sound in systole + diastole? Dressler's syndrome (autoimmune pericarditis) o Ddx of pericarditis: recent viral syndrome (MCC), renal failure/uremia, radiation therapy to chest (also a huge RF for thyroid CA!) +chest pain radiating to jaw, nitroglycerin leading to hypotension: RCA infarct (pre-load dependent!) o EKG: II, III, AVF o Tx: fluids *steroids are not generally helpful in MI bc prevent wound healing*

-------------------------------------------------------------------------------------------------------------------------------

Episode 135: The Clutch Electrolytes Episode 135 notes were graciously provided by Divine Intervention from an anonymous contributor.

Calcium ●

Hypocalcemia 276

o

Sx: Chvostek sign @ jaw, Trousseau sign @ upper extremities Hypercalcemia|-- 1. fluids 2. calcitonin o Pathophys: high Ca messes with signaling cascade of ADH --> ~ nephrogenic DI ~ --> low volume o @hospital: think about malignancy! o @outpt: think about primary hyperparathyroid! Pt placed on ACEi o Sx, dx: high renin, high K, metabolic acidosis ● ACEi --> decreased conversion of AT1-AT2 --> low aldosterone --> volume down --> positive feedback on RAAS ● --| Aldosterone --> H+ pump Hypercalcemia o Short QT interval o DDx ● Primary hyperparathyroidism ▪ Parathyroid adenoma: PTH secretion --> osteoclasts --> reabsorb bone ● Vs parathyroid hyperplasia (MEN) ▪ NBS ● Check ionized Ca, PTH ● Tch-99 sestamibi scan ● Sx/dx ▪ High Ca, low PO4 (PTH trashes phosphate) ▪ High urine Ca ● Tx: parathyroidectomy ● "CHIMPANZEES" Familial hypocalciuric hypercalcemia o Pathophys: calcium-sensing receptor does not respond to right signals o Sx/dx: high PTH, high Ca, low PO4, low urine Ca ● Receptor does not function well --> reabsorb tons of Ca from urine o Tx: benign! Chronic kidney disease o Pathophys: no activity of 1-alpha-hydroxylase --> impaired conversion of calcidiol to calcitriol (active VitD) --> cannot reabsorb Ca, PO4 in gut o Sx ● High PTH ● High Ca, high PO4 (cannot excrete through kidneys) o Tx: sevalamer = binds PO4 o What if they get a transplant? ● Glands of parathyroid undergo hyperplasia due to CKD (they're "always on!") --> no longer response to normal signals --> ● Sx/dx: high PTH, high Ca, low PO4 (new kidneys) = tertiary hyperparathyroidism ▪ Bad habits are hard to break. True. ▪ Tx: resect 3.5 parathyroid glands, re-implant 0.5 gland; cinacalcet (if poor sgy candidate) Chronic liver disease o Pathophys: no calcidiol (comes from the liver) --> decreased calcitriol --> cannot reabsorb Ca, PO4 o Sx/dx ● High PTH ● High Ca, low PO4 (kidneys are still functioning!) AA with bilateral hilar adenopathy: sarcoidosis o Pathophys: noncaseating granulomas --> 1-alpha-hydroxylase --> calcidiol conversion to calcitriol --> reabsorption in gut --> high Ca --> low PTH Heavy smoker, cavitary lesion in lungs, hypercalcemia: lung squamous cell carcinoma 277

o o o

● ●

● ● ●

Pathophys: paraneoplastic PTHrP --> high PTH --> high Ca, low PO4 --> low PTH Sx: central mass, cavitary lesions Dx: "keratin pearls" on histology H/o MEN syndrome, diarrhea, prolonged QT interval: MEN2B o Sx: medullary thyroid cancer, Marfanoid habitus, mucosal neuromas, pheochromo ● Vs, MEN2A primary hyperparathyroid, pheo, MTC ● Vs. MEN1 primary hyperpara, pituitary adenoma, pancreatic neuroendocrine (VIPoma, insulinoma, glucagonoma, gastrinoma) ▪ Tx: insulinoma - diazoxide (open K channels to prevent insulin secretion) o Dx: calcitonin --> tones down calcium! Low Ca o Ppx: resect thyroids Large volume blood transfusion, carpopedal spasms, seizures: EDTA o = ~anticoag that chelates Ca --> hypocalcemia o Sx: dilutional thrombocytopenia --> bleeding o Tx: calcium gluconate ● Also used for hypermagnesemia (i.e. PEC), hyperkalemia Rhabdomyolysis o Sx/dx: low Ca ● Necrosed skeletal muscle fibers sequester Ca Pancreatitis o Sx/dx: low Ca ● Saponification reaction by fatty acids Infants of diabetic mothers o Sx/dx: low Ca, low glucose Congenital heart disease, hypocalcemia: diGeorge syndrome o Pathophys: failure of 3rd/4th pharyngeal pouches o Sx: low Ca, recurrent infections, hypocalcemic seizures Hypervitaminosis D o Sx/dx: high Ca, variable PO4, low PTH ● Increased reabsorption of Ca, PO4 in gut Hypercalcemia of malignancy o Tx: biphosphonates Multiple myeloma --> hypercalcemia o Pathophys: plasma cells secrete IL-1 (osteoclast-activating factor) --> reabsorbs bone --> high Ca Mg x Ca o High Mg --| PTH --> low Ca o Low Mg --> PTH --> high Ca o Very low Mg --| PTH --> low Ca ● Won't respond to Ca (or K) depletion w/o Mg repletion! Thiazide diuretics o Sx/dx: high Ca, low urine Ca o Pathophys: --| Na/Ca exchanger @ DCT ● Good for kidney stones, osteoporosis! Loop diuretics o Sx/Dx: low Ca, high urine Ca o Pathophys: --| NKCC transporter @loop of Henle --> no extrusion of K --> no reabsorption of Ca o C/i: kidney stones Nephrotic syndrome, Menetrier's disease (protein-losing gastropathy): low albumin --> low Ca, normal ionized Ca

Zinc 278

Low zinc o Sx: impaired taste, impaired wound healing, loss of hair o DDx ● Nutritional ● Trientene = zinc chelator ▪ @ Wilson's dz tx

Magnesium ●

Low Mg o Sx/dx: prolonged QT interval ● "hypo___" causes prolonged QT interval! o DDx ● Alcoholics: hypomagnesemia, folate deficiency, B1/thiamine deficiency ● Refeeding syndrome ▪ *Hypophosphatemia is the killer in refeeding syndrome! o Tx: Mg High Mg o Sx/dx: 1. decreased DTR 2. respiratory depression 3. CV collapse o DDx ● PEC with Mg infusion o Tx: calcium gluconate Tx for premature labor, preterm infant

Potassium 1. 2. 3. 4.

Hypoperfusion @ JG cells Renin released AT1 converted to AT2 @ ACE receptor in lung capillaries AT2 a. Vasoconstrictor @ type1 receptors b. Stimulates production of aldosterone from zona glomerulosa c. Stimulates release of ADH from supraoptic nucleus of hypothalamus d. Efferent arteriolar constriction to maintain GFR Low K o Sx: flat T waves, U waves, prolonged QT interval o DDx: anything that increases RAAS activity! ● Hypovolemic ▪ Pathophys: decreased perfusion of afferent arteriole --> JG cells see this --> renin --> AT1 --> AT2 --> aldosterone --> principal cell of collecting duct --> reabsorb Na, dump K ● Conn syndrome ▪ Sx: resistant hypertension, metabolic alkalosis ▪ Pathophys: adrenal adenoma making aldosterone --> reabsorb Na, waste H, waste K --> volume expansion, metabolic alkalosis ● Hyperperfusion --| renin ▪ Dx: increased plasma aldosterone:renin ratio ● Vs: renal artery stenosis/FMD/NSAID = hypoperfusion of afferent arteriole --> RAAS --> low K, high Na, metabolic alkalosis, normal aldosterone:renin ratio ● Diuretic --> decreased volume --> hypoperfusion --> RAAS ● Sx/dx: low K, metabolic alkalosis (proton pumps @ alpha-intercalated cells) ● *low K does not always mean alkalosis -- see below! ● Acetazolamide, dorzalamide 279

Pathophys: --| carbonic anhydrase --| reabsorption of HCO3 --> dumping HCO3 in urine, water follows --> non-anion gap metabolic acidosis (Type 2 RTA) with volume depletion/diuresis --> RAAS ● Sx/dx: low K, metabolic acidosis, volume depletion ● Uses ▪ Volume overload ▪ Central sleep apnea: encourages ventilation ▪ Idiopathic intracranial hypertension/pseudotumor cerebri: inhibition of CSF production ▪ Glaucoma: inhibition of aqueous humor production ▪ Altitude sickness Hyperventilation o Pathophys: respiratory alkalosis --> ● pulls H+ from cells --> potassium enters cells ● Binds up positive charges in the body i.e. Ca --> low Ca

High K o Sx:

o

o

o

o

o o

peaked T waves widened QRS Can also be caused by TCA! sinusoidal EKG flat line/asystole death

Tx: calcium gluconate insulin (activate Na/K pump) + glucose albuterol/beta agonist (activate Na/K pump) sodium bicarb (induce alkalosis) kayexalate, loop diuretic Addison's disease Sx: orthostatic hypotension for a while, low Na, high K, non anion-gap metabolic acidosis (Type IV RTA, positive urine AG), eosinophilia ▪ Vs. diarrhea non anion-gap: urine AG (negative/"neGUTive") Pathophys: autoimmune destruction of adrenal glands ▪ Decreased perfusion --> RAAS --X--> cannot produce aldosterone --| reabsorption of Na … Rhabdomyolsis Pathophys: muscle cells are dying --> release of K Ppx: telemetry to track fatal arrythmia Tx: hydration, monitor EKG Tumor lysis syndrome Pathophys: lymphoma/leukemia cells dying --> release of K Potassium sparing diuretics spironolactone/eplerenone (--| aldosterone receptor) amiloride/triamterene (--| ENAC) ▪ Can treat nephrogenic DI 2/2 lithium

Sodium ●

Low Na o Serum osmolarity = 2(Na) + glucose/18 + BUN/2.8 ● ^Na is the greatest contributor o + Low serum osmolarity (280-290) ●

+ Low volume 280

DDx: ●

▪ ●

diuretics (--| NKCC, losing more Na than water bc of power of diuretic) ● Forcing a kidney to work outside its normal physiology! High urine Na ● vomiting (loss of electrolyte-rich fluid) ● Low volume --> RAAS --> reabsorb Na, low urine Na Tx: fluids

+ Normal volume ▪

DDx: ●

Small-cell lung cancer: SIADH ● Also seen with early gen sulfonylurea, clozapine, SSRI ● ADH --> keep reabsorbing water in the blood --> low serum osmolarity, high urine osmolarity ● Tx: fluid restriction, ADH antagonist (-vaptan) ● Hypothyroidism ● Psychogenic polydipsia ● Also seen in MDMA ● Excess water intake --> dilution of plasma --> low osmolarity --> decreased ADH --> pee out all the water you just drank --> low serum osmolarity, low urine osmolarity 4. Tea and toast diet ●

+ High volume ▪

DDx: cirrhosis, nephrotic syndrome, renal failure ● Low oncotic pressure --> not much fluid in vasculature --> not perfusing vasculature -->--> RAAS DDx: CHF ● Effective blood volume is low i.e. not perfusing kidneys well (cannot pump forward) --> RAAS ● High aldosterone: increase Na + H2O ● High ADH: increase H2O ● Dx: low urine Na (*high urine Na in renal failure) d/ t aldosterone ● Tx: restrict fluid

High Na o DDx: diabetes insipidus ● Not enough ADH activity! Cannot retain water --> peeing out a ton --> high serum osmolarity, low urine osmolarity ● DDx ● Central: closed head injury ● Nephrogenic: lithium, demeclocycline, hypocalcemia ● Dx: ▪ water deprivation test ● Urine osmolarity goes up: normal ● Barely increases: DI! ▪ Desmopressin ● Urine osmolarity goes up? Central DI ● Barely increases: nephrogenic DI ● Also used for: central DI, vWD (increased release of vWF from WP bodies), mild hemophilia A, nocturnal enuresis, kidney-assoc coagulopathy (uremia prevents degranulation of platelets)) ● Vs. coagulopathy of liver disease (tx: FFP) ● Ae: hyponatremic seizure o Tx: ● Normal saline 281

D5W High --> low: cerebral edema! o *Think about this in overtly rapid correction of glucose in DKA, HHNK Low --> high: pons will die!

● ●

-------------------------------------------------------------------------------------------------------------------------------

Episode 137: Next Best Step In Management Series 1 ●

55 yo M with unilateral vision loss, sxs resolved within 5 mins → TIA ○ NBSDx? Carotid ultrasound ○ NBSM? Antiplatelet agent (e.g. aspirin, clopidogrel) ■ NOT anticoagulant unless obviously has A-fib 55 yo M with WHOL, started 30 mins ago + nuchal rigidity + fever 100.9F → SAH ○ NBSDx? NCCT ■ If negative, do LP → xanthochromia ○ NBSM? ■ Labetalol (control BP) ■ Nimodipine (prevent ischemic stroke) 50 yo F with unilateral HA + pain with chewing 2 weeks ago → temporal arteritis ○ NBS? High-dose corticosteroids ○ Temporal artery biopsy later ○ Polymyalgia rheumatica ■ Proximal shoulder pain & stiffness ■ NBS? Low-dose corticosteroids ■ Temporal artery biopsy later 60 yo M w/ AMS for past 24 hrs + temp 102F + nuchal rigidity → meningitis ○ If signs of elevated ICP → give abx ○ Otherwise, obtain LP then immediately afterward give abx ○ Antibiotics: ■ Neonate or over 50 → ceftriaxone + vancomycin + ampicillin ■ Otherwise → ceftriaxone + vancomycin Red, hot, swollen, painful knee + temp 102F → possible septic joint ○ NBS? Arthrocentesis ■ 50k → septic arthritis ● Potentially crystalline arthropathy ■ Yellow needle-shaped, negatively birefringent crystals → gout ■ Blue rhomboid, positively birefringent crystals → CPPD Lifelong smoker with 40 lbs weight loss + nodule in LUL on CXR + large L-sided pleural effusion → lung cancer ○ NBS? Thoracentesis ○ Malignant cells in pleural fluid → stage 4 → not surgical candidate Alcoholic M with AMS for past few days + spider angiomas/gynecomastia + asterixis + fever 100.9F + mild diffuse abdominal tenderness + shifting dullness & fluid wave → SBP ○ NBS? paracentesis 282

● ●

■ >250 PMNs → SBP ○ Tx? Ceftriaxone + FQ prophylaxis after recovered ○ Associations? ■ Peritoneal dialysis ■ VP shunts ■ Budd Chiari syndrome 50 yo M with MI 3 days ago or hx A-fib + severe, sudden onset leg pain + cold, mottled leg → ALI ○ NBS? ■ 1st: IV heparin ■ 2nd? Vascular surgery consultation 50 yo M with MI 3 days ago or hx A-fib + severe abdominal pain → acute mesenteric ischemia ○ NBS? Angiography to find vessel ■ Most likely vessel? SMA Lifelong smoker with severe bilateral leg pain → peripheral arterial disease ○ NBS? ABI ■ 1.4 → likely Mönckeberg medial calcific sclerosis ● NBS? Toe-brachial index ○ Tx? ■ 1st: Supervised walking program ■ 2nd: Cilostazol (PDE inhibitor, helps improve sxs) ■ 3rd: Bypass ● But arteriography first! Pt with hx hearing voices + multiple hospitalizations + improves on antipsychotics in hospital, then stops taking them → schizophrenia ○ NBSM? Monthly haloperidol decanoate injections Pt on antipsychotic with head “stuck” → acute dystonia ○ NBS? Give diphenhydramine or benztropine Pt on antipsychotic feeling restless, pacing in exam room → akathisia ○ NBS? ■ 1st line: beta blocker ■ 2nd line: benzodiazepine Pt with hx schizophrenia with resting tremor + bradykinesia → parkinsonian ○ NBS? ■ 1st line: benztropine or trihexyphenidyl ■ 2nd line: dopamine agonists Pt been on antipsychotic for a while, now with weird tongue movements → tardive dyskinesia ○ NBS? Stop drug ■ Switch to atypical antipsychotic 35 yo F with BMI 40 with morning HAs + blurry vision for last 3 weeks → IIH ○ NBS? LP ○ Tx? ■ Acetazolamide ■ serial therapeutic LPs 283

● ●

● ●

● ●

Stop potential offending meds ■ Vit A analogs ■ Doxycycline 35 yo M with daily pulsatile headaches for past 3 months, feels better resting in dark room → migraines ○ NBS? Chronic migraine medication ■ Beta blockers ■ Topiramate ● Adverse effect? nephrolithiasis ■ TCA ● Signs of toxicity? Wide QRS 35 yo M hx of migraines, having severe HA right now ○ NBS? Abortive therapy ■ Sumatriptan ■ Ergo derivative 35 yo M with severe unilateral headache + rhinorrhea + conjunctival injection → cluster HAs ○ NBS? 100% O2 30 yo M hx of tx-resistance schizophrenia, started on new med 2 weeks ago, over last 24 hrs has had fever to 103F + WBC 2k with 20% neutrophils → neutropenic fever 2/2 agranulocytosis 2/2 clozapine tx ○ NBS? Stop clozapine ○ Tx? ■ Antipseudomonal agent: e.g. meropenem, imipenem ○ Similar presentation to pt on chemo or pt with Graves’ on methimazole Lifelong smoker presents with 7 days of SOB + increased sputum production → COPD exacerbation ○ NBS? IV corticosteroids + bronchodilators (e.g. Duoneb) + azithromycin ■ Azithromycin is used for its anti-inflammatory properties 60 yo M with hx AR presents with 3 days of profound SOB + orthopnea + S3 heart sound → CHF exacerbation ○ NBS? IV furosemide (loop diuretic) ■ Can also gives nitrates to relieve dyspnea 2/2 pulmonary edema Pt with terminal cancer with dyspnea ○ NBS? Morphine 75 yo M lifelong smoker presents with 6 weeks of joint pain (fingers, legs) not improved with NSAIDs + finger clubbing on exam → Hypertrophic pulmonary osteopathy ○ NBS? CXR 57 yo F with insomnia due to “crawling sensation in leg” → restless leg syndrome ○ NBS? Dopamine agonist (e.g. pramipexol, ropinirole) ■ Alternatively, primidone ○ Association? IDA 30 yo M with 6 months of diffuse worries → GAD ○ NBS? Consider SSRI or buspirone (partial serotonin receptor agonist) 25 yo F with hx depression + placed on SSRI/SNRI 4 weeks ago + want to stop drug due to sexual side effects ○ NBS? Switch to bupropion (no sexual side effects) ■ Other indications: smoking cessation, weight loss ■ Contraindication: anorexia & bulimia, hx seizures 50 yo F with 2 months of guilt + loss of interest + hypersomnia + low energy + HR 49 → pseudodepression 2/2 hypothyroidism ○ NBS? TSH 284

● ●

● ●

45 yo M with 2 weeks palpitations + 10 lb weight loss + BP 180/90 + HR 190, then suddenly becomes somnolent → thyroid storm ○ NBS? Propranolol then PTU then Lugol’s solution (supersaturated solution of KI) 75 yo M over past year forgets to turn off stove, forgets longtime friends → Alzheimer’s disease ○ NBS? Brain MRI ○ Tx? AChE inhibitor ■ Donapozel ■ Galamine ■ Rivastigmine 35 yo M alcoholic presents with 2 days severe mid abdominal pain, radiates to back → alcoholic pancreatitis ○ NBS? IVF & NPO ○ DX? Lipase (more specific than amylase) 40 yo F with Grave’s on methimazole presents with 2 hrs palpitations + HR 220 + pulse irregular, then appears confused & BP drops 70/40 → A-fib with RVR, now HDUS ○ NBS? Synchronized cardioversion = DC cardioversion ■ Because pt is hemodynamically unstable ○ What if the patient was HDS? ■ Beta blocker ■ CCB (verapamil, diltiazem) ● Verapamil associated with hyperprolactinemia ○ Rate control strategy: beta blocker, non-dihydropyridine CCB, digoxin ○ Rhythm controls strategy: amiodarone 25 yo M with palpitations + HR 250 + irregularly irregular + hx WPW ○ NBS? Procainamide ■ Do NOT use AV nodal blocking agent (BB, CCB, digoxin, adenosine) because this enhances use of accessory tract → V-fib likely 30 yo F presents with 3 months severe abdominal pain + globular, boggy uterus → adenomyosis ○ NBS? Hormonal IUD 40 yo F presents with 3 months severe abdominal pain + globular, boggy uterus, doesn’t want any more children, fed up with pain → adenomyosis ○ NBS? Hysterectomy ○ Dx? Uterine MRI ○ Gold-standard test for adenomyosis? Diagnostic laparoscopy 45 yo F with BMI 40 + severe pain RUQ + temp 101F + WBC 12k? ○ NBS? RUQUS ■ If equivocal → HIDA scan ● “Non-visualization of GB after 4 hrs” = positive for cholecystitis ○ Tx? Proceed with cholecystectomy Pt severely ill in hospital for 2 weeks now with RUQ pain + fever + leukocytosis + RUQUS showing GB wall thickening & pericholecystic fluid but NO cystic duct dilation or obstructing stones → acalculous cholecystitis ○ NBSM? Percutaneous cholecystostomy ■ Do NOT perform cholecystectomy (high mortality!) 17 yo M with BMI = 40 with morning HAs + RR 8 + PCO2 60 → OSA/OHS ○ NBS? Polysomnogram 6 month old with 2 weeks of frequent GTCs + hypopigmented macules on exam ○ NBSM? ACTH ■ Alternative = vigabatrin 285

● ●

○ EEG findings? Hypsarrhythmia ○ Association? Tuberous sclerosis 33 yo M with BMI = 18 with 3 days of severe blood diarrhea + severe distention, rebound, guarding on exam → toxic megacolon 2/2 UC ○ NBS? Ex-lap Ex-lap is NBSM ○ Toxic megacolon (2/2 C. diff, UC, Chagas) ○ Penetrating trauma to abdomen ○ Free air under diaphragm Nurse stuck by needle from Hep B positive pt, had Hep B vaccine 2 years ago with confirmed immunity ○ NBS? No further action ○ If non-immune? Hep B vaccine + Hep B immune globulin HIV pt on HAART, CD4 = 600 ○ NBS? ■ In fall, give influenza vaccine ■ Pneumococcal vaccine ● Adults 65 yo 22 yo G2P1 F at 18 weeks, has elevated AFP, decides to undergo amniocentesis ○ NBS? Give Rhogam 37 yo F with 3 months of heavy menstrual bleeding, BMI = 35, menses every 70 days ○ NBS? OCPs ○ Endometrial biopsy may also be the right choice due to high risk of endometrial hyperplasia/carcinoma 25 yo M with high # partners and inconsistent use of condoms. Painless lesion with heaped up borders on penis ○ NBS? RPR or VDRL ○ After RPR/VDRL, do a treponemal test (e.g. FTA Abs) ○ Tx? Penicillin G 35 yo M going for eye surgery, 20 minutes after induction has stiffness in extremities and fever 105F → malignant hyperthermia ○ Pathophys? Abnormal ryanodine receptors permits tons of Ca++ to be released from SR → muscle contraction ○ NBS? Administer dantrolene ■ Blocks ryanodine receptor (which is a Ca++ channel that enables Ca++ release from the sarcoplasmic reticulum) ○ Inheritance? AD ○ Gene? Ryanodine receptor gene

-------------------------------------------------------------------------------------------------------------------------------

Episode 138: The Clutch Genetic Disease Episode 138 notes were graciously provided by Divine Intervention from an anonymous contributor.

286

Chromosomal/trisomy ●

Newborn w/ flat face, epicanthal folds, Brushfield spots on iris, single palmar crease: Down syndrome ● Et: Trisomy 21 ● Pathophys: 1. Maternal nondisjunction (MCC) 2. Robertsonian translocation = 1 part of one chromosome goes to another --> teeny-tiny chromosome + big boy chromosome ● Dx: elevated bHCG + inhibin, low AFP + estriol, karyotyping ● Sx (mostly from neural crest issues): ● Endocardial cushion defects w septa + valves --> atria and ventricles not separated properly ● Duodenal atresia --| recanalization ▪ Vs. jejunal atresia d/t vascular insult in utero ● Hirschsprung disease --| Auerbach/Meissner plexi ▪ Sx: meconium ileus ● C/b ALL, Alzheimers (amyloid-precursor protein on chr21) ● RF: advanced maternal age = less eggs to fertilize, higher likelihood of fertilizing nondisjunction egg Second most common trisomy: Edwards Syndrome ● Et: Trisomy eighteen ● RF: advanced maternal age ● Sx: prominent occiput, micrognathia, overlapping digits, rocker-bottom feet, death by age1-2 Least common trisomy: Patau syndrome ● Et: Trisomy 13 ● RF: advanced maternal age ● Sx: microcephaly, micropthalmia, holoprosencephaly, multiple digits, cleft lip/palate, rocker-bottom feet, death cannot put mannose-phosphate on things, so things are improperly transported FA oxidation disorders ● Medium chain, long chain FA: metabolized @ mitochondria ● MCAD/LCAD --| beta-oxidation --> impaired FA oxidation --> elevated acylcarnitine ▪ Vs. carnitine deficiency: low acylcarnitine ● Very long chains: metabolized @ peroxisome ● Adrenoleukodystrophy: demyelination, cerebellar ataxia, death hemolytic anemia: G6PD ● Sx: heinz bodies, bite cells ● Dx: G6PD assay weeks after episode

X-linked recessive (sx @ boys) 289

Immunodeficiencies! ● Recurrent infections, hypopigmented skin (eczema), low platelets: Wiskott Aldrich syndrome ● Chronic Granulomatous Disease ● Et: deficiency in NADPH oxidase ● Sx: recurrent infections w Staph aureus (abscesses) ● Dx: tetrazolium, dihydrorhodamine test ● Tx: interferon-gamma ▪ Vs. ● Interferon alpha --| HepC ● Interferon beta --| MS ● Kiddo w/ gout, chews off fingers/toes: Lesch-Nyhan syndrome ● Et: deficiency of HGPRT (purine salvage pathway) ● Sx: hyperuricemia --> toxicity @ neurons, self-mutilation ● Bruton's agammaglobulinemia ● Et: mutation in Bruton's tyrosine kinase (B-cells) ● Sx: small tonsils, bacterial infections ● Tx: IVIG Hemophilia A --| Factor 8 ● Sx: elevated PTT ● Dx: correct w mixing studies (but sometime create Ab-factor8) Hemophilia B --| Factor 9 ● Sx: elevated PTT ● Dx: correct w mixing studies (but sometime create Ab-factor9) Meinke's disease/kinky hair disease ● Et: ATP7A mutation (copper metabolism) ● Vs. ATP7B Wilson's disease (copper @ basal ganglia, liver dz, psych, Kayser, c/I copper IUD, tx penicillamine/trientine) ▪ *penicillamine/trientine = chelators; ae: zinc deficiency (also chelated) ● Sx: death will not clear cholesterol ● Type 1: No LDL receptor ● Type 2: LDL receptors do not reach surface of the cell (--| exocytosis pathway) ● Type 3: LDL has trouble binding to LDL receptor; MCC ● Type 4: LDL binds to LDL receptor but not endocytosed ● Type 5: --| recycling LDL receptor ● Sx: accelerated atherosclerotic dz ● Tx: statin, PCSK9 inhibitors (-cumab) 6'5", hyperextensible joints, displacement of lens, tearing chest pain radiating to back: Marfan's syndrome ● Et: mutation on chr15 fibrillin gene/FBN1 (maintains integrity of elastic fibers) ● Sx: ● Mitral valve prolapse/myxomatous degeneration ● Impaired elasticity of arteries (@intima) --> dilation without collapse --> aneurysm --> ascending aortic dissection (highest pressure vs. abdominal aorta) --> pull leaflets apart --> aortic regurgitation ● Circle of willis aneurysm --> subarachnoid hemorrhage ● Lens subluxation/ectopia lentis: up and out 290

Marfan's syndrome

hom*ocystinuria

Aut dom

Aut rec

Fibrillin mutation

Cystathionine beta-synthase deficiency

Lens up and out

Lens down and out

Normal intelligence

ID

*B6/B9/B12 elevate hom*ocysteine*

● ●

● ●

Problems w/ wound healing, hyperextensible joints: Ehlers-Danlos syndrome ● Et: mutation in Type 3 collagen ● Sx: aortic dissection, thoracic aortic aneurysm, wound dehiscence, rupture of colon (thicker fecal material d/t water absorption --> wall of colon w type3 collagen struggles) Kiddo w blue sclera, fractures, short: osteogenesis imperfecta ● Et: defect in Type 1 collagen (COL1) Kiddo w hematuria, hearing problems, cataracts: Alport syndrome ● Et: defect in Type 4 collagen COL4A5 (@eye, ear, kidney) ● *X-linked dominant ● Vs. Goodpasture syndrome: Ab-Type4collagen Kiddo w short stature, dwarf, problems w long bone growth: achondroplasia ● Et: mutation in fibroblast-growth-factor3 Café au lait spots, growths on body: NF1 ● Et: chr17 @ neurofibromin = tumor suppressor of p21 (RAS cascade) ● Sx: neurofibromas, meningiomas, pheochromo, Lisch nodules (hamartomas @ iris) NF2 ● Et: chr22 @ merlin = tumor suppressor that tells cells when they're too close (contact inhibition) ● Sx: bilateral acoustic neuromas Posterior fossa mass, hematocrit 60%: VHL ● Et: chr3 @ VHL gene = ubiquitination of proteins that cause growth/proliferation ● Sx: hemangioblastoma (esp @cerebellum, inhibited downregulation of HIF1 vascularization), EPO production, cysts @ liver/pancreas, bilateral renal cell carcinoma Multiple polyps in colon: FAP ● Ppx: colonoscopies encephalopathy, lactic acidosis, stroke

Multifactorial inheritance ● ● ●

Neural tube defects Congenital heart disease T2DM

-------------------------------------------------------------------------------------------------------------------------------

Episode 141: Rapid Review Series 13 (IM) ●

● ●

40 yo F, BMI = 40, presents with pain in upper abdomen + fever 102F + positive Murphy’s sign → acute cholecystitis ○ Dx? RUQUS w/ GB wall thickening + pericholecystic fluid ■ If negative, HIDA scan ● Don’t see gallbladder at 90 mins → acute cholecystitis ○ Tx? Lap chole Pt on TPN or in ICU, RUQUS shows characteristic findings but no stones → acalculous cholecystitis ○ Tx? Percutaneous cholecystostomy ■ Do NOT perform cholecystectomy (increased mortality) Pt with stone in cystic duct + dilation of intrahepatic bile ducts → Mirizzi syndrome RUQ pain + fever + AMS + scleral icterus + BP 80/48 → ascending cholangitis ○ Charcot triad = RUQ pain, fever, jaundice ○ Reynolds pentad = Charcot triad + AMS + hypotension ○ Dx? ERCP ○ Tx? ERCP + abx Pt s/p spleen repair for traumatic injury, now with referred pain to L shoulder & fevers → subphrenic abscess 292

● ●

● ● ● ● ●

● ●

○ Tx? Drain abscess Pt recently treated for acute pancreatitis, isolated gastric varices found on imaging or endoscopy → splenic venous thrombosis ○ Weird thromboses: ■ Renal vein thrombosis ● Association: nephrotic syndromes (esp membranous nephropathy) ■ Hepatic vein thrombosis = Budd Chiari syndrome ● Associations: OCPs, polycythemia vera Pt with hx OA presenting with epigastric pain → peptic ulcer disease Alcoholic pt with pain radiating to back → acute pancreatitis ○ Drug that cause pancreatitis: ■ Didanosine ■ Stavudine ■ GLP-1 agonists (e.g. exenatide, liraglutide) ■ DPP4-inhibitors (e.g. sitagliptin) ○ Associations: ■ Alcoholism ■ Gallstones ■ Sickle cell ■ IgG4-related dz ● RP fibrosis ● Pancreas looks like a sausage on imaging Periumbilical pain progressing to RLQ → acute appendicitis Flank pain radiating to groin + hematuria → nephrolithiasis 23 yo F with severe RLQ pain + inconsistent condom use → ectopic pregnancy 15 yo F with 5 cm simple ovarian cyst, now with sudden-onset severe abdominal pain → ovarian torsion ○ Contrast with ruptured cyst: will have free fluid in peritoneal cavity on USMLEs Cervical motion tenderness + vagin*l discharge → PID ○ Fitz-Hugh-Curtis syndrome = involvement of hepatic capsule ○ Bugs? CT & NG ○ Tx? Ceftriaxone + azithro/doxy ○ Don’t give IUD recently after PID episode 80 yo F with LLQ pain + fever → diverticulitis ○ Dx? CT w/ contrast ○ Colonoscopy weeks later to r/o CRC 80 yo F with LLQ pain + pneumaturia or fecal matter in urine → colovesical fistula 2/2 diverticulitis Pt on chemo with thickened cecum → typhlitis ○ Pathophys: severe inflammation of cecum (similar to necrotizing enterocolitis) MAHA + thrombocytopenia + renal failure → Hemolytic uremic syndrome ○ Bugs? O157H7 E. coli or Shigella ○ Platelet count low ○ Bleeding time high ○ PT/PTT normal Hemophilia A/B ○ Platelet count normal ○ Bleeding time normal 293

○ ○ ○ ○

● ●

● ●

PT normal PTT high Inheritance? X-linked recessive Factors? ■ Hemophilia A → factor VIII ■ Hemophilia B → factor IX Von Willebrand disease ○ Platelet count normal ○ Bleeding time prolonged ○ PT normal ○ PTT high ■ Pathophys: factor VIII is bound to vWF in circulation, vWF helps prevent its degradation Risocetin test used to assess 1st step in primary hemostasis (integrity of Gp1b-vWF interaction) ○ Abnormal in VWD & Bernard–Soulier syndrome DIC ○ Platelet count low (consumption) → prolonged bleeding time ○ Coagulation factors being consumed → PT & PTT high Warfarin ○ Platelet count normal ○ Bleeding time normal ○ PT & PTT both high (affects factors 2, 7, 9 10) ○ Reversal? ■ Vit K ■ Four-factor PCC (prothrombin complex concentrate) Heparin ○ Platelet count normal ○ Bleeding time normal ○ PT & PTT both high (affects factors 2 & 10 in common pathway) ○ Reversal? Protamine sulfate Aspirin ○ Platelet count normal ○ Bleeding time prolonged ■ Pathophys: irreversibly inhibits COX → impairs platelet function ○ PT & PTT normal Liver ○ PT & PTT elevated due low factor production Splenomegaly ○ Platelet count low (due to sequestration) 45 yo F with hx HIV at annual visit, with bilateral pedal & periorbital edema on exam, decreased breath sounds + dullness to percussion lower lung fields + UA with 4+ protein + CXR with bilateral blunting of costophrenic angles → transudative effusion 2/2 FSGS ○ Pathophys: loss of protein in urine → decreased oncotic pressure → transudative effusions ○ Causes of transudative effusions ■ CHF ■ Cirrhosis (low albumin) ■ Nephrotic syndromes 294

■ Menetrier disease (protein-losing enteropathy) ■ PE **can cause both Causes of exudative effusions ■ Malignancy ■ PE Light’s criteria ■ Pleural protein/serum protein < 0.5 ■ Pleural LDH/serum LDH < 0.6 ■ Effusion LDH < ⅔ ULN serum LDH

55 yo M s/p type A aortic dissection 2 days ago, has developed progressive dyspnea + pH 7.47 / pCO2 44 / PaO2 58 + CVP 8 + PCWP 14 + BNP 97 + bilateral crackles + CXR with pulmonary edema (“batwing appearance”) → ARDS ○ Pathophys? Increased pulmonary vascular permeability ○ PCWP must be < 18 to make ARDS diagnosis! ■ Indicates that it’s a non-cardiogenic pulmonary edema ○ ARDS diagnostic criteria? ■ PaO2/FiO2 < 200 ■ CXR with bilateral infiltrates ■ PCWP < 18 ○ Tx? Ventilation with high PEEP & low tidal volumes

DIARRHEA EXTRAVAGANZA ● Pt with recent travel to Mexico, now watery diarrhea → ETEC ● Pt with rice-water stools → Vibrio cholerae ○ Tx? ■ Rehydration with OHS (salt & sugar) ■ Macrolide or tetracycline ● Watery diarrhea in hiker/camper, pt with IgA deficiency or Bruton’s agammaglobulinemia → Giardia ● Watery diarrhea on cruise ship → norovirus ● Watery diarrhea in infant → rotavirus ○ Vaccine can’t be given if hx of intussusception, Meckel’s, HSP ● Watery diarrhea in AIDS pt → cryptosporidium ○ Special stain? Acid-fast oocysts ○ Tx? ■ Paromomycin ■ Nitazoxanide ● Bloody diarrhea after beef consumption → EHEC, Shigella, Campylobacter ○ Campylobacter is most common cause of bloody diarrhea in US ○ Campylobacter associated with GBS ■ Dx? Albuminocytologic dissociation in CSF ■ Tx? Plasmapheresis or IVIG ● Diarrhea after abx → C. diff ○ Tx? Oral vancomycin ● Bloody diarrhea + RLQ pain + hx pork consumption → Yersinia enterocolitica ○ “Pseudoappendicitis” ● Bloody diarrhea + liver abscess → Entamoeba histolytica 295

● ● ● ● ●

Tx? ■ Metronidazole ■ Inject iodoquinol into abscess Blood diarrhea + low Hgb + low platelets + elevated Cr → HUS ○ Bugs? Shigella or EHEC ■ Shigella has very low microbe requirements for infection Diarrhea after oyster/seafood consumption → Vibrio parahaemolyticus Pt with liver dz, consumes oysters, now severely ill → Vibrio vulnificus Pt swam in freshwater then died soon thereafter → Naegleria fowleri Diarrhea & vomiting after eating fried rice → Bacillus cereus Vomiting 2 hrs after consuming potato salad → Staph aureus

POISONING ● 44 yo M brings 4 yo son to ED, 4 hr history of profuse sweating + drooling + watery diarrhea, BP 89/58 + HR 33 + T99.1 + RR 8, PE with diffuse wheezing + bilateral pupillary constriction, mother with myasthenia gravis → AChE inhibitor poisoning ○ Similar presentation to organophosphate poisoning ■ Organophosphates inhibit AChE ○ Tx? Atropine (anticholinergic, blocks muscarinic receptors) ○ Other overdose tx ■ Physostigmine (cholinesterase inhibitor) ● Jimson weed (anticholinergic properties) ● atropine overdose ■ Flumazenil ● Benzos overdose ● Zolpidem, zaleplon, eszopiclone ■ Naloxone ● Opioid overdose IRRITABLE BOWEL SYNDROME TX ● IBS will not have any PE or lab abnormalities ● IBS-D treatment ○ Loperamide (synthetic opioid) ○ TCA (anticholinergic activity) ● IBS-C ○ Stool softener ○ Lubiprostone (chloride channel activator → secretory diarrhea)

-------------------------------------------------------------------------------------------------------------------------------

Episode 143: Biostats Review 296

FYI: Here’s a copy of the powerpoint slides found on the Divine Intervention website formatted to a word document. This is NOT the audio transcribed. Q1 A new serum test is created to screen for peripheral arterial disease. The sensitivity of the test is 80%. The most accurate interpretation of this statement is? a. Patients with positive test results have an 80% chance of having the disease. b. In patients with negative test results, 80% do not have the disease. c. In patients who have the disease, 20% will have a negative test result. d. Patients with negative test results have an 80% chance of not having the disease.

Q1 Key -The best answer here is C. -In my experience, answering NBME questions rarely depends solely on doing math. Understanding is the way to go! -Sensitivity essentially answers the Q-Of all the population with a given disease, what % have +ve test results? That’s it! -The other % that you don’t detect that TRULY have disease are the false negatives. The 2nd word is “negative” but the word in front of it is “false” so you know that they are in fact +ve. I use this 2nd-1st word mantra to keep things straight. Highly seNsitive tests have a low fNr.

Q2 A study is done on 1000 patients with a history of glioblastoma (GBM). A new serum test (ST) is done to screen for recurrent GBM. 100 patients have a positive ST test and 900 have a negative ST test. Brain imaging with biopsy is done on all these patients and 30 recurrences of GBM are found. 10 patients with positive ST tests have GBM and 20 patients with negative ST tests have GBM. Which of the following best represents the sensitivity of ST tests? a. 92% b. 35% c. 75% d. 50%

Q2 Key -The best answer here is B. The sensitivity is 33%. This is the closest # to 35%. The NBME occasionally plays this trick where inexact answers are posted. When this occurs, pick the answer that is closest to your math. -The Q here sounds nebulous but simple math based on understanding will save the day. -Sensitivity essentially answers the Q-Of all the population with a given disease, what % have +ve test results? 297

-The total diseased population is 30 people. The # with +ve test results were 10. So sensitivity = 10/30 = 33%. You’re welcome to test a 2 by 2 table. Q3 A new serum test for glioblastoma (GBM) has a specificity of 90%. The most accurate interpretation of this statement is? a. 90% of patients with GBM have positive test results. b. 10% of patients with GBM are missed by this test. c. 10% of patients without GBM have positive test results. d. 90% of patients without GBM have positive test results.

Q3 Key -The best answer here is C. -Again, simple math + understanding = clutch on this Q. -Specificity essentially answers the Q-Of all the population without a given disease, what % have -ve test results? That’s it! -The specificity of this test is 90%. So of the people w/o GBM, 90% test -ve. So 10% that should have tested -ve, ultimately end up testing +ve (aka false +ve’s). -A highly sPecific test has a low fPr. Sidebar 1-SPin and SNout principle -If a test is highly sensitive, people with disease should have a +ve test result. -If the test is -ve, then disease should be absent (aka a low FNR). A -ve test should rule OUT disease. -If a test is highly specific, people w/o disease should have a -ve test result. -If the test is +ve, then disease should be present (aka a low FPR). A +ve test should rule IN disease. Sidebar 2-Screening and Confirmatory Tests -In tests with high sensitivity, people with disease should have +ve test results. -High sensitivity tests make good “screening” tests so you don’t inadvertently miss out on people with disease. For example, you’d hate to miss out on people with HIV. This is why you use the ELISA test. -In tests with high specificity, people w/o disease should have -ve test results. -High specificity tests make good “confirmatory” tests so you don’t inadvertently label people w/o disease as having a disease. Tests that are highly specific are very good at labeling people w/o disease so if the test is +ve (and by definition, high specificity tests have a low FPR), you very likely have disease. This is why Western Blots are undertaken after a +ve ELISA so you don’t tell a patient they have HIV based on a +ve ELISA when they don’t! -Note however, that the WB is no longer done in most places as a confirmatory test.

Q4 Which of the following points best represents the region of the graph with the highest positive predictive value (PPV) for the detection of Type 2 Diabetes Mellitus (T2DM)?

298

Q4 Key -The best answer here is C. -These Q’s have a high tendency to be annoying. To beat them, remember the following; The highest PPV region on a graph, corresponds to the region with the highest sPecificity, which corresponds to the region that DOES NOT miss anyone w/o disease. If you remember this, you’re golden. -Said another way, the highest PPV is achieved if the test when +ve, only includes people that have the disease. -PPV simply means the % of people with +ve tests who have disease. Sidebar-Do not mix this up! Sensitivity of a test represents the % of people with disease who have +ve test results. PPV of a test represents the % of people with +ve test results who have disease. DO NOT MIX THIS UP! If you switch the words before and after “who have”, you should be able to keep things straight. Learn one side and remember that the other one is the other one. Q5 Which of the following points best represents the region of the graph with the highest negative predictive value (NPV) for the detection of Type 2 Diabetes Mellitus (T2DM)?

Q5 Key 299

-The best answer here is B. -These Q’s have a high tendency to be annoying. To beat them, remember the following; The highest NPV region on a graph, corresponds to the region with the highest seNsitivity, which corresponds to the region that DOES NOT miss anyone with disease. If you remember this, you’re golden. -Said another way, the highest NPV is achieved if the test when -ve, only includes people that don’t have the disease. -NPV simply means the % of people with -ve tests who don’t have disease. Sidebar-Do not mix this up! Specificity of a test represents the % of people w/o disease who have -ve test results. NPV of a test represents the % of people with -ve test results who don’t have disease. DO NOT MIX THIS UP! If you switch the words before and after “who have”, you should be able to keep things straight. Learn one side and remember that the other one is the other one. Q6 A clinical trial is conducted to measure the effectiveness of the IM test as a screening tool for the detection of testicular cancer. 500 IM tests are obtained. 20 men have positive IM tests and are found by testicular biopsy to have testicular cancer. 180 men have positive IM tests and are negative for testicular cancer by biopsy. 290 men have negative IM tests and are negative for testicular cancer by biopsy. 10 men have negative IM tests and are found to be testicular cancer positive by biopsy. What is the NPV of this test for the detection of testicular cancer? a. 97% b. 10% c. 33% d. 40% e. 90% Q6 Key -The best answer here is A. No need to panic on these questions with tons of numbers. Simply define the qty that is being tested AND then abstract the #s you need. Many times the #s given are not useful. -NPV of a test represents the % of people with -ve test results who don’t have disease. -There are 300 people with -ve IM test results. Of these people, 290 DO NOT have testicular cancer. So the NPV is basically 290/300 which is 97%.

Q7 If the cutoff for a positive IM test result for the detection of testicular cancer (TC) is 5, which of the following best represents the outcome of adjusting the test cutoff value to 1? a. PPV would increase but NPV would decrease. b. Specificity would decrease but sensitivity would increase. c. PPV and NPV would both increase. d. Sensitivity and specificity would both increase.

Q7 Key -The best answer here is B. 300

-The name of the game with biostats Q’s is to first define what is being tested (doing your analysis first) before picking out an answer. When you look at the answers first, your mind is swayed in -ve directions. -The prior cutoff is 5 (above 5, you have TC). If you bring it down to 1, you vastly increase your chances of catching every single person with TC. In other words, you don’t miss anyone. -This increases the sensitivity of a test. Whenever seNsitivity goes up, Npv goes up. sPecificity and Ppv also go in the same direction.

Q8 A medical student at Johns Hopkins invents a drug that improves survival in patients with Glioblastoma Multiforme (GBM) by 7 years. Which of the following changes would be seen a few years after drug FDA approval? a. The sensitivity of screening tests for detecting GBM would decrease. b. The prevalence of GBM would increase in the population. c. The PPV of GBM detection tests would decrease. d. The incidence of GBM would increase in the population. e. The specificity of screening tests for detecting GBM would increase. f. The NPV of GBM detection tests would increase

Q8 Key -The best answer here is B. -By having this awesome Hopkins invented drug, we would keep more people who have already been diagnosed with GBM alive, which is great, so the # of people with GBM in the population would increase. -Therefore, prevalence increases. As Prevalence goes up, Ppv should increase, hence C is wrong. NPV would decrease, so F is wrong (look at next slide). -Changes in prevalence do nothing to test sensitivity and specificity so A and E are wrong. The only things that change these qties are changes in the actual test (like modifying the cutoff values). -We will likely still be diagnosing GBM at the same rate, so incidence stays the same.

Sidebar 1-Why does PPV increase with prevalence? Think of this, if a person comes to the ED in December with fevers, rhinorrhea, and myalgias, they likely have the flu. If you got a -ve flu swab result, would you believe this? The prevalence of the flu goes up in December so NPV goes down, but PPV goes up. You are less likely to believe the results of a -ve test during this “high prevalence” period. Stated another way, you are a lot more likely to believe the results of a +ve test if the disease is common! Sidebar 2-Incidence vs Prevalence -Incidence represents the # of new cases of a disease that have been diagnosed within a specific time period. -Prevalence is the # of people that are alive AT a given time period. Q9 An M2 (2nd year med student) researcher at The Gifted Medical Students Institute plans to study the effects of consuming high amounts of kale on the development of pheochromocytoma. He plans to publish the results of 301

his study prior to graduation. Which of the following study designs presents the most appropriate means of completing the study? a. Randomized control trial. b. Prospective cohort study. c. Crossover study. d. Case-control study. e. Case report.

Q9 Key -The best answer here is D. -The phenomenon the researcher is trying to measure here is exceedingly rare and he has a limited time frame. -Approaching this by way of a prospective cohort study/RCT would literally take as much time as a 60+ year medical career. -To study rare phenomena, case-control studies are typically the best option on NBME exams. -Results generated from the CCS can then be used to formulate research Q’s that can be examined in a cohort study/RCT.

Sidebar-Case-Control Studies -In a CCS, you need 2 groups of people with similar characteristics. -Group 1 have the disease in Q (pheo), Group 2 do not have the disease in Q (no pheo). -You then ask about exposures they may have had back in the day. You should already imagine that recall bias may be a prominent issue with CCS. -It is HY to know that CCSs give rise to data pertaining to odds ratios. Q10 A professor and 2 medical students undertake a case control study over the course of a year and publish their results in a high impact journal. Which of the following best represents an example of a possible conclusion from their study? a. Duloxetine decreases pain scores in patients with fibromyalgia. b. A combination of Sofosbuvir and Ledipasvir cures hepatitis C with high fidelity. c. Asbestos exposure causes mesothelioma. d. Ursodiol administration improves survival in patients with primary biliary cholangitis.

Q10 Key -The best answer here is C. -In option C, the researchers essentially looked at people with mesothelioma and compared them to people w/o mesothelioma. They likely determined that a good # of people with mesothelioma had prior exposure to asbestos. -Option A, B, and D are wrong because they involve “interventions” which are things you’d ordinarily do in a RCT.

302

-As is evident with this Q, you can’t just memorize facts and do well on these USMLE exams. You actually need to understand concepts. This is the central principle behind doing well regardless of Q difficulty on these exams. -CCS/Cohort studies deal with exposures, RCTs deal with interventions. DETOUR

Q11 The average normal CD4 count is 1000 per mm3 of blood with a standard deviation of 100/mm3. Which of the following best represents the normal percentage of individuals who would be measured to have a CD4 count > 1200/mm3 of blood? a. 2.51% b. 95% c. 5% d. 16% e. 68.2%

Q11 Key -The best answer here is A. -The key principle to realize here is that 95% of the population will fall within 2 SDs (2*100 = 200) of the mean-from 800-1200. -So 5% must fall “outside” this range on “either side”. Either side here means < 800 or > 1200. -Therefore, half of this 5% must have a CD4 count that is < 800/mm3 and the other half must have a CD4 count that is > 1200/mm3. -So the best answer is 2.51%. Make sure you know this for the USMLEs!

Sidebar-P Values (Statistical Significance) -P values are used to express the probability that the results of a study occur from chance events. -The lower the number, the more confident we are in the results of that test. In other words, a P value of 0.05 (5% probability of obtaining results by chance or 1 in 20) is worse than a P value of 0.01 (1% probability of obtaining results by chance or 1 in 100). -Unless you’re told otherwise, use a P value of 0.05 in every NBME question.

Q12 4 separate drug trials are conducted to test the relative effectiveness of 4 different 3-beta hydroxysteroid dehydrogenase agonists in raising libido. The mean “libido” levels in the study (with confidence intervals) are graphed below. Which of the following statements are true? Q12 contd. (multiple answers may be correct) a. Drug 1 is more effective than Drug 2. b. Drugs 3 and 4 are similar in effectiveness. c. Drug 4 is more effective than Drug 2. d. Drugs 1 and 4 show similar effectiveness. 303

Q12 Key -Statements A, B, and D are all true. -The general principle is that when 2 confidence intervals cross each other (lines overlap), there is no difference b/w those treatments. -These scenarios are unfortunately very common on the USMLEs. -Another critical way this can be tested is to give you confidence intervals (CI) of epidemiological quantities that are ratios or differences; A ratio driven qty (like relative risk) will have non-significant results if the CI crosses 1. A difference driven qty (like absolute risk reduction) will have non-significant results if the CI crosses 0. Why???

Q13 A study is done to assess the relationship between vaping in college and the future need for lung transplant. The study yielded a relative risk of 3.5 with a p value < 0.05. Which of the following represents a possible 95% confidence interval from this study? a. 0.5-3.5 b. 2-4.5 c. 3.5-6.0 d. 3.9-7.1 e. 0.71-3.68

Q13 Key -The best answer here is B. -A and E are wrong b/c the CI includes 1 but this study is measuring a relative risk (which is a ratio), so you cannot have significant results and have the CI cross 1. -A and C are wrong b/c the RR derived from the study either begins or ends the CI. This is not possible. Results obtained from a study have to be WITHIN the CI, they cannot BEGIN or END the CI. -D is wrong b/c it does not include the value obtained from the study. -Pls be absolutely sure you understand this.

Q14 A study is done to assess the effectiveness of a new drug (D) for the treatment of GBM. All patients enrolled in the study received the current standard of care (SOC). In addition to receiving SOC, Group A received drug D; Group B received SOC and a sham drug (Y). Of the 40 patients receiving D, 8 die over the course of the study. Of the 40 patients receiving Y, 20 die over the course of the study. What is the NNT for drug D? a. 2.7 b. 3.3 c. 13.3 d. 5.0 e. 15.5

Q14 Key 304

-The best answer here is B. -To calculate the NNT, you need to find the difference in risk b/w patients exposed to D and the patients exposed to Y (placebo). You then divide the answer obtained into 1. That’s it! -Stated another way, NNT is 1/Absolute Risk Reduction. -40 people got D, 8 died (20%). 40 people got Y, 20 died (50%). The difference here is 30% (or 0.3). -Dividing this into 1 gives (1/0.3) which yields 3.3. -The NNH is a qty that has a similar calculation but follows the mantra that the rate of harm in the “exposed/treatment” group exceeds that in the placebo group. -To make things even easier (and only remember 1 formula), take 1/the difference in risk b/w any 2 groups given. Just always write the higher risk # first in the difference.

Sidebar-Relative Risk -To calculate relative risk, take the risk in the exposed population and divide it by the risk in the unexposed population. -For example, if a cohort study comparing smokers and non-smokers is done with 100 of 500 people in the smoking group developing lung cancer and only 50 of 500 people in the non-smoking group developing lung cancer. The RR is 20%/10% (risk of LC in smokers/risk of LC in non-smokers) which is 2. -The smokers have a 2-fold increased risk of LC compared to nonsmokers.

Q15 If the presence of dysmorphic erythrocytes in the urine has a sensitivity of 90% and a specificity of 45% for the detection of IgA nephropathy, what is the likelihood ratio of having IgA nephropathy if the patient has dysmorphic erythrocytes detected on urinalysis? a. 1.35 b. 0.45 c. 4.55 d. 2.33 e. 1.67

Q15 Key -The best answer here is E. -Likelihood ratios occasionally pop up on the USMLEs. The classic worry of many students is to decipher when to use the +ve LR formula (Sensitivity/1-specificity) vs the -ve LR formula (1sensitivity/specificity). Here’s the rule; If the patient has a +ve test result, use the +ve LR formula. If the patient has a -ve test result, use the -ve LR formula. In this Q, we need to use the +ve LR (0.9/1-0.45) = 1.67

305

Sidebar-Likelihood Ratios -When calculated, +ve LRs tell you how much more likely a phenomenon is given a +ve test result. -When calculated, -ve LRs tell you how much less likely a phenomenon is given a -ve test result. Q16 In a study examining the relationship b/w exposure to ketamine and the subsequent development of neutropenia, medical records of 300 children were reviewed. 100 children who were exposed to ketamine were found to have neutropenia, 50 children who were exposed to ketamine were found to not have neutropenia, 80 children who were not exposed to ketamine were found to not have neutropenia, and 70 children who were not exposed to ketamine were found to have neutropenia. What is the odds ratio for this study? a. 3.29 b. 2.29 c. 5.67 d. 2.23 e. 7.16

Q16 Key -The best answer here is B. -Odds ratios compare the odds of a person with disease being exposed to a risk factor (RF) to the odds of controls being exposed to the same RF. -To calculate OR, take the logical people product (LGP)/weird people product (WPP). -LGP = (exposed and affected, unexposed and unaffected)/WPP(exposed and unaffected, unexposed and affected). -In this case our OR = 100 * 80/70*50 = 8000/3500 = 2.29.

Q17 The mean blood glucose level of a group of 81 medical students was 170 mg/dL with a SD of 15 mg/dL. Calculate the 95% CI and in words interpret your results.

Q17 Key Mean = 170 mg/dL. Std error of the mean = 15/sq.rt of 81 = 1.67 mg/dL. Z-score for the 95% CI = 2 (1.96 is more accurate but doesn’t matter). Therefore, CI = 170 +/- (2*1.67) = 170 +/- 3.34 = 166.66-173.34 You can say with 95% confidence that the real mean BP of the medical student population falls between 166.66 and 173.34 mg/dL. Alternatively, you can say that the mean BP of any randomly selected group of 81 medical students will fall b/w 166.66 and 173.34 mg/dL 95% of the time if the same experiment is repeated on multiple occasions. HY to know the calculation and the interpretation in words!

306

Other HY Concepts -For ROC curves, the best test (highest combined sensitivity and specificity) lies at the upper left corner of the graph. -Cohort studies essentially involve looking at 2 groups of people with differential exposures and following them into the future for the development of some outcome. They could be prospective or retrospective. -68%, 95%, and 99.7% of a normal population lie b/w 1, 2, and 3 SDs of the mean respectively. Other HY Concepts contd. -To compare means of 2 groups, use the T test. For > 2 groups, use the ANOVA (or F) test. -When you incorrectly reject the null, you are committing a Type 1 error (alpha error). When you incorrectly accept the null, you’re committing a Type 2 error (beta error). Remember that power = 1beta. -Tighter CIs tell you that a study is more precise. However, you should be a lot less confident in the results of the study as the CIs are too narrow (less room for error). Other HY Concepts contd-Increasing power To increase the power of a study; -Recruit more people for a study (more closely approximates the population). -Have a large difference b/w 2 qties you’re trying to measure (aka larger effect size). The power of a study comparing people with test scores of 99 and 100 as a means of comparing intelligence has less power than one comparing test scores of 25 and 100. -Have a lot of your data for a measured qty cluster around 1 value. Increasing the precision of your measurements also increases the power of a study. -Stated another way, a study that uses a P value of 0.01 has more power than one using a P value of 0.05. Other HY Concepts contd. -The fact that something is statistically significant does not mean that it is clinically significant. A BP drug that lowers BP by 1 mm Hg from baseline even at a p value < 0.01 is a useless drug. -Mean is the average. Median represents the middle # (if you have an odd # set of data) OR the mean of the 2 middle #s (if you have an odd # set of data). Mode represents the most frequent qty in the data set. Arrange these in order before making these determinations. The mean is affected by extreme values.

Other HY Concepts contd. -For a normal distribution, mean = median = mode Remember that mean precedes median which precedes mode when taken in alphabetical order. -This should help you remember that in a -vely skewed curve (flat portion at left), mean < median < mode. -In a +vely skewed curve (flat portion at right), mean > median > mode. -HY bimodal distributions to be aware of include the distro for Hodgkin’s lymphoma and slow/fast acetylators in the metabolism of some drugs. Other HY Concepts contd. -Lead time bias involves erroneously thinking that survival has been improved when in fact the “apparent survival improval” arose primarily from detecting a disease early.

307

Cross checked: No -------------------------------------------------------------------------------------------------------------------------------

Episode 145: Rapid Review, Series 14, Surgery ●

● ●

● ●

● ● ● ● ● ●

Kid w/mild RLQ pain guaiac positive stool → Meckel’s ○ Failure of obliteration of the vitelline duct/omphalomesenteric duct ○ Tech-99/Pertechnetate scan for diagnosis ○ Gastric mucosa secretes acid that erodes the GI tract and causes bleeding ■ Not in most cases, just symptomatic cases ○ Treated with resection Pt w/ hx of Crohn’s has n/v and no bowel mvmts → small bowel obstruction ○ Obstructed d/t strictures (GERD can also cause strictures) Pt w/ hx of UC has severe abdominal pain, distended, transverse colon → toxic megacolon ○ Next best step = ex lap ○ Other things that can cause toxic megacolon: c.diff, chagas ○ Screening colonoscopies 8 yrs after diagnosis and every 1-2 yrs after ○ PSC and UC diagnosed at the same time → screening colonoscopy at time of diagnosis Pt w/ 3 mos of fatigue, low mood, bradycardic, carpal tunnel → hashimoto’s thyroiditis ○ Most common cause of hypothyroid in US: hashimoto’s w/ anti TPO abs ○ Most common cause in newborn: thyroid dysgenesis ○ Radioactive iodine uptake: minimal to none ○ Other causes of hypothyroid: ■ de Quervain's thyroiditis (painful, tender thyroid, hx of recent viral URI, decreased radioactive uptake) (can also cause hyperthyroid in early thyrotoxic phase) ■ Factitious hyperthyroid → low TSH, low radioactive uptake, low thyroglobulin ○ Grave’s → autoab’s against TSH receptor, diffusely increased uptake on radioactive scan ○ Struma ovarii → elevated T3/T4, low TSH, decreased uptake on radioactive scan ○ Toxic adenoma → only 1 hotspot on radioactive scan ○ Multinodular goiter → multiple hotspots interspersed w/cold spots ○ Papillary thyroid cancer is popular aka most common ■ Psammoma bodies and orphan Annie eye nuclei ○ Follicular thyroid cancer spread hematogenously 80 yo pt w/profound wt loss and enlarging thyroid → anaplastic thyroid cancer Pt had multiple kidney stones, recurrent ab pain, enlarging thyroid → medullary thyroid cancer ○ MEN2A- hypercalcemia ○ Calcitonin tumor marker → can cause prolonged QT ○ Apple green birefringence on congo red b/c it becomes amyloid Rock hard thyroid gland → Reidel’s thyroiditis (Fibrosis of the thyroid) ○ Associated w/ IgG4 related dz → Autoimmune pancreatitis (sausage shape), retroperitoneal fibrosis (BL hydronephrosis), autoimmune gallbladder dz Psammoma bodies: papillary thyroid cancer, meningiomas (parasagittal, dural tail), mesothelioma (ferruginous bodies) Pt w/ n/v, coffee bean sign w/head toward RUQ → sigmoid volvulus ○ Coffee bean sign w/head toward LUQ → cecal volvulus Pt w/ heavy smoker, wt loss, dullness to percussion, decreased breath sounds, larger pleural effusion, coin lesion ○ Next best step = thoracentesis w/cytology (Malignant cells → stage 4 cancer) #1 cause of lung cancer → adenocarcinoma Squamous cell carcinoma → keratin pearls, hypercalcemia (PTHrP) Small cell lung cancer → SIADH (hypoosmolar serum, hyperosmolar urine), Lambert Eaton (Ab against presynaptic Ca channels → proximal muscle weakness improves w/use), and Cushing’s (ACTH does not suppress w/dexamethasone) ○ Neuroendocrine origin, already metastatic at diagnosis Lung cancer complications: SVC Syndrome (next best step is radiation), pancoast tumor (Horner’s syndrome), hypertrophic pulmonary osteopathy (get imaging)

308

● ●

● ● ●

Partial Horner’s syndrome (ptosis and miosis) → cluster headache Xanthochromia → subarachnoid hemorrhage ○ Worth HA of pt life, often mimics meningitis but is sudden onset ○ Next best step = non contrast head CT → LP ○ Give nimodipine to prevent superimposed ischemic stroke Herpes encephalitis/meningitis → hyperintense signalling in temporal regions on MRI ○ RBCs in CSF → cover w/IV acyclovir (only give IV if admitting pt) 6 wk period of memory, myoclonus, and proprioception loss → CJD ○ Elevated protein 14.3 3 Narcolepsy → low levels of orexin/hypocretin in CSF ○ Diagnose w/ sleep study, treat w/scheduled naps, stimulants (modafinil), sodium oxybete (for cataplexy)

These are my personal notes from when I originally listened to the podcast. I apologize for anything I overlooked or any mistakes! Cross Checked: No -------------------------------------------------------------------------------------------------------------------------------

Episode 153: Rapid Review, Series 15, Surgery ● Long term smoker in ED b/c on increased urine, ab pain, low BP → squamous cell cancer ○ PTHrP → hypercalcemia causes stones, bones, groans, psychiatric overtones ○ First step = give IV normal saline then loop diuretic to decrease Ca, can also give calcitonin (tones down calcium) ○ Hypercalcemia causes short QT interval ● small cell lung cancer: euvolemic hypoNa, SIADH, ACTH, hypercortisolism not suppressed with high dose dexamethasone. ● smoker for long time, trouble standing from chair, proximal muscle weakness that gets better with repetitive nerve stimulation – lambert Eaton ○ autoantibodies against presynaptic VG Ca channels ● lung cancer and pleural effusion thoracentesis and if malignant cells found, drain fluid or pleurodesis. bad sign :/ ● Polymyositis and dermatomyositis can be paraneoplastic syndromes for lung cancer ● smoker and 4-5 weeks, severe arthritis in LE hypertrophic pulmonary osteoarthropathy. no tx, treat cancer ● loss of sensation below nipple line (T4) prostate cancer with mets ● How to diagnose prostate cancer bone mets → ○ Gets there by Batson’s plexus ○ If causing spinal cord compression → high dose dexamethasone and MRI ● Pt w/hx of lupus and 7 days of nosebleeds and heavy menstrual bleeding → ITP ○ Can be idiopathic but is often d/t autoimmune hx ○ Abs against Gp2b3 (low platelets, high megakaryocytes) ○ Mild → no treatment Severe → steroids, IVIG, rituximab, splenectomy (cure) ● Deficiency of Gp2b3 → Glanzman’s ○ Normal ristocetin assay 309

● Bernard-soulier → Gp139, increased bleeding time, normal PTT ○ Abnormal ristocetin assay ● VWD → increased bleeding time and PTT (less production of Factor 8) ○ Abnormal ristocetin assay ● Esophageal rupture ○ few hours ago pt got pneumatic dilation for achalasia OR pt came in with dysphagia, losing weight, so EGD with biopsy was done, now is sick, hypotensive. ○ Next best step = gastrografin enema aka water soluble contrast enema ● Pt recently started dialysis and now is volume overloaded, SOB → high output HF ○ Mechanism → new AV fistula decreases time in capillaries and venules ● Peritoneal dialysis complications → SBP (ab pain, low grade fever) ○ Next best step = paracentesis (>250 neutrophils → start 3rd gen ceph) ● ESRD patient on dialysis uremia, coagulopathy. Tx desmopressin ● Other causes of high output HF: paget dz, severe anemia, trauma AV fistula (pulsatile mass at region of stab wound), hereditary hemorrhagic telangiectasia, osler-weber-rendu dz ● Pt had recent ERCP w/biopsy and now has severe ab pain with high lipase → acute pancreatitis ○ Tx: NPO, IV fluids, pain management (opioids) ○ Can also have hypocalcemia 2/2 saponification (prolonged QT) ● Pt had parathyroidectomy then has seizures and spasms → hypocalcemia ○ Tx: IV calcium gluconate (also used for symptomatic hyperkalemia and hypermagnesemia) ○ PS. if it’s a kid consider DiGeorge syndrome (murmur, seizures, 34rd/4th pharyngeal pouch not developed, no thymus → fungal, bacterial infections ○ Infants of diabetic mothers (seizures due to hypocalcemia or hypoglycemia) ○ Hypocalcemia also seen in rhabdo ● Transplant rejection: ○ Hyperacute → rejection in OR, Ab mediated ○ Acute → rejecting days to weeks later, MHC2 to CD4 T cells ○ Chronic → years later ● Woman with bulge in inguinal canal → femoral hernia ● Hx of MEN1 w/ potassium of 2.5 and diarrhea → VIPoma (watery diarrhea, hypokalemia, achlorhydria) ○ Pancreatic tumors can be neuroendocrine (gastrinoma = jejunal ulcers, ZES lots of acid) ○ Whipple triad → symptomatic hypoglycemia relieved w/glucose administration (insulinoma – high insulin, high C peptide levels) ○ new onset diabetes, new rash = necrolytic migratory erythema from glucagonoma ● Insulin and c peptide are elevated with sulfonylurea use ○ Differentiated using secretagogue screen – positive with overdose of sulfonylurea use ○ Same is true for meglitinides b/c they both block K dependent insulin channels ○ PS Don’t give B-blockers to diabetics b/c it masks symptoms of hypoglycemia

310

These are my personal notes from when I originally listened to the podcast. I apologize for anything I overlooked or any mistakes! Cross Checked: YES (added missed text in different font) -------------------------------------------------------------------------------------------------------------------------------

Episode 156: Rapid Review, Series 16, OB/GYN ●

● ● ● ● ●

● ● ● ●

● ●

52 yo woman with breast mass → mammogram ○ 50 yo (give aromatase inhibitor ex. anastrozole) ○ If not in menopause aromatase inhibitor will start menopause Her2 positive and triple negative breast cancers have poor prognosis ○ Prior to starting trastuzumab get an echo! Suspect metastatic breast cancer to bone then next step = bone scan (sensitive but not specific) LCIS tx = lumpectomy with radiation (equivalent of mastectomy) ○ Increases risk of local recurrence of cancer ○ Cannot repeat lumpectomy if it recurs d/t fibrosis → mastectomy Pt had radiation for breast cancer, rapidly growing neck mass, cervical lymphadenopathy → papillary thyroid cancer Positive sentinel lymph node biopsy → proceed to axillary node dissection ○ At increased risk for lymphangiosarcoma

Cross checked: No -------------------------------------------------------------------------------------------------------------------------------

Episode 158: Rapid Review, Series 17, Surgery ●

Most likely complication of axillary lymph node dissection → lymphedema ○ Now the limbs are large (usually upper limb) and they’re losing wt → lymphangiosarcoma ○ Most common cause of lymphedema → filariasis ○ Congenital lymphedema is found in Turner syndrome (cystic hygroma) ○ Most common cause of lymphedema in US → lymph node dissection/removal Pt w/leg pain that gets worse by the end of the day w/ulcer on medial malleolus → varicose veins ○ Risk factors: prolonged standing, combined OCPs, pregnancy, obesity ○ Stasis dermatitis and painless ulcers above medial malleolus ○ First step in management = compression stockings ○ Refractory therapy = sclerotherapy

311

● ● ●

■ Diagnostic test prior to sx → venous doppler ultrasound of lower extremity ○ Arises from incompetent valves Mini NBME Pathophysiology Buzzwords Review ○ Chronic venous insufficiency: incompetence of the valves in the lower extremity ○ Aortic stenosis: calcification of valves ○ Mitral Valve Prolapse: myxedematous degeneration ○ Aortic Dissection: cystic medial necrosis ○ Duodenal atresia: failure of recanalization ○ Jejunal atresia: vascular assault in utero Pt loses vision for 5 min and it comes back → amaurosis fugax ○ Next best step dx = carotid duplex ultrasound (Embolus usually arises from carotids) ○ Next best step tx = antiplatelet agent (e.g., clopidogrel) Who gets an anticoagulant if they have carotid artery disease as well? → A. Fib When to do carotid endarterectomy → >70% stenosis and symptomatic (never acute!) Pt w/severe chest pain and large pleural effusion, hx of uncontrolled HTN → aortic dissection ○ Dx: CT angio or TTE (TransThoracic Echocardiogram) if unstable ○ Type A → involves the ascending aorta ■ Tx: B-blocker, nitroprusside, surgery ○ Type B → limited to descending aorta ■ Tx: B-blocker (first line), nitroprusside (consider, but not first line) ○ Cause is cystic medial degeneration Genetic dz w/aortic dissection → Marfan ○ Mutation in chr 15 fibrillin dz, AD inheritance ○ Could also be Ehlers Danlos but that’s less common AAA ○ Male >65-74 yo with ANY hx of smoking → get 1x screening ab ultrasound (or CT) ○ Male >50 yo with any fam hx of AAA → get 1x screening ab ultrasound (or CT) ○ Most commonly located in the infrarenal aorta ○ Draped aorta sign → aorta is hanging around the posterior vertebrae ○ Tx: open surgical vs endovascular repair ■ Complications of endovascular repair → ● endoleak (leakage around the graft) ● paraplegia b/c of artery of adamkiewicz ● fatigue/heme pos stools/microcytic anemia: aortoenteric fistula

These are my personal notes from when I originally listened to the podcast. I apologize for anything I overlooked or any mistakes! Cross checked: YES -------------------------------------------------------------------------------------------------------------------------------

Episode 159: Rapid Review Series 18 (IM) ●

65 yo M with 2 weeks of hematuria + recent 20 lb weight loss. Worked as a plumber. PMH of EGPA. Nonsmoker. UA with hematuria, no dysmorphic RBCs. → Bladder cancer ○ RF in this case? Meds (likely cyclophosphamide for EGPA) ○ Cyclophosphamide ■ Adverse effects? Hemorrhagic cystitis, bladder cancer ■ Prevention? Mesna ○ Dx? Cystoscopy w/ bx 312

RF for bladder cancer ■ Smoking ■ Schistosoma haematobium ■ Aniline dyes Flank mass + hematuria + new-onset varicocele in longtime smoker → RCC ○ Most common type? Clear cell ○ Dx? Do NOT biopsy ○ Tx? Nephrectomy ○ Paraneoplastic syndrome? Polycythemia 2/2 epo production

23 yo F, roommates note behavior change, working on multiple projects to cure poverty, sleeps less than 2 hrs but lots of energy, med student can’t get word in during interview, urine tox is negative → bipolar disorder ○ Tx? ■ Lithium ● Ebstein’s anomaly ■ Valproic acid ● Liver toxicity ● Teratogen

ACE-I contraindications ○ Bilateral renal artery stenosis ■ Already low GFR ■ Pathophys? ATII constricts efferent arteriole. ACE-I → low ATII → efferent arteriole constricts → possible renal failure ○ Hereditary angioedema ■ Pathophys? C1 esterase inhibitor deficiency. C1 esterase and ACE both breakdown bradykinin. Block ACE → can’t break down bradykinin ○ Pregnancy

50 yo F with large breast mass, nipple retraction, skin changes. Hx of depression on paroxetine, currently well-controlled. Says that she is not interested in knowing diagnosis. NBS? Ask pt why she doesn’t want to know diagnosis ○ Always explore the patient’s mental model! 45 yo F Crohn’s on etanercept. Comes to ED with dyspnea + nonproductive cough + fever + 12 days malaise. Na 131. PaO2 = 57. CXR with “diffuse ground glass interstitial infiltrates.” PPD 3 years ago was negative. No sick contacts. → PCP pneumonia ○ Many brain or lung pathologies can cause SIADH ○ Labs? Elevated serum LDH ○ Dx? BAL w/ silver-stain positive organisms ○ RF? ■ HIV ■ TNF inhibitors ○ Before you start a TNF inhibitor, screen for what? ■ TB ■ Hep B ○ Contrast with Strep pneumo → more rapid onset, severe sxs, lobar consolidation ○ Contrast Legionella → pneumonia + diarrhea/abdominal pain + hyponatremia + exposure to water source (e.g. hotel, fountain, air conditioning) → Legionella 313

○ ●

● ● ●

Contrast with TB → fever + night sweats + hemoptysis + cavitary lesion (not interstitial infiltrates)

33 yo F with fatigue + loss of interest + 10 lb weight gain + bilateral LE edema + missed last 2 periods → Hashimoto’s thyroiditis ○ Dx? TSH ○ Ab? anti-TPO ○ Histology? Lymphocytic infiltrate of thyroid gland ○ Associations? Other immune diseases ■ Vitiligo ■ Pernicious anemia ○ Complications ■ Initial thyrotoxic phase ● Pathophys: release of preformed thyroid hormone ■ Hx Hashimoto's + rapidly enlarging thyroid → thyroid lymphoma ○ Why the missed periods? ■ Low T3/T4 → TRH release → TSH & prolactin release → suppresses HPG axis Low TSH + low T3/T4 → secondary hypothyroidism ○ Causes? ■ Craniopharyngioma ● Derived from? Rathke’s pouch ■ Sheehan’s syndrome Painful thyroid + hx viral URI → De Quervain’s subacute thyroiditis ○ Hypothyroid or hyperthyroid (2/2 release of preformed hormone) ○ RAIU? Low uptake (not producing thyroid hormone) ICU pt + mildly low T3/T4 + mildly low TSH → euthyroid sick syndrome High estrogen → increases TBG → higher total T4 but free T3 normal → clinically euthyroid Thyroid cancer ○ Labs? Elevated TSH (thyroid cancers tend by hypofunctional) ○ Dx? US with biopsy ○ #1 RF? Radiation to head & neck ○ Papillary ■ Most common type ■ Spread? lymphatic ■ Histology? Psammoma bodies + Orphan Annie eyes ○ Follicular ■ Spread? Hematogenous ○ Medullary ■ Tumor marker? Calcitonin ● Calcitonin → hypocalcemia → prolonged QT ■ Association? MEN2A/2B ● Mutation? RET gene ● Inheritance? AD ● Prevention? Prophylactic thyroidectomy (risk = 100%) ■ Histology? Apple green birefringence on congo red stain

-------------------------------------------------------------------------------------------------------------------------------

314

Episode 161: The Clutch Antibody Episode 161 notes were graciously provided by Divine Intervention from an anonymous contributor.

Sx

Dx

Specific

Morning stiffness, better w exercise, ulnar deviation

Rheumatoid arthritis

Anti-RF (sensitive), Anti-CCP (specific)

Malar rash

Lupus

ANA (sensitive), Anti-smith, anti-dsDNA (specific)

Autoimmune hemolytic anemia

Lupus

Pregnant woman w Antiphospholipid size less than dates, antibody syndrome recurrent pregnancy losses

Anti-phospholipid, anticardiolipin, lupus anticoag, anti-beta2-glycoprotein

Marital problems d/t pain w sexual intercourse, dry mouth

Sjogren's

Anti-ro (SSA), anti-la (SSB)

Wegner's

c-ANCA

Ab can cause 3rd degree heart block by crossing placenta

Ab can cause 3rd degree heart block by crossing placenta!

Ulcerative colitis, PSC, p-ANCA polyarteritis nodosa, Churg-Strauss/EGPA, microscopic polyangiitis Prox muscle, shoulder weakness; elevated CK

Polymyositis, dermatomyositis

Anti-Jo1 (--| tRNA synthetase), anti-mi 2, anti-SRP

Bleeding gums, plt >20,000

ITP

Anti-Gp2b3a

45F, jaundice, Primary biliary pruritus, conjugated cholangitis hyperbili Systemic scleroderma

Vs. Glanzmann (complete deficiency of Gp2b3a)

Anti-mitochondrial

Anti-Scl 70/topoisomerase 315

CREST

Anti-centromere

calcinosis + Raynaud + esophageal + sclerodactyly + telangiectasia

Drug-induced lupus

Anti-histone

Sulfonamides, etanercept, procainamide, hydralazine, isoniazid

Mixed-connective tissue disease

Anti-U1RNP

Post-infectious glomerulonephritis

Antistreptolysin O, anti-DNAse B

17Y w/ diabetes

T1DM

Anti-islet cell antibodies = GAD, IA2

Hypothyroidism

Hashimoto's

Anti-thyroglobulin, anti-TPO

Hyperthyroid

Graves

Thyroid-stimulating immunoglobulin

40F, Droopy eyes

Myasthenia gravis

Anti-nicotinic ach receptor

Hemolytic disease of newborn

Anti-Rh, anti-Kell

Encephalitis, post URI/vaccine/VZV

Autoimmune encephalitis

Anti-NMDA

H/o breast cancer, ataxia

Limbic encephalitis

Anti-Hu, anti-Yo

Autoimmune hepatitis

Type 1: anti-smooth muscle; Type 2: antiliver/kidney/microsomal

TB tx with new malar rash

Painful vision loss that resolves after few weeks, lose all sensation below T4/T10

Neuromyelitis Anti-NMO optica/Devic's disease

Hematuria, hemoptysis

Goodpasture syndrome

Anti-GBM

Pemphigus vulgaris

Anti-desmoglein

*MCC of hypothyroidism in US

"lupoid-hepatitis"

MS-variant

316

(desmosomes) Bullous pemphigoid

Anti-hemi-desmosomes

Fat malabsorption, Celiac disease low weight, rash on extensor surfaces of upper/lower extremities w/o silver scale

Anti-gliadin, anti-TTG, antiendomysial

Smoked for long time, trouble rising from a char

Lambert-Eaton syndrome

Anti-presynaptic voltage gated Ca channel

Loss of LE vibratory sensation, Babinski+, (UMN + LMN sx)

B12 deficiency 2/2 pernicious anemia

Anti-IF (parietal cells)

Sx

Recent HIV tx, profound anaphylaxis + death!

Dx

HLA

Seronegative spondyloarthropathy

HLA-B27

Abacavir-anaphylaxis

HLA-B57

Iran, painful sores on buccal Behcet's disease mucosa

HLA-B51

Less dangerous than pemphigus, nikolskynegative

Dx: pathergy test

--------------------------------------------------------------------------------------------------------------------------

Episode 163: Rapid Review, Series 19, Surgery (Abdomen) 1. Penetrating trauma to the abdomen → ex lap 2. Blunt trauma to abdomen with rebound and guarding (signs of peritonitis) → ex lap a. No rebound or guarding and stable → CT scan of abdomen b. Unstable patient → FAST scan or DPL c. FAST scan equivocal → get DPL 3. Cardiac cath and hours later patient has back pain, hypotensive, unstable → Dx is retroperitoneal hematoma/hemorrhage → get emboli angiogram for NBSIM 317

4. Pt with MVC with minimum urine output. PE shows suprapubic tenderness or fullness. Dx? Urinary retention or cauda equina syndrome. NBSIM is urinary cath a. Mets cancer with hx of prostate cancer? suprapubic fullness, breast cancer, prostate cancer, spinal cord compression → give IV dexamethasone b. Lady with breast cancer and subrapubic tenderness. Give IV dexamethasone 5. Bladder rupture a. Intraperitoneal rupture (dome) with diffuse pain, signs of peritonitis → straight to urology surgery b. Extraperitonal rupture of neck of bladder/trigone → conservative treatment 6. Pt in recent MCV and has left shoulder referred pain. Dx? rupture of spleen (Kehr sign) a. Vaccinate these patients for SHiN organisms 7. Pt in MVC gets blunt abdominal trauma and has R shoulder pain. Dx? liver laceration with diaphragmatic rupture on left side typically (abdominal contents in thoracic cavity) 8. Recent abdominal surgery, fever, abdominal pain for days. Dx? Subphrenic abscess 9. Newborn with abdominal folds/contents in the thoracic cavity. Dx? Diaphragmatic hernia a. Embryo: diaphragmatic hernia arises from pleural peritoneum membrane 10. Pancreas a. Presentation: Injury, MVC, child epigastric pain due to handlebar injury b. Labs: High lipase (most sensitive) 11. Kid with handlebar injury, epigastric or back pain, pulsatile mass, bilious vomiting, low Hgb. a. Dx? Duodenal hematoma 12. Pt started on warfarin and has epigastric pain, low hgb. a. Dx? Duodenal hematoma 13. Peptic ulcer disease with sudden hypotension, low Hgb, severe abdominal pain. Dx? Perforation of peptic ulcer due to gastroduodenal artery laceration (this can bleed like crazy!) and get EGD 14. Massive GI Bleeds management a. 2 large IV bore needles b. IV fluids, blood products c. Consider surgery 15. Acute onset RLQ pain, fever, leukocytosis, psoas sign → acute appendicitis a. Next step in mgmt? Ex lap (or laparoscopic appendectomy) b. Pathophys of appendicitis is fecolith that obstructs appendiceal lumen. c. If perforated appendicitis → rapid appendectomy! d. Appendeceal abscess → give abx, drain. THEN weeks later get appendectomy 16. Tricky! 7-9 days with gradually worsening RLQ pain, fever, leukocytosis → psoas abscess a. Dx CT scan with contrast b. Tx ciprofloxacin + metronidazole or cefazolin c. Notice this presentation sounds like appendicitis but its DAYS later not ACUTE 17. Know psoas sign, obturator sign, rovsing sign, Mcburney point for appendicitis - look this up! 18. FEMALE with RLQ pain, high fever, B-HCG positive, leukocytosis. a. Dx? Ruptured ectopic pregnancy 318

19. Pt with hx of recent MI, irregularly irregular interval. a. Dx? Acute mesenteric ischemia - superior mesenteric artery is messed up b. Get angio c. May need to resect bowel 20. Chronic mesenteric ischemia: LUQ pain with eating, weight loss, systemic hypotension 21. Hx of ulcerative colitis with severe abdominal pain with rebound, guarding and distention with massive dilation of colon. a. Dx? Toxic megacolon. NBSIM? Ex lap b. Toxic megacolon also complication of C diff (abx use) + T cruzi (pt from developing country) 22. Female with severe lower abdominal pain, BL adnexal tenderness a. Dx PID. b. Tx broad spectrum abx to cover N gonorrhea, and Chlamydia. Tx ceftriaxone + doxycycline/azithromycin 23. PID and now pt ℅ RUQ pain → fitz curtiz syndrome (hepatic capsule inflammation) 24. Appendicitis - usually go straight to surgery a. But if NBME wants you to pick imaging do US for child or pregnant female or CT scan for everyone else 25. Diabetic stable pt with abdominal pain and needs to get CT scan with contrast. NBSIM: stop use of metformin because this causes lactic acidosis in chronic kidney dz or AKI pt especially if pt has profound hypotension with prerenal AKI 26. Smoker with severe mid abdominal pain or back pain with profound hypotension. Pulsatile mass on abdominal exam. Think AAA that’s ruptured a. Calcifications anterior to vertebral body = ruptured or impending AAA b. Get surgery NOW for AAA (can get abdominal US if NBME asking for imaging) 27. AAA Tx Options: Open Surgical Repair or EVAR 28. Complications of AAA repair: a. Aorto-enteric fistula: Pt: Microcytic Anemia weeks-months after AAA repair, maybe formed fistula between aorta and GI tract -> slowly leeching blood into GI tract. Colon cancer will be an incorrect answer choice. b. Anterior Spinal Artery syndrome: Paraplegic, cauda equina style symptoms, or urinary retention after surgery due to Adam of Adamkiwetz (anterior 2/3 of spinal cord) ischemia. Pt: Multiple episodes of profound hypotension during surgery c. Acute Tubular Necrosis or Prerenal AKI: Rising Cr after AAA repair -> ischemia of kidneys during surgery case d. Endoleak: Pt: Person will have had AAA repair, Hg is slowly dropping, microcytic anemia symptoms, on CT Angio: see contrast going beyond margins of excluded aneurysm 29. Biggest risk factor for AAA is smoking 30. Screen for AAA in: a. male smokers b/w age 65-75 b. male smokers or nonsmokers above age 50 with first-degree relative who had AAA rupture 31. Screening: AAA > 5.5cm OR AAA >0.5 cm/6 months -> repair asap 319

a. Prior u/s (3-4 cm): repeat ultrasound screen in 2 years b. Prior u/s (4-5 cm): repeat ultrasound screen in 1 year c. Prior u/s (5-5.5 cm): repeat ultrasound screen in 6 months 32. Young female with sudden onset abdominal pain, weeks earlier found cyst on imaging → ovarian torsion. Tx surgery asap. Risk factors: PCOS, dermoid cyst, teratoma, prior ovarian cyst a. If free fluid in peritoneum → pick ruptured ovarian cyst b. If no mention of ^^ pick ovarian torsion 33. Pt w bulging abdominal mass for a long time, mass no longer reducible, severe abdominal pain → strangulated hernia. Tx surgery 34. Pt with RUQ pain, shoulder pain, fever, leukocytosis → acute cholecystitis. Dx RUQ US - thickening of GB wall, pericholecystic fluid. Tx laparoscopy cholecystectomy with abx a. RUQ US is equivocal → get HIDA scan → if negative and GB fills → r/o acute cholecystitis

35. Cholecystitis and no obstructing stone seen on imaging = acalculous cholecystitis. NBSIM? Percutaneous cholecystostomy (NOT cholecystectomy) a. These patients are very sick - on ventilators, ICU level care, etc

36. Pt w s/s of acute cholecystitis (RUQ pain, fever, leukocytosis) AND profound jaundice with signs of ascending cholangitis. imaging shows CBD is not distended → think Mirizzi's syndrome a. Obstructing stone in cystic duct → cystic duct dilation bulges and presses common hepatic duct → conjugated hyperbilirubinemia b. Complication: cholecsytodocofistula - connection between cystic duct and common hepatic duct

c. Dx RUQ US or MRCP 37. RUQ pain, fever, jaundice, hypotension, high white count → ascending cholangitis. NBSIM? ERCP (diagnostic and therapeutic) this is an emergency! 38. Choledocliathsisi, no fever, RUQ pain, dilation of common bile duct. Dx RUQ US or MRCP if equivocal

a. Not as sick patients 39. Blunt trauma to abdomen involving kidney. NBSIM? Non operative management a. Don't do nephrectomy! 40. Kidney transplant - new kidney in the pelvic fossa and bad kidney is not removed 41. Pt has hx of peritoneal dialysis, mild fever (100.7), acting crazy (AMS), low grade abdominal pain. a. Dx? spontaneous bacterial peritonitis b. NBSIM? paracentesis (>250) c. Tx ceftriaxone, cefotaxime. fluoroquinolone prophylaxis 42. Peritoneal dialysis vs hemodialysis - no difference in outcome a. Insert cath for peritoneal dialysis at least one month prior b. Hemodialysis - insert AV fistula at least 2 months prior 43. Pt with recent AV fistula for hemodialysis. Pt has chest pain, hypotension, crackles in lungs with heart failure symptoms. Echo shows EF 75% a. Dx? high output heart failure

44. Paget dz - high output heart failure because bone marrow gets hypervascular. 45. Pancreatitis a. Tx NPO, IVF, pain control 320

46. Pancreatic pseudocyst - weeks later from pancreatitis. Patient has abdominal pain, epigastric tenderness and fullness. 47. Chronic pancreatitis: Chronic epigastric pain, fat malabsorption. Imaging shows white dots in the walls of pancreas. Risk factor alcohol 48. Diverticulitis - RLQ pain in older pt, fever, leukocytosis.

a. Get CT scan with IV contrast b. Tx abx c. Weeks later - get colonoscopy 49. Pt with hx of diverticulitis, now poop or air in urine - colovesical fistula 50. Severe abdominal pain, coffee bean sign. Head of coffee bean sign is oriented to RUQ - sigmoid volvulus a. If head of coffee bean sign is oriented to LUQ - cecal volvulus

51. Old guy with bad abdominal pain. Abdomen is severely distended and not passing gas. Colon is distended. Colonic pseudo obstruction = ogilvie's syndrome. NBSIM neostigmine (ACHesterse inhibitor) to decompress bowl insert rectal tube. a. s/s of obstruction but colon is not distended

52. Appendectomy in past or c section now has bilious vomiting, abdominal pain - not passing gas - small bowel obstruction due to adhesions (followed by hernias, cancer) ~I finally finished these notes and if someone can cross check that would be great!~CROSS CHECKED? No --------------------------------------------------------------------------------------------------------------------------

Episode 164: The Clutch Toxicology Podcast (Drugs of abuse, overdose, withdrawal, toxidromes) ●

Pt with a hx of DM found unresponsive → likely hypoglycemia ○ NBSM? Administer dextrose solution OR give glucagon injection

Pt found unresponsive with RR = 4 and bilateral pupillary miosis. ABG with pH 7.21, PaO2 55, CO2 70 → opioid overdose ○ NBSM? Naloxone ■ NOT naltrexone

Pt with hx of multiple episodes of acute pancreatitis or calcified pancreas on imaging. Pt comes to ED with nystagmus and trouble walking. → Wernicke’s encephalopathy ○ NBSM? IV thiamine (B1) ○ Triad = confusion + ophthalmoplegia + ataxia ○ What if there is amnesia + confabulation? Korsakoff’s psychosis ○ Neuroanatomical association? Hemorrhagic infarction of mamillary bodies ○ Pathophys? Lack of B1 cofactor → dysfunction of transketolase enzyme (rate-limiting enzyme of nonoxidative phase of pentose phosphate pathway) ○ Patient population ■ Alcoholics ■ Anorexic ■ Hyperemesis gravidarum ■ Chronic malnutrition 321

Pt on nitroprusside drip that develops metabolic acidosis with pH 7.19 and bicarb 10 → cyanide poisoning ○ Pathophys? Lactic acidosis 2/2 anaerobic metabolism ○ NBSM? Hydroxocobalamin OR amyl nitrate + sodium thiosulfate ■ CN- binds to Fe3+ (oxidized state). Amyl nitrate oxidizes hemoglobin so CN- binds. Sodium thiosulfate converts the CN- to thiocyanate, which can be excreted.

Pt that consumes antifreeze, now complaining of flank pain radiating to the groin → oxalate nephrolithiasis 2/2 ethylene glycol poisoning ○ Other population that gets oxalate nephrolithiasis? Crohn’s ○ NBSM? Fomepizole ■ Mechanism? Alcohol dehydrogenase inhibitor ■ Indications: methanol, ethanol, or ethylene glycol poisoning ○ Lactic acidosis + visual difficulty → methanol poisoning

Respiratory depression + no pupillary findings +/- psych disorder? benzodiazepine overdose ○ NBSM? Flumazenil ■ Mechanism? GABA receptor antagonist ■ Indications: ● Benzo overdose ● Z drug overdose (zolpidem, zaleplon, eszopiclone)

Body temp 103F + dilated pupils + dry red skin + suprapubic fullness → anticholinergic toxicity ○ Causes ■ Atropine ■ TCAs (also look for wide QRS on EKG, give sodium bicarb) ■ 1st gen antihistamines (e.g. diphenhydramine) ■ 1st gen low potency antipsychotics ■ Clozapine ■ Belladonna plants/Jimson weed ○ Tx? physostigmine ■ Mechanism? AChE inhibitor

Diarrhea + miosis + sweating + lacrimation + bronchospasm → cholinergic toxidrome ○ Causes? ■ Child that takes AChE pills ■ Organophosphate poisoning ● E.g. terrorist attack with nerve gas ○ Tx? Atropine (1st) + pralidoxime (2nd) ■ Atropine - block muscarinic receptors ■ Pralidoxime - helps regenerate AChE

Hx of DM with gastroparesis. Pt is having muscle rigidity + temp 105F → NMS 2/2 metoclopramide ○ Tx? ■ Benzodiazepine ■ Dopamine agonist (e.g. cabergoline, bromocriptine) ○ NBSM 322

Pt that was on an SSRI for a while switched to MAOI. Now with fever + myoclonus + diarrhea → serotonin syndrome 2/2 inadequate washout time ○ Causes? ■ SSRI / SNRI / TCA / MAOI ■ Trazodone ■ St. John’s wort ■ Linezolid ■ Ondansetron ■ Triptans ■ Ergots ■ Tramadol ■ Ecstasy/MDMA ○ Which SSRI has the longest half life? Fluoxetine ○ Tx? ■ Benzo ■ Cyproheptadine (antihistamine with antiserotonergic properties) ○ Why diarrhea? Oversecretion of serotonin causes diarrhea ■ Recall carcinoid syndrome mnemonic (mnemonic “Be FDR”) ● Bronchospasm ● Flushing ● Diarrhea ● Right-sided heart lesions

Pt recently started on psych drug is at dinner and starts to have HA and vision changes. BP is 240/120 → hypertensive crisis 2/2 tyramine consumption while on MAOI ○ Tx? Phentolamine (reversible alpha-1 antagonist) ○ General tx for hypertensive urgency/emergency ■ Nitroprusside ■ Nicardipine ■ Clevidipine ■ Labetalol ■ Fenoldopam ○ Do NOT use hydralazine for hypertensive emergency

Pt with diabetic gastroparesis has rigid UE → acute dystonia ○ Pathophys? Dysfunction of nigrostriatal pathway (extrapyramidal symptom) ○ Tx? ■ Diphenhydramine ■ Benztropine (anticholinergic) Pt started on antipsychotic feels really restless → akinesthesia ○ Tx? Beta blocker Pt started on antipsychotic now has cogwheel rigidity, trouble walking at normal pace ○ Tx? ■ Dopamine agonist (e.g. bromocriptine, cabergoline) ■ Benztropine Pt that’s been on a psych drug for a while now has repetitive movements of the tongue → tardive dyskinesia ○ NBS? STOP the drug ○ Switch to an atypical antipsychotic

● ●

323

Pt was a hx of Parkinson’s. Carbidopa/levodopa dose was recently increased. Now they’re having psychosis. NBS? ○ NBS? reduce dose of carbidopa/levodopa ■ If they don’t tolerate dose reduction → quetiapine ● Why? Quetiapine has weaker antidopaminergic activity ● Adverse effect? Cataracts Pt with HTN + tachycardia + mydriasis → sympathomimetic toxidrome ○ Causes? ■ If nasal septum perforation → cocaine toxicity ● Tx for HTN? ○ Alpha-1 blocker (phentolamine) ○ Alpha-beta blocker (carvedilol) ○ Benzodiazepine ■ Amphetamines ■ Benzo/alcohol withdrawal ■ Hyperthyroidism

Pt found down at home in the winter, now with bad lactic acidosis → CO poisoning ○ Causes? Space heater use ○ Tx? Hyperbaric oxygen

Pt going to developing country and taking malaria ppx, now hypoxic with cyanosis → methemoglobinemia ○ Pathophys? Fe2+ in Hgb is converted to Fe3+. O2 can’t bind to Fe3+ ○ Common drug causes ■ Dapsone ■ Primaquine ■ TMP-SMX ■ Lidocaine/benzocaine ■ Nitrates & nitrites ○ Tx? Methylene blue ○ When do we induce methemoglobinemia? To treat cyanide poisoning

Pt with mitral stenosis and A-fib presents with bloody BMs → warfarin toxicity ○ Manifestations ■ GI bleeds: duodenal hematomas/rectus sheath hematomas ○ NBSM? Give four-factor PCC (prothrombin complex concentrate) ○ Tx for other cases ■ High INR but no sxs → stop warfarin + give oral Vit K

Pt with psych disorder tried to overdose on something. Now LFTs are high and Cr is rising → acetaminophen poisoning ○ Pathophys? Production of NAPQI, which is toxic to hepatocytes ○ Tx? N-acetylcysteine to prevent hepatic damage ■ If within minutes → give activated charcoal ○ Other weird presentations ■ Hypoglycemia 324

■ Metabolic acidosis (mitochondrial poisoning → lactic acidosis) ■ Encephalopathy Pt that tried to overdose on something. RR = 30 and PaCO2 = 20, pH 7.51, bicarb 13 → aspirin toxicity ○ Pathophys? Respiratory alkalosis + metabolic acidosis ○ Tx? Sodium bicarb ■ Mechanism? Alkaline the urine and increases excretion of aspirin ○ Tx if very severe? dialysis

Profound bradycardia + hypotension in pt on an antihypertensive drug → beta blocker overdose ○ Tx? Glucagon

Pt that overdoses on a CCB. NBS? Calcium-containing agent (e.g. calcium chloride, calcium gluconate)

Hyporeflexia + slow RR → Mg toxicity ○ NBS? calcium gluconate

Pt is admitted for CHF exacerbation and is put on a loop diuretic. Now they’re having a lot of ventricular arrhythmias → digoxin toxicity ○ Pathophys? Hypokalemia predisposes to digoxin toxicity ○ Tx? anti-digoxin Fab fragments ○ Other adverse effects of digoxin ■ Yellow vision ■ Hyperkalemia

Reversal agents ○ Dabigatran → idarucizumab ○ Unfractionated heparin → protamine sulfate ■ Does NOT tx LMWH toxicity ○ Warfarin → Vit K or PCC

Pt found down, unresponsive. Hx of T2DM and not being treated with insulin. → hypoglycemia ○ Oral agent with the highest risk of hypoglycemia? Glyburide (sulfonylureas) ○ Tx? ■ Give glucose (best choice) ■ Glucagon ■ Octreotide

Pt was partying at a club and took a drug. Now with hyperkalemia + hypocalcemia + rising Cr. → MDMA (Ecstasy) use & rhabdomyolysis ○ Other problems with Ecstasy use ■ Serotonin syndrome ■ Heat exhaustion ■ Psychogenic polydipsia

Pt is undergoing surgery and gets rigid and febrile → malignant hyperthermia ○ NBS? Dantrolene (CCB) ■ Mechanism? Ryanodine receptor antagonist 325

○ ○

Inheritance? AD Pathophys? Ryanodine receptor mutation → excessive Ca++ released from the sarcoplasmic reticulum

Pt that has been on a ventilator for a while gets pancreatitis → propofol ○ Pathophys? Propofol contains a ton of lipid → hypertriglyceridemia ○ Contraindications ■ Familial hypertriglyceridemia

● ●

Pt with Parkinson’s disease presents with myoclonus + hyperthermia → MAO-B inhibitors Pt on a cancer drug that starts having blood clots in urine → hemorrhagic cystitis 2/2 cyclophosphamide ○ Pathophys? Acrolein metabolites ○ Prevention? Mesna

Pt on a cancer drug with rising Cr and hearing difficulties → cisplatin ○ Prevention of renal toxicity? Amifostine ○ Renal + ototoxicity drugs ■ Aminoglycosides ■ Vancomycin ■ Ethacrynic acid

Pt with breast cancer that now has SOB or PND → irreversible dilated cardiomyopathy 2/2 anthracyclines (doxorubicin, daunorubicin) ○ Prevention? Destrozame (iron chelator) ○ What if the pt has HER2-positive breast cancer? reversible dilated cardiomyopathy 2/2 trastuzumab

Megaloblastic anemia in pt with alcoholism → folate deficiency ○ NOT B12 deficiency ○ Folate stores don’t last as long as B12 stores --------------------------------------------------------------------------------------------------------------------------

Episode 166: Rapid Review Series 20 (Psych) ●

Pt thinks they are president, has slept 1 hr/day for past 2 weeks, feels like superman → bipolar I disorder ○ Manic episodes ■ D = distractibility ■ I = indiscretion/impulsivity ■ G = grandiosity ■ F = flight of ideas ■ A = increased goal-directed activity ■ S = decreased need for sleep ■ T = talkativeness ○ Depressive episodes ○ After ONE manic episode → can make diagnosis! Even w/o depressive episode! ■ Episode must last at least 1 week ■ Exception: if sxs are so severe they are hospitalized 326

○ ○

● ●

● ●

Tx if acutely manic? Antipsychotic & start lithium 1st line maintenance tx? Lithium ■ Decreased risk of suicide! ■ Adverse effects? ● Hypothyroidism ● Nephrogenic DI (high serum osmolarity, low urine osmolarity) ○ Gains entry to principal cell via ENaC channel ○ Tx? ENaC blockers, e.g. amiloride, triamterene ○ Thiazide will worsen lithium toxicity by causing RAAS activation and increasing ENaC activity ● Tremors ● Ebstein’s anomaly = downward displacement of tricuspid valve, “atrialization of LV” ■ If a woman is well-controlled on lithium and gets pregnant, NBS? Continue lithium! ■ If a woman is diagnosed with bipolar disorder in pregnant, NBS? ● Start haloperidol Person with bipolar disorder, sxs not controlled on lithium, NBS? Add atypical antipsychotic (e.g. quetiapine)

Pt with HTN is also on a psych drug. Which drug? SNRI (especially venlafaxine) Pt with low libido, recently started on psych med. Which drug? SSRI ○ Adverse effects? ■ Sexual dysfunction ■ Weight gain ○ Indications? ■ MDD ■ GAD ■ PMDD ■ PTSD How to make GAD diagnosis? Need sxs for 6 months How to make MDD diagnosis? 5/9 of SIGECAPS + low mood for at least 2 weeks ○ S = sleep ○ I = loss of interest ○ G = guilt ○ E = low energy ○ C = poor concentration ○ A = appetite ○ P = psychom*otor ○ S = suicidal ideation What if they don’t meet 5/9 or 2 week criteria for MDD? adjustment disorder with depressed mood

How long must sxs by present to make PTSD diagnosis? 1 month ○ What’s the diagnosis if they have sxs for < 1 month → acute stress disorder ○ Tx for PTSD nightmares? Prazosin

Pt with an eating disorder is taking a psych med and had a seizure? Bupropion (NDRI) ○ Indications? ■ MDD, if patient has been sexual side effects with SSRI 327

■ ■

Weight loss Smoking cessation ● Varenicline is more effective (partial agonist at nicotinic receptors) ● Nicotine patch + variclean is MOST effective Contraindications ■ Anorexia ■ Bulimia ■ Seizure disorder

How to distinguish between anorexia, bulimia, binge eating? ○ BMI < 18.5 → anorexia ○ Binge eat & compensatory behavior → bulimia ○ Binge eat & NO compensatory behavior → binge eating disorder

-------------------------------------------------------------------------------------------------------------------------------

Episode 167: Rapid Review Series 21 Cardio Episode 167 notes were graciously provided by Divine Intervention from an anonymous contributor. ●

75M passes out while mowing lawn: aortic stenosis o NBS: echocardiogram (dx) o Sx: SEM radiating to carotids o Where is it heard best? RUSB/2nd intercostal space o Pathophys: "calcification of valve"* ● Vs. "myxomatous degeneration" (MVP) vs. "cystic medial degeneration/cystic medial necrosis" (aortic dissection) ● *Prior to 70Y, d/t bicuspid aortic valve AS HOCM Age

Older

Younger

Heard best?

RUSB

LLSB

Murmur

SEM, radiates to carotids SEM

Pulse

Pulsus parves et tardus ("late") d/t trouble getting blood out of ventricle (PMI…carotid)

Pulses bisferiens d/t mitral valve moving towards LVOT mid-systole ("bifid carotid pulse") = 1. small 2. big carotid pulse

Increase preload Louder murmur (squat, standing --> supine): more blood in LV

Softer murmur (Anterior mitral leaflet occludes LVOT)

Increase afterload

Softer murmur (less gradient for flow across valve)

Softer murmur

Tx

Surgically replace valve

Implant defibrillator, beta blocker (more time in diastole = LV fills) 328

● ●

Diastolic murmur heard best at apex: mitral stenosis o Biggest RF: Rheumatic fever o MCC complication: Afib ● d/t LA contracting against increased resistance --> dilation --> disruption of electrical conductivity ▪ + compression of esophagus (dysphagia) Holosystolic murmur @ apex, radiating to axilla: mitral regurgitation Mid-systolic murmur @ apex, w/ click: mitral valve prolapse o RF: ADPKD, Marfans, Ehlers-Danlos

MR

MVP

Increase preload

Louder murmur

Unprolapse the valve by filling LV --> decrease regurg --> softer murmur

Increase afterload

Louder murmur (more blood to regurgitate)

Softer

Diastolic blowing murmur heard at RUSB: Aortic regurgitation o RF: Takayasu, tertiary syphilis (@vasa vasorum of aortic arch), connective tissue disorder, retrograde aortic dissection o Sx: wide pulse pressure (also seen in PDA) o c/b: chronic volume overload of LV --> eccentric hypertrophy (sarcomeres in series) --> dilation of LV --> systolic dysfunction --> HFrEF ● Vs. AS --> pressure overload --> concentric hypertrophy (sarcomeres in parallel) --> decreased luminal size of LV --> diastolic dysfunction Severe sudden onset chest pain with radiation to back: Aortic dissection o Sx: differences in blood pressure between arms, mediastinal widening, unilateral pleural effusion + chest pain, elevated BP o Dx: ● Stable: CTA (c/I renal insufficiency) ● Unstable: TTE o Tx ● Propranolol ● Avoid increased heart rate (place more shear stress on aortic valve) i.e. don't use hydralazine d/t ae: reflex tachycardia s/p baroreceptor activation by vasodilation ● : ▪ Stanford Type A: involves ascending aorta --> surgery ▪ Stanford Type B: does *not* involve ascending aorta --> medical therapy o RF: HTN IVDU, fevers, 10lb weight loss, LLSB murmur: tricuspid valve endocarditis o NBS: blood culture o Dx: echocardiogram o Tx: IV abx for 6wks o Etiology: Staph aureus (MCC IVDU endocarditis) into blood stream, veins drain to R side of heart (tricuspid valve) ● Vs. recent dental procedure: Strep viridans --> subacute endocarditis ▪ Increased risk with valvular issues ▪ Ppx: oral amoxicillin ● Vs. culture-negative: Coxiella Burnetti ● Vs. + hemolytic anemia, malar rash on face: SLE Libman-Sacks endocarditis ● Vs. Strep bovis: colon cancer! --> colonoscopy ● Vs. 70d s/p recent valve replacement: Staph epidermidis (makes biofilm) 329

● ●

Vs. recent immigration, URT w/o treatment: GAS Rheumatic fever 70Y, heavy smoker, severe pain when walking to YMCA: PAD o NBS: ABI 1.1: Monckeberg arteriosclerosis 2/2 T2DM (calcified blood vessels) ▪ NBS: *toe* brachial index o Tx: supervised walking program, cilostazol (PDE inhibitor leading to increased cAMP, smooth muscle relaxation), arteriography --> bypass o @ ● Femoral artery occlusion: pain @ thigh ● Popliteal artery occlusion: pain @ leg ● Leriche syndrome d/t occlusion of internal+external iliac branches of aorta + impotence, buttock pain #rip o RF: smoking Long-term smoker, past 30min severe back/abdominal pain, pulsatile abdominal mass, hypotensive: ruptured AAA o RF: smoking o Dx: bedside ultrasound or CTA o Ppx: 65-75M w/ history of smoking or FMHx AAA? One time screening abd ultrasound ● >5.5 cm, growing >0.5 cm/6mo or >1cm/yr, symptomatic --> surgery o Complications of surgery: ● AKI d/t transient hypoperfusion of kidneys ▪ AAA @ Infrarenal aorta near renal artery ● Anterior spinal artery syndrome d/t infarction of Artery of Adamkiewitz ▪ Sx: paraplegic/quadriplegic; all of spinal cord knocked out except dorsal columns (vibration/fine touch) ● Endo-leak = did not properly exclude aneurysm --> contrast going past contours of aneurysm stent ● Aortoenteric fistula ▪ Sx: dropping Hgb s/p weeks or months, heme-occult positive stools Bed-bound hospitalized patient, severe unilateral leg pain/tenderness: DVT o RF: Virchow's triad = stasis, hypercoagulability, endothelial dysfunction o NBS and Dx: duplex scan of LE o Tx: heparin or IVC filter (prevent PE) Chest pain, ST elevations on EKG, radiation to jaw: STEMI ACS Unstable angina NSTEMI STEMI

o

o o

Chest pain

+

+

+

Troponin

+

+

ST elevation + Unstable angina ● NBS: Coronary angiography! (not exercise stress test) d/t high probability ▪ Vs. Stable angina: NBS = stress test ● Echo stress test if known cardio anomaly ● Exercise stress test preferred vs pharmacological stress test (adenosine/dipyrimadole/dobutamine for obese) ● *adenosine c/I in reactive airway disease ● *dobutamine c/I in arrythmias NSTEMI STEMI ● Complications 330

▪ ▪

MCC death s/p 48h: Vfib Diffuse STE, chest pain, worsened with inspiration: pericarditis (post-myocardial vs Dressler's) ● Tx: NSAID (c/I renal failure), steroid, colchicine (c/I renal failure) Short PR segment, wide QRS, palpitations: WPW o Pathophys: bundle of Kent that allows direct communication atria --> ventricles o Tx: procainamide Severe leg pain worse at end of day, painless ulcer above medial malleolus: chronic venous insufficiency o Dx: duplex of LE o Pathophys: "incompetence of venous valves" o Sx: painless ulcer, stasis dermatitis (d/t blood pooling) o Tx: leg compression (Unna boots), leg elevation Wide-complex regular tachyarrythmia: Vtach o Pulse? ● Yes: ▪ Hemodynamically stable? Amiodarone ▪ Hemodynamically unstable? Synchronized cardioversion ● No: ACLS -- defibrillation

-------------------------------------------------------------------------------------------------------------------------------

Episode 169: Clutch Nephrotic/Nephritic Syndrome ●

Nephrotic → protein excretion more than 3.5 in 24 hrs ○ Urine protein to creatinine ratio, low albumin, edema, hyperlipidemia, maltese cross bodies ○ Systemic diseases: DM, sarcoidosis ○ Lost antithrombin III = hypercoagulable ○ Tx: statins, anticoagulate, loop diuretics ○ FSGS ■ Risk factors: A. american, IV drug users, HIV (collapsing), obesity, NPHS1 and NPHS2 mutations ■ Dx: biopsy w/podocyte effacement ■ Tx: steroids or cyclosporine ■ Less than 50% effected and not continuous ■ More than 50% progress to renal failure in 5 yrs ○ Membranous glomerulopathy ■ White person with ab against phospholipase A2 receptor ■ Subepithelial spike and dome ■ Risk factors: solid cancers, HBV, HCV, malaria, lupus, NSAIDs ■ Can develop renal vein thrombosis → hypercoagulable ■ Dx: biopsy ■ Tx: steroids + cyclophosphamide or cyclosporine, treat coexisting HBV (emcitabine, lamivudine, tenofovir) ○ Minimal change dz ■ Usually in kids w/podocyte effacement ■ Risk factors: URI, NSAIDs, liquid cancers ■ Tx: steroids ○ Diabetic nephropathy ■ Microalbuminuria in pt with longstanding DM, clinical diagnosis ■ Tx: ACE/ARB ■ Check kidney function at time of diagnosis for type 2 and 5 years after diagnosis for type 1 Nephritic → protein excretion less than 3.5 ○ Dysmorphic erythrocytes, periorbital edema, HTN, RBC casts ○ Goodpasture aka anti-GBM dz

331

■ Auto ab against a3 chain of type IV collagen → young male with hematuria and hemoptysis ■ Linear pattern rather than granular ■ Type 2 hypersensitivity rxn ■ Tx: steroids and cyclophosphamide ○ Wegener’s aka GPA ■ Triad of hematuria, hemoptysis, and sinusitis ● Might have mastoiditis, eustation tube, or otitis media ■ c-ANCA (antiproteinase 3 abs) - only c-ANCA dx ■ Normal complement levels ○ Churg Strauss aka EGPA ■ Asthma hx w/eosinophilia and hematuria ■ Normal complement levels ○ Microscopic polyangiitis ■ >60 yo, hematuria, fevers, weight loss - nonspecific sx so likely will not ask to dx ■ p-ANCA (myeloperoxidase abs) ○ RPGN → progress to bad renal function over days to weeks ■ Histo: crescents b/c of leakage into bowman’s space ○ IgA nephropathy ■ Hematuria 1-2 days after URI or GI infection ■ Dx: biopsy w/ Ag-Ab complexes Tx: don't need to treat ■ Normal complement, HSP ○ MPGN ■ Subendothelial tram tracks ■ Associated with lupus, HBV and HCV ■ Low complement → low C3 with normal C4 ■ DDD → dense deposits around BM, C3 nephritic factor, super low C3 ○ Post infectious GN ■ Subepithelial humps, granular pattern ■ Low complement, different strains than what cause RF ■ 1-6 wks after strep infection ■ Titers for ASO and anti DNAseB ■ 40% of adults gets RPGN vs 10% of kids (diff in incidence) ○ DPGN ■ Low complement, associated with lupus ■ Anti smith abs, ANA abs ○ Alport ■ XLD, can’t see, can’t pee, can’t hear a bee (sensorineural) ■ Mutations in a2 chain of type IV collagen ■ COL4A5 mutation These are my personal notes that I took while listening to the podcast a while ago. I apologize for any errors! Cross Checked: No

-------------------------------------------------------------------------------------------------------------------------------

Episode 169: Clutch Nephrotic/Nephritic Syndrome (Version 2) Episode 169 (Version 2) notes were graciously provided by Divine Intervention from an anonymous contributor.

Nephrotic syndrome

Nephritic syndrome 332

Dx

>3500mg proteinuria (or protein:Cr) in 24h

sudden onset flank pain + fevers) ● Dx: biopsy, "subepithelial spike and dome" ● Tx: cyclophosphamide +/- steroids ▪ Also treat underlying disease i.e. HepB (|-- emtricitabine, lamivudine, tenofovir "HepBELT") Minimal change disease ● RF: kiddos, also found less commonly in adults, URI, heme malignancy ● Dx: "podocyte foot process effacement" ▪ --> loss of negative charges @ foot processes --> albumin leaks through ▪ Tx: steroids ● Good response, rarely progress to renal failure Diabetic nephropathy ● Dx: clinical, microalbuminuria ● Ppx: @ time of DM diagnosis --> measure albumin:Cr ratio ▪ If not done at time of diagnosis, start 5y after dx is made ● Tx: ACEi/ARB, glucose + BP control

Nephritic syndrome o

Goodpasture's syndrome/anti-GBM disease ● Path: Ab-alpha 3 chain of Type4 collagen = T2HS ● Sx: hematuria, hemoptysis ● Dx: linear pattern of Ig deposition ● Tx: plasmapheresis, steroids 333

o

o

o

o

o

o

Pauci-immune: Wegeners/GPA, microscopic polyangiitis, Churg-Strauss/EGPA ● Sx: rapidly progressive glomerulonephritis = normal renal function --> failure over days-weeks ▪ Wegener's: hematuria, hemoptysis, sinusitis/mastoiditis, otitis media, eustachian tube dysf(x) ● Path: c-ANCA (Ab-proteinase3) ▪ Microscopic polyangiitis: >60Y, fever, hematuria, p-ANCA (Ab-myeloperoxidase) ▪ Churg-Strauss/EGPA: asthma/allergies, tons of eosinophils, hematuria, p-ANCA ● Dx: crescent-shaped = monocytes leak through GBM into Bowman's space and proliferate ▪ Complement levels are normal ● Tx: steroids, cyclophosphamide, rituximab IgA nephropathy/synpharyngitic nephropathy ● RF: recent URI ● Pathophys: Ab against glycosylated-IgA (from N-acetylgalactosamine on bugs) --> deposits in mesangium of kidneys --> inflammation ● Systemic manifestation = HSP (purpura x abdominal pain x hematuria) ● Sx: hematuria ● Dx: biopsy, complement levels are normal ● Tx: supportive Lupus nephritis ● Sx: malar rash, anti-Sm Ab, anti-dsDNA, low complement levels ("full-house pattern") ● Subtypes ▪ Diffuse proliferative glomerulonephritis Post-infectious glomerulonephritis/PSGN ● RF: URI, Strep infection (nephritogenic strains) ▪ --> abx for Strep throat do not decrease chance of PSGN ● Pathophys: Ab against GAS cross react with antigens in glomerulus --> complement "destroys everything in its wake" ● Sx: 1-6wks post-URI, hematuria, low complement ● Dx: anti-DNAseB, anti-streptolysinO, anti-hyaluronidase, "granular pattern" ● C/b: chronic renal failure (kids 1% vs adults 40%) Membranoproliferative glomerulonephritis (MPGN) ● RF: lupus, HepB/C ● Dx: low C3, "subendothelial tram tracks" ● Subtype: dense-deposit disease ("ribbon-shaped" deposits around basem*nt membrane of glomerulus, C3 nephritic factor = C3 convertase --> low C3) Alport syndrome ● Sx: "can't pee, can't see, can't hear a bee" = cataracts/dislocation of lens, nephritic syndrome, sensorineural hearing loss ● Path: X-linked dominant mutation in alpha2* chain of Type4 collagen/COL4A5 ▪ *A comes before G[oodpasture] in the alphabet! ▪ (same inheritance as Fragile X syndrome)

Disease

Light microscopy findings

Minimal change disease

Podocyte effacement

FSGS

Podocyte effacement

PSGN

Subepithelial humps, granular

Membranous nephropathy

Subepithelial spike and dome

Membranoproliferative glomerulonephritis

subendothelial tram tracks 334

Lupus nephritis

wire-loop appearance

Goodpasture's syndrome/anti-GBM disease

Linear pattern of IgG deposition

IgA nephropathy

Mesangial proliferation, IgA deposits

Complement levels

Disease

Low

PSGN, lupus, membranoproliferative (C3)

Normal

IgA nephropathy, pauci-immune entities

High

-------------------------------------------------------------------------------------------------------------------------------

Episode 173: The Clutch Immunodeficiency Diseases Podcast ●

3 yo M with multiple Strep pneumo pneumonia + giardia infections → Bruton’s agammaglobulinemia ○ Recurrent infections after 6 months old (when antibodies from mom wane) ○ Cell affected? B-cells ○ Pathophys? Mutation in tyrosine kinase that allows for B cell differentiation → no mature B cells → issues with antibody production / humoral immunity ○ Classic organisms? Encapsulated organisms (Strep pneumo, H. flu) ○ Inheritance? X-linked dominant ○ Mutated gene? BTK ○ PE findings? ■ Poorly developed tonsils ■ Tiny spleen ■ No lymphadenopathy with infections ○ Lab findings? ■ NO B cells on peripheral smear ■ Low levels of immunoglobulin ○ Tx? Monthly IVIG

Similar presentation to Bruton’s? CVID ○ Inheritance? AR or AD ○ Pathophys? Issues with B cell differentiation → low levels of immunoglobulins ○ Presentation? Boy or girl that has recurrent respiratory or GI infections after age 10 ○ Alternate presentation? Pt gets pneumococcal vaccine, but still gets multiple pneumococcal infections. Check titers and they’re not immune. ○ Associated problems: ■ Skin disease (e.g. alopecia, vitiligo) ■ Autoimmune dz (e.g. ITP, autoimmune hemolytic anemia, RA) 335

○ ○

■ Lymphoma Lab findings? ■ B cells on peripheral smear Tx? IVIG

Pt getting blood transfusion that becomes hypoxia, has stridor/wheezing → IgA deficiency ○ Presentation? Recurrent respiratory and GI infections ○ Pathophys? ■ Failure of differentiation of IgA-producing B cells. Recall that IgA is the “guardian of the mucosa” ■ Since they are deficient in IgA, they can form Ab to IgA. ○ Associated problems ■ Food allergy ■ Atopic disorders ○ Lab findings? ■ Normal WBC count ■ Low IgA levels, other Ig normal ○ Tx? ■ Keep treating recurrent infections ■ Washing of blood transfusions (to remove plasma)

Girl with progressive neurologic impaired + problems walking + telangiectasias in the skin or eye + mild ID → Ataxia-telangiectasia ○ Classic presentation? ataxia w/ negative Romberg → age 3-6 telangiectasias on skin & eye → then recurrent sinopulmonary infections ■ Freidrich’s ataxia tends to have positive Romberg ■ Use name of the disease to remember order ○ Inheritance? AR ○ Pathophys? ATM gene regulates response to ds DNA breaks → sensitivity to ionizing radiation ○ Mutated gene? ATM ○ Chromosome? 11 ○ Lab testing? ■ Low WBC count ■ Low IgA level ○ Associated problems? ■ Increased risk of cancer, especially hematologic cancers

SCID ○ Presentation? Bad infections since birth ○ Pathophys? ■ Common cause: deficiency of adenosine deaminase → apoptosis of lymphocytes ■ Another cause: Mutation in IL-2 receptor ○ Cells affected? B-cells & T-cells ○ Classic organisms? ALL (bacterial, fungal, viral) ○ Inheritance? Can vary (AR, X-linked) ○ Lad findings? ■ Low WBC count ■ Low immunoglobulins 336

○ ○ ○

Imaging? Absent thymic shadow on CXR ■ Also seen in DiGeorge Histology? Hypoplasia of the paracortex (T-cell zone) of lymph node Tx? ■ Abx ■ Bone marrow transplant Vaccination restrictions? NO live-attenuated vaccines

Boy with petechiae on skin + platelet count 30k + eczematous lesions + recurrent infections → Wiskott-Aldrich syndrome ○ Inheritance? X-linked recessive ○ Mutated gene? WASP ○ Pathophys? WASP gene mutation → issues with signaling cascade allowing for the appropriate movement of actin filaments ○ Lab findings? ■ Thrombocytopenia ■ Tiny platelets ■ High IgE → eczema ■ High IgA ■ Low IgM ○ Associated problems? ■ Autoimmune disease

Skin infections w/o pus + delayed separation of umbilical cord → leukocyte adhesion deficiency ○ Pathophys? Neutrophils cannot adhere to endothelium & exit the bloodstream ○ Mutated proteins? Integrins (e.g. CD-18), E-selectin, sialyl Lewis X ○ Lab findings? ■ Leukocytosis w/o signs of infection (large % leukocytes are in a demarginated state) ○ Dx? Flow cytometry

337

Pt with lymphadenopathy + recurrent Staph aureus abscesses → CGD ○ Inheritance? X-linked recessive ○ Cells affected? Neutrophils ○ Pathophys? NADPH oxidase deficiency → neutrophils cannot generate oxidative burst ○ Classic organisms? Catalase-positive ■ Staph aureus ■ E. coli ■ Candida ○ Dx? ■ Dihydrorhodamine test (DHR) ■ Nitroblue tetrazolium test (NBT)--OLD! ○ Tx? Interferon-gamma

Recurrent infections + albinism (“diffuse skin hypopigmentation”) w/ blond hair & blue eyes + intellectual disability → Chediak-Higashi disease ○ Inheritance? AR ○ Cells affected? Neutrophils ○ Mutated gene? LYST or CHS1 ○ Pathophys? Problems with intracellular protein transport → problems with transporting secretory granules to appropriate place ■ Issues with melanin granule transport → albinism ■ Issues with neutrophil granules → can’t use them to kill bacteria ○ Associated problems? ■ Hematologic malignancies ○ Lab findings? ■ Peripheral smear with giant granules in neutrophils

Recurrent infections + no thymic shadow on CXR + hypocalcemia → DiGeorge ○ Pathophys? Failure of the 3rd and 4th pharyngeal pouches to develop 338

○ ○ ○ ○

○ ○

■ No thymus → T cells can’t mature ■ No parathyroids → hypocalcemia → prolonged QT & seizures Gene? 22q11 deletion Chromosome? 22 Dx? FISH CATCH-22 ■ C = cardiac issues (e.g. tetralogy, truncus arteriosus, VSD) ■ A = abnormal facies ■ T = thymic aplasia ■ C = cleft palate ■ H = hypoparathyroidism ■ 22 = chromosome 22 Most common cause of death? Heart disease Velocardiofacial syndrome vs. DiGeorge ■ Doesn’t have the hypoparathyroidism

Terminal complement deficiency (C5-C9) makes you susceptible to what bug? Neisseria meningitidis ○ Prevention? Neisseria meningitidis vaccine

-------------------------------------------------------------------------------------------------------------------------------

Episode 174: Rapid Review Series 22 (IM) ● ●

Cattle farmer with suspected endocarditis, cultures are negative → Coxiella endocarditis ○ Also consider HACEK organisms Hx carcinoid syndrome, with chronic diarrhea + forgetfulness + 20/30 MMSE + skin rash → pellagre 2/2 niacin deficiency ○ Pathophys? Small bowel or appendiceal tumor makes a ton of serotonin, which requires drains tryptophan stores and niacin can’t be produced Hx abdominal surgery, now with fevers + vomiting + no flatus + distention (SBO), suddenly loses consciousness & becomes hypotensive, imaging shows bowel contents throughout abdomen → small bowel rupture 2/2 EhlersDanlos ○ Pathophys? Collagen defect ○ Inheritance? AD ○ Association? ■ Aortic dissection ■ Aortic aneurysm ■ MVP ■ Intracranial aneurysms → SAH Pt with of hx of hereditary spherocytosis, now with MVC 110 → folate deficiency ○ Run out of folate (B9) much faster than B12 ○ Associations: ■ Hx hemolytic anemia (pathophys? burn through folate stores) ● Sickle cell disease ● Thalassemias ■ Alcoholic 339

● Pathophys? Alcohol poisons the enzymes that helps absorb B12 Prevention? Daily folate supplement

35 yo M smoked 0.5 ppd x 10 years, increased sputum production + cough, CXR with mild hyperinflation. Uncle died of liver problem → alpha-1 antitrypsin deficiency ○ Inheritance? Autosomal codominant ○ Pathophys? alpha-1 antitrypsin is antiprotease that protects lung structural proteins from degradation from proteases ○ Lung manifestation? Panacinar emphysema ○ Liver manifestation? Cirrhosis

53 yo M with 30 pack-year hx, fatigue for 3 months, Hgb 8, MCV 65. NBS? colonoscopy ○ Microcytic anemia in 50+ yo, NBS = colonoscopy ○ LLQ pain + fever → diverticulitis ■ Dx? CT scan with contrast ■ Colonoscopy 6 weeks after episode to r/o colon cancer ■ Do NOT do when acutely ill Lung cancer patient started on chemo, now has pins & needles sensation. What drug is responsible? Vinca alkaloids (vincristine, vinblastine) ○ Mechanism? Inhibit polymerization of microtubules ○ Other pharm causes of peripheral neuropathy ■ Taxanes (e.g. pacl*taxel) ■ Cisplatin

Pt on chemo for colon cancer, now with SOB + drug cough, CT scan shows increased interstitial markings → pulmonary fibrosis ○ What drug is responsible? Busulfan, bleomycin or methotrexate ○ Restrictive pattern (FEV1/FVC ratio normal or mildly increased) ○ Bleomycin ■ Mechanism: G2 phase, free radicals → strand breaks ○ Methotrexate ■ Mechanism: inhibits dihydrofolate reductase ■ If profound bone marrow suppression, NBS? Give leucovorin (folinic acid analog)

Pt with HIV, with HA & seizures, brain MRI with multiple ring-enhancing lesions → toxoplasmosis ○ Tx? Sulfadoxine/pyrimethamine ■ Can also cause bone marrow suppression

Pt with latex allergy. #1 RF? Healthcare worker or prolonged exposure to healthcare system ○ Type HSR? Type 1

Type 1 HSR (IgE mediated) ○ Pathophys? Antigen binding to IgE → cross linking of IgE on mast cell → mast cell degranulation ○ Examples? ■ Anaphylaxis ■ Asthma ■ Latex allergy Type 2 HSR (antibody-mediated)

340

○ ○

Pathophys? Antibodies against a particular antigen Examples? ■ Autoimmune hemolytic anemia Type 3 HSR (Ab-Ag complex-mediated) ○ Pathophys? Antibody-antigen complexes accumulate & deposit ○ Examples? ■ Vasculitis ■ Serum sickness ■ Most nephrotic/nephritis Type 4 HSR (delayed) ○ Examples? ■ Contact dermatitis ■ Tuberculin skin testing

-------------------------------------------------------------------------------------------------------------------------------

Episode 175: Rapid Review Series 23 (Psych) ●

● ●

Pt BMI = 16, has been vomiting, now hypotensive & HDUS → anorexia ○ NBS? Admit (even involuntarily) ○ MUST be underweight (BMI < 18.5) to make dx ○ Electrolyte abnormalities ■ Hypochloremic due to vomiting ■ Hyponatremic due to loss of electrolyte-rich fluid & hypovolemia → nonosmotic release of ADH → free water reabsorption ■ Hypokalemia due to RAAS activation ■ Alkalotic due to aldosterone effect on alpha-intercalated cell ○ If comorbid depression, consider which drug? Mirtazapine Binge + compensatory behavior (e.g. vomit, work out), BMI normal → bulimia nervosa Binge but no compensatory behavior → binge eating disorder

BP 210/120 + pupillary mydriasis → cocaine intoxication ○ Pathophys? Sympathomimetic ■ Alpha-1 agonist → increased SVR → increased BP ○ Tx? ■ Benzodiazepine ■ Alpha + beta blocker (e.g. carvedilol) ■ Alpha-1 antagonist (e.g. phentolamine) ○ What drug should NOT be given? Beta blockers ■ Will lead to unopposed alpha

Pt with hx Parkinson’s consumes lots of cheese, wine, smoked meats and has a hypertensive crisis. Why? ○ Combination of tyramine in food & MAO-B inhibitors (e.g. selegiline)

Pt with depression has MRSA infection and develops fever, myoclonus, & hyperreflexia → serotonin syndrome ○ Antidepressant + MAO-I activity of linezolid → serotonin syndrome 341

Cocaine vs. methamphetamine intoxication ○ Both are sympathomimetics ■ Both has pupillary mydriasis ○ Meth → milder BP elevation, poor dentition, +prominent hallucinations

70 yo M with hx Parkinson’s, parkinsonian sxs has gotten worse so you increase his meds. Now he’s acting weird. → medication-induced psychosis ○ Pathophys? Pro-dopaminergic effects of Parkinson’s meds ○ NBS? ■ 1st: try to lower dose of drug ■ If Parkinson’s sxs come back, put back on dose that control sxs and add quetiapine

Sxs of MDD/GAD but doesn’t meet criteria (# of sxs or time) + recent stressor → adjustment disorder

Veteran with nightmares, flashbacks, etc for 2 weeks. He acts suspicious during interview. → acute stress disorder ○ sxs for 1 months = PTSD ■ Tx for general sxs? SSRI ■ Tx for nightmares? Prazosin

22 yo M with episodes of tremulousness, sweating, dry mouth, feeling of loss of control. Has happened multiple times over the past few months. → panic disorder ○ Panic attack = one episode ○ Panic disorder = multiple episodes + fear of having another one ○ Tx? SSRI

Pt with bipolar disorder in acute manic episode. NBSM? ○ Lithium + antipsychotic Pt with bipolar disorder diagnosed 6-7 weeks ago, has been on lithium, sxs still poorly controlled. NBSM? ○ Add atypical antipsychotic ○ E.g. lithium + quetiapine ○ Adverse effects of lithium ■ Tremors ■ Hypothyroidism ■ Nephrogenic DI ● Tx? Amiloride or triamterene

41 yo M with trisomy 21, forgetting way home from store, 15/30 on MMSE → early-onset Alzheimer’s ○ NT? Low ACh ○ Neuroanatomical association? Basal nucleus of Meynert ○ Deficiency of what enzyme can cause AD? Choline acetyltransferase (rate-limiting enzyme in ACh synthesis) ■ Congenital myasthenic syndrome ● Presentation: similar to MG but doesn’t improve with AChE inhibitor (e.g. edrophonium = “Tensilon test”) 342

● ● ●

Pathophys? Choline acetyltransferase deficiency

Cogwheel rigidity + wide gait + bradykinesia → Parkinson’s ○ NT? Low dopamine Parkinsonism + visual hallucinations + syncopal episodes → Lewy Body dementia ○ Dementia BEFORE Parkinsonian sxs

Pt treated with 9 months of INH for latent TB, starts to have seizures. Why? ○ INH w/o B6 (pyridoxine) → decreased activity of glutamate decarboxylase, which converts glutamate to GABA → high glutamate, low GABA → seizures

● ●

If you suspected serotonin syndrome, what metabolite can help in diagnosis? urine 5-HIAA In MDD, what NT are affected? ○ Low serotonin ○ Low norepi ○ Low dopamine In pheo, what NT are affected? ○ High catecholamines ○ High urine metanephrines

In Huntington’s, what NT are affected? ○ Low GABA ○ Low ACh ○ High dopamine ■ Tx? Tetrabenazine

-------------------------------------------------------------------------------------------------------------------------------

Episode 177: Rapid Review Series 24 (OGBYN) ●

30 yo F with 15 months of infertility, BMI = 30, irregular periods → PCOS ○ Diagnostic criteria (must meet ⅔): ■ Hyperandrogenism (clinical or biochemical) ● E.g. hirsutism ■ Polycystic ovaries on US ■ Oligo-amenorrhea ○ Tx if trying to get pregnant? Clomiphene (SERM--partial agonist on estrogen receptors in CNS, removes negative feedback → promote ovulation) ○ Tx for hirsutism? ■ OCPs ■ Spironolactone (aldosterone & androgen-receptor antagonist) ● Inhibits 5-alpha reductase in the skin CERVICAL CANCER ● #1 RF? HPV exposure ○ Early coitarche ○ Multiple partners 343

● ●

● ● ● ● ●

MC cause of death in pts with cervical cancer? Renal failure 2/2 obstructive nephropathy Screening ○ Start at age 21 (never before age 21!!) ○ If =30 yo: ■ Pap q3years ■ Pap + cotesting q5years **preferred** ○ Stop at 65 if multiple negative Pap smears ■ If CIN2+, need to continue past 65 ○ If s/p hysterectomy? ■ For benign reasons → not needed ■ For CIN2+ → do Pap of vagin*l cuff ○ If hx of HIV? Pap q1year Pap f/u ○ Indeterminate/inadequate sample → repeat now ○ ASCUS → reflex HPV testing or repeat Pap in 1 year ■ If hrHPV present → colposcopy ○ Atypical glandular cells → colposcopy & endometrial bx ○ If CIN2+ → colposcopy ○ ASC-H (atypical squamous cells, cannot exclude high-grade) → colposcopy Prevention? HPV vaccine ○ Males & females age 9-26 HPV 16, 18, 30s → cervical cancer HPV 6, 11 → genital warts HPV 1, 2, 4 → plantar warts M with lower abdominal pain + urinary incontinence + new parter 2 weeks ago → cystitis ○ Tx? TMP-SMX or cipro ■ Nitrofurantoin never used in men Fever + flank pain + urinary sxs → pyelo ○ Dx? CT abdomen with contrast ○ Tx? ■ Ceftriaxone ■ Cipro ■ TMP-SMX ○ What if pt is diabetic & not improving on abx? ■ NBS? Another CT scan to r/o complication (e.g. emphysematous pyelo, perinephric abscess) UTI in pregnant woman ○ Tx for cystitis? Nitrofurantoin ○ Tx for pyelo? ceftriaxone UTIs associated with sexual activity? ○ Ppx abx prior to intercourse ○ Urinate after sex Weird diabetic infections ○ Gangrenous cholecystitis ○ Mucormycosis of face 344

■ Tx? Amphotericin B + aggressive debridement Fournier’s gangrene = necrotizing fasciitis of perineum ■ Can start with lower reproductive tract infection

Classic uses of an amnioinfusion ○ Cord compression → variable decels ○ Treat or prevent meconium aspiration syndrome (evidence mixed!) ■ RF? Post-term fetus

Key tests during pregnancy ○ Kleihauer–Betke test ■ Purpose: detects degree of fetal–maternal hemorrhage, used to determine dose of Rhogam after delivery to prevent Rh isoimmunization ■ Usually give Rhogam at 28 wks + after delivery ○ Fetal fibronectin ■ Purpose: used to determine probability of preterm delivery ○ Nitrazine paper test ■ Purpose: determine if the water has broken ■ Ferning pattern → fluid is amniotic fluid

-------------------------------------------------------------------------------------------------------------------------------

Episode 180: Hematology ● ●

Megaloblastic anemia is anemia secondary to aberrant DNA synthesis (B12 or folate def) Pt w/fatigue, SOB, Hb 8, MCV 88, MCHC high → hereditary spherocytosis ○ Mutations in spectrin and ankyrin proteins → AD ○ RBCs lack central pallor ○ Dx: osmotic fragility test or eosin-5-maleimide test ○ Tx: Splenectomy b/c this removes the splenic macs Child of missionaries that takes primaquine, central cyanosis → methemoglobinemia ○ Can also be from sulfonamides or nitrates ○ Fe3+ iron → cannot bind oxygen → hypoxia ○ Tx: methylene blue 16 yo A. american kid w/severe pain in one extremity, non-palpable spleen → Sickle Cell ○ AR inheritance ○ Tx: opioids ○ Daily meds: hydroxyurea (ribonucleotide reductase inhibitor), penicillin prophylaxis, folate supplements ○ S. pneumo is the most common cause of sepsis in these patients! ○ Salmonella is the most common cause of osteomyelitis in these patients! ○ Painless hematuria → renal papillary necrosis Pt started TB tx, fatigued, low Hb, HCV 68 → sideroblastic anemia ○ Isoniazid depletes Vit. B6 → decreased ALAS → no heme synthesis ○ Basophilic stippling on blood smear ○ Can lead to seizures b/c glutamate decarboxylase needs B6 to turn glutamate to GABA ○ Other causes: B6 def, myelodysplastic syndromes (hyposegmented neutrophils and ringed sideroblasts) Pt comes in for surgery and is anticoagulated, platelets drop → HIIT

345

● ●

● ● ●

○ Can occur with reg heparin or LMWH ○ Ab from factor IV activates platelets → depletes platelets and forms clots (hypercoagulable) ○ First step = stop heparin then resume direct thrombin inhibitor (argatroban) Warfarin toxicity ○ Next best step = 4F-PCC (has II, VII, IX, X) NOT FFP anymore Valve replacements ○ Porcine valve → lasts 5 years ○ Mechanical valve → lasts 10 yrs, anticoagulate w/warfarin for life Pt w/ mechanical valve now has fatigue, low Hb, high LDH → hemolytic anemia 2/2 hemolysis ○ See schistocytes on smear and indirect bilirubinemia Pt w/ aortic stenosis, microcytic anemia, bloody stools → Heyde Syndrome ○ vWF multimers are cleaved so primary hemostasis does not happen → GI bleeds Platelet disorders: ○ vWD → no vWF → heavy menses, bleeding gums ■ AD inheritance, increased bleeding time, increased PTT, positive ristocetin ■ Tx: desmopressin → increases vWF release from endothelial cells ○ Bernard-Soulier Syndrome → Gp1b3 ■ Positive ristocetin, normal PTT, increased bleeding time ○ ITP → Ab against Gp2b3 ■ Elevated bleeding time, isolated thrombocytopenia, normal ristocetin ■ Type 2 hypersensitivity reaction ○ Antithrombin 3 deficiency → hypercoagulability ■ Factor X and XII work unopposed ■ Give heparin w/o increase in PTT ■ Thrombosis of a random vein b/c of kidney dz ○ Protein C and S deficiency → hypercoagulable ■ Cannot neutralize factor VIII or V ○ Factor V Leiden → Factor V resists degradation by protein C ■ Dx: Activated protein C resistance assay ■ Most common, forms lots of clots ○ Antiphospholipid Ab syndrome → many miscarriages b/c of uteroplacental artery thrombosis ○ Prothrombin G20210a → overproduces factor II ■ Increased production of clots Hemophilia ○ A → XLR, Factor VIII deficiency, normal PTT, long bleeding time (< mistake?) ○ B → XLR, Factor IX deficiency, elevated PTT, normal bleeding time ○ C → AR, Factor XI deficiency, elevated PTT Hypofibrinogenemia → hypercoagulable ○ AR mutation in fibrinogen genes ○ Form clots b/c you won’t form fibrin → no clot breakdown

These are my personal notes from when I originally listened to the podcast. I apologize for anything I overlooked or any mistakes! Cross Checked: No

-----------------------------------------------------------------------------------------------------------------------------

Episode 181: Comprehensive NBME Emergency Medicine Shelf Review Series 1

346

Pt with CKD 2/2 FSGS with sudden, severe CP and palpitations. HR 125. CP is worse with deep breath → pulmonary embolism ○ Pathophys? Pts with nephrotic syndrome lose AT-III in the urine, so they’re hypercoagulable ○ Risk factors? Virchow’s triad ■ Stasis ■ Hypercoagulability ■ Endothelial injury ○ Associations: ■ Malignancy (especially mucin-secreting malignancies) ■ Nephrotic syndrome ■ Genetic hypercoagulable states ● Factor V Leiden ● Prothrombin 20210 Mutation ■ Antiphospholipid antibody syndrome ○ Classic presentation? Pleural CP + SOB ○ Lab findings? ■ Respiratory alkalosis (2/2 hyperventilation) ■ Hypocalcemia (negative charges bind Ca++) ■ Increased A-a gradient ○ MC EKG finding in PE? Sinus tachycardia ■ Other findings? ● New RBBB ● S1Q3T3 (uncommon) ○ Risk stratification done with Well’s score ■ 5 → high-risk → straight to CT angio ○ Dx? ■ CT angio w/ contrast ● Avoid in pts with CKD ● Acceptable in pregnancy if V/Q scan isn’t an option ■ V/Q scan ○ Gold standard test? Pulmonary angiogram (BUT NOT DONE!) ○ Classification of PE ■ R heart strain → submassive PE ■ HDUS (SBP < 90, required pressors) → massive PE ○ Tx? ■ Heparin ● Unfractionated heparin ONLY in renal failure pts ■ Submassive PE w/ mild sxs → just heparin ■ Submassive PE w/ severe sxs → tPA ■ Massive PE → tPA ■ Embolectomy if: ● Contraindications to tPA ● tPA was administered & pt continues to deteriorate ○ Long-term tx? 347

■ ● ● ● ●

● ●

If unprovoked → chronic anticoagulation (warfarin, factor X inhibitors, direct thrombin inhibitor)

Sudden-onset, severe flank pain in pt with nephrotic syndrome → renal vein thrombosis ○ Strongest association with which nephrotic syndrome? Membranous nephropathy Pt with hx DVT that gets a stroke → some connection b/t R and L heart ○ E.g. PFO, ASD Woman that just delivered baby that becomes comatose and is bleeding from IV sites → amniotic fluid embolism Pt that goes scuba diving and rises quickly → air embolism ○ Pathophys? Gas solubility rises with high pressures. When you come up rapidly, pressure rapidly decreases and nitrogen undissolved. Nitrogen bubbles can occlude blood vessels. Pt that recently had an IV placed has SOB & CP → air embolism Pt with recent fracture or orthopaedic surgery procedures now with SOB → fat embolism ○ Pathophys? Fat in the bone marrow gets into the bloodstream ○ Presentation? Hypoxia + petechiae + AMS

Unilateral leg pain & swelling → DVT ○ Dx? LE doppler ○ Tx? Heparin

35 yo M with palpitations, home meds include methimazole, EKG with narrow-complex tachyarrhythmia, spaces b/t QRS complexes are not constant → Afib ○ EKG findings? Irregularly irregular rhythm ○ Tx? AV nodal blocking agent ■ Beta blockers ■ Nondihydropyridine CCBs ○ Management of HDUS pt with Afib? ■ Synchronized cardioversion = direct current cardioversion = direct current countershock ○ Management based on duration of sxs? ■ Symptomatic for 48 hrs or unknown ● Start anticoagulation, TEE, cardiovert if no clot in LA ● Warfarin for 3 weeks, TEE, then cardiovert ○ Complications? ■ Embolic stroke ○ Long Term anticoagulation? caclulate stroke risk with CHA₂DS₂-VASc ■ Options for anticoagulation ● Warfarin ● Factor Xa inhibitors ● Direct thrombin inhibitors ○ Reversal agents ■ Heparin → protamine sulfate ■ Warfarin → 4-factor PCC for life-threatening bleed ■ Factor Xa inhibitors → andexanet ■ Dabigatran → idarucizumab ○ Rhythm control drugs? ■ Amiodarone ■ Propafenone 348

Pt that ran marathon yesterday, now with malaise and palpitations. EKG with peaked T waves. → hyperkalemia 2/2 rhabdomyolysis ○ Pathophys? Myocyte necrosis → release intracellular K+ ○ Other causes of hyperkalemia: ■ Tumor lysis syndrome ■ Missing dialysis ○ EKG findings in hyperkalemia (as it increases) ■ Peaked T waves ■ Wide QRS ■ Sine wave ■ V-fib ○ First step in management? calcium gluconate ○ Tx? ■ Insulin + glucose ● Drives K+ into cells ■ Albuterol ● Drives K+ into cells ■ Fluids + furosemide ● Capitalize on K-depleting properties ■ Kayexalate = sodium polystyrene sulfonate ● Reduce total body K+

Pt with depression taking imipramine. Notice wide QRS on EKG. → TCA toxicity ○ First step in management? Sodium bicarb

Hypokalemia EKG findings? ○ U waves ○ Prolonged QT Hypocalcemia EKG findings? ○ Prolonged QT Hypercalcemia EKG findings? ○ Short QT Hypomagnesemia EKG findings? ○ Prolonged QT

● ● ●

Digoxin causes what electrolyte abnormality? Hyperkalemia ○ Pathophys? Inhibits the Na+-K+ ATPase pump, so prevents K+ from getting in the cell ○ BUT hypokalemia predisposes to digoxin toxicity ■ Digoxin binds to K+ binding sit on Na+-K+ pump ■ Hypokalemia → more spots for digoxin to bind ○ If person has K+ abnormality, don’t give them digoxin Pt with CHF on diuretic comes in with CHF exacerbation c/b cardiogenic shock. Pt is given digoxin and they get lots of PVC and dies. → pt had hypokalemic 2/2 diuretic and was susceptible to digoxin toxicity QT-prolonging drugs ○ Azithromycin 349

○ ○

Haloperidol Many antipsychotics

Prolonged QT → Torsades → sudden cardiac death ○ Tx of Torsades? IV mag

Hypocalcemia refractory to appropriate repletion → hypomagnesemia ○ Tx? Replete Mg then other electrolytes can be repleted

Corrected Ca+ ○ Every 1 mg/dL drop in albumin reduces Ca+ by 0.8

Pt with K+ 8, but no sxs and no EKG findings → likely hemolyzed sample ○ NBS? Repeat lab draw

Hypercalcemic crisis ○ First step in management? Give NS

Pt with severe, sudden-onset CP. Profoundly hypoxia, BP 40/palp, +JVD. Central line was recently placed. → tension pneumothorax ○ NBS? Needle decompression ■ 2nd intercostal space in midclavicular line ■ 5th intercostal space in the mid-axillary axillary line ○ Tx? Chest tube ○ PE finding? Unilateral decreased breath sounds, hyperresonant to percussion

Primary spontaneous pneumothorax ○ Population? Tall thin male ○ If 180/110 without sxs → hypertensive urgency Tx for hypertensive urgency/emergency ■ Labelol ■ Esmolol ■ Nitroprusside ● Adverse event? Cyanide toxicity ○ Tx? Amyl nitrate + thiosulfate OR hydroxocobalamin ■ Nicardipine ■ Clevidipine ■ “Observing LENT in North Carolina” ○ How fast to lower BP? Not by >25% in 1st hour BP 210/150. Pt complaining of WHOL. → SAH ○ NBS? NCCT ○ If NCCT negative, do LP → look for xanthochromia ■ Will be thousands of RBCs ○ Contrast with meningitis, which will have a less sudden onset ○ Other causes of RBCs in CSF? HSV (hundred of RBCs) ○ Pathophys? Rupture of berry aneurysm, most often in ACom ○ Tx? ■ Lower BP with labetalol or nicardipine ■ Give nimodipine to prevent post-SAH vasospasm ■ Phenytoin/fosphenytoin for seizure ppx ■ If rapidly increasing ICP, can ventilate pt and hyperventilate them ● Hyperventilation is fastest way of relieving increased ICP Pt with long hx of uncontrolled HTN that presents with sudden-onset, severe tearing CP that radiates to the back. ○ Pathophys? Cystic medial necrosis/degeneration ○ Other cause of aortic dissection? ■ Marfan’s ■ Ehlers-Danlos ■ Syphilis (destroys vasa vasorum) ■ Bicuspid aortic valve ○ First step? Give beta blocker ■ Alternative: nitroprusside ○ Imaging? Widened mediastinum on CXR ■ Also seen in anthrax hemorrhagic mediastinitis ○ Dx? ■ If stable → CT angio w/ contrast ■ If HDUS → TEE ○ Tx? ■ Stanford type A (ascending involved) → surgery ■ Stanford type B (descending aorta only) → medical management ○ Complications ■ Extension of proximal dissection → occlusion of RCA → inferior MI

351

Pt with hx thyroid problems. Gets CT scan with IV contrast. On the 2nd day after the scan, the pt is markedly tachycardia, mild HTN, temp 103F, AMS → thyroid storm ○ Causes? ■ Hx hyperthyroidism (especially if missed meds) ■ Iodinated contrast ■ Amiodarone ○ Lab findings? ■ TSH low ■ T3/T4 high ○ First step? Propranolol ■ Blocks conversion of T4 → T3 in the periphery (inhibits 5’-deiodinase) ○ Second step? PTU ■ Also blocks peripheral conversion ○ Third step? Wait 1 hr then give Lugol’s solution (supersaturated solution of potassium iodide) ■ Inhibits production of new thyroid hormone via the Wolff-Chaikoff effect ■ Iodine load prevents organification step in thyroid hormone synthesis ○ Fourth step? Can give dexamethasone/hydrocortisone ■ Prevents development of adrenal insufficiency ○ MC arrhythmia in thyroid storm? Afib

Pt with neck mass. Multiple relatives that died from thyroid cancer. Pt has episodes of severe HA & very high BPs (SBP ~ 250) → pheochromocytoma 2/2 MEN2 ○ Inheritance? AD ○ Mutated gene? RET ○ First step? Block alpha receptors ■ phenoxybenzamine (irreversible) ■ Phentolamine (reversible) ■ Labetalol (alpha-beta blocker) ○ Second step? Block beta ○ Associated syndromes? ■ VHL ■ MEN2A/B ■ NF-1

Pt with hx depression, taking MAO-I. Goes to restaurants and during meal starts to feel altered. SBP 250. → tyramine crisis ○ List of MAO-Is ■ Phenelzine ■ Tranylcypromine ■ Isocarboxazid ○ Tx? ■ Phentolamine ■ Labetalol

Pt with hx IVDU presents with fevers. New murmur present at LLSB. → tricuspid endocarditis ○ Embolic phenomena on exam ■ Osler’s nodes ■ Janeway lesions 352

○ ○ ○

First step? Draw blood cx Dx? TEE to look for vegetations Empiric tx? Vancomycin (cover MRSA)

ACUTE CORONARY SYNDROMES ● Pt comes into ED with 2-weeks of CP that occurs when they walk 2 blocks. Pain goes away with rest → stable angina ○ Stable angina is NOT an ACS ○ Do NOT need to take this pt to cath lab ○ NBS? Refer for stress test ● Pt with hx stable angina, but now sxs are worsening. Now has CP at rest → unstable angina ○ EKG findings? ■ T-wave inversions ■ ST depression ○ Troponin? NORMAL ● NSTEMI ○ EKG findings? ■ T-wave inversions ■ ST depression ○ Troponin? elevated ● STEMI ○ EKG findings? ■ ST elevation ○ Troponin? Elevated ● Risk stratification ○ HEART score ■ Predicts 6-week risk of major adverse cardiac event ○ TIMI score ■ Estimates mortality for patients with unstable angina and non-ST elevation MI ● Leads & vessel relationships ○ Inferior leads (II, III, aVF) → RCA infarct ○ V1-V4 → anterior septal infarct → LAD ○ V5-V6 → lateral wall infarct → LCX ○ New heart block → RCA infarct ■ RCA supplies AV node ● Drug contraindicated in RCA infarct? Nitrates ○ Pathophys? These pts are preload dependent. Nitrates are venodilators, so giving nitrate would take away their preload → cardiogenic shock ● Other pts that shouldn’t get nitrates? Pts take PDE-inhibitor ○ E.g. sildenafil for erectile dysfunction ● ACS management ○ First diagnostic step? Get EKG ○ Hyperacute T waves → ST depressions/elevations → Q waves ○ First therapeutic step? give 324 mg aspirin within 10 minutes of arrival ■ Has a mortality benefit 353

Dx? ■

Troponins ● Repeat 6 hrs later ■ CK-MB for reinfarction ● Troponin takes 7-10 days post-MI to fall ○ Initial Tx? ■ 324 mg aspirin ASAP ■ P2Y12 blockers/ADP receptor blockers (e.g. clopidogrel, prasugrel, ticagrelor) ■ Unfractionated heparin ■ For pain: ● Nitrates ● Morphine ■ O2 therapy ■ If going for PCI → GpIIbIIIa antagonists (e.g. eptifibatide, tirofiban, abciximab) ○ Definitive Tx? ■ PCI ● Door-to-balloon time venous graft ● Artery used? Internal mammary ● Vein used? Saphenous ○ MONA BASH mnemonic ■ M = morphine ■ O = O2 ■ N = nitrate ■ A = aspirin ■ B = beta-blocker ■ A = antiplatelets (P2Y12 blockers) ■ S = statin ■ H = heparin ○ Pt with cardiogenic shock 2/2 MI. NBS? intra-aortic balloon pump ■ How does it work? Inflates is diastole, prevents blood from going past it, which encourages perfusion of the coronary vessels and cerebral vessels ● Recall that coronary vessels fill during diastole ○ Discharge meds for pts with recent MI ■ Beta blocker ● Avoid in: decompensated HF ■ ACE-I ■ High-intensity statin ■ Aspirin ■ If stent → give dual antiplatelet therapy (e.g. aspirin + clopidogrel) MI complications ○ Pt with MI 3 days ago. Now with flash pulmonary edema & new holosystolic murmur at LLSB → interventricular septal rupture 354

Pt with MI 3 days ago with flash pulmonary edema. Echo shows lots of mitral regurg but normal ventricular size → acute mitral regurg 2/2 papillary muscle rupture ■ Contrast with chronic mitral regurg, which will caused volume overload and systolic dysfunction ○ Pt with recent MI that becomes responses. +JVD and BP 60/palp. Difficult to hear heart sounds → free wall rupture ■ Often presents as tamponade ■ Beck’s triad = JVD + hypotension + muffled heart sounds ○ Pt with hx MI s/p PCI 7 days ago. Noticed blue toes and petechiae on lower extremities. Elevated Cr. → blue toe syndrome/cholesterol embolization syndrome ■ Classic after cardiac catheterization ■ Pathophys? When stenting open vessels, cholesterol can embolize and obstruct small arteries in kidneys and LEs ■ Alternate PE finding? Livedo reticularis ○ Pt s/p PCI a few hours ago. Access was established through the femoral artery. Pt is profoundly hypotension and complaining of severe back pain. → RP bleed ○ Pt is progressively hypotensive immediately after PCI → cardiac tamponade 2/2 puncture during PCI ■ NBS? Surgery ○ Pt with MI 3 days ago, presenting with sudden-onset severe abdominal pain → acute mesenteric ischemia ■ Dx? Mesenteric angiography ○ Pt with recent MI s/p PCI, presenting with severe LE pain → acute limb ischemia ■ NBS? Heparin ■ Dx? LE arteriography ■ Tx? Vascular surgery ○ Pericarditis a few days after MI → fibrinous pericarditis ○ Pt with MI 6 weeks ago, now presenting with CP better with leaning forward → Dressler’s syndrome = autoimmune pericarditis ■ EKG findings? Diffuse ST elevations & PR depressions ■ Tx? NSAID + colchicine ● If renal insufficiency → consider steroids ● Colchicine is better than steroids, but it decreases the recurrence risk (whereas steroids increase it) ○ MC cause of death with 24 hrs of MI? V-fib Pt with multiple episodes of CP that tend to happen at night. Pt has ST elevations & troponin elevations but no occlusion found in cath lab → Variant angina = Prinzmetal angina ○ Tx? CCB → diltiazem or amlodipine ○ Contraindicated meds? Sumatriptan (causes vasoconstriction)

ARRHYTHMIAS ● Definition of prolonged QT? ○ QTc > 440ms in men or > 460ms in women ○ QTc = QT/sqrt(RR interval) ● Things that prolong the QT interval ○ Electrolyte abnormalities ■ Hypokalemia ■ Hypocalcemia ○ Antibiotics 355

■ Azithromycin ■ FQ ○ Antiarrhythmics ■ Class Ia: procainamide, quinidine ■ Class III: amiodarone, sotalol Congenital long QT ○ Romano-Ward ■ Inheritance? AD ■ Get ventricular tachyarrhythmias ○ Jervell and Lange-Nielsen syndrome ■ Inheritance? AR ■ Congenital hearing loss ○ Tx of congenital long QT? ■ 1st line: beta-blocker for life ● Prevents sympathetic discharge that can cause arrhythmias ■ ICD to shock them out of ventricular tachyarrhythmias Common cause of inappropriate ICD discharge? Magnet exposure ○ E.g. MRI scan ○ Tx? Use magnet to disable device, cardiology will have to reset it Young pt complaining of palpitations. EKG shows a short PR interval, delta wave, and wide QRS. → WolfParkinson-White ○ Pathophys? Antidromic atrioventricular reentrant tachycardia (AVRT) ○ Pathway? Signal goes down Bundle of Kent to ventricles, then up the AV node to the atrium ○ Tx? Procainamide ■ Adverse event? Drug-induced lupus (anti-histone Ab) ○ Contraindicated meds? AV blocking agents ■ Beta-blocker ■ Verapamil or diltiazem ■ Digoxin (muscarinic receptor agonist) ○ Contrast with orthodromic AVRT ■ Pathway? down the AV node to ventricles then up the accessory pathway ■ EKG findings? Narrow QRS Heart blocks ○ 1st degree AV block ■ Prolonged PR interval, no dropped beats ○ 2nd degree Mobitz I = Wenckebach ■ Progressively prolonging PR interval, then dropped beat ○ 2nd degree Mobitz II ■ Fixed PR interval w/ dropped beats ■ High risk of progressing to 3rd degree ○ 3rd degree ■ No relationship b/t P waves and QRS complexes ■ P-P intervals and R-R intervals are constant ○ When are pacemakers indicated? ■ Mobitz II or 3rd degree ○ Acute tx? ■ Transcutaneous pacing ■ Atropine 356

● ●

Pt with regular narrow-complex tachyarrhythmia → SVT ○ Narrow complex because it arises from above the ventricles ■ Exception: SVT w/ aberrancy can be wide ○ NBSM? ■ Vagal maneuvers ■ Adenosine - stops conduction down AV node ● 3 doses: 6 mg → 12 mg → 12 mg ○ If above fails, do what? Attempt to control HR ■ Beta blocker ■ Verapamil or diltiazem ○ If pt with SVT is HDUS? Proceed to synchronized cardioversion ■ Avoids R on T phenomenon (shock while the ventricles are depolarizing can throw the person into Torsades → V-fib) ○ Which rhythms are treated with unsynchronized cardioversion (defibrillation)? ■ V-fib ■ Pulseless V tach Sawtooth pattern → atrial flutter ○ Often seen best in leads II, III, aVF 3 or more different P wave morphologies → multifocal atrial tachycardia ○ Population? ■ Pt with COPD ○ Tx? ■ Stop smoking ■ Verapamil or diltiazem Regular wide-complex tachyarrhythmia → ventricular tachycardia ○ If no pulse → initiate ACLS ■ Epi / amio / epi / amio every 2 minutes ○ If pulse & HDUS → synchronized cardioversion ○ If stable → amiodarone ■ 2nd line: lidocaine Management of PEA or asystole? Chest compressions + epi

VALVULAR DISORDERS ● Athlete brought to ED because he collapsed during basketball game? HOCM ○ Inheritance? AD ○ Mutated proteins? Sarcomere proteins (e.g. beta myosin heavy chain) ○ HOCM murmur ■ Best heard at LLSB ■ Does not radiate ■ Pulsus bisferiens (small upstroke then big upstroke) ■ Increase preload → increase EDV → reduce LVOT obstruction → softer ■ Increase afterload → increase EDV → softer ○ AS murmur ■ Best heard at RUSB ■ Radiates to carotids ■ Pulsus parvus et tardus ■ Increase preload → increased EDV → more flow across stenotic valve → murmur louder 357

● ●

● ● ●

Tx of HOCM? ■ Beta-blocker ● Mechanism? More time in diastole → increased filling ■ If arrhythmias → place ICD Pt with Marfan’s has murmur with midsystolic click → MVP ○ Pathophys? Myxomatous degeneration of valve Holosystolic murmur at apex that radiates to axilla → mitral regurg ○ Complication? Volume overload → systolic dysfunction ○ Cause of acute MR? Papillary muscle rupture Systolic ejection murmur at RUSB, radiates to carotids → aortic stenosis ○ Cause of early AS? Bicuspid aortic valve “Blowing” diastolic murmur heard best at LSB + wide pulse pressure → aortic regurg Holosystolic murmur at LLSB → VSD ○ Cause in adults? Interventricular septal rupture after MI Pt with hx CHF with crackles, JVD, hepatomegaly, LE edema. Hypervolemic hypernatremia. → CHF exacerbation ○ Triggers? ■ Nonadherence to meds ■ Missing dialysis ■ Metabolic stress (like infection) ○ R heart failure ■ Hx COPD causing cor pulmonale or CF ■ +JVD, +hepatomegaly, +LE edema ■ No crackles ■ Elevated CVP ■ Normal PCWP (proxy for LA pressure) ○ L heart failure ■ All signs of R heart failure + pulmonary edema ■ PCWP > 18 mmHg ○ Tx for dyspnea? ■ Nitroglycerin ● Mechanism? Venodilator that decreases preload → reduces myocardial O2 demand ■ Furosemide (loop diuretic) ● If not responding adequately? Add thiazide ■ If pt is still dyspneic? NIPPV = non-invasive positive pressure ventilation (e.g. BiPAP) ● Mechanism? Decreases WOB by keeping alveoli open. Also increases intrathoracic pressure, so decreases preload. ○ Tx of cardiogenic shock in s/o CHF exacerbation? inotropes ■ Dobutamine ■ Milrinone (PDE-inhibitor, “inodilator”) ● Mechanism? PDE inhibitor → increased cAMP → increased cardiac contractility & vascular SM relaxation ○ Young pt with URI sxs a few days ago. New S3 heart sound. +JVD. Pulmonary edema on CXR. Echo shows global enlargement of ventricles → viral myocarditis ■ Virus? Coxsackie B ■ Idiosyncratic exam finding? Tachycardia not proportionate to fever 358

■ ●

Treat like a CHF exacerbation

Syncope ○ Young pt that was sweating & pale, then passed out → vasovagal syncope ○ No prodrome, pt loses consciousness, comes back quickly w/o sequelae → cardiogenic syncope ■ Likely due to dangerous arrhythmia ○ Pt with neuro deficit or postictal period → neurogenic syncope (seizures or stroke)

-----------------------------------------------------------------------------------------------------------------------------

Episode 182: Comprehensive NBME Emergency Medicine Shelf Review Series 2 ●

Pt with 2 days of n/v, not eating well. Glucose 300, bicarb 5. Na+ 130. → DKA ○ Pathophys of DKA? ■ Absolute insulin deficiency → no inhibition of glucagon → glucagon stimulates production of ketone bodies ■ Often type 1 diabetics (no insulin) ■ Type 2 diabetics have enough insulin to inhibit glucagon ○ Dx? ■ Ketone bodies (e.g. acetoacetate, beta-hydroxybutyrate) ● Other causes? ○ Starving ○ Hyperemesis gravidarum ○ Alcoholics ○ Electrolyte abnormalities? ■ Pseudohyponatremia = actual Na+ is normal, but measured Na+ is low due to very high glucose ● Add 1.6 to Na+ for every hundred of glucose above 100 ● Corrected Na+ = 1.6 x (glucose - 100)/100 ● Tx? Will correct itself as you treat DKA/HHNS ■ Hyperkalemia on labs BUT depleted total body K+ ○ Tx? ■ 1st: Fluids ● 2 L bolus of NS @ 1 L/hr ● Then switch to 0.45% NS ■ 2nd: Insulin ● Potassium requirements for insulin therapy ○ K+ must be ABOVE 3.3 ○ If between 3.3-5.3 → add potassium to fluids ○ If > 5.3 → don’t need to add potassium ● Type of insulin? IV Regular insulin (rapid acting) ● Rate? 0.1 u/kg/hr ● When to add dextrose? When glucose is 38 ○ RR > 20 ○ HR > 90 ○ WBC < 4k or > 12k ● SIRS + source (e.g. pneumonia, UTI) → sepsis ● Sepsis + sign/sxs organ dysfunction → severe sepsis ○ Ex: elevated Cr, elevated LFTs, AMS, lactic acidosis ● Pt that’s hypotensive despite volume resuscitation → septic shock ● Most likely pulmonary complication? ARDS ● Management? ○ Recognize it early ○ Give IVF (2 L bolus) ■ If not responding to IVF, give norepinephrine through central line ■ If not responding to norepi, give vasopressin ○ Start broad-spectrum abx ASAP ■ Ex: vanc + carbapenem ■ If pulmonary cause, vanc + ceftriaxone + azithromycin + FQ ■ If neutropenic pt, also cover pseudomonas ● Ceftazidime ● Cefepime ● Carbapenem ● Pip-tazo ■ If pt not improving on vanc + ceftazidime → add antifungal (e.g. caspofungin) ○ Source control ■ Remove infected line ■ I&D abscess ●

Pt with cellulitis. ○ Bugs? Staph (including MRSA) + Strep ○ Dx? Clinical! ■ Don’t need to swab ○ Tx? ■ To cover MRSA: Clinda or TMP-SMX ■ To cover Strep: cephalexin ○ Admit if: systemic sxs/signs or hypotensive ■ Tx with vancomycin Pt with fever + well-demarcated, tense, erythematous rash on face → erysipelas ○ Bug? Strep pyogenes ○ Dx? Clinical ○ Tx? ■ Penicillin/amoxicillin 364

● ● ●

● ●

■ Cephalexin ■ If systemic signs/sxs → admit & give IV cefazolin Pt with something that looks like cellulitis. Area is exquisitely tender to palpation. There are areas of skin discoloration. Bullae or crepitus on exam. Temp 100.9, HR 150 → necrotizing fasciitis ○ Imaging? Gas within soft tissue ○ Labs? ■ Elevated creatinine kinase → C. perfringens ■ Elevated Cr ■ CBC abnormalities (e.g. anemia, thrombocytopenia) ○ Tx? ■ Early abx ● Vanc + carbapenem + clindamycin (inhibit toxin synthesis) ■ Surgical debridement ■ If due to tetanus vaccine, give it Pt with fluctuant mass → abscess ○ Tx? I&D, often don’t need to give extra abx Woman with fluctuant mass on labia → bartholin gland abscess ○ Tx? Drain it + insert Ward catheter Pt with painful masses in axillae → hidradenitis suppurativa ○ Pathophys? Inflammation of apocrine sweat glands ○ Tx? surgery Pt with painful mass in intergluteal fold → cyst ○ Tx? Surgery Pt that is a gardener with subcutaneous nodules following the course of a lymph node chain → sporotrichosis ○ Bug? Sporothrix schenckii ○ Tx? Itraconazole for weeks Pt with abrupt-onset fever/chills, HA, myalgias in December → influenza ○ Presentation in a kid? Diarrhea ○ Pt with flu that was improving then got febrile again. Consolidation on CXR → secondary bacterial pneumonia ■ Bug? MRSA ■ Pathophys? Flu causes impaired mucociliary clearance → predisposed to superimposed bacterial infection ○ Tx that is contraindicated in kids? Aspirin due to the risk of Reye syndrome ○ Neurologic complication? GBS ○ Dx? ■ If low-risk pt w/ classic sxs → none needed ■ If high-risk pt (e.g older person, chronic conditions) → rapid antigen test ■ Rapid antigen detection test ● NOT sensitive ● Pretty specific ○ Tx? Neuraminidase inhibitor ■ Timeline to initiate? Within 48 hrs of symptom onset ■ If POing → oseltamivir ■ Zanamivir (inhaled) ● Contraindications? Airway disease (COPD, asthma, CF) ● Bronchospastic agent 365

■ If not POing → IV Peramivir Tx of secondary bacterial pneumonia → vancomycin or linezolid (cover MRSA)

Herpes manifestations ○ Oral herpes ○ Genital herpes ○ Herpetic whitlow (finger) ○ Lesion on other part of skin (often a wrestler or body builder) ○ Herpes keratitis ■ Slit lamp exam shows dendritic lesion w/ fluorescein staining ○ HSV encephalitis ■ Temporal lobe enhancement ■ LP with RBCs + lymphocytic predominance ■ Tx? IV acyclovir

Fever + vesicular rash, lesions in different stages of healing → chicken pox (primary varicella) ○ Contagious until all lesions have crusted over ○ Contrast with smallpox ■ Lesions in same stage ■ Lesions on extremities ■ Lesions tend to be larger

Pt with skin tingling/pain then vesicular lesions in a dermatomal distribution → Shingles (varicella reactivation) ○ Tx? Acyclovir within 72 hrs rash 24 yo F with new partner presenting with sore throat. Temp 102F. Exam with cervical lymphadenopathy, exudative pharyngitis, splenomegaly. ○ Dx? Monospot ■ If negative, consider CMV ○ Associations ■ Hogdkin’s lymphoma ■ Burkitt’s lymphoma ● Starry sky on histology ● t(8/14) ■ Nasopharyngeal carcinoma ○ Histology? Atypical lymphocyte (Downey cell) ○ Tx? ■ Supportive care ■ Avoid contact sports for 3-4 weeks ● Avoid splenic rupture ○ Pt with mono has signs/sxs of airway compromise? Give steroids ■ Steroids for airway compromise, brain (AMS), or CBC abnormality

Pt brings in a child with temp 103F, runny nose, cough, conjunctival injection. Exam shows white spots on buccal mucosa (Koplick spots). Rash that started on the head and spread down the body → measles ○ Management? ■ Contact health department ■ Give Vit A 366

Pt with high fevers. Recent travel to Africa. Pt has been bleeding from ears & nose → hemorrhagic fever (e.g. Ebola) ○ Pathophys? Inflammation → increased vascular permeability ○ NBS? Isolation + appropriate PPE ○ Tx? Supportive

Pt with exposure to rodents now with fever + hypotension + AKI → hantavirus hemorrhagic fever w/ renal syndrome

24 yo MSM with 3 weeks of low-grade fevers + sore throat + rash. Temp 101F + generalized lymphadenopathy → HIV ○ Dx? Antigen (p24)/antibody testing ○ Tx? HAART ■ 2 NRTIs + one other drug (e.g. protease inhibitor, integrase inhibitor) Opportunistic infections in HIV ○ Esophagitis → candida, CMV, HSV ■ Tx for candida? fluconazole ○ Thrush → candida ■ Tx? Nystatin or fluconazole ○ Pt with low CD4 gets treatment for HIV then starts having fevers + flu-like illness → Immune reconstitution inflammatory syndrome (IRIS) ○ Retinitis → CMV ■ Tx? Ganciclovir ■ If resistant to ganciclovir? foscarnet ○ Pneumonia ■ MC cause? Strep pneumo ○ Profound hypoxia + increased LDH + interstitial infiltrates on CXR → PCP ■ Tx? IV TMP-SMX ● Alternatives? Pentamidine or primaquine ■ Steroid criteria? ● PaO2 < 70 ● A-a gradient > 35 ○ Seizures + multiple ring-enhancing lesions on imaging → Toxo ■ Dx? Brain CT or MRI ■ Tx? Pyrimethamine-sulfadiazine ○ High fevers + nuchal rigidity + LP with high OP → cryptococcal meningitis ■ Dx? Latex agglutination test ■ Tx? Amphotericin B + flucytosine (then 1 year maintenance with fluconazole) ○ Chronic watery diarrhea + acid-fast oocysts → cryptosporidium ■ Tx? Nitazoxanide ○ Pt with late-stage AIDS (e.g. CD4 40) with diarrhea → MAC or CMV diarrhea ■ Tx for MAC? REC = rifabutin + ethambutol + clarithromycin ○ Rash with brown-purple nodules → Kaposi’s sarcoma ○ Shingles in a non-dermatomal distribution Prophylaxis in HIV ○ CD4 < 250, lives in southwest US → cocci ppx with itraconazole ○ CD4 < 200 → PCP ppx with TMP-SMX

367

○ CD4 < 150, lives in midwest → histoplasma ppx with itraconazole ○ CD4 < 100 → toxo ppx with TMP-SMX ○ CD4 < 50 → MAC with azithromycin Healthcare worker that gets stuck with needle from HIV-positive pt → start post-exposure ppx within 2 hrs of being stuck ○ PEP = HAART ○ RF that increase risk of transmission ■ High viral load ■ Deep injury ■ Stick goes into blood vessel

IVDU with 2 weeks fever + new heart murmur → endocarditis ○ MC bug? Staph aureus ■ #2 MC bug = Strep viridans ■ #3 MC bug = Enterococci ○ Bug in pt with new prosthetic valve (< 60 days)? Staph epidermidis ○ Bug in pt s/p GI or GU procedure? Enterococcus ○ Which valve most commonly affected? ■ Tricuspid in IVDUs ■ Bicuspid in other pts ○ Exam findings? ■ Osler’s nodes ■ Janeway lesions ■ Splinter hemorrhages ○ Dx? ■ 3 sets of blood cx from 3 sites (before abx) ■ TEE ○ Empiric tx? ■ Vanc + gent +/- rifampin ○ Abx ppx? ■ What abx? amoxicillin ■ What procedures? ● dental procedures ● GI/GU procedures in the setting of GI/GU infection ● Respiratory tract procedures ● Procedures on infection skin or MSK tissue ■ Who? ● Hx of endocarditis ● Prosthetic heart valve ● Heart transplant with abnormal valve function ● Unrepeated cyanotic congenital heart defect

Pt with puncture wound OR animal bite OR unsterile abortion. Pt reports jaw stiffness and weird muscle spasms. On exam, rigid extremities and back in extension → tetanus ○ Pathophys? Tetanus toxin cleaves SNARE proteins → can’t release glycine & GABA (inhibitory NTs) → spastic paralysis

368

Mom with poor prenatal care. Unsanitary delivery conditions or poor care of umbilical stump. When baby is a week old, it is having poor feeding and clenched fists → neonatal tetanus ○ Tx? ■ Admit pt ■ Quiet room (environmental stimuli can cause muscle spasms) ■ Tetanus immune globulin ■ Tetanus toxoid vaccine (different arm) ■ Debride necrotic tissue ■ Benzos to reduce muscle spasms ■ If autonomic hyperactivity, give beta-blocker ■ Often intubate these pts & paralyse muscles (e.g. vecuronium, rocuronium) Pt that is bitten by a dog. Weeks later, he starts behaving weird. Hyperactive episodes + hallucinations + hypersalivation + autonomic instability + doesn’t want to drink water that progresses to coma → rabies ○ Causes of rabies ■ Bites from dog, bat, shunk, raccoon ○ Very poor prognosis ○ Dx? Postmortem brain pathology ○ Post-exposure prophylaxis? ■ What? rabies immune globulin + rabies vaccine (2 different sites) ■ When is PEP indicated? ● Any contact with bat ● Stray animal ● Animal that can’t be observed ■ If it’s a domestic animal that can be observed, quarantine it and observe for 10 days

Pt that is a missionary that just came back from Africa. Has been having fevers every 3-4 days. PE shows splenomegaly. Labs show low Hgb and low platelets → malaria ○ Vector? Anopheles mosquito ○ Dx? Thick & thin blood smear ○ Plasmodium falciparum - worst ■ Complications: ● Cerebral malaria ● Jaundice ● Renal impairment ○ Plasmodium vivax/ovale - hypnozoites can be dormant in the liver ■ Tx? Primaquine ● Contraindication? G6PD deficiency ○ Plasmodium malariae ○ Tx? ■ Artemether-lumefantrine ■ Atovaquone-proguanil ■ Quinidine + doxy ● Quinidine adverse effect? QT prolongation

Pt that is longtime alcoholic presenting with severe LLE cellulitis. He recently swam in a coastal area or consumed shellfish. → Vibrio vulnificus ○ Tx? ■ Doxycycline + cefepime 369

● ●

● ●

■ Debridement Pt exposed to freshwater with cellulitis → Aeromonas hydrophila Pt with folliculitis in the shape of their swimsuit, recently went in hot tub → Pseudomonas ○ Tx? It will usually clear on its own ■ If they need abx, give FQ or aminoglycoside Pt that uses hot tubs a lot. Now with respiratory sxs, crackles on exam → hypersensitivity rxn 2/2 thermophilic actinomycete Pt that works with water presents with skin lesions. Granulomas on biopsy → mycobacterium marinum

Watery diarrhea ○ Cruise ship exposure → norovirus ○ Kid → rotovarius ○ Potato salad/picnic food then vomiting with 6 hrs → Staph aureus ○ Profound watery diarrhea → Vibrio cholera ■ Tx? Oral rehydration solution ○ HIV pt → cryptosporidium

Blood diarrhea ○ EHEC ○ Shigella ○ Campylobacter ○ Salmonella ○ Pseudoappendicitis → Yersinia enterocolitica ○ Also live abscess → Entamoeba histolytica

Hemolytic uremic syndrome ○ Causes? E. coli O157:H7 or Shigella ○ MAHA (schitoscytes) + thrombocytopenia + elevated Cr

Diarrhea after recent abx → C. diff ○ Can be watery OR bloody ○ Tx? Oral vancomycin

Tx of diarrhea ○ Rehydrate ○ Antiemetic (e.g. ondansetron, metoclopramide) ○ If question is making you pick abx? FQ ○ When are abx contraindicated? HUS

Pt that consumed exotic fish now with flushing, HA, autonomic instability → scombroid poisoning ○ Pathophys? Histamine toxicity ○ Tx? Antihistamine (e.g. diphenhydramine) ○ Contrast with ciguatera poisoning, which will have HAs,myalgias, numbness/tingling of lips/tongue/hands/feet, reversal of hot-cold sensation (“hot things feel cold and vice versa”)

Pt that breeds dogs that has fever that cycles (“undulant fever”) → brucellosis ○ Tx? Doxycycline 370

○ Pt with osteomyelitis that was recently bitten by dog → Capnocytophaga canimorsus ○ Tx? ■ Amox-clav ■ clindamycin ○ Pt with flu-like sxs or pneumonia (more rarely, endocarditis) that breeds birds → Chlamydia psittaci ○ Tx? ■ Doxy ■ Azithromycin

Pt that rears cattle/sheep/goats that has been fevers, pneumonia, or endocarditis → Q fever 2/2 Coxiella burnetii ○ Tx? Doxycycline

Pt with recent tick bite that has fever, HA, myalgias, rash. CBC with anemia & thrombocytopenia. → Ehrlichiosis ○ Vector? Lone star tick ○ Tx? Doxycycline for everyone (including kids + pregnant women)! ○ Can progress to sepsis

Pt with recent tick bite that has high fevers, HA, rash that started on the palms/soles → Rocky mountain spotted fever 2/2 Rickettsia rickettsii ○ Geographic association? North Carolina ○ Dx? Weil-Felix test ○ Tx? Doxycycline for children and adults ■ Chloramphenicol for pregnant women

When should kids < 8 yo get doxy? ○ Rocky mountain spotted fever ○ Ehrlichiosis

Pt that was swimming in Hawaii that now has conjunctival injection + generalized lymphadenopathy + fevers → leptospirosis ○ Visible on dark field microscopy ○ Tx? ■ Penicillin ■ Doxycycline

Pt with rabbit exposure now with ulcer + swelling of regional lymph nodes → tularemia 2/2 francisella tularensis ○ Alternate presentation? ■ Eye inflammation + preauricular lymphadenopathy → oculoglandular tularemia ○ Possible bioterrorism agent ○ Tx? ■ Doxycycline ■ Aminoglycosides

Pt with bullseye rash Lyme disease ○ Vector? Ixodes tick 371

○ ○

Geographic association? Northeastern US Stage 1 = erythema chronic migrans (bullseye rash) ■ Tx? ● Children age 8+ and adults → doxy ● Younger than age 8 → amoxicillin Stage 2 = heart block, bilateral Bell’s palsy, disseminated rash, arthralgias ■ Tx? ● Ceftriaxone or cefotaxime (3rd gen cephalosporin) Dx? ■ ELISA (screening) ■ Western blot (confirmatory)

Pt that works for the FBI/national security OR works in wool-sorting facility that’s been having hemoptysis. Widened mediastinum on CXR → pulmonary anthrax ○ Alternate presentation? ■ Macule that becomes an ulcer and then a black eschar → cutaneous anthrax (good prognosis) ○ Pathophys? Spore-forming bacteria ○ Causes? ■ Inhalation of spores from wool ■ Bioterrorism ○ Tx? Cipro + meropenem + linezolid

Necrotic draining lymph nodes → bubonic plague caused by Yersinia pestis ○ Tx? Aminoglycoside ■ Toxicity? Ototoxicity + nephrotoxicity ■ Other drugs with ototoxicity + nephrotoxicity ● Vancomycin ● Cisplatin ○ Prevention? Amifostine ● Ethacrynic acid (no sulfa loop diuretic)

Pt with severe bone pain + massive hepatosplenomegaly + high fevers. Recent travel to Southeast Asia → Dengue ○ Vector? Aedes mosquito ○ Labs to follow? CBC ■ They can have hemolysis ■ May require RBC or platelet transfusion ○ Avoid aspirin & NSAIDs because they may have thrombocytopenia

Dengue-like illness → chikungunya

Pt with fever + rash + conjunctivitis + joint pain. On exams, often a pregnant woman that delivers NB with microcephaly → Zika ○ Neuro complication? GBS

Pt with very high fever + severe abdominal pain and distention + bradycardia. A few days after these sxs, a salmon-colored lesion develops on the abdomen → typhoid fever ○ Bug? Salmonella typhi (not salmonella enteritidis!) 372

○ ○

Tx? FQ If severely ill? Add dexamethasone

Pt that returns from international travel. Did not get any vaccines prior to travel. Pt had a flu-like illness that progressed to hemoptysis + black emesis + proteinuria → yellow fever ○ Bug? Flavivirus ○ Tx? Supportive care

Pt that consumed pork that is having seizures → neurocysticercosis ○ Tx? ■ Antiepileptic for seizures ■ Albendazole (+/- praziquantel) ■ Corticosteroids ○ Dx? Calcified lesions on non-contrast head CT ○ Pt that immigrated to the US from Africa with painless ulcer somewhere on skin. Starts having fevers that come and go. Family says that pt is very somnolent. Pt becomes comatose and dies. → Sleeping sickness = African trypanosomiasis ○ Vector? Tsetse fly

Pt with unilateral periorbital edema + painful swelling around bite site → Chagas ○ Vector? Riduvid bug ○ Complications? ■ Dilated cardiomyopathy ■ Achalasia ■ Megacolon (2/2 destruction of enteric nervous system) ○ Tx? ■ Benznidazole ■ Nifurtimox

Egyptian pt with hematuria → schistosomiasis ○ Vector? Snail ○ Complication? Bladder cancer

Bloody diarrhea + elevated eos + liver abscess → entamoeba histolytica ○ Tx? Metronidazole

Pt that traveled abroad and ate lots of street food. Dry cough + severe constipation + elevated eos → Ascaris lumbricoides ○ Pathophys? ■ Worms travel from small intestine up to lungs → pneumonitis (Loeffler's syndrome) ■ Large worm burden → SBO ○ Tx? Albendazole

Little kid with perianal itching → pinworm (Enterobius vermicularis) infection ○ Dx? Scotch tape test ○ Tx? Albendazole or pyrantel pamoate 373

Pt with IDA that works in construction. See a bug burrowing until that pt’s skin → hookworm (Ancylostoma duodenale or Necator americanus) infection ○ Bug? ○ Tx? Albendazole

Nematode that causes myalgias? Trichenella spiralis ○ Pathophys? Cysts in striated muscle ○ Tx? albendazole

Tapeworms ○ megaloblastic anemia 2/2 B12 deficiency → diphyllobothrium latum ○ Undercooked beef → Taenia saginata ○ Undercooked pork → Taenia solium ○ Tx? Praziquantel

Pt s/p stem cell transplant comes in with rash + diarrhea → GVHD ○ Tx? Steroids

Pt s/p recent liver transplant now complaining of severe abdominal pain. Signs/sxs of peritonitis → bile leak ○ NBS? Surgical consult ○ Dx? HIDA scan if not clear

Infection in recent kidney transplant recipient → BK polyomavirus

Pt with recent heart transplant presenting with rejection ○ NBS? High-dose methylprednisolone (IV corticosteroid) ○ If bradyarrhythmia → isoproterenol ■ Mechanism? Beta-1 and beta-2 agonist ○ If cardiogenic shock → dobutamine or milrinone

-------------------------------------------------------------------------------------------------------------------------------

Episode 184: NBME weird Most common complications of: 1. cleft lip/palate → Chronic OM/ speech problems 2. BM transplant → GVHD 3. Pregnancy related complication of Anti-phospolipid antibody → recurring spontaneous abortions. 4. first 20 wks of pregnancy → spontaneous abortion 5. terminal disseminated malignancy → Cachexia. 6. AAA → rupture 7. Chronic HTN → LVH 8. first few hours of life in baby of diabetic mom → hypoglycemia 9. meckel’s diverticulum → bleeding 10. diverticulosis → diverticulitis. 374

11. Appendicitis → periappendiceal abscess 12. cholilithiasis → biliary colic 13. Upper UTI → hydronephrosis 14. BPH → obstructive uropathy 15. DM (esp T1) → insulin induced hypoglycemia 16. Infectious complication of chronic granulomatous disease → Pneumonia 17. systemic amyloidosis → renal failure 18. left atrial thrombus → embolization 19. ganglion cyst resection → recurrence. 20. acute pancreatitis within 1-2 weeks → pancreatic pseudocyst. 21. Influenza → primary influenza pneumonia or secondary bacterial superinfection Abbreviations for the rest of the text: MC = most common MCC = most common cause MCCOD = most common cause of death MSC = most serious complication ML = most likely MLM = most likely mechanism MLCP = most likely clinical presentation 1. MCC of infx in burn pts → P. aeruginosa sepsis 2. Most imp intervention associated w/ decreasing risk of ocular complications in measles pt → Vit A supplementation. 3. MCCOD in hx of chronic HTN → Acute MI 4. MCC of arrhythmia in STEMI → premature ventricular contraction 5. Most serious complication (MSC) + MCCOD of ARF → myocarditis 6. 3 complications of O2 therapy → bronchopulmonary dysplasia, retinopathy of prematurity, IVH (Divine doesn’t consider this a complication but he still mentions it) 7. MCC of esophageal varices → Portal HTN 8. MSC of hirschsprung disease → death due to enterocolitis from super dilated bowel. 9. MSC of UC → toxic megacolon. 10. Most likely outcome of actinic keratosis → resolution of disease 11. most important mechanism underlying hyperglycemia in T1DM → gluconeogenesis 12. MSC of ICP → Brain herniation 13. MCC of cyanide poisoning → house fire 14. MCC of fatty change in liver → chronic alcoholism 15. MCC of skin abscess → S. aureus 16. MCC of death in Ehlers danlos syndrome → aortic dissection 17. Most likely barrier to proper healing of wound → persistent infection 18. most likely cause of rejection in transplant pt → acute rejection 375

19. most likely malignant complication in pt on chronic immunosuppressant → SCC of skin. 20. MCC of infx overall in transplant recipient → CMV 21. MCC of infx in solid organ transplant (heart, kidney, lung, pancreas) → candida 22. MCC of infx in BMT → aspergillus (put on voriconazole). #2 is candida. 23. Most likely organ to dev complication in pt with lupus → kidney 24. MC cardiac finding in SLE→ fibrinous pericarditis 25. MCC of Drug induced lupus → procainamide 26. MCCOD in SLE → Infx from immunosuppression 27. MC presenting complaint in pt dx w/ scleroderma → raynaud's phenomenon (skin most commonly involved). 28. MCCOD in Systemic sclerosis → respiratory failure 29. MC HIV serotype in the US → HIV-1 30. Most likely mechanism(MLM) of transmission of HIV in US → men having sex with men 31. Most likely cause of HIV in HCW → Needle stick injury 32. Most likely cause of infx in HIV pt CD4 count of 25 → PCP pneumonia or systemic candida infx. 33. MLM behind weight increase in hospitalized pts → increased total body sodium → due to increased blood volume and hydrostatic pressure in blood stream. 34. MLM in Pt with hyponatremia and increased urine osmolality → ectopic ADH production 35. Most likely clinical presentation in pt on HCTZ w/ EKG showing prolonged QT interval + U wave → muscle weakness and fatigue due to hypokalemia 36. MSC of perineal cellulitis → necrotizing fasciitis. 37. MCCOD in ICU → septic shock 38. Boy with a genetic mutation most likely pathogenesis → Enzyme defect 39. mechanism of disease in 25 yo 6ft male, infertile, micropenis → Klienfelters (genetic nondisjunction) 40. MCCOD 1mo – 1 year → SIDS. 41. MCCOD 1 – 44 years → Motor vehicle accidents 42. MCC of blindness in elderly → macular degeneration 43. MC valvular abnormality in elderly → AS 44. No.1 RF for development of pressure ulcers → increased pressure on capillaries 45. MCC of anemia in alcoholic → anemia of chronic disease (be careful not to pick megaloblastic) 46. MCC of thrombocytopenia in healthcare setting → use of heparin 47. MCC of HTN in young reproductive age female → OCP use. 48. MCCOD in pt struck by lightning → cardiorespiratory arrest. 49. ML malignant complication of pt exposed to radiation → acute leukemia. (papillary thyroid only if radiation is to head and neck) 50. Most damaging UV radiation to skin → UVB 51. MSC in low BMI pt → ventricular arrhythmia from hypokalemia 52. MCC of Vit K deficiency in hospitalized pt → use of Abx. 53. Most likely bone to have mets from cancer elsewhere in body → vertebral column 54. No.1 cause of bone mets to vertebrae → Breast Ca. 2. Prostate ca (osteoblastic mets). 55. If there are mets in liver, most likely primary malignancy → lung Ca 376

56. Brain mets, most likely primary malignancy → lung ca 57. Lung mets, most likely primary malignancy → breast ca. 58. Malignancy that may develop in the future in retinoblastoma pt → osteosarcoma 59. Most common mutation overall that gives rise to malignancy → P53 mut. 60. MC valvular complication in marfan’s syndrome → Mitral valve prolapse. 61. Most common neoplastic complication of HIV → Kaposi sarcoma 62. Pt admitted to hospital in US over 65 yrs most likely dx on admission → CHF 63. Most likely initial presentation in RF → migratory polyarthritis. 64. Most common mutation (chromosome) that gives rise to HOCM → Chromosome 11. 65. ML finding on further eval of 55 yo male presenting with Hb of 8, mcv 60 → Polyps/CRC 66. MSC of SCD → acute chest syndrome 67. ML trigger of hemolytic episode in G6PD → acute infection 68. Plt dysfxn most likely presenting clinical complaint → nose bleeds 69. ML infx due to blood transfusion → CMV infection (don’t choose HEP C) 70. MC infectious precipitant of an acute COPD exacerbation → H. influenza 71. ML clinical complication of barett-s esophagus → Esophageal ulcers w/ strictures. 72. MCCOD in cirrhosis → rupture of esophageal varices. 73. ML vessel involved in bleeding gastric ulcer→ Left gastric A. 74. Bleeding duodenal ulcer → gastroduodenal A. 75. MCC of cirrhosis in kids → Alpha 1 AT def. 76. no. 1 RF for hepatic adenoma → OCP use. 77. no.1 RF for liver angiosarcoma → Vinyl chloride exposure. 78. no. 1RF for cholangiocarcinoma in US → Primary sclerosing cholangitis 79. No. 1 RF for chronic pancreatitis in a child → cystic fibrosis 80. No. 1 RF for pancreatic adenocarcinoma → smoking 81. No.1 RF for ESRD in US → Diabetes 82. ML renal complication associated with aminoglycoside → ATN 83. ML Renal complication associated w/ SCD → symptomatic hematuria. 84. ML complication of VHL → bilateral RCC. 85. ML site of obstruction in nephrolithiasis → ureter or UVJ. 86. ML cause of sepsis in hospitalized pt → indwelling catheter. MC bug is e coli. 87. MCC of impotence >50 years → vascular insufficiency 88. most likely mechanism behind Endometriosis → reverse menses thru fallopian tubes 89. No.1 RF for female infertility/ectopic pregnancy → PID + scarring from previous PID 90. MC ovarian mass in pregnancy → Corpus luteum cyst 91. MSC of acromegaly → Death from dilated cardiomyopathy. 92. MLCP in pt dx with primary hyperparathyroidism → nephrolithiasis 93. #1 RF for hyperphosphatemia → Chronic renal failure 94. #1 RF for peripheral neuropathy in US → diabetes 95. #1 RF for fasting hypoglycemia in US → Alcoholism 96. MLCP in pt with paget’s disease → bone pain 377

97. MSC of tetanus → pneumonia and cardiac arrest 98. MLCP in pt with MG → ptosis 99. MCC of impetigo → S. Aureus 100. MLCP of superficial dermatophytosis → tinea pedis (ps. If pt has nail fungus or tinea capitis → tx with oral agent, oral terbinafine/itraconazole) 101. MCC of noncommunicating hydrocephalus in neonate → stricture/ obstruction in cerebral aqueduct of sylvius 102. Any pt with CNS infx MLM of disease → hematogenous spread 103. 2 biggest RF for carpal tunnel syndrome → pregnancy and RA. 104. #1 RF for optic neuritis → MS 105. MCC of blindness in HIV → CMV retinitis 106. Favorable prognostic factor in an individual with ALL → presence of t(12;21) translocation. 107. Most important prognostic factor in a pt with Hodgkin’s L → clinical stage of disease not type. 108. Ranson’s criteria for pancreatitis (criteria for admission to ICU) - GA LAW: o glucose >200 (Divine says 100 but 200 according to Uptodate) o AST >250 o LDH > 350 o Age >55 o WBC >16000 - 48 hours post admission → C and HOBBS o HypoCalcemia 10% o O2 sat 4 mg/dL o Six liters of fluid needed in 48 hr period 109. Most important prognostic factor in pt with RCC → invasion of renal vein 110. Most important prognostic factor in pt with melanoma → depth of invasion 111. Pt with NF2 requires regular hearing screening + annual brain MRIs from ~10 yrs – 4 th decade of life. 112. Pt with NF1 → screen for optic nerve gliomas 113. Common complication of sturge-weber syndrome →seizures, developmental delay 114. Most likely primary malignancy in child with multiple bone mets → neuroblastoma 115. ML reproductive complication of receiving systemic chemotherapy → hypogonadism 116. #1 RF for uterine sarcoma → radiation therapy to pelvis 117. MLM behind increased insulin resistance in pregnancy → human placental lactogen 118. A pt about to die, family comes in and you must speak to them → don’t ask close ended questions, ask how much they know, how much they want to know. Pick an answer that encourages conversation. 378

119. Diabetic patients with pyelonephritis not responsive to Abx → obtain a repeat CT to rule out complications of pyelonephritis (i.e perinephric abscess/emphysematous pyelonephritis) 120. In a person with necrotizing fasciitis include clindamycin in the Abx regimen. 121. Diabetic with RUQ may have gangrenous cholecystitis → emergent cholecystectomy 122. Diabetic pt with mucormycosis of sinuses → give Ampho B + debride extensively 123. MC pancreatic neuroendocrine tumor in pt with Hx of MEN1 → gastrinoma 124. ML finding on ECG in pt with cardiac amyloidosis → low voltage. 125. #1 RF for MR → mitral valve prolapse. 126. MCC of folliculitis → S. aureus 127. ML subtype of melanoma in a pt with a dark complexion → acrolentigenous melanoma 128. ML inciting factor in pt with erythema multiforme → recurrent herpes simplex virus infx. 129. ML inciting factor for TEN/SJS → drugs 130. MLM of disease in pt presenting with signs of hypopituitarism → pituitary adenoma 131. In a hospitalized pt with “bones, groans, psychiatric overtones” (hypercalcemia) → malignancy 132. ML symptom that would trigger initial presentation in pt with MEN1 → Sx of hypercalcemia 133. MLCP in MEN2 → medullary thyroid cancer 134. MC infectious cause of odynophagia (painful swallowing) → candida albicans 135. MCC of osmotic diarrhea → lactase deficiency 136. MLM of treatment failure in pt placed on gluten free diet for celiac’s disease → non-adherence. 137. ML cause of abnormal LFTs on routine labs → non-alcoholic fatty liver disease. 138. ML cause of knee pain in pt I think it means most serious complication where most likely cause of death is CVD) 151. MC malignancy in kidney transplant recipient → SCC of the skin. 152. MLM of disease in status epilepticus → low level of antiepileptic drug. 153. #1 RF for intracerebral hmg → HTN 154. MCC of rapidly progressive dementia → Creutzfeldt-Jakob Disease 155. ML initial clinical presentation of Cervical Ca → Abnormal vagin*l bleeding. 156. MC anterior mediastinal mass → thymoma 379

157. MC middle mediastinal mass → Lymphadenopathy 158. MC posterior mediastinal mass → neurogenic tumor/ schwannoma 159. A pt that is nauseous/vomiting → don’t place on bipap or cpap. 160. MC extra-articular manifestation of ankylosing spondylitis → anterior uveitis 161. Lupus Ab that has the strongest association with kidney disease → Anti ds-DNA Ab 162. MLCP of relapsing polychondritis → red hot painful ear. CROSS CHECKED? No

---------------------------------------------------------------------------------------------------Episode 187: Rapid Review Series 25 ●

6 month old w/ CXR showing interstitial infiltrates + BAL with PJP + low IgG, IgE, IgA + high IgM → hyper IgM syndrome ○ Pathophys: problems with class switching (IgM → IgG & IgA) ■ Requires CD40-CD40L interaction ○ PJP in child ddx? ■ SCID ■ DiGeorge ■ Hyper IgM Child with recurrent Staph aureus abscesses → chronic granulomatous disease ○ Inheritance? X-linked recessive ○ Pathophys? NADPH oxidase deficiency ○ Cell involved? Neutrophil ○ Dx? ■ dihydrorhodamine (DHR) test ■ nitroblue tetrazolium (NBT) test - older ○ Tx? Interferon-gamma & IVIG ■ Interferon-alpha → hep B and C ● MDD is a relative contraindication ■ Interferon-beta → MS ■ Interferon-gamma → CGD ■ IVIG → Bruton’s, Hyper IgM, Guillain-Barre Newborn w/ hypocalcemic seizures + tetralogy of Fallot → DiGeorge syndrome ○ Pathophys? 3rd and 4rd pharyngeal pouches fail to form → no thymus and no parathyroid glands → viral/fungal infections + hypocalcemia ○ Cell involved? T cells ○ Imaging? No thymic shadow on NB CXR ■ Also seen in SCID Child with infections where WBC count is high but no pus + delayed separation of umbilical cord stump → Leukocyte adhesion deficiency ○ Pathophys? Lack CD-18 or beta-2-integrin → neutrophil cannot adhere to endothelial cells ○ Cell involved? Neutrophil ○ Inheritance? AR ○ May also have bleeding problems. Why? Glanzman thrombasthenia-like defect that causes problems with primary hemostasis Pt given enalapril, now c/o severe abdominal pain → hereditary angioedema 380

○ ○

Pathophys? C1 esterase inhibitor deficiency → cannot shut down kallikrein pathway → increased production of bradykinin → vascular permeability → edema of lips, airway, GI tract ■ ACE breaks down bradykinin, so ACE-Is remove this breakdown pathway Dx? C2 & C4 levels low (due to unchecked consumption of complement) Tx? ■ Ecallantide (direct inhibitor of kallikrein) ■ Icatibant (bradykinin receptor antagonist)

Child was hypotonic at birth + now 15 yo boy that is very obese + almond-shaped eyes → Prader-Willi syndrome ○ Chromosome? 15 ○ Genetic mechanism? Imprinting → maternal gene is turned off ■ Maternal uniparental disomy ■ Paternal deletion Girl with inappropriate laughter + intellectual disability → Angelman syndrome ○ Chromosome? 15 ○ Genetic mechanism? Imprinting → paternal gene is turned off ■ Paternal uniparental disomy ■ Maternal deletion

Drugs that improved survival in HF? ○ ACE-I ○ Spironolactone ■ AE? Gynecomastia (due to androgen receptor antagonism) ○ eplerenone ○ Beta-blockers ■ Metoprolol, carvedilol, bisoprolol ○ Hydralazine / Isosorbide dinitrate (BiDil) **in African-Americans**

Pt on antipsychotic now with… ○ dilated cardiomyopathy → clozapine ○ hyperprolactinemia → risperidone OR aripiprazole 381

○ ○ ○ ○

Torsades → ziprasidone (causes QT prolongation) increased Hgb A1c → olanzapine (causes obesity & metabolic syndrome) neutropenic fever → clozapine Visual difficulty + lens opacification on exam → quetiapine (causes cataracts)

55 yo M smoked 2 ppd x 40 years, presents with 3 months chronic cough + hard time rising from seated positions → LEMS 2/2 small cell lung cancer ○ Pathophys? Ab to the presynaptic voltage-gated Ca++ channel ○ Dx? EMG/nerve conduction study ■ Incremental pattern with repeated stimulation ○ Other paraneoplastic phenomena of small cell lung cancer? ■ Ectopic ACTH ● Doesn’t suppress with high-dose dex ■ SIADH

Choriocarcinoma ○ #1 RF? Gestational trophoblastic dz (especially complete mole) ○ Arises from? Trophoblastic tissue ○ Marker? beta-hCG ○ Metastasizes to? Lungs ○ Tx? Methotrexate ○ What if pt with choriocarcinoma now has A-fib, lid lag, hyperreflexia → hyperthyroidism ■ Pathophys? beta-hCG and TSH have similar structures, so beta-hCG can stimulate the TSH receptors and trigger thyrotoxicosis ■ Dx? Elevated T3/T4 + suppressed TSH

---------------------------------------------------------------------------------------------------Episode 189: Rapid Review Series 26 382

Focal nodular hyperplasia

Hepatic adenoma

Has central stellate star CentriFugal enhancement (peripheral) on arterial phase Enhanced on HIDA scan (contains biliary epithelium)

Women taking COCs Does NOT have central scar Centripetal enhancement (edges → center) on CT arterial phase Non-enhancing on HIDA scan (don’t contain biliary tree)

Sliding hiatal hernia

Paraesophageal hernia

Femoral hernia

GE junction herniates through diaphragm Associated with GERD Tx? Can observe

Fundus of stomach herniates Tx? Surgery! High risk of incarceration → strangulation

Female Under inguinal ligament Medial to femoral vessels Tx? surgery!

Chronic inflammation in any hollow organ can cause strictures. Examples: GERD → strictures in esophagus → dysphagia Crohn’s → strictures in small bowel → SBO Recurrent PID → strictures in fallopian tube → infertility Dx? Hysterosalpingogram

Psoriasis

Acne

Silver scale on extensor surfaces Tx? 1st line: Topical steroids, coal tar, Vit D analog (calcipotriene) 2nd line: add methotrexate or cyclosporine 3rd line: TNF-alpha inhibitors or UV-B phototherapy/PUVA Mental health association? Depression

Tx? 1st line: topical benzoyl peroxide, salicylic acid, or retinoid 2nd line: topical abx (e.g. clinda) 3rd line: oral abx (e.g. doxy) 4th line: isotretinoin (need to be on 2 forms of birth control, check LFTs)

Cardiogenic shock

Septic shock

Hypovolemic

Pathophys: pump doesn’t work CO low PCWP increased (>18) BP low SVR high Tx? Inotrope (e.g dobutamine, digoxin, milrinone)

Pathophys:inflammation → systemic vasodilation SVR low CO high (d/t reduced afterload) PCWP low Pulm complication? ARDS Tx? 1st line: abx + fluids 2nd line: norepi

Pathophys: low preload CO low (d/t low preload → low EDV) PCWP low (pump still works) SVR high Tx? 1st line: crystalloid fluids 2nd line: blood transfusion

● ●

#1 RF breast cancer? Age ○ dsDNA break repair problems → BRCA ○ Mismatch repair problems → Lynch #1 RF squamous cell carcinoma penis/vulva/anus/cervix? HPV #1 RF endometrial cancer? Unopposed estrogen

383

● ● ● ● ●

● ● ●

○ Estrogen + progestin is OKAY! #1 RF lung cancer? Smoking #1 RF ovarian cancer? Age + family hx #1 RF papillary thyroid cancer? Radiation to head/neck (e.g. for lymphoma in childhood) ○ Recall Orphan Annie eye nuclei + Psammoma bodies #1 RF Burkitt’s lymphoma? EBV Translocations to know: ○ t(8, 14) → Burkitt’s lymphoma ○ t(9,22) → CML (BCR-ABL protein) ○ t(15,17) → APML ○ t(11,22) → Ewing’s sarcoma (EWS-FLI protein) #1 RF RCC? Smoking ○ Bilateral RCCs → VHL #1 RF bladder cancer? Smoking #1 RF pancreatic cancer? Smoking

Growing pains

Osteoid osteoma

Bilateral Pain worse at night Pain worse with physical activity during day

Unilateral Pain at night, relieved with NSAIDs (Pathophys: increase in prostaglandins → pain) Bony prominence on PE or imaging

Cholecystitis

Ascending cholangitis

Choledocholithiasis

Fever + RUQ pain only Obstruction at level of cystic duct, not biliary tree Dx? RUQUS, if needed HIDA Cholecystectomy NOW

Fever + RUQ pain + jaundice (AMS + hypotension) Obstruction of CBD Dx? ERCP Cholecystectomy LATER

Mild RUQ pain + jaundice Obstruction of CBD Dx? 1st RUQUS 2nd ERCP or MRCP

HELLP

Intrahepatic cholestasis

Acute fatty liver of pregnancy

Elevated indirect hyperbili (hemolysis) Schistocytes High AST/ALT Low platelets

Obstructive jaundice Direct (conj) hyperbili Elevated alk phos Normal AST/ALT Itching → excoriations

VERY HIGH AST/ALT Hypoglycemia PT/PTT elevated

Gestational hypertension

Pre-eclampsia

Eclampsia

After 20 weeks BP >140/90

BP >140/90 +proteinuria (>300 mg/24 hrs) Severe features criteria? - AMS - Low platelets ( 160/110 - Elevated Cr - Hepatic dysfunction Tx? Delivery if >34 wks ga Drugs for HTN in pregnancy? Hydralazine, alpha-methyldopa, labetalol, nifedipine “Hypertensive Moms Love Nifedipine” Ectopic pregnancy

Blighted ovum

Fertilized egg implants OUTSIDE uterus

Fertilized egg but embryo does NOT develop Implants INSIDE uterus +gestational sac w/o embryo inside

Hyperemesis gravidarum ○ 1st trimester ○ Admit to hospital if dehydrated ○ Tx? IVF + IV B1 (thiamine) + IV B6 + IV antiemetics (e.g. ondansetron) ○ Labs? Ketones in blood/urine

Cervical incompetence ○ RF? Hx LEEP or cervical conization, connective tissue disorder (e.g. Ehlers-Danlos), DES exposure, hx preterm delivery ○ Presentation? No contractions, but can see bag or fetal parts emerging from cervix ○ Tx? Cerclage ○ Prevention in future pregnancies? vagin*l progestin

Cervical ectopy/cervical ectropion ○ 2 parts of cervix: endocervix (columnar epithelium) + ectocervix (stratified squamous keratinized epithelium) ○ Columnar epithelium of endocervix more susceptible to infection ○ Ectropion (more common in young females & OCP users) makes them more susceptible to STIs

Amnioinfusion indications ○ Variable decels → suggest cord compression ■ Can help cushion the cord ○ Severe oligohydramnios ■ Risk of cord compression ○ Decrease risk of meconium aspiration syndrome in post-term fetus ■ “Soft call recommendation”...not part of guidelines, but on NBMEs ■ Inhalation of meconium → meconium aspiration syndrome → persistent pulmonary hypertension ■ Amnioinfusion can help avoid this (think of it as diluting/solubilizing meconium)

Weird bug: Ureaplasma urealyticum ○ Potential cause of: ■ UTIs in sexually active young female ● #1 cause = E. coli ● #2 cause = Staph saprophyticus ● High urine pH (urease-positive → hydrolysis of urea into ammonia) ■ Chorioamnionitis ● Increased risk of preterm labor, prolonged intubation/mechanical ventilation in neonate, etc. ■ PID ■ Miscarriages ○ Tx? ■ Doxycycline ■ Azithromycin

385

Doesn’t have a cell wall, so can’t use a cell wall agent! (e.g. penicillins)

Chorioamnionitis is NOT an indication for C-section ○ Tx? Induce to deliver baby soon

Women in immediate perinatal period becomes unresponsive + low platelets + elevated fibrin split products + elevated PT/PTT → amniotic fluid embolism

---------------------------------------------------------------------------------------------------Episode 195: Rapid Review Series 27 ●

Given 3 RFs, with NNH = 5, NNH = 10, NNH = 20. Which is the most important RF? ○ The one with NNH = 5 ○ Low NNH → significant RF ○ Low NNT → very effective drug/intervention

Pre-op patient, what drug would you give for abx ppx? Cefazolin (“Ancef”) ○ Decreases the incidence of surgical site infections ○ Administer 30-60 mins BEFORE incision ○ Covers Staph aureus, Staph epidermidis, Strep, some gram-negatives ○ What if hx of beta-lactam allergy? ■ Clinda ■ Vanc

Infection after shoulder surgery. Bug? ○ Cutibacterium acnes (formerly Propionibacterium acnes)

ENDOCARDITIS PPX ● Who gets abx ppx for endocarditis? ○ Hx prosthetic heart valve ○ Hx infectious endocarditis ○ Unrepaired cyanotic congenital heart dx ○ COngenital heart defect fixed 5% prepregnancy weight ○ Labs? Elevated ketones in blood/urine ○ Tx? ■ Inpatient: ● IVF ● IV thiamine/B1 ● IV antiemetic (e.g. ondansetron) ■ Outpatient: ● Vit B6 + doxylamine ● Small frequent meals Pt from Turkey with painful sores on buccal mucosa mouth & on genitals → Behcet’s disease ○ Association? Mediterranean descent ○ HLA? HLA-B51 ○ Dx? Pathergy test ■ Needle prick on arm → they will form an ulcer Other HLAs to know ○ HLA-B27 → seronegative spondyloarthropathies ○ HLA-B57 → HSR to abacavir

MICROCYTIC ANEMIAS ● Iron deficiency anemia ○ Serum iron? Low ○ Ferritin? Low ○ TIBC? High 387

○ Transferrin saturation? Low ○ RDW? Increased Anemia of chronic disease ○ Pathophys? Chronic inflammatory state → increased hepcidin → sequestration of iron in bone marrow macrophages ○ Serum iron? Low ○ Ferritin? High ○ TIBC? Low ○ Transferrin saturation? Low ○ RDW? Normal Lead poisoning ○ Pathophys? Inhibition of ALAD and ferrochelatase → can’t synthesize protoporphyrin and can’t add iron to protoporphyrin to make heme ■ Like an iron overload state ○ Serum iron? High ○ Ferritin? High ○ TIBC? Low ○ Transferrin saturation? High ○ RDW? Normal Other causes: ○ Thalassemia ○ Sideroblastic anemia (can be part of myelodysplastic syndrome) Myelodysplastic syndrome ○ Sideroblastic anemia ○ Hyposegmented neutrophils ○ Progression to acute leukemia

Two things to increase power of study (type II error/beta error = incorrectly accept null hypothesis) ○ Increase study population ○ Increase the effect size

If you increase sensitivity what happens to PPV/NPV? ○ PPV decreases ○ NPR increases (because false negatives are less likely)

---------------------------------------------------------------------------------------------------Episode 196: Rapid Review Series 28 Case control study

Retrospective cohort study

Start at outcome and look back to see if there was exposure Example: Look at people who have lung cancer now and see how many were had a certain exposure in the past *Susceptible to recall bias

Start at exposure & go forward to outcome Example: Look at people who smoked in 1995 and see how many have lung cancer now

388

Data reported as odds ratio Odds ratio that crosses 1 is NOT stat sig ●

Pt with sudden-onset severe CP radiating to back → aortic dissection ○ Dx ■ If HDS → CT chest ■ If HDUS → Bedside TEE ○ Stanford type A = involves ascending aorta (may involve descending too) ■ Beta blocker + surgery ○ Stanford type B = only involves descending aorta ■ Medical tx only: beta blocker ○ Complications ■ Acute AR ■ RCA occlusion → inferior MI ● ST elevations in II, III, aVF ● Preload dependent! Don’t get nitrates!

Drug A lowers SBP by 3 mmHg and DBP by 1.5 mmHg. Conventional tx lowered SBP by 1.5 mmHg and DBP by 0.75 mmHg. P-value is 70 4. 40+ y.o. + ASCVD risk > 7.5%

Which 2 statins are “high-intensity statins”? Atorvastatin, Rosuvastatin ○ High-intensity statins given in specific dosing and reduce LDL by ≥ 50% ■ Atorvastatin (40 or 80 mg qday) ■ Rosuvastatin (20 or 40 mg qday) 406

Vitamin & Mineral Deficiencies: ●

Vitamin B1 aka thiamine deficiency = Wernicke-Korsakoff syndrome or wet beri-beri ○ Wernicke-Korsakoff syndrome ■ Pathophys? Depletion of Vit. B1 → issues with transketolase in TCA cycle and pentose phosphate pathway ● Transketolase enzyme uses Vit. B1 as cofactor ■ Triad: confusion + ataxia + ophthalmoplegia → Wernicke encephalopathy ● Tx? IV thiamine then glucose ● Reversible ● Note: “ophthalmoplegia” can be any eye problem, e.g. nystagmus, lateral gaze palsy, etc. ■ Confusion + ataxia + ophthalmoplegia + amnesia + confabulation → Korsakoff syndrome ● Complication of untreated Wernicke encephalopathy ● Irreversible ■ Populations at risk? ● Alcoholics ● Pregnant woman with hyperemesis gravidarum ○ Hypokalemic hypochloremic metabolic alkalosis ○ Pathophys? Vomiting triggers RAS activation → H+ & K+ excretion ■ Neuroanatomical association/pathology? Hemorrhagic infarction of mammillary bodies ○

Wet beri-beri = dilated cardiomyopathy

Vitamin B2 = riboflavin

Vitamin B3 aka niacin deficiency ○ Pellagra = diarrhea, dermatitis, dementia, death ■ Dermatitis on hands + around neck (necklace-like rash) ■ Etiologies: ● Dietary deficiency ● Carcinoid syndrome ○ Pathophys? Carcinoid tumor produces excess serotonin → shunts tryptophan to serotonin production instead of niacin production ■ Note: tryptophan = precursor for serotonin and niacin ● Hartnup disease ○ Pathophys? Defect in neutral amino acid transporter in PCT → cannot reabsorb neutral amino acids (tryptophan is a neutral a.a.)

Vitamin B6 aka pyridoxine deficiency ○ What drug causes Vit B6 deficiency? Isoniazid treatment for Tb ○ What 3 pathologies result from Vit B6 deficiency? Sideroblastic anemia, seizures, abnormal LFT’s ■ Sideroblastic anemia ● Pathophys? ALAS enzyme rxn cannot run → heme production halted 407

○ ■

ALAS = rate-limiting enzyme for heme synthesis and requires Vit B6 as cofactor

Seizures ● Pathophys? GABA deficiency, i.e. lack of inhibitory n.t’s → unopposed excitatory n.t. activity ○ Normally: glutamate decarboxylase requires Vit B6 as a cofactor to convert glutamate to GABA ○ Glutamate decarboxylase cannot run In Vit B6 deficiency → GABA deficiency Abnormal LFTs ● Pathophys? Transaminases unable to function properly (b/c transaminases normally use Vit B6 as cofactor) ● Results in AST/ALT > 2 ○ Note: same finding as LFTs seen in alcoholics b/c alcoholics often have Vit. B6 deficiency ☺

Vitamin Vit B9 aka folate deficiency ○ What 2 pathologies result from Vit B9 deficiency? Megaloblastic anemia, neural tube defects ■ Megaloblastic anemia ● High hom*ocysteine, normal MMA ■ Neural tube defects ● Folate is in prenatal vitamins to prevent NTD’s ○ Populations at risk? ■ Tea & toast diet/malnutrition ■ Alcoholics ■ Pts on anti-seizure drugs, esp. valproate

Vitamin B12 aka cobalamin deficiency ○ What 2 pathologies result from Vit B12 deficiency? Megaloblastic anemia, subacute combined degeneration ■ Megaloblastic anemia ● Labs show? Elevated hom*ocysteine + elevated methylmalonic acid ● In odd-chain fatty acid pathway, methylmalonyl-CoA mutase converts methylmalonyl-CoA to succinyl-CoA by using B12 as a cofactor ○ So, MMA build ups up in Vit. B12 deficiency ■ Subacute combined degeneration of spinal cord ● Loss of dorsal columns → loss of vibration, proprioception, light touch ● Loss of corticospinal tract → UMN lesion signs (e.g. hyperreflexia) ○ Populations at risk? ■ Vegans (B12 comes from animal products)

Dysgeusia + alopecia → zinc deficiency ○ Etiologies? ■ TPN use (total parenteral nutrition) ■ Wilson’s disease tx’ed with trientine ● Trientine chelates copper and zinc

Mucosal bleeds + fatigue/weakness + muscle aches → Vit C deficiency (scurvy) 408

Pathophys? Body unable to do collagen synthesis (specifically, unable to do hydroxylation of proline & lysine)

Night blindness → Vit A deficiency

Osteoporosis + secondary hyperparathyroidism → Vit D deficiency ○ Pathophys? Decreased Vit. D causes decreased Ca2+ absorption from intestines → increased PTH levels in response to low serum Ca2+ levels

Acanthocytosis on blood smear + ataxia → Vit E deficiency ○ Pathophys? Demyelination of spinocerebellar tracts ■ Myelination of spinocerebellar tracts requires Vit E Bleeding problems → Vit K deficiency

● ●

Fat malabsorption causes deficiency of fat-soluble vitamins (Vitamins ADEK) ○ Etiologies? ■ Crohn’s, esp. if terminal ileum is resected ■ Cystic Fibrosis → pancreatic insufficiency ■ Celiac disease ■ Chronic pancreatitis

-------------------------------------------------------------------------------------------------------------------------------

Episode 207: Geriatrics 1. 2. 3. 4. 5. 6.

Check vision, hearing, fall risk, ur. incontinence Polypharmacy- review meds BEERS criteria Look out for Elderly abuse Assess functional ability- assess ADLs Basic and instrumental ADLs (things that you need to live independently vs things that you need to interact w/ society) 7. MMSE to evaluate cognitive functioning ( abN tell pt a series of words then draw a clock then recall the words 9. Check TSH, B12, meds, S. Na, BNP if they have cognitive dysfunction 10. MCC of hearing loss in elderly → presbycusis (high freq hearing loss first) 11. Best screening test to determine hearing loss-> take history then whispered voice test then audiometry 12. Access for Presbyopia, macular degeneration, cataracts 13. Rule out depression in the elderly esp in CKD, stroke, CAD pts; older pts have a greater risk of suicide than younger 14. We don’t always use SIGECAPS for elderly; instead ask have you felt depressed/sad/low and ask have you lost interest in the past 2 wks? 15. SSRI is D.O.C. do NOT pick TCA (coz of S/E profile) 16. Fall risk assessment→ ask for h/o problems w/ gait or balance, do a get up and go test (normal is 1011sec, abN is >20 sec) 17. Interventions that dec fall risk-> exercise, physiotherapy, Vit D supplementation 409

18. Urge incont./ overactive bladder (hypertonic detrusor)-> bladder training is the best Tx, biofeedback, oxybutynin, tolterodine (best avoided in elderly) 19. Elderly women, G6P6 (multiple deliveries)-> Keigel’s exercise, sling procedure 20. Overflow incont. (hypotonic detrusor)-> self catheterization, timed voiding 21. Functional incont. (cognitive problems or arthritis where they can’t move fast enough)--> need caregiver 22. Living will aka Advanced Directive 23. Health care proxy (DPOA) 24. POLST forms--> a health care provider has to be present when pt is signing this 25. Palliative care < 6 mos prognosis--> initiate Hospice care 26. Elderly pt driving h/o mild dementia or is taking benzo, TCA or h/o vision problems or h/o falls, ambulation issues, degenerative dz who can have trouble moving neck side to side-> report to DMV 27. Pressure ulcer--> old pt, sensory issues, chrn illness, stool or ur incontinence, vascular dz all RF for developing Pressure/ Decubitus Ulcer. To prevent it, reduce pressure points by using air beds, reduce friction, reduce shearing forces; 4 stages: stage 1- erythema on skin, non blanchable; stage 2- lose partial thickness of skin epi and dermis; stage 3- subcut tissue is exposed but no involvement of fascia/bone/musc/tendon; stage 4 – involvement of fascia, muscle, tendon or bone; give them as much nutrition as possible, consider debridement for stage 3 and 4 ulcer, apply occlusive dressings 28. Surrogate decision maker--> use next of kin (spouse then adult children then parents then siblings then other relatives) Cross checked? NO -------------------------------------------------------------------------------------------------------------------------------

Episode 207: Geriatrics (Version 2) Episode 207 (Version 2) notes were graciously provided by Divine Intervention from an anonymous contributor. ● ● ● ●

Comprehensive Geriatric Assessment: hearing, urinary incontinence, vision, cognition, fall risk, functional capability Elderly person taking tons of medications NBS to reduce falls: review medications! ● Polypharmacy Beers criteria = high-risk medications in elderly Elderly abuse ● Mandated reporting: elderly abuse + child abuse

Functional capability ●

Dx: ADL = activities of daily living 1. Basic = things you need to live independently ● i.e. shower, dress, bathroom, feed, pee/poop on your own 2. Instrumental = necessary for interaction with society ● i.e. medications, clean apt, manage finances, make food

Cognitive function ●

Dx: MMSE ● > 24 ● < 24: cognitive dysfunction 410

Vs. mini-cog: words, clock, recall words NBS: r/o reversible causes = B12, TSH, medications, BMP (hyponatremia)

Et: macular degeneration, presbyopia, cataracts, etc

Vision Hearing ● ●

MCC presbycusis = sensorineural hearing loss --> loss of high-frequency hearing Dx: 1. Any issues w hearing? 2. Whispered voice test

Depression ● ● ● ● o

RF: chronic disease Have you felt hopeless, depressed over past 2wk? Have you lost interest in things over past 2wk? SIG-E-CAPS Tx: SSRI ● c/I TCA (ae: sedation, orthostatic hypo-falls, delirium)

Fall risk o

Dx:

o

History of falls in the past year? Get up and go test: get up from chair --> walk 10 feet --> come back to chair ● >20 sec = abnormal ● Assoc w/ impaired ADL ● r/o vision loss Ppx: exercise regularly, physical therapy, supplement w/ vitD

Urinary incontinence o

o o o

Urge incontinence = hypertonic detrusor muscle --> overactive bladder ● Tx: bladder training exercises/timed voiding, oxybutynin/tolterodine (generally c/I in elderly d/t anticholinergic-delirium) Stress incontinence = incontinence with effort ● Tx: kegel exercise, sling procedure Overflow incontinence = neurologic --> hypotonic detrusor muscle --> high post-void residual ● Tx: self-catheterization, timed voiding Functional incontinence = normal exam, but still cannot get to toilet in time ● Tx: caregiver, timed voiding, commode by bedside

EOL care o o

o

Living will/advanced directive: measures to prolong life, ventilator? Healthcare power of attorney/healthcare proxy ● Healthcare proxy limited to health decisions; negated if decision-making capacity returns ● Vs. power of attorney: health, finances, etc POLST = physician orders for life-sustaining treatment ● Signed when pt is superrrrr old/terminal illness in the case of sudden decline ● *healthcare provider MUST be present when pt fills this out

Driving o c/i: opioids, benzos, MSK pain that prevents proper head movement o --> report to DMV Pressure ulcers 411

o o o

RF: old age, limited mobility, sensory inability, severe PAD, malnourishment Ppx: do not apply continuous pressure, reduce friction w surface, reduce shear forces, nutrition Dx: 1. Erythema on skin, non-blanchable 2. Loss of partial thickness of skin @epidermis/dermis 3. Loss of entire layer of skin -->--> necrosis of subcutaneous tissue; no fascial involvement (bone, tendon, muscle) 4. Fascial involvement ● Tx: debridement! Occlusive dressings, etc. prevent cellulitis. Surrogate decision maker? o Next of kin 1. Spouse 2. Adult children 3. Parents 4. Siblings

---------------------------------------------------------------------------------------------------Episode 208: Transfusion Reactions 1. Acute Hemolytic a. MCC: ABO incompatibility due to clerical errors* (HY) b. Ex:Type O person gets Type A or B blood c. Presentation: immediately as transfusion is started complains of : back/flank pain, hematuria d. Diagnostic test: Direct coombs test e. Type 2 Hypersensitivity reaction: hemolysis due to preformed antibodies f. Tx: give fluids! (similar to rhabdomylolysis tx) g. May go into DIC- give platelets, FFP 2. Delayed hemolytic a. MCC: DUFFY/KEL/KIDD antigen present not due to Abo incompatibility b. Rxn does not occur during transfusion usually days after! c. + direct coombs test d. Much less severe reactionsà e. Type 2 HS rxn but days later 3. Allergic transfusion (anaphylaxis) a. MCC: cross linking of IgE on the surface of mast cells b. ONSET: Get blood transfusion- blood contains something you have IgE antibodies to c. Presentation: airway problems- wheezing, bronchospasm, normal vitals d. Type 1 HS reaction e. Tx: give antihistamine or IM epinephrine if severe 4. Febrile nonhemolytic a. Pathophys (2 mechanisms) i. Blood elements specifically Platelets stored at room temp contain WBCs that make cytokines and activate. 412

ii.

5.

6.

7.

8.

Antibodies from the RECIPIENT attack WBCs in DONOR blood and form an antigenantibody complex( typ 3HS rxn)à release of cytokines b. Present: during transfusion develop chills, fevers, mild tachy, no significant hemodynamic instability c. ONSET: ***either during the transfusion OR 1-4 hrs after the transfusion. d. Tx: fever reducer (cytokine mediated mechanism) TRALI: transfusion related acute lung injury (ARDS) a. Pathophys: i. Antibodies from DONOR blood attack WBCs in RECIPIENT blood à antigen- antibody complex ii. Basically identical to what happens in the kidneys in nephritic/nephrotic syndromes but in the lungs! b. Non-cardiogenic pulmonary edema c. Presentation: given normal PCWP ( 18mmhg d. Tx: give diuretics e. Exam Presentation to determine how CVP, BNP and PCWP is affected : **everything goes up! i. ↑CVP: proxy for right atrial pressure- fluid overload that the heart can’t pump ii. ↑PCWP: proxy for Left atrial pressure- fluid overload iii. ↑BNP: whenever chambers are distended for extended period of time they release ANP and BNP to shut down renin-angiotensin system Transfusion associated Graft vs Host Disease a. Pathophys: Donor T cells attack recipient tissue b. Very high mortality c. RF: recipient is immunocompromised d. Symptoms: hepatosplenomegaly, diarrhea, pancytopenia, rash e. Type 4 HS rxn f. prevention: irradiate the donor blood to kill the T cells Leukoreduction a. Reduce WBCs in donor blood b. Effective for febrile nonhemolytic rxn and possibly in allergic rxn

CROSS CHECKED? No ------------------------------------------------------------------------------------------------------------------------------413

Episode 209: Family Medicine Shelf Review Series 2 Episode 209 notes were graciously provided by Divine Intervention from an anonymous contributor.

Depression ● ●

● ● ● ● ● ●

"Down, depressed, hopeless, little interest over past two weeks?" Sx: o Depression o Sleep o Interest (loss) o Guilt o Energy (less) o Concentration o Appetite o Psychom*otor o Suicidality + Mania = bipolar disorder o Tx: lithium (ae: nephrogenic DI |-- amiloride/triamterene, hypothyroid), SGA + varies with seasons = seasonal affective disorder o Tx: SSRIs + phototherapy + loss of a loved one, bereavement (for multiple days, horrible guilt) = MDD +/- psychosis >>> grief reaction (no tx) + assoc 1 week before menses = PMS --> PMDD o Dx: symptom diary DDx: hypothyroid, stroke, Parkinson's, Cushing's, interferon (HepC), BB Tx o SSRI if… ● CHF, CVD: sertraline ● Sexual dysfunction, smoking: bupropion (c/I bulimia) ● Weight loss: mirtazapine ● C/I pregnancy: paroxetine o 1st episode --> SSRI for 6-9 mos --> 2nd episode --> SSRI for 6x2 = 12 months --> 3rd episode --> tx for life (or if 2nd episode comes w/I 1 year of 1st) ● Serotonin syndrome: SSRI, MAOi, linezolid, dextromethorphan/cough syrup, tramadol, buspirone --> myoclonus, hyper-reflexic o ***ensure no history of mania

Anxiety ● ● ● ● ●

Sx: irritable, disturbances of sleep, worriers C/b: psych co-morbidities Tx: SSRI, SNRI, buspirone, CBT o If anxiety attack, flight, etc |-- one-time benzo + fear of poor performance in social settings = social anxiety disorder o Tx: propranolol (c/I asthma), benzo Panic attack = palpitations, sweating, SOB, losing control o Tx: one-time benzo o Multiple attacks --> panic disorder o Tx: SSRI, CBT + somatic symptom which causes distress in life = somatic symptom disorder o Tx: same physician 414

o

DDx: malingering (+ incentive-based), factitious (+primary gain), factitious by proxy (+imposed on another), conversion disorder (+non-localizable neuro) + worry about general health, preoccupation with health-related activities = illness anxiety disorder/hypochondriasis

PTSD ● ● ●

Sx: re-experiencing traumatic events, avoidance of assoc stimulus, hyper-arousal w stimuli Dx: screen for comorbid psych, abuse Tx: SSRI, CBT, prazosin --| flashbacks o NEVER pick benzos as tx for PTSD!

OCD ●

Sx: obsession +/- compulsion o Obsession = intrusive idea, thought --> anxiety o Compulsion = things that you do to relieve ^^^ Tx: CBT exposure & response prevention, “OCD” dSSRI, clomipramine, olanzapine DDx: OCPD (do not recognize as unreasonable)

● ●

Eating disorders ●

Anorexia nervosa: BMI < 18.5; distorted body image, amenorrhea o c/b osteoporosis, arrythmias, refeeding syndrome (mcc: hypophosphatemia) o Tx: CBT Bulimia: BMI > 18.5; compensatory behaviors (laxative, vomit) o Tx: CBT, fluoxetine, imipramine o DDx: binge eating disorder (no compensatory) --> hypochloremic hypokalemic metabolic alkalosis

Schizophrenia ● ●

Sx: disorganized speech, illogical thoughts, disorganized behavior, catatonia, flat affect, alogia, avolition Tx: SGA (olanzapine, risperidone, aripiprazole, quetiapine)

ADHD ● ● ●

Sx: @childhood, inattention +/- hyperactivity in 2 settings RF: substance abuse, mood disorder Tx: stimulant (methamphetamine, methylphenidate), atomoxetine (SNRI) o Stimulants c/I CVD -------------------------------------------------------------------------------------------------------------------------------

Episode 210: Rapid Review Series 31 ●

Pt with a 40-day long menstrual cycle. How long is the follicular phase? ○ Luteal phase is always 14 days, follicular phase is 26 days ○ Dominant follicle makes estrogen → proliferative endometrium ○ LH surge → ovulation ○ Corpus luteum makes progesterone → converts proliferative endometrium to secretory endometrium ○ If egg is fertilized, zygote implants in endometrium and makes beta-hCG and this maintains the corpus luteum ○ If egg is not fertilized, the corpus luteum degrades after 14 days (FIXED #) ○ Progestin withdrawal test: supply progestin and withdraw it, the person should bleed. ■ If they get a period, it means that progestin deficiency was the problem, so corpus luteum did not form so they probably didn’t ovulate. ● Consider PCOS Lesion in the MLF vs PPRF ○ MLF = medial longitudinal fasciculus 415

■ ■ ■

● ● ●

Location? Pons Function? Makes horizontal conjugate gaze work Looking to the R ● R eye abduct (R lateral rectus → R CN6) ● L eye adduct (L medial rectus → L CN3) ● MLF allow for communication between CN6 & CN3 ○ CN6 → MLF → CN3 ● If L MLF lesion, R eye abducts but L is stuck ■ Impaired in INO (often MS pt) ■ MLF that is impaired is the same side as the eye that can’t adduct ■ Only 1 eye affected ○ MLF vs. CN3 lesion ■ MLF - cannot adduct when doing horizontal conjugate gaze but CAN adduct for accomodation (normal convergence response) ■ CN3 lesion - cannot adduct for horizontal conjugate gaze or accomodation ● E.g. compression from PCOM aneurysm ○ PPRF ■ Location? Pons ■ Function? Drives CN6 to create horizontal conjugate gaze ■ Looking to the R: ● Activate the R PPRF → activate R CN6 → activate L MLF → activate L CN6 ● If R PPRF lesion, the L PPRF is acting unopposed → eyes deviate to the L ■ For PPRF lesion, both eyes deviate AWAY from the side of the lesion ■ Both eyes affected Pt with Hartnup disease. What is the most likely psych comorbidity? Depression ○ Pathophys? Defect in transporter that helps reabsorb neutral amino acids ○ Can’t absorb tryptophan → can’t make serotonin or niacin Pulmonary contusion ○ Presentation? Pt who was in MVC with bilateral interstitial infiltrates + hypoxia Cardiac contusion ○ Presentation? Pt who was in MVC with CP, elevated PCWP, mildly elevated troponins Blunt trauma to kidney ○ 1st step? UA (to look for hematuria) ○ If blood in urine → CT abdomen ○ If UA negative → stop there!

Which is the biggest RF for C. diff? Recent abx/recent hospitalization

Elevated alk phos. Causes? ○ Obstructive biliary process ■ Choledocholithiasis ■ Ascending cholangitis ■ Primary biliary cholangitis (destruction of small bile ducts of liver) ○ Bone ■ Paget’s disease of bone ○ Check GGT. If GGT elevated too → biliary cause 416

---------------------------------------------------------------------------------------------------Episode 211: Rapid Review Series 32 LOCALIZE THE LESION! ● Pt that has trouble forming new memories (anterograde amnesia) → hippocampus ○ Temporal lobe ● Non-fluent aphasia (can’t get the words out, but comprehension is good) → Broca’s area (inferior frontal gyrus) of dominant hemisphere ○ L hemisphere in most people ○ Supplied by MCA ● Fluent aphasia (can say the words, but comprehension is terrible) → Wernicke’s area of dominant hemisphere ○ Supplied by MCA ● Pt is neglecting one side of the world (e.g. putting make up only on one side of face) → non-dominant parietal lobe lesion ○ In most people: L sided neglect (because R side is non-dominant) ● Agraphia, acalcula, finger agnosia, left-right disorientation → Gerstmann syndrome → dominant parietal lobe lesion ○ Will also tend to have a fluent aphasia (Wernicke’s area involved) ● Cortical blindness → primary visual cortex in occipital lobe ○ Contralateral hom*onymous hemianopia (lose same visual field in both eyes) ○ L visual cortex lesion → lose R visual field in both eyes ● Bitemporal hemianopsia / Heteronymous hemianopsia → optic chiasm compression ○ E.g. due to prolactinoma, craniopharyngioma ● Eyes deviating in one direction ○ Toward side of lesion → frontal eye field ○ Away from side of lesion → PPRF lesion ● Executive dysfunction + disinhibition + contralateral UMN signs (Babinski, pronator drift, hyperreflexia) → frontal lobe ○ Can have reemergence of primitive reflexes ○ Brain tumor in frontal lobe that is calcified w/ fried egg appearance on histology → oligodendroglioma ● Tremors → basal ganglia problem ● Loss of all motor function on one side of body → posterior limb of internal capsule ○ Likely a rupture of a Charcot-Bouchard microaneurysm of lenticulostriate artery (HTN is #1 RF) ● Parkinsonian sxs → substantia nigra ○ Causes of parkinsonism sxs in young person? ■ MPTP contamination of heroin ■ Wilson’s disease (movement sxs + psych sxs + liver problems) ● Copper deposits in basal ganglia ● Person cannot be aroused → reticular activating system (midbrain) ○ Midbrain injury → comatose state ● decorticate posturing → problems above level of red nucleus (in the midbrain) ○ Think problem in CORTEX with deCORTicate ● decerebrate posturing → problems below level of red nucleus (in the midbrain) ● Huntington’s disease → atrophy of caudate ● Loss of all motor function, can only wink eyes (locked-in syndrome) → pons 417

○ ○

● ● ● ● ● ●

● ●

Basilar artery stroke Osmotic demyelination syndrome ■ 2/2 rapid correction of hyponatremia ■ “From low to high, the pons will die. From high to low, the brain will blow” Vertical diplopia → Parinaud’s syndrome → compression of superior colliculus (often due to pinealoma) Truncal ataxia → cerebellar vermis lesion ○ Causes? Medulloblastoma, pilocytic astrocytoma Limb ataxia → IPSILATERAL cerebellar hemisphere lesion Flailing movements of one arm (hemiballismus) → contralateral subthalamic nucleus (part of basal ganglia) Obstructive hydrocephalus / non-communicating hydrocephalus. MC Location of the obstruction? Cerebral aqueduct of Sylvius Communicating hydrocephalus? Arachnoid granulations (where CSF is reabsorbed) ○ Causes? ■ meningitis ■ malignancy Schizophrenia → enlarged lateral ventricles Azheimer’s → basal nucleus of Meynert ○ Alternatively, CHAT (=choline acetyltransferase) enzyme dysfunction

418

-------------------------------------------------------------------------------------------------------------------------------

Episode 212: Family Medicine Shelf Review Series 3 (GI) 419

Episode 212 notes were graciously provided by Divine Intervention from an anonymous contributor. ●

Hepatocellular

Cholestatic

Marker elevation

AST/ALT > AP or GGT

AP > AST/ALT, direct bili

DDx

Drug-induced injury @Biliary tree, hemolytic (acetaminophen, which is anemia, UDPGT issues (i.e. not an NSAID lol), Gilbert syndrome) alcoholic hepatitis (AST:ALT > 2:1), HepA w/ travel

Tx

Liver transplant esp when synthetic function declines w/ MELD score >18 OR Marjorie score > 32 (give steroids to help too!)

Acute liver injury o Sx: acute liver failure o Dx: abrupt increase AST/ALT o Et: acetaminophen, acute viral hep, mushrooms (?!), systemic hypotension "shock liver" Hepatitis o Sx: jaundice, fatigue, pallor, AST/ALT in 1000s o Dx: IgM, AST/ALT ● HepB ▪ Causes: sex, needle-stick (+IVIG) ● *no PEP if already vaccinated ▪ Dx: ● HepB surface antigen (+) = infected i. Acute: IgM ii. Chronic: IgG ● HepB surface antibody (+) = not infected (vaccine OR exposed but recovered = core Ab) ● "Window period": everything is negative except IgM core antibody ▪ Tx: usually cleared in adults on its own! vaccine after birth; entecavir or tenofovir (c/I pregnancy), pegylated interferon (c/I depression) ● If +HIV = emtricitabine + tenofovir ▪ Assoc: polyarteritis nodosa (abd pain), membranous nephropathy (MCC nephrotic in Caucasians) ● HepC ▪ Dx: ● 1. HCV antibody ● 2. HCV RNA (Type 1 in US) ▪ RF: 1945-1965, blood transfusions, IVDU, mixed cryoglobulinemia (Raynauds w/ cold agglutinins), porphyria cutanea tarda (UROD deficiency) ▪ Tx: sofosbuvir ▪ Complication: leukocytoplastic vasculitis (palpable purpura on feet), HCC (US q6m), HepB superinfection w/ tx ● Autoimmune hepatitis 420

▪ ▪ ▪

o

RF: other autoimmune dz Sx: elevated AST/ALT, RUQ pain, ANA+ Dx: anti-Sm, anti-LKM microsomal ● NAFLD ▪ RF: diabetes, HLD ▪ Tx: tx underlying ^^ Complications ● Ascites ▪ Tx: lasix, serial paracenteses (if >5L then also give albumin) ● Spontaneous bacterial peritonitis --> paracentesis for >250 PMNs ▪ Tx: albumin + 3rd gen cephalo ▪ Ppx: fluoroquinolones ● HSM ● HCC q6m US ● Low albumin ● Bleeding problems ▪ Varices ● Ppx: beta blocker, EGD with band ligation (if c/I to bb) ● Active bleed |-- fluids, EGD, IV octreotide, ppx fluoro ● If refractory: TIPS (ae: hepatic encephalopathy) ● Hepatic encephalopathy |-- lactulose, rifaximin ● +SOB, high A-a gradient (transudative): hepatopulmonary syndrome ▪ Dx: echo ● +SOB, high RV pressure: portal pulmonary hypertension ▪ Dx: echo ● +rising Cr: hepatorenal syndrome = perfusion away from kidneys ▪ Sx: ~ prerenal azotemia ● Low calcium, low phosphate, high PTH ▪ Et: cannot make 25-hydroxy-vitD ▪ Sx: osteopenia, osteoporosis, "secondary hyperpara" ▪ Tx: calcitriol, bisphosphonate ● GI bleed!! ▪ Large bore IV ▪ Fluids, blood if 1.1 (low oncotic, high hydrostatic) ▪ SAAG < 1.1 (malignancy?!)

Primary biliary cirrhosis

Primary sclerosing cholangitis

Sx

@woman 40-60Y

@younger man w/ UC

Et

intrahepatic ducts

Intra and extrahepatic ducts

Dx

elevated alk phos/GGT (cholestatic), anti-mitochondrial Ab, US

elevated alk phos/GGT (cholestatic) 1. US 2. MRCP, ERCP annually "string of beads" to dx + dilate

Tx, f/u

UDSA, fat-soluble vitamin deficiency (ADEK)

Screen for colon cancer w colonoscopy at time of diagnosis & q1-2y; screening US 421

for cholangiocarcinoma ● ●

● ●

● ● ●

Autoimmune pancreatitis o Dx: elevated IgG4, "sausage-shaped" Hemochromatosis = deposition of iron in various organs o Autosomal recessive o Sx: ED, skin hyperpigmentation, new onset DM, calcium pyrophosphate deposition disease, arthritis o Dx: transferrin o RF: C282Y +/- HFE, H62D mutation o Tx: Phlebotomy (also for polycythemia, porphyria cutanea tarda) Wilson's disease o Sx: Kayser-Fleisher rings, elevated AST/ALT o Tx: trientine (ae Zinc def), D-penicillamine Mushroom poisoning o Tx: penicillin G Cholecystitis o Sx: fever, mildly elevated AST/ALT, thickened gallbladder o Tx: US then cholecystectomy Ascending cholangitis o Sx: RUQ, jaundice, fever o Tx: ERCP immediately Merizzi syndrome o Sx: jaundice, stone in cystic duct Cholecysto-enteric fistula/gallstone ileus o Sx: SBO! Acalculous cholecystitis o Sx: super septic, in ICU o Tx: cholecystostomy Fitz-Hugh-Curtis syndrome o Sx: RUQ pain, vagin*l discharge, adnexal tenderness o Tx: ceftriaxone Heyde syndrome o Sx: aortic stenosis, lower GI bleeds, angiodysplasia

Pregnancy ●

Hyperemesis gravidarum o Sx: 1st trimester woman, Wernicke's o Tx: vitb6 + doxylamine Intrahepatic cholestasis o Sx: 2nd/3rd trimester, pruritus o Dx: elevated ALT/AP Pre-eclampsia o Sx: 3rd trimester, elevated BP, proteinuria o c/b HELLP o Tx: magnesium (esp for seizures in eclampsia) Acute fatty liver of pregnancy o Sx: fulminant liver failure !!! Comatose, many liver fx abnormalities, encephalopathy

-------------------------------------------------------------------------------------------------------------------------------

422

Episode 213: Family Medicine Shelf Review Series 4 (GI) Episode 213 notes were graciously provided by Divine Intervention from an anonymous contributor. ● Difficulty swallowing, lost weight NBS: EGD o Alarm symptoms: lost weight, dysphagia, odynophagia, max medical therapy w/o improvement, >50Y ● Difficulty initiating swallowing NBS: video fluoroscopy ~ barium swallow o Dx: Oropharyngeal dysphagia (assoc w neuromuscular dz) ● Regurgitation of undigested food, halitosis: Zenker's diverticulum o Et: cricopharyngeal muscle dysfx o Dx: Barium swallow o c/I EGD d/t r/o perforation ● Dysphagia to solids and liquids (decreased peristalsis): achalasia = esophageal motility disorder o RF: Chagas disease ("traveled to S America") o Dx: barium swallow ("birds-beak sign") 2. esophageal manometry 3. EGD (r/o pseudo-achalasia!) ● c/b SCC of esophagus ^^ ● c/I pneumatic dilation --> perforation of esophagus! o Tx: CCB, botox, surgical myotomy + PPI ● Heartburn, cough worsened in morning, hoarseness, asthma-like sx: GERD o NBS/Tx (if no alarm sx): PPI ● *GERD + ANY other sx --> EGD o Dx: 1. EGD 2. Esophageal pH monitor o c/b: Barrett's esophagus ● Tx: endoscopic resection + ablation, PPI ● F/u: routine EGD q6-12mo ● Esophageal adenocarcinoma o RF: ● 1. Barrett's esophagus ● 2. GERD ● *pick the answer most directly related to question at hand! o Alcohol, smoking, long-standing GERD in a man --> screening EGD ● Odynophagia, HIV: Candidal esophagitis o *may not always present with thrush o Tx: -azole lozenges, nystatin swish 'n swallow o MCC esophagitis! ● Candida ● CMV ● TX: ganciclovir, foscarnet ● HSV ● Tx: acyclovir/valacyclovir, foscarnet ● Bisphosphonates, tetracycline, ibuprofen, NSAIDs, potassium, iron tablets: pill-induced esophagitis o Tx: drink lots of water/supportive care ● Young pt with dysphagia, GERD: eosinophilic esophagitis o Dx: EGD "crap ton of eosinophils" o Tx: corticosteroid therapy ● Bloating, burning, gnawing in epigastrium, feel full early (!): peptic ulcer disease o Et: H. pylori, NSAIDs ● Dx: urea breath test, stool antigen test, (antibody test suboptimal) ● Tx: "CAP" or "MBTP" o NBS: test for H. pylori o C/b: perforation, hemoptysis (esp if ulcer erodes gastroduodenal artery yikes!) 423

o

+ alarm sx! --> EGD Chronic steroid therapy? Requires bisphosphonates + PPI ppx o Et: steroids --> gastric acid production --> ulcers o Et: steroids --> avascular necrosis Diabetic, early satiety: gastroparesis o Other causes: hypothyroidism, scleroderma, chronic opioid therapy, daily diphenhydramine o Tx: erythromycin (motilin-receptor agonist), metoclopramide (dop-antagonist ae: parkinsonism) ● -LOL this is why macrolides cause diarrheaWeight loss surgery, cramping pain, nauseous, sweating, light-headed: dumping syndrome o Tx: smaller meals more frequently, low-carb (prevent hyper-insulinemia) Sudden-onset pain in epigastrium going to back: pancreatitis! o RF: EtOH, gallstones (4 F's), familial hypertriglyceridemia (>1000), stavudine, didanosine, valproic acid, CF, hypercalcemia, GLP-1 agonist o Dx: 2/3 severe abd pain, lipase 3x normal, CT o Tx: pain control, IVF o F/u: Ranson's criteria (CA HOBBES)

● ●

● o

C/b: pancreatic pseudocyst, chronic pancreatitis (tx: supplementation of fat-soluble vitamins + enzymes), peri-pancreatic abscess Hypothyroidism, Graves disease, vitiligo, Addison's///autoimmune "crap" with recurrent epigastric pain: autoimmune pancreatitis o Et: IgG4 = pancreatitis, cholecystitis, retroperitoneal fibrosis o Dx: CT scan "sausage-shaped pancreas" o Tx: corticosteroids

--------------------------------------------------------------------------------------------------

Episode 214: Family Medicine Shelf Review Series 5 - GI ●

Pt with 1 week of bloody diarrhea + RLQ pain → Yersinia enterocolitica ○ Can cause an pseudoappendicitis

AIDS pt with chronic watery diarrhea → cryptosporidium ○ Histology? Acid-fast oocysts in stool

Diarrhea in pt with recent abx, recent hospitalization, lives in nursing room → C. diff

MC cause of diarrhea? Viral (norovirus)

Pt who is avid hiker or has IgA deficiency, now with foul-smelling watery diarrhea → Giardia ○ Dx? Stool antigen testing 424

Tx? metronidazole

Pt from developing country with blood diarrhea + liver abscess → Entamoeba histolytica ○ Tx? Metronidazole

EHEC ○ Strain? O157:H7 ○ Tx? No abx!

For any bloody diarrhea, don’t give antidiarrheals (e.g. loperamide) ○ Give to pt with EHEC → increases risk of HUS ○ Give t pt with C. diff → increases risk of toxic megacolon

Chronic diarrhea + fat malabsorption → consider Crohn’s

Stool osmotic gap = 290 - 2*(Na+K) ○ Low (100) stool osmotic gap → osmotic diarrhea ■ Lactase deficiency Pt with low BMI, body-image issues. Stool osmolarity < 250 → laxative use

● ●

Pt with recurrent bouts of pneumonia/sinusitis + chronic diarrhea 2/2 giardia → IgA deficiency, CVID, or CF ○ CF - sxs from birth ○ IgA deficiency - sxs can start in 20s/30s

Pt eating lots of sugar-free candy has bloating + diarrheal episodes → diarrhea 2/2 artificial sweeteners (osmotic agents)

Pt who has bloating & diarrhea after eating. No alarm sxs. → consider lactase deficiency

Young pt with bloating/dyspepsia + sxs improve with defecation + changes in BMs. No weight loss. No lab abnormalities → IBS ○ IBS-C tx ■ Lubiprostone ■ Linaclotide ○ IBS-D tx ■ TCA (e.g. imipramine) ■ Loperamide ■ Alosetron ● Serotonin antagonist ● Effective, but associated with ischemic colitis

425

Middle-aged women with chronic diarrhea. No association with food. Colonoscopy normal. → microscopic colitis ○ Associated meds? ■ NSAIDS ■ PPIs ○ Tx? ■ Oral budesonide ■ Loperamide

25 yo pt with chronic diarrhea + fat malabsorption + microcytic anemia → celiac disease ○ Pathophys? Autoimmune attack → damaged villi → impaired absorption of iron in the duodenum ○ Ab? ■ anti-tTG ■ anti-gliadin ■ anti-endomysial ○ Dx? tTG IgA ■ If positive, EGD with bx ■ If negative, get tTG IgG (IgA deficiency is common in pts with celiac) ○ Tx? Gluten-free diet ■ MC cause of failure to respond to therapy? Non-adherence ■ Decreases the risk of small bowel lymphoma! ○ Skin finding? Dermatitis herpetiformis (vesicular rash on extensor surfaces) ■ Tx? Dapsone ○ MSK complication? Osteoporosis ■ Pathophys? Can’t absorb Vit D and calcium ■ Dx? DEXA ○ Neoplastic complication? small bowel lymphoma, i.e. EATL (enteropathy-associated T-cell lymphoma) ○ Other comorbid autoimmune diseases? ■ T1DM ■ Hashimoto’s ● Presentation? Pt taking Synthroid but isn’t getting better because it’s not getting absorbed well ○ Ileum involvement → fat-soluble vitamin deficiencies ■ Vit A deficiency → night blindness ■ Vit D deficiency → Rickets/osteomalacia ■ Vit E deficiency → acanthocytosis, ataxia ■ Vit K deficiency → bleeding issues

Pt with heavy alcohol use hx. Stopped drinking 3 years ago. High blood glucose. → chronic pancreatitis / pancreatic insufficiency ○ Dx? can check stool for fat ○ Imaging? Calcifications in pancreas on imaging ○ Tx? Pancreatic enzyme replacement

Pt with hx bowel resection / scleroderma / DM gastroparesis + macrocytic anemia → consider small intestinal bacterial overgrowth (SIBO) 426

Middle-aged man with diarrhea/abdominal pain/weight loss + joint pain + cardiac problems + eye problems → Whipple’s disease ○ Dx? EGD w/ bx ○ Tx? 1 year of abx

Pt travels to Puerto Rico and returns. Has been having chronic diarrhea + fatty stools + weight loss → tropical sprue ○ Tx? ■ Abx? Bactrim or tetracyclines ■ Folate

Young person with blood diarrhea. Sxs improves when they’re smoking → ulcerative colitis ○ Histology? Crypt abscesses ○ Affected areas? Colon + rectum ○ Dx? colonoscopy ○ Colonoscopy findings? ■ Pseudopolyps ■ continuous areas of inflammation ○ Tx? ■ Acute flare: steroids ■ Maintenance: salicylates (e.g. sulfasalazine, mesalamine) ■ If medical management fails? Proctocolectomy (remove colon and rectum) ○ Associated hepatobiliary disorder? PSC ■ Pathophys? Affected intrahepatic & extrahepatic bile ducts ■ Lab findings? Direct hyperbilirubinemia, alk phos high, GGT high ■ Marker? p-ANCA

Pt with watery diarrhea + abdominal pain + fevers + fistulas (stool coming out from skin, stool in urine) → Crohn’s ○ Dx? Colonoscopy ■ Which study is contraindicated? Barium swallow ○ Tx? ■ Acute flare: steroids ■ Maintenance (step up approach): ● salicylates ● immunomodulators (e.g. azathioprine, 6-mercaptopurine, methotrexate) ○ AZA & 6-MP are metabolized by xanthine oxidase (which is inhibited by gout drugs, like allopurinol) ○ Check TPMT activity because deficiency is common ● TNF inhibitors (e.g. infliximab, adalimumab) ○ Test for TB & Hep B before starting these ○ Histology? Granulomas ○ Colonoscopy findings? ■ Skip lesions ■ Cobblestoning ■ Strictures ○ Depth of involvement? transmural 427

○ ○ ○ ○ ○

Affected areas? MC is terminal ileum, but anywhere from mouth to anus Skin findings? erythema nodosum & pyoderma gangrenosum Mucosal findings? aphthous ulcers Eye findings? uveitis Associated rheumatic disorder? ankylosing spondylitis

Pt with hx chronic diarrhea + colonoscopy shows black mucosal → melanosis coli (benign, secondary to laxative use)

Old person with LLQ pain + fevers → Diverticulitis ○ Dx? CT with IV contrast ○ Tx? ■ Clear liquid diet ■ Cipro + metronidazole ○ Colonoscopy? NO during acute episode ■ Perform 6 weeks later to r/o colon cancer ○ When is surgery appropriate? After 2nd episode

Pt with hx Afib or recent MI with sudden-onset, severe abdominal pain → acute mesenteric ischemia ○ Dx? CT angiography ■ Catheter angiography is gold-standard, but is more invasive ○ Tx? Bowel resection if necrotic bowel Pt with hx CAD or PAD presents with abdominal pain that is worse with eating → chronic mesenteric ischemia ○ Dx? CT/MR angiography

-------------------------------------------------------------------------------------------------------------------------------

Episode 214: Family Medicine Shelf Review Series 5 - GI (Version 2) Episode 214 (Version 2) notes were graciously provided by Divine Intervention from an anonymous contributor.

Diarrhea ● ● ● ●

● ● ● ● ●

+ Bloody, RLQ pain "pseudo-appendicitis": Yersinia enterocolitica o Tx: supportive care, >1 wk --> abx + Chronic watery, AIDS: cryptosporidium "acid-fast oocysts" + Recent abx, nursing home, hospitalization: C. diff + Hiking, IgA deficiency, chronic: Giardia lamblia o Dx: stool antigen o Tx: metronidazole + Bloody, liver abscess: Entamoeba histolytica o Tx: metronidazole EHEC O157:H7: NO ABX (increased risk of HUS) *loperamide generally c/I + Chronic, malabsorption: Crohn's disease Osmotic gap o = 290 - 2(stool Na + K) o Lots of electrolytes, low osmotic gap 100: osmotic diarrhea i.e. lactase deficiency/malabsorption 428

● ● ● ● ● ● ● ● ● ●

● ●

Sx: improves with fasting + Flushing sx: carcinoid syndrome o Dx: 5-HIAA + Low BMI, self-conscious: laxative abuse o Dx: stool osmolarity < 250 + black colon: melanosis coli o No tx needed + recurrent pneumonia, sinusitis, Giardial diarrhea: Selective IgA deficiency o Ddx: CVID (all Ig low) + pancreatic insufficiency, diarrhea from birth: Cystic fibrosis + sugar-free, sour candies, cough drops, bloating: artificial sweetener-diarrhea! + bloating, w/o alarm symptoms: lactase deficiency + bloating, better sx with bowel movement: IBS o Tx: TCA, serotonin-agonists (alosetron ae: ischemic colitis), loperamide, rifaximin + 45-60Y, chronic, no temporal assoc, NSAIDs, normal scope: microscopic colitis o Tx: oral budesonide, laxative + fat malabsorption, microcytic anemia, dermatitis herpetiformis (extensors), osteoporosis (NBS: DEXA): Celiac disease o Dx: 1. anti-TTG 2. EGD w/ biopsy ● Can also see anti-gliadin or anti-endomysial Ab o Tx: gluten-free diet, dapsone for dermatitis ● F/u non-adherence to diet, ADEK deficiency o Assoc with Hashimoto's, T1DM, IgA deficiency, small-bowel lymphoma ● Ex. Thyroid replacement therapy not working d/t lack of absorption in small bowel + EtOH, hyperglycemia: chronic pancreatitis o Dx: stool fat, white streaks on imaging o Tx: pancreatic enzyme replacement + resected bowel, scleroderma, diabetic gastroparesis, macrocytic anemia: bacterial overgrowth o Et: bacteria blue --> red"

Et

Medium-vessel vasculitis

Tx

Protect from cold (i.e. gloves), dihydro CCB

Dihydro CCB (--| L-type Ca2+ channel)

3. Small vessels = capillaries, arterioles, venules o

"tumor" component of blood vessels = "purpura" ● Vs. "petechiae" d/t low platelets, collagen dysfx o Goodpasture syndrome ● Pathophys: antibodies against Type IV collagen GBM ▪ Linear immunofluorescence of C5a binding ● C5a = chemotactic for neutrophils o Child with palpable purpura under buttocks, joint pain, abdominal pain: Henoch-Schoenlein Purpura ● Tx: supportive/reassurance, check renal labs, *severe sx --> corticosteroids* ● c/I rotavirus ssDNA vaccine (also c/I w intuss, Meckel's) ● T3HS reaction (deposition of complement-antibody complexes): Iga-Ab --> activation of neutrophils --> damage to kidneys, skin, abdomen Pt from NC, migrating rash from palms/soles to inwards: Rocky Mountain Spotted Fever o Lesions on palms and soles = vasculitis! ● Rickettsia rickettsii (carried by dog tick) invades endothelial cells --> inflammatory response o Tx: doxycycline (even if under 8 years old!) ● *pregnant women get chloramphenicol Meningitis, purpura: Meningococcal meningitis o Neisseria penetrates blood vessels --> purpura o C/b adrenal insufficiency: Waterhouse-Friedrichsen syndrome ● Tx: replace mineralocorticoids, glucocorticoids (prednisone, dexamethasone, etc) o Tx: ceftriaxone Female, HepC, renal-like sx, stroke, moved to colder climate: Mixed cryoglobulinemia o RF: HepC, Sjogrens, SLE ● HepC also a/w membranoproliferative glomerulonephritis, porphyria tarda o Pathophys: IgMs agglutinate at cold temperatures 464

o o

Sx: palpable purpura, sx better in warmth Tx: tx underlying disease, cyclophosphamide

-----------------------------------------------------------------------------------------------------------------------

Episode 233: Shock This episode provides an overview of septic, neurogenic, cardiogenic, and hypovolemic shock. Septic shock ● Who gets septic shock? -> person with some kind of infection/inflammatory process ● What gets released? -> histamine, heparin, bradykinin, leukotrienes, prostaglandins ● This release leads to overall -> vasodilation ● If vasodilation occurs that means systemic vascular resistance (SVR) goes DOWN ● If SVR goes down that means afterload goes DOWN ● If afterload goes down that means it's easier for blood to eject from heart ● If it's easier to eject blood from heart that means Cardiac Output goes UP ● If cardiac output goes up, what happens to the pressures in both heart atriums? -> Down because blood moves forward through the heart. ● The surrogate for right atrial pressure is -> Central Venous Pressure (CVP) ● The surrogate for left atrial pressure is -> Pulmonary Capillary Wedge Pressure (PCWP) ● Therefore the CVP (i.e., right atrial pressure) and PCWP (i.e., left atrial pressure) in septic shock will both go -> DOWN ● What represents - for the most part - the oxygen saturation of blood that returns from the body to the right atrium? -> Mixed venous oxygen saturation (MVOS) ● Expected mixed venous oxygen saturation (MVOS) in pt with septic shock? HIGH ● Why is MVOS high in Septic shock? Think back to Step 1 with Fick's principle on the cardiac output:Cardiac Output = Oxygen delivery/ (O2 content at artery - O2 content at vein) ● If we said Cardiac output goes UP in septic shock that means the whole denominator (O2 content at artery - O2 content at vein) has to go down. ● How does the denominator go down? -> the O2 content at the vein is increased. ● In review, what is the expected mixed venous oxygen saturation (MVOS) in pt with septic shock? HIGH ● What are the four SIRS criteria to dx sepsis? ○ Meet two or more of following criteria: a. Temperature (>38) b. HR > 90 bpm c. RR > 20 min or partial pressure of CO2 (PCO2) < 32 mmHg d. WBC > 12K or pt is considered septic ● If pt is hypotensive or organ failure (increased LFT, troponin, creatinine, low bicarbonate) -> severe sepsis ● What are bicarbonate levels in lactic acidosis? low ● What broad spectrum (that covers pseudomonas & MRSA) antibiotics are typically given to pt with sepsis? ceftazidime & vancomycin or piperacillin and tazobactam ● ●

Mean arterial pressure = 2/3 diastolic + 1/3 systole (blood spends most of its time in diastole) What is the vasopressor of choice in hypotensive pts with sepsis? Norepinephrine 465

Key principles in septic shock: Cardiac Output and SVR go in opposite directions in shock. CO goes up while SVR goes down, exception: If a patient has neurogenic shock!

Neurogenic shock ● Who? High spinal injury (anesthesia - epidural) or spinal cord surgery or IVDU with spinal epidural abscess (i.e anything that impacts spinal cord at thoracic levels). ● What happens to the sympathetic system if the spinal cord is impacted at thoracic levels? -> Knocked out. ● If you lose sympathetic tone that means not only does your Systemic Vascular Resistance (SVR) go DOWN, but also your Cardiac Output (CO) goes DOWN ● Example, if you see a vignette pt. with bradycardia ( Mixed venous oxygen saturation (MVOS) ● This means that mixed venous oxygen saturation (MVOS) goes DOWN in cardiogenic shock. ● If cardiac output goes down. spread between arterial and venous content goes up (the denominator). ● What is the treatment for cardiogenic shock? positive inotrope ○ Digoxin and Dobutamine (beta-1 agonist) ○ Milrinone (PDE inhibitor that increases cardiac contractility but decreases SVR) ○ What happens to cAMP levels if you inhibit PDE? increases cAMP ■ High cAMP in cardiac muscle = increase contractility ■ High camp in smooth muscle (e.g., blood vessels ) = decrease SVR ■ Why does pulse pressure go up in milrinone? CO goes up so SVP goes up; SVR goes down so DVP goes down. ■ Pulse pressure: the spread in systolic and diastolic pressure gets wider. Hypovolemic shock ● Who? pt. is bleeding out (can be internally or externally) ● If patient is bleeding out the preload goes DOWN ● If preload goes down, CO goes DOWN ● Therefore, your SVR should go UP ● The surrogate for right atrial pressure is -> Central Venous Pressure (CVP) ● The surrogate for left atrial pressure is -> Pulmonary Capillary Wedge Pressure (PCWP) ● If patient has low CO because they have low fluid in body, their CVP and PCWP will both go DOWN ● What represents the oxygen saturation of blood that returns from the body to the right atrium?->Mixed venous oxygen saturation (MVOS) ● This means that Mixed venous oxygen saturation (MVOS) goes DOWN in hypovolemic shock. ● Tx. Fluids 466

15:39 Additional information (not seen on NBME) "What do you mean by qSOFA criteria Divine?" -> quick Sequential Organ Failure Assessment 1. Altered Mental Status 2. RR >22/min 3. Systolic is < 100 mmHg If you meet at least one of those criteria, the patient is septic. CROSS CHECKED? YES -------------------------------------------------------------------------------------------------------------------------------

Episode 233: Shock (Version 2) Episode 233 (Version 2) notes were graciously provided by Divine Intervention from an anonymous contributor.

Septic shock ●

Pathophys: infection/inflammatory state --> release of mediators (heparin, bradykinin, LTE, PG) --> vasodilation --> SVR decrease --> afterload decrease --> easier for blood to be ejected from the heart --> CO increases --> LA/RA = PCWP/CVP pressures decrease (blood is moving forward through the heart) o Mixed venous oxygen saturation = increased ● = O2 sat of blood that is in the RA = blood that has returned from all the veins in the body ● Fixed principle: CO = O2 delivery/(O2 content of arteries - O2 content of veins) ▪ Septic shock: CO increases --> difference in O2 artery-venous should decrease --> O2 content of veins goes up (MVO2 goes up) o *CO and SVR go in opposite directions in shock* ● only exception is neurogenic shock SIRS+ criteria: >= 2 criteria o Temperature 38C o HR >90 bpm o RR >20 bpm o PaCO2 12000 or @spinal cord thoracic levels --> take out sympathetic nervous system --> lose vascular tone --> SVR decrease, CO decrease o *the only kind of shock where SVR AND CO are low*

Cardiogenic shock ●

Pathophys: cardiac tamponade, CHF, post-MI --> CO decreased (heart cannot pump fluid forward), SVR increased --> fluid backs up in the heart --> CVP/PCWP increased o CO is low --> tissues get very efficient at extracting oxygen --> O2 sat of blood returning to RA is very low (MVO2 decreased)

467

Tx o

Positive inotropic = digoxin, dobutamine (b1 agonist), milrinone ● Milrinone = PDE inhibitor --> increased cAMP --> increased cardiac contractility, decreased vascular resistance ▪ Pulse pressure increases due to increased SBP + decreased DBP

Hypovolemic shock ●

Pathophys: bleeding out --> body volume goes down --> preload goes down --> CO decreased, SVR increased --> PCWP/CVP decreased, MVO2 decreased Tx: fluids

● ●

*QSOFA criteria (1+): AMS, RR >=22, SBP poison ivy Psoriasis ● See rash at extensor surfaces, think psoriasis Xerotic eczema ● 79 y/o (old people in general) that presents to dermatologist bc really dry skin on the left shin (treebark appearance at lower extremities). On PE skin is red/dry, neck-like fissures. Skin reaction gets worse in the cold, dry winter months. Treatment for eczema, general terms 477

● Tx emollients. Topical glucocorticoids ● Special cases: ○ Do NOT use topical glucocorticoids on dermatitis of the face ○ One of the MCCs of acne on the face on NBME exams? topical corticosteroids Plaque psoriasis ● 36 y/o Mexican male with thick erythematous plaques with silver scales on elbow (Extensor surface) ● Show on elbow, knees, scalp, ears, genitalia ● Classic exam findings in psoriasis -> very thick NAILS. yellow, nail-pitting, Hispanic ● Things that worsen: beta-blocker, NSAIDS, ace inhibitor, oral steroids, tetracycline Guttate psoriasis ● Pt. with papules and plaques on their trunk. Looks like tiny tiny drops. ● Patient recently developed group A skin infection (i.e., strep pyogenes) ● Tx. Vitamin D analog - topical calcipotriene; retinol; anthralin; or tar prep NEVER give oral/IV steroids for psoriasis. Systemic steroids can worsen psoriasis. Erythroderma ● Pt with a hx of psoriasis and is taking steroids. Skin turns really red. ● Complication: electrolyte abnormality (loss of fluid) Lichen planus ● Pt. has noticed a lot of pruritus these past few days on wrists and ankles. Noticed purplish papules shaped like polygons. ● Tx. topical corticosteroid Pityriasis rosea ● Pt over the last two weeks has pruritus. Noticed circular or rectangular lesion on trunk (Herald patch), under axillae and lasts for weeks. ● Tx. topical steroids and antihistamine for pruritus Seborrheic dermatitis ● Pt. has oily, scaly lesion on eyebrow on scalp, nasolabial folds, chin or perineal cysts ● Tx. selenium sulfide shampoo ● If pt is young person sexually active with tons of lesions that resemble seborrheic dermatitis -> screen for HIV ● Typically also seen in patients with Parkinson's disease Rosacea ● 35 y/o female with redness of cheeks/nose whenever she eats spicy food or drinks alcohol has facial flushing. Can see papules/pustules. ● Usually >30 y/o female ● Don't confuse this patient with malar rash on lupus. Rash of lupus SPARES nasolabial folds. People with lupus DO NOT have papules, pustules on face or flushing episodes. 478

Adrenal Tumor (DHEA) or Gonadal (Sertoli-leydig tumor) ● Woman who suddenly develops severe acne and hirsutism with menstrual problems or signs of virilization. Hidradenitis suppurativa ● Pt. with a history of diabetes who have had chronic lesions under axilla, breasts or in gluteal clefts. Nodules, cysts, comedones, a lot of scarring. ● Pathophys: apocrine sweat glands. ● Tx. Clindamycin or Rifampin also Infliximab ● Definitive Tx. excision of lesions Acne vulgaris ● Open and closed comedones, papules, pustules, nodular lesions. Distribution at face, neck or upper trunk ● Tx. topical retinoid or salicylic acid or benzoyl peroxide. ○ Not work? Add a topical antibiotic (e.g., erythromycin, clindamycin). ■ Not work? Add an Oral antibiotic (tetracycline) ● Not work? Give isotretinoin ● ● ● ●

If pt has acne, visual headaches worse in the morning -> idiopathic intracranial hypertension. What labs to order before giving isotretinoin? LFT's, B-HCG Don't give preggos isotretinoin or tetracycline Pt with PCOS + acne (hyperandrogenism) DOC = OCP's.

Bacterial folliculitis ● Athletic patients with pustules and papules on the scalp (anywhere with hair) centered around hair follicles. ● MCC: staph aureus ● Tx. mupirocin Hot tub folliculitis ● Pt in a community pool/hot tub liquidly chlorinated ● MCC: pseudomonas Hot tub lung ● MCC: mycobacterium avium intracellularly complex Dermatophyte Skin Infections (Tinea ● MCC trichophyton tonsurans > microsporum species > epidermal phyton species Tinea pedis ● Scaling from toes to areas of achilles heel (athlete's foot) Tinea corporis (ringworm) 479

● Erythematous circular red lesion with vesicles with "Central clearing" Onychomycosis / nail fungus ● Yellow, thick nails or really white. Distal edge (farthest away from skin is elevated) Cutaneous candidiasis. ● Red, itchy skin with red satellite lesions. After scraping- > KOH prep see spores and pseudohyphae. Tinea versicolor or pityriasis versicolor ● Hypopigmented macules on upper trunk or back. On prep: "spaghetti and meatball pattern" ● MCC: malassezia furfur Tx general ● All tinea: topical antifungals: Clotrimazole (any -azole) ○ Special exception: ■ Tinea Capitis (head): oral medication - terbinafine, griseofulvin ■ Griseofulvin (penetrates keratin containing tissue) ■ cutaneous candida: topical nystatin or other azole ■ Tinea versicolor: selenium sulfide or topical azole Molluscum contagiosum ● Pt. with umbilicated papule on skin (can de adult or kid) ● cryotherapy or curettage ● Remember assoc. with HIV! CROSS CHECKED? YES -------------------------------------------------------------------------------------------------------------------------------

Episode 243: Water Soluble Vitamins ●

Pt found on street by police, brought into ED. Swaying side to side + nystagmus + doesn’t know how he got to hospital → Wernicke’s encephalopathy ○ Population? ■ Alcoholics ■ Hyperemesis gravidarum ■ Eating disorders ■ Starvation ○ Triad? Confusion + ophthalmoplegia + ataxia ○ Pathophys? Thiamine deficiency ■ B1 is part of the TLCFN cofactor group (cofactor for pyruvate dehydrogenase complex, alpha-ketoglutarate dehydrogenase, branching ketoacid dehydrogenase) ● Defect in branching ketoacid dehydrogenase → maple syrup urine disease ■ B1 is a also a cofactor for transketolase ● Transketolase dysfxn implicated in Wernicke’s ○ Neuroanatomical association? Hemorrhagic infarction of the mammillary bodies 480

○ ○

○ ○

● ●

What if they have confabulation + amnesia → Korsakoff syndrome Prognosis? ■ Wernicke’s → reversible ■ Korsakoff → permanent Tx? Give thiamine BEFORE glucose Other presentations of thiamine deficiency? ■ Generalized edema + other signs of CHF → Wet beriberi ■ Ataxia + paralysis + sensory sxs → Dry beriberi

B2 = riboflavin ○ Required for production of FADH2 ■ Part of the TLCFN cofactor group B3 = niacin ○ Required for production of NADH/NADPH ○ Presentation of deficiency? ■ dermatitis + chronic diarrhea + dementia → pellagra ○ Causes of niacin deficiency? ■ Hartnup disease ● Can’t reabsorb neutral AAs (e.g. tryptophan, which is used to make niacin & serotonin) ■ Carcinoid syndrome ● All the tryptophan is being shunted towards serotonin production, so there’s not a lot available to produce niacin ● Presentation? Flushing episodes + chronic diarrhea + holosystolic murmur at LLSB that increases w/ inspiration (likely tricuspid regurg) ● Sxs only occur once metastasized ○ Liver metabolizes the serotonin ● Why only R-sided heart murmurs? ○ Lungs also metabolize serotonin ○ Therapeutic use of niacin? Best way to raise HDL ■ Better than statins! ■ AE? Flushing + itching ● Tx? NSAIDs B5 = pantothenic acid ○ Used to make coenzyme A B6 = pyridoxine ○ Cofactor for transaminases ○ Cofactor for glutamate decarboxylase (glutamate → GABA) ○ Cofactor for ALAS (1st step in heme synthesis) ○ Presentation of deficiency? ■ Sideroblastic anemia ■ Seizures ○ Drug that causes B6 deficiency? Isoniazid ■ Other AE? drug-induced lupus B7 = biotin ○ Cofactor for carboxylase enzymes ○ Very rare, it’s difficult to get biotin deficiency 481

Odd cause of deficiency? egg whites contain avidin protein, which binds biotin and can cause deficiency B9 = folate ○ Necessary for DNA synthesis ○ Converts hom*ocysteine → methionine ○ Causes of deficiency? ■ Alcoholism ● Alcohol inhibits conjugase, which helps us reab ■ Small bowel reabsorptive disorders ■ Poor nutrition (body stores of folate only last months) ■ Chronic hemolytic anemia (e.g. sickle cell, hereditary spherocytosis) ■ Drugs ● Phenytoin ● Methotrexate ○ Rescue agent? Leucovorin (folinic acid analog) ● TMP-SMX ○ Presentations of deficiency? ■ Megaloblastic anemia ■ Neural tube defects in fetus ○ Lab findings? Elevated hom*ocysteine B12 = cobalamin ○ Converts hom*ocysteine → methionine ○ Converts methylmalonyl-CoA → succinyl-CoA ○ Presentations of deficiency? ■ Megaloblastic anemia ■ Dementia ■ Subacute combined degeneration ● Damage to dorsal columns + lateral corticospinal tract ○ Causes of deficiency? ■ Pernicious anemia ■ Crohn’s affecting terminal ileum ■ Strict vegan diet ■ Diphyllobothrium latum (fish tapeworm) ○ Lab findings? Elevated hom*ocysteine AND MMA Vitamin C ○ Cofactor for synthesis of collagen ○ Presentation of deficiency? ■ Bleeding gums + poor wound healing → Scurvy ○ Therapeutic use? Tx of methemoglobinemia ■ Keeps iron in the Fe2+ form

CROSS CHECKED? NO ----------------------------------------------------------------------------------------------------------------------------

482

Ep 244: (Cardiac Valvular Disorders)

483

-

Q1 79 m with syncopal episodes, and shortness of breath when mowing lawn or with physical activity - Think Aortic Stenosis - These pts usually above 65, old guy with syncopal episodes almost always do this. Angingal syncopal HF symptoms. - Point of Maximal impulse laterally displaced causes concentric hypertrophy of L ventricle. - Old guy + Syncopal episodes = AS - Heard best at R upper sternal border (imagine the anatomy to help) - Radiates to carotids. Pulsus parvus atardis = tardy means late takes a long time to feel carotid pulse even though you feel heart pumping - In Aortic Stenosis the valve is calcified! - You can see it in a younger person, when? If they have a Bicuspid aortic valve. Normally there are 3 cusps/leaflets, if only 2 cusps they have to do more work and wear out and calcify. Turner syndrome pts classically has a bicuspid aortic valve. - Intensity increases with anything that puts more blood in the L ventricle, like giving fluid bolus. - Replace valve to treat. Think “ASH” when considering replacement, angina, syncope, HF, HF meaning you definitely need to replace. 484

-

You have to replace you can't do valvotomy cause valve is too calcified to balloon You can usually answer a question like this even without the audio if you consider the scenario (but it is crescendo-decrescendo systolic ejection murmur over 2nd R-ICS)

-

-

Aortic Regurg -

-

Heard best at the left sternal border OR left lower sternal border, think of the direction of regurg anatomically to help remember. Diastolic Murmur If the aortic regurg is really bad it can also be heard at the right sternal border, rarer Hints to diagnose: if bp is 120/40 or if 150/50 (ie wide pulse pressures), it's a giveaway to aortic regurg. Mechanism of widened pulse pressure: The systolic pressure increases because it is correlated to cardiac output. Cardiac output is increased in aortic regurgitation because preload increases when the volume is regurgitated back. Diastolic pressure is related to systemic vascular resistance. In aortic regurg blood can relax and go back into the heart, lowering SVR and hence diastolic BP (see divines explanation it's a lot better). Another example of widened pulse pressure is Patent Ductus Arteriosus, systolic is up because there is an extra source of preload (the PDA), and also a conduit for blood to go during the relaxing phase of diastole, lowering SVT and diastolic BP Head Bobbing, hyperdynamic pulse, are a result of a wide pulse pressure phenomenon. Treat pts with something that will lower afterload, as it will lessen the amount of regurgitation. If less afterload blood has more motivation to move forward and not regurg back.

485

-

Other notes:

-

-

Q: Patient had URI 30 years ago, now having palpation, EKG shows afib, what's your diagnosis? Mitral Stenosis - Diastolic murmur with opening snap - Heard best at apex (like all mitral murmurs) - #1 RF for Mitral Stenosis is rheumatic fever - #1 RF For AFib is Mitral Stenosis. Left Atrium dilates as it is pushing against resistance, causing AFib - If patient is Afib due to a valvular problem (like mitral stenosis), the only anticoag you can use to treat them (chadsvasc) is WARFARIN , you CAN'T use a NOAC

-

Q: Pt is 6’5 with hyperflexible joints, they are known to have Ehlers-Danlos syndrome, they also have a midsystolic murmur heard best at the apex with a click, what is the cause of the murmur? Mitral Valve

Prolapse -

-

It is very similar to mitral regurg, MVP is kind of like a subset of mitral regurg - Lets compare the two. Mitral Regurg is holosystolic, while MVP is midsystolic with a click. - How to make MR louder? By putting more blood in the left side of the heart, increasing preload via squatting, going standing to supine, or bolus of saline. Can also be made louder through the handgrip maneuver, which increases afterload, blood will be like hey it's harder for me to go forward, so i guess i'll just regurg back through the mitral valve MVP also has blood regurg, this is because the valve leaflets do not overlap well. - How do we make the leaflets overlap better? Also by putting more blood in the left ventricle of the heart, essentially erase the prolapse and decrease the amount of regurg. This also makes the murmur sound LESS intense. So if squat will increase preload, 486

-

make murmur less intense. If you increase afterload via handgrip, this will also make murmur less intense. So anything that increases preload or afterload makes MR louder, and MVP less intense MVP is caused by myxomatous degeneration of the valve. MVP associated with Marfan's and Ehlers Danlos (and ADPKD). Also in people with psych disorders, mostly in younger people on NBME exams.

-

Holosystolic Murmur at the left lower sternal border? VSD - VSD is most common congenital heart defect - Blood initially flows from left to right, but eventually after pulmonary hypertension develops blood flows from R to L shunt, this is called Eisenmenger syndrome

-

Fixed Split S2? ASD - But this is too buzz wordy, NBME will probably be like, you hear split S2 on inspiration and expiration, what is murmur? ASD. - Normally only S2 split in inhalation, reason being inhalation causes increased blood return to the right side of heard, so it takes a bit longer for it to pump it compared to the left side of the heart, so valve closes a bit later, hince S2 normal physiological split in inhalation - So if you see person with S2 split on EXPIRATION think ASD - [My notes are not divines: ASD has fixed split S2 meaning the split does not widen in inhalation or exhalation, its constant.] - ASD commonly caused by secundum defect, but in kids with Downs syndrome its cause of primal defect

-

Pt has loud P2, what does that indicate? Pulmonary Hypertension

-

What is going on if A2 is occurring after P2? - Paradoxical splitting, splitting is occurring only in exhalation. That's weird, why is that? - Occurs in any instance that slows down left ventricle, like LBBB, or really bad aortic stenosis

-

Q: Patient has syncopal episode while playing sports, he had an uncle die at 35, what is the diagnosis? Hypertrophic Obstructive Cardiomyopathy, a genetic disease, has autosomal dominant inheritance - Caused by mutations in myosin binding protein C and B - Heard best at left lower sternal border - It's a systolic ejection murmur that does NOT radiate to carotids (unlike AS which does) - Bifid pulse seen in HOCM pts - What makes it louder? In HOCM there is systolic anterior motion of mitral valve leaflet (imagine what's going on here anatomically) this motion causes obstruction, as well as regurgitation. So blood not only has to go through thickened septum, but also through the mitral valve leaflet that is being pulled forward in systole. This is what causes the bifid pulse. - So if you move this mitral valve leaflet out of the way the symptoms wouldn't be as bad and the murmur would be less loud - This can be done by increasing amount of blood in left ventricle (anything that increases preload squatting etc)(Also anything that increases afterload like handgrip)

487

-

-

Treat via B blockers, which cause heart to slow down, allowing it to fill more in diastole, the increased volume in the left ventricle will then decrease the bad movement of the mitral valve leaflet Increasing Preload (squatting) make murmur LESS intense

-

Aortic Dissection is caused by cystic medial degeneration (buzzword) - Tertiary Syphilis, Marfans, Ehlers Danlos pts can all get AD

-

Carcinoid Syndrome - Can have tricuspid insufficiency, pulmonic stenosis (TIPS mnemonic) - Also Be FDR mnemonic, ie bronchospasm flushing diarrhea right sided heart problems

-

IV Drug users who inject drugs into their VEINS, what valve will be affected? Tricuspid

-

What Murmurs do you investigate? - If systolic and equal or louder than Grade III (ie louder then s1s2) - ANY Diastolic murmur no matter how loud - Any symptomatic murmur - Holosystolic also investigated on NBME exams - In peds world, if musical qualities to murmur dont worry thats benign you don't need to investigate it

-

Life Advice: Keep calm, step back, think what you can solve, what is out of your control don't worry about it :) don't worry, God loves you :) God Bless :)

-

CROSS CHECKED? Yes! ----------------------------------------------------------------------------------------------------------------------------

Ep 245: (Rapid Review Series 39) Pt. with severe pain on finger mcp dip knee or something. Red erythematous -> septic arthritis ● NBS : Arthrocentesis ○ WBC count > 50k ○ Need a washout ○ What bug? -> Staph aureus ■ How to treat? Vancomycin ● Sickle cell? - > Salmonella Same as above plus Petechiae on skin -> gonococcal septic arthritis. Tx: ceftriaxone or cefotaxime · If negative gram stain or not sure, can give both vanc and ceftriaxone · How differentiate between septic arthritis and osteomyelitis? o Septic: pain over a joint o Osteo: tender over a bone 488

Cervical ca screening: 21-29 pap q3 yrs, don’t start before 21 even if sexually active. · 30 yo, continue Paps but preferred to do q5yrs with HPV co-testing · Immunocomp (HIV, immunodeficiency like DiGeorge, history of high-risk lesions): Paps annually · In utero DES: Paps annually · Hysterectomy for benign reason: can stop Paps · If Hysterectomy due to endometrial ca or hyperplasia: need Paps of vagin*l cuff o Usually stop screening at 65 if no history of abnormal findings for last 3 Paps or last 10 years if co-testing, ● unless history of CIN or cervical lesion resected then need Paps for 20 year period after lesion resected ● Remember: conizations increase risk of cervical insufficiency Pt that has 2-PPD smoking history for 5 years, in past 3 weeks has severe HA and face appears “bulging” and have neck pain and veins appear distended -> SVC syndrome · Cannot do chemo for symptom relief, instead must radiate lesion (Tx of choice) · Assoc with Small cell lung cancer Pt with Family hx of pancreatitis, presenting with epigastric pain to back but no hx of alcohol or just socially, no hx of gallstones -> familial hypertriglyceridemia · Type 4 familial dyslipidemia: AD pattern · Tx: give fibrate (gemfibrozil), better than statins to lower TG o MOA: activate PPAR-alpha (transcription factor to make more LPL to help clear out TGs) o Side effects: myopathy, hepatotoxic, increase risk of gallstones Pt with DM and HFrEF, do not give thioglitazones o PPAR-gamma activator *different from above!* ● (PPAR-gamma receptor also in kidney so increase water absorption and increase volume retention and worsens HF) Pt with 2 weeks hx of gen edema, UA with 4+ proteinuria, then pt complaints of chest pain, SOB and super tachy, CT angiography shows filling defect think membranous nephropathy ● MOA? ○ Acquired deficiency of clotting inhibitor o ATIII peed out in urine, cannot inhibit factor X or factor II so then these go make clots in body and cause renal vein thrombosis (sudden onset severe flank pain, hematuria) or PE o Membranous Nephropathy: Ab against phospholipase A2-receptor ● Also assoc with solid organ cancers (colon, etc.) and SLE ● If lupus and nephritic diffuse membranoproliferative GN Pt is a 32 yo female sex active with HTN -> OCPs 489

·

MCC HTN in reproductive age females

If given scenario of PaO2 saturations of various areas around heart, example – SVC PaO2 sat 70% and right atrium sat 71% but right ventricle sats 80% and pulm artery is 81% think of blood from left side of heart is mixing with right side of heart which increases oxygenation · Think ASD or VSD: ask where have biggest jump in saturation? o If SVC to RA: ASD o If RA to RV: VSD ----------------------------------------------------------------------------------------------------------------------------

Ep 246: (Derm Part 2 of 3) Pt. is a 31 y/o male who is active military who came back from Afghanistan (Iraq, Saudi Arabia, Peru etc.) 2 to 3 weeks ago. On his arm (or anywhere typically on upper extremities) there is a painless, purplish ulcerating papule. ● Dx ○ Leishmaniasis ■ Transmission ● Sandfly ■ How to diagnose? ● Skin Biopsy ■ Treatment ● Amphotericin B or Paromomycin Pt is 50 or older there is a recombinant zoster vaccine ● NOT live attenuated. ● Can start administer 50 y/o ○ Eligible ● Reduce risk of post-herpetic neuralgia ● Reduce incidence of zoster Pt is over 60 y/o there is a live-attenuated zoster vaccine ● live -attenuated given to those over 60 ● Immunocompetent ○ If has HIV or CLL or immunodeficiency, do NOT give a live-attenuated vaccine Typical vignette for zoster: pain, rash dermatomal distribution. ● But, if patient has a “zoster explosion” in body ○ NBS: Screen for HIV Pt has a sudden outbreak of molluscum contagiosum ● NBS: Screen for HIV 490

Pt has porphyria cutanea tarda ● NBS: Screen for Hep C Pt. with zoster with vesicular rash spread in dermatomal distribution on first branch of trigeminal nerve, tip of nose and eye (Opthalmic branch) ● Refer to ophthalmologist ○ Zoster ophthalmicus Pt has vesicles in the ear and anterior sensation of taste of ⅔ tongue gone. Paralyzed upper and lower part on one side of face (like bell’s palsy) in the Cranial 7 pattern. ● Ramsay Hunt Syndrome (herpes zoster oticus) ○ Treatment ■ Acyclovir Post-herpetic neuralgia ● Treatment ○ Gabapentin ○ nortriptyline, amitriptyline (be careful in elderly) Do not give steroids in herpes zoster!! Pt is a young kid, homeless who comes with referral. Itchy rash between finger webs, penis, scrotum. ● Dx ○ Scabies (sarcoptes scabies) ○ How? ■ Mite burrows in the upper layer of skin ○ Disseminated scabies ■ HIV, immunocompromised ○ How to diagnose? ■ Swab tissue and find mites and eggs on KOH prep ○ Treatment ■ Permethrin (also family members) ■ Ivermectin ■ Wash everything in hot water ■ DO NOT pick Lindane lotion ● Neurotoxic, induces seizures in children Pt. with an itchy lesion in skin. On exams, grouped papules that are very itchy. “Breakfast lunch and dinner lesions” Red circles in very close approximation. Usually in the morning. ● No real treatment (do antihistamines topical steroid etc) Pt. with a history of HIV with brown lesions that look like a tan. Well demarcated plaques, papules with a “Stuck-on” appearance ● Dx 491

○ Seborrheic keratosis ● What to do with it? ○ Excision ○ Liquid nitrogen Pt with sudden onset with tons and tons of stuck on appearances. What to screen for? ● GI malignancy (colonoscopy, EGD etc etc) 17 yo female not sexually active. Has warts. Flesh colored papules. Genital warts. Those are the things known as? ● Condyloma acuminatum [Do not confuse with Condyloma latum (syphilis) - do not confuse!] ● Treatment ○ Topical salicylic acid (works for acne) ○ Cryotherapy ○ Podophyllin ● Most likely sequelae? ○ Spontaneous resolution ● Red lesion on sun exposed spots (Face, back) lesions with “rough sandpaper appearance/ texture; bad rough spot” ○ What is it? ■ Actinic keratosis ● Precursor to? ○ Squamous cell carcinoma ● Treatment ○ Topical agent (5-FU) ○ Imiquimod ● Biggest RF for skin cancer -> sun exposure ○ UV-A vs. UV-B light ■ UVB light is worse ● Thymidine-thymidine dimers form ● Primary preventive strategy for skin cancer? ○ Use clothes that will protect you from the sun ■ Pick sunscreen if there is no answer choice that gives you sun protective clothing or sun avoidance ● Pt was rescued from a fire. Has healed over time with plastic surgery. On his scalp there are lesions that haven’t resolved. Have been slowly evolving. ○ Dx ■ Squamous cell cancer (usually bottom lip, but you can get it anywhere - like the scalp, ear, and neck) ● Pt has a red nodule that has continued quickly growing over time, and looks like a volcano. Contains a lot of keratin, debris at the center. Looks like it’s going to erupt. 492

○ Dx ■ Keratoacanthoma ○ Treatment ■ Excision of lesion ○ Keratin indicates what? ■ Squamous malignancy ● Pink pearly, translucent lesions with telangiectasias on upper lip ○ Dx ■ Basal cell carcinoma ○ Spread? ■ Likes to spread horizontally ■ Very rapidly destructive ○ Treatment ■ Resection ■ Sometimes Mohs surgery (same as micrograph surgery) ● Pt with a lesion on skin with many different colors (black, brown), which is not round or oval, more irregular borders ○ Dx ■ Melanoma ● Criteria ○ Asymmetry ○ Borders - irregular ○ Color variation (brown, red, black, blue) ○ Diameter - > 6mm we get worried ○ Evolution - changing over time ● Pt with a history of dysplastic nevus. ○ Risk factor for melanoma ○ Looks a lot like melanoma. ■ Dysplasia leads to cancer ● Pt with a family history of melanoma. With a ton of dysplastic nevi. ○ Dx ■ Familial melanoma dysplastic syndrome ● Inheritance ○ Autosomal Dominant ● Melanoma ○ Different types ■ Nodular ● Worse prognosis ■ Acral lentiginous ● African american with melanoma under nail bed 493

● Not as bad prognosis as nodular ■ Lentigo maligna ● Pt has a melanoma that is on the face, upper-trunk, prominently exposed to sun ■ Superficial spreading melanoma ● Best prognosis ● Shows up on back in men. Legs in women. ● Good prognosis ○ Treatment ■ Complete excision ■ More than 1 mm thick, send a sentinel lymph node biopsy ■ Prognosis ● Breslow depth/thickness ● Pt sat on couch and has been itching with wheeling of skin ○ Treatment ■ Antihistamine ● Pt with angioedema do NOT have hives! ● What is the most common medication that people report an allergy to? ○ Penicillin ■ If they try to test patient on allergy ● Do skin testing (not RAS(?) or ELIZA test) ■ Pt with anaphylaxis? ● Anti-staph, cephalosporins should be avoided ● Pt. is a 6 y/o male with lyme disease. Given doxycycline (or adult that gets treated for syphilis, lyme disease) develops fever, headache, myalgia, malaise, sweating, headache, hypotensive ○ Dx ■ Jarisch-Herxheimer reaction ● Treponema pallidum or borrelia etc. when you treat spirochetes they will explode and release endotoxins (penicillins are cell wall inhibitors) ● Resolves quickly ● Supportive care ● Continue antibiotic ● Not an allergic reaction ● Pt recently took TMP-SMX for cystitis. Last two days the patient has an edematous face. Generalized skin reaction. Person AST/ALT and eosinophil elevated, elev. Lymphocytes and generalized lymphadenopathy ○ Dx ■ Hypersensitivity syndrome (Type IV) ■ Dress Syndrome (same thing) ■ Treatment 494

● IVIG

----------------------------------------------------------------------------------------------------------------------------

Ep 247: (Rapid Review Series 40) Pt is a 51 y/o fEMA with right sided headache for the past four days with diffuse muscle aches and pain. On labs ESR and CK are both elevated. ● Dx: Temporal arteritis ● Associ: proximal muscle tenderness (polymyalgia rheumatica), jaw claudication ● NBS: High Dose Corticosteroid Therapy (eventually you biopsy) Pt is a 27 y/o m with hx. of multiple suicide attempts. Over the last 24 hours his close acquaintances have noticed he has had bloody bowel movements, hemoptysis with nasal bleeds and abdominal pain. ● Most likely ingestion? Rat poison ● Why? Rat poison contains warfarin. ● MOA? Inhibit Vit K epoxide reductase (Protein C, S and Factors 2, 7, 9, 10) Pt is a 22 y/o Male with 6 months of trouble breathing and low back pain. On PFT, noticed to have a restrictive pattern of lung disease. ● Dx: Ankylosing Spondylitis ● Why? They have spinal problems. Can cause restricted expansion of the lungs. Cavity is kind of contorted. What is the lung problem? Will have restrictive lung disease with normal DLCO. Pts are a couple of gardeners. Over the last 12 hours they've had diffuse muscle weakness, blurry vision, ptosis on a fundoscopic exam. Been having constipation, feeling very hot, hyperthermic. ● Dx: Botulism ● MOA: can't release ACH at neuromuscular junction Pt. is a 23 y/o male that is sexually active. Over the last 3-4 weeks flesh colored growths on genitals. ● Dx: Condyloma acuminatum ● Cause? HPV 6,11 (NOT 1,6 which cause plantar warts) (16, 18 cause cervical cancer) ● Tx: Podophyllin (topical agent) Pt is a 27 y/o male with hx of DMT1. Over the past 2 days has had a severe headache. Bilateral babinski sign. Neurodeficits. Glucose at 700. Bicarb at 10. pH at 7.16. On imaging has ring-enhancing lesion in brain. ● DX: Diabetic Ketoacidosis ● Bug? Mucormycosis 495

Pt is a 62 y/o female presents with pain in her mid-back over the last three days. T 98.6, P:70bpm, RR14. PE is unremarkable other than tenderness in the mid thoracic spine. Labs normal except for Alkaline Phosphatase, which is elevated. ● Dx: Osteoporosis (compression fracture) ● Common location: vertebral location or hip. ● Why does this happen? post-menopausal women don't make as much estrogen. Decrease osteoprotegerin. RANKL and RANK receptor interactions cause more activation of Osteoclasts. Increased osteoclast activity! Gives rise to resorption of bone. Pt that recently bought a used dehumidifier at an antique store int he past 2-3 weeks. Pt. has had SOB, diarrhea, high fevers with Plt count at 30,000 and Sodium at 127. ● Dx: Legionella ● NBS: Urine antigen test ● Tx: Macrolides ● MOA of hyponatremia: thinking about interstitial nephritis. Acute Kidney Injury. Can't excrete potassium correctly. May have hyperkalemia. Like dialysis patients who miss their dialysis appointment. High potassium. ● Stain: Silver-stain (pcp is also silver stain, think HIV, tx with TMP-SMX) Pt. that has had bad epigastric pain that is worsened by meals. ● Dx: H. Pylori (silver stain + staining organism) Pt is a weight-lifter who over the last two weeks with numbness, weakness in right hand, forearm with barely perceptible radial and ulnar pulse. ● Dx: thoracic outlet syndrome ● MOA: (super hypertrophied muscles cause they're super fit. Compress subclavian or brachial plexus injury. Almost claudication of extremity) Pt. is a 71 y/o male who over the last three hours has had chest pain radiating to jaw. On way to hospital patient deceases. ● Dx: Arrhythmia (think VFIB) ----------------------------------------------------------------------------------------------------------------------------

496

Ep 248: [New Free 120 Q1-10 (2020)]

497

498

----------------------------------------------------------------------------------------------------------------------------

Ep 249: (Blood Oxygen Content and the USMLEs) ●

Formula for Oxygen content of blood ○ 1.34(mL) x Hemoglobin x SaO2 ■ SaO2 ● How much of Hgb is saturated with oxygen ■ paO2 ● Amount of oxygen dissolved in plasma ● Does NOT involve any oxygen attached to hemoglobin ■ pAO2 ● Amount of oxygen entering alveoli Pt. goes to higher elevation? ○ Amount of oxygen in the atmosphere? ■ Less oxygen present. ■ Oxygen tension goes down ■ Less atmospheric pressure ○ Proportion of oxygen at atmosphere ■ Remains the same (21%) ■ Same percent. ○ MOA of low paO2? ■ O2 goes to alveoli diffuses capillaries membranes and becomes paO2. The O2 dissolved in plasma saturates Hb with Oxygen. ■ Dec. pAO2 leads to paO2 and leads to SaO2. ● A chain reaction ○ Low pAO2 leads to low paO2 leads to SaO2. ■ Think about NBME arrow questions. SaO2 ○ Percent of hemoglobin saturated with oxygen ○ If paO2 then your SaO2 is also low. ○ What affects SaO2? ■ Heme contains iron ■ Oxidation # of iron ● Fe 2+ ○ Ferrous iron ● Fe 3+ ○ Ferric iron ○ No ability to bind oxygen (already maximally oxidized) ○ Heme with iron at 3+? ■ Methemoglobin ■ How to measure? ● Pulse oximeter 499

● ●

Pt. with headache and history of (car exhaust, space heater, house fire, uses a stove). Blood has a cherry red color. ○ Dx: carbon monoxide (CO) poison ○ Primary site of action of CO? ■ Hemoglobin (Doesn't do much in blood) ■ PAO2 and PaO2 is normal, but SaO2 will be decreased. ■ Shift on the oxyhemoglobin curve? ● Left-shift ○ How to treat poisoning? ■ 100% hyperbaric oxygen ○ Pt. with chronic exposure to CO with cogwheel rigidity etc with most common etiology? ■ Necrosis of the Globus pallidus ● Imaging? ○ Hyper-intensity of Globus Pallidus Pt took nitrate for angina, or sulfa drug (TMP-SMX) - PCP prophylaxis, TMP-SMX for toxo prophylaxis. Appears cyanotic. And blood appears “Chocolate color” ○ Dx: methemoglobin ■ Iron at 3+ = ferric form ● No ability to bind oxygen anymore ● Pathophys? Fe2+ in Hgb is converted to Fe3+. O2 can’t bind to Fe3+ ■ SaO2 is decreased. ○ Don’t expect to see headache ○ Tx: ■ Methylene blue ● Enzyme methemoglobin reductase. Converts Fe3+ to Fe2+ ■ Vitamin C When do we induce methemoglobinemia? ○ To treat cyanide poisoning ■ Cyanide inhibits ETC ● Complex IV ○ Prevents ETC from being functional ■ Depend more on anaerobic metabolism ● Increase conc. Of lactic acid. ■ Give Amyl Nitrate ● Fe 2+ -> 3+ ○ Thiosulfate ■ Thiocyanate ● Pee / poop out to excrete cyanide ■ Give hydroxocobalamin (B12) ● Combines with Cyanide and becomes safely excreted from the body Recap: Don’t confuse Carbon Monoxide, Methemoglobinemia, and Cyanide Poisoning Pt who is anemic. Are they hypoxemic? ○ Yes ○ When pt. has anemia there’s no problem with lungs, therefore: ■ PAO2 is fine ○ When pt has anemia there’s no problem with blood vessels, therefore: ■ PaO2 is fine 500

○ ○

When pt has anemia there is no problem with saturation, therefore: ■ SaO2 is fine Anemic patients have decreased hemoglobin ■ Decreased buses for oxygen ■ Decreased oxygen content is due to decreased Hemoglobin When oxygen content decreases in blood what happens to the heart? ■ To maintain more oxygenation heart increases cardiac output ● Heart eventually craps out. Can’t work at an elevated rate forever. ■ Leads to HIGH OUTPUT HEART FAILURE ● Mechanism of hydrops fetalis ○ Rh incompatibility. ○ Baby develops profound anemia. ■ Heart tries to keep up with increased Cardiac output ■ Baby develops heart failure, which leads to edema if heart stops working. ● Mechanism of Parvo ○ Parvo torches RBC precursors ■ Can’t produce RBC. ■ Develop anemia. ● May lead to high output cardiac failure ■ High output cardiac failure ● Speed of blood in vessels is elevated. ● Preggo with Rh antibodies and the baby is at risk. NBS? ○ Measure the velocity of MCA via Ultrasound. ○ Increased velocity = surrogate for anemia ■ Blood less viscous (less hemoglobin) Diffusibility of gas ■ Diffusibility of gas is related to the area available of diffusion divided by thickness of membrane. ■ Direct relation with spread of pressures between Point A to Point B ■ Diffusibility of gas is proportional to area available for gas diffusion and inversely proportional to thickness available for diffusion. ■ Pt with emphysema ● Has proteases that have chewed up alveolar surface area/parenchyme ○ Pt has decreased surface area then there's reduced diffusibility of gas, therefore: ■ There’s hypoxemia with increased A-a gradient ● PAO2 can’t match PaO2 ○ No alveolar membrane ○ Decreased area of diffusion which leads to hypoxemia ■ Pt with pulmonary fibrosis ● (Ankylosis?) ● Increased thickness of alveolar membrane ○ Inverse relationship means less diffusibility of gas ○ Distance between PAO2 and paO2 is larger. ■ Systemic scleroderma ● Pulmonary hypertension 501

■ ■ ■ ■

Pt has interstitial lung disease ■ Pulmonary fibrosis ● Increased thickening Pt with UTI who takes nitrofurantoin and pt develops chronic hypoxia? ● Nitrofurantoin induced pulmonary fibrosis Pt with Rheumatoid arthritis and pt develops chronic hypoxia? ● Methotrexate induced pulmonary fibrosis Pt on chemo and develops chronic hypoxia ● Bleomycin or Busulfan Pt with chronic rhythm control for AFIB develops chronic hypoxia ● Amiodarone

----------------------------------------------------------------------------------------------------------------------------

Ep 250: [HY Vaccine] ● Two big types of vaccines ○ Live attenuated ■ Bug that has been severely weakened, but retains ability to infect cells. ■ Generates a T-Cell response ■ No real need to give boosters ■ Ex ● MMR ● Varicella ● Intranasal influenza vaccine ○ Killed/Inactivated vaccine ■ Destroyed bug but you keep antigenic parts, which generates a response from immune system ■ Generates Humoral response ■ Need to give boosters ○ Toxoid vaccine ■ Toxin from bug that can act on receptors in the body to generate antibody response ● Pt is a newborn with a mom that has HepB surface antigens + ○ NBS ■ HepB vaccine AND HepB immunoglobulin ● Administered In opposite extremities ● Rotavirus vaccine is live attenuated (with a twist) ○ Live attenuated are not usually given to those < 1y/o ■ Rule does NOT apply to newborns. ■ Kids receive a vaccine at 2,4,6 months. ● Child that receives rotavirus vaccine. Which of the following is the most dangerous complication? ○ Intussusception 502

■ If pt has had it in the past intuss ■ Hx of meckel’s ■ IgA nephropathy (or HSP) What do these vaccines have in common: TDAP, HepB or PCV-13, polio, HepA, neisseria meningitidis? ○ Inactivated Two types of influenza ○ Inactivated ■ Intramuscular ○ Live-attenuated ■ Intranasal. Inhaled as mist PCV-13 and PPSV-23 ○ PCV-13 ■ Conjugated. Pneumococcal “C”onjugated ● B + T cell response ○ PPSV 23 ■ Polysaccharide ● Humoral response (No T cells) ○ PCV 13 always taken FIRST before 23 ■ Mnemonic aid: 13 comes before 23. ○ PCV13 is taken by everybody! ○ PPSV 23 ■ Given for those > 65 y/o ■ Those under 65 y/o that should have vaccine include (think of organs with chronic disease): ● Smoker ● Diabetes ● Cochlear implant ○ Older person on test. ○ Alport syndrome ● Chronic CSF Leak ● Immunodeficiency (HIV, Bruton, SCID, CVID) ● Those with no spleen Who does not get live attenuated vaccines? ○ Pregnant women ○ < 1 y/o (exception: rotavirus) ○ CD 3 after starting medication. ○ ACE inhibitor normally elevate Cr a little bit ■ Prevent conversion of angiotensin I to angiotensin II ● Angiotensin II is a potent constrictor of efferent arteriole ● Effect of medication? ○ Dilate efferent arteriole ■ Blood will not pool in glomerular capillaries ● Will drain away ■ Hydrostatic pressure goes down in glomerular capillaries. This does NOT favor filtration. ■ GFR goes down. Creatinine goes up. With Renal Artery Stenosis, the RAAS system is activated. Increased renin-->Angiotensin 2 constricts efferent arteriole, raising hydrostatic pressure in glomerular capillaries hence favoring filtration. Hydrostatic pressure favors fluids going out of the vessel (fluid extravasation). Oncotic pressure favors fluid staying in the vessel. If you take ACEi/ARB, the RAAS system is no longer activated and the opposite occurs. Blood is no longer flowing to afferent capillaries due to ischemia in the setting of Renal Artery stenosis. The addition of ACEi/ARB causes a dilation of efferent arteriole (instead of vasoconstriction as in RAAS activation in the setting of unilateral Renal Artery Stenosis). This decreases GFR, and increases Cr. ○ Summary: ACEi in the setting of unilateral RAS, dilates efferent arterioles, thus decreasing GFR, and ultimately increasing Cr. This is why ACEi/ARB are contraindicated in bilateral Renal Artery Stenosis. Kidneys have a portal system. Why does ACEi slow down the rate of diabetic nephropathy? In diabetic nephropathy, there is non enzymatic glycosylation of efferent which is the same as constricting the efferent arterioles, which leads to increased hydrostatic pressure, this increases GFR. GFR is increased (sounds like a good idea, analogous to working out (exercising), but if you overwork yourself (exercising 20 hours a day), your body will die because at some point your heart will give out), if GFR is chronically elevated, over time the capillaries will be injured (known as hyperfiltration injury). ○ If you give ACEi in the setting of diabetes, efferent arterioles will be dilated, decreased GFR, decreasing hydrostatic pressure and intraglomerular hypertension. BPH patient: If you have obstruction out of the uterers, this create a backup of fluid, increasing hydrostatic pressure surrounding the glomerular capillaries, hence more fluids will go into the glomerular capillaries, will not favor filtration, Creatine will not rise. Hydrostatic pressure always favors fluid movement out of a compartment. Oncotic pressure favors keeping fluids within a compartment. Calcium Channel Blockers (ex: amlodipine): Lowers BP. Amlodipine is a direct arteriolar dilator. This will send more fluids to the capillaries, increasing hydrostatic pressure, causing extravasation of fluid. Hence the peripheral edema. ○ How do you treat the peripheral edema? Lower hydrostatic pressure via ACEi/ARBs ■ What drug can dilate post capillary venules (efferent arterioles)? ACEi/ARBs In ARDS (as in the setting of COVID-19): cytokines increase vascular permeability, causing more fluid to drain into the interstitium of lungs, impairing gas exchange leading to non-cardiogenic pulmonary edema. PCWP18. Nephrotic syndrome: urinating albumin (excretion), decreased oncotic pressure (cannot keep fluids within compartment), resulting in extravasation aka edema/periorbital edema/anasarca. ○ Liver disease similar pathway: cirrhosis ○ Menetrier disease: protein losing gastropathy aka nephrohitic disease of the stomach, pooping out albumin ○ Protein Malnutrition: Kwashiorkor.

----------------------------------------------------------------------------------------------------------------------------

Ep 255 [Legendary Step 2CK/3 Pharmacology] ●

● ●

Blood drugs ○ antiplatelet/ anticoagulants ○ Anti-platelet ■ Who? ● Stroke or carotid stenosis (TIA) ○ 1st line drug? ■ Aspirin ○ 2nd drug ■ Clopidogrel ○ 3rd drug ■ Dipyridamole ● Peripheral Arterial disease ○ Aspirin for life ● Percutaneous coronary intervention (MI) ○ Dual antiplatelet agents for a few months after the stent is placed ● Acute MI / Chest pain ○ 1st line drug? ■ Aspirin ● Kawasaki ○ Aspirin + IVIG ● Preeclampsia in previous pregnancy and prevent in future pregnancy ○ Aspirin prophylaxis Woman with cervical insufficiency. How to prevent insufficiency in future pregnancy? ○ Progestin suppository Primary Hemostasis ○ Adhesion step ■ GP1B - VWF binding ● Begin to eject ADP from platelet ○ ADP binds to ADP receptor (P2Y12) ■ GPCR triggers signaling cascade ○ Aggregation step 513

■ GpIIb/IIIa binds to other GpIIb/IIIa ○ Formation of platelet plug What does aspirin do? ○ Irreversible COX-1 and COX-2 inhibitor ■ Can’t make TXA2 (helps with platelet aggregation) so you can’t aggregate GpIIb/IIIa inhibitors ○ Abciximab, eptifinabie, terafibine ■ Prevent GpIIb/IIIa from binding to another GpIIb/IIIa can’t form platelet plug ADP receptor antagonists (P2Y12 receptor blockers) ○ clopidogrel , ■ Block receptor ADP won’t bind to it and can’t activate the receptor. PD3 inhibitors ○ Cilostazol, dipyridamole ○ Increase cAMP ■ Increase cAMP in platelet INHIBIT signaling cascade downstream of p2y2 receptor ■ P2Y2 is inhibitory GPCR. less camp = more platelet activation. So if you inhibit PD3 leads to more cAMP and decreased platelet activation. ○ When cAMP is high it causes smooth muscle? ■ To Relax! ● Vasodilation ○ Cilostazol and NBME? ■ Give after trying a supervised exercise program in patients with Peripheral Arterial Disease. ○ Dipyridamole ■ Pharmacological stress test ■ Boosts levels of adenosine ● Adenosine deaminase inhibitor ○ Causes vasodilation ● Theophylline antagonizes Adenosine

■ ~12 mins Warfarin ■ Prosthetic Heart Valve indication ● Target INR 2.5 - 3.5 ● INR 6 and 7 ○ Stop the Warfarin, give oral vitamin K ○ Bleeding Out? Reverse with 4FPCC ○ Reverse Heparin with Protamine Sulfate ■ A. Fib from valvular cause i.e. MS cause ■ Risks ● Don’t give to pregnant woman ● Duodenal hematoma (SBO), rectus sheath hematoma, intracranial bleed ● Don't’ give tPA with warfarin (they’ll bleed out and die) ■

Inhibits Vitamin K Epoxide Reductase ● Can’t gamma carboxylate 2,7,9, 10, Protein C, S ● Protein C and S are anticoagulation ● 2,7,9,10 are coagulants 514

Bridge with Heparin First! ● Activates antithrombin III ○ Can inhibit factors 10 and 2 ● Protein C and S have a shorter half life ○ Fall out of circulation first ○ 2,7,9,10 are left for temporary coagulation ○ So there is a temporary hypercoagulable state in patients with Warfarin, that’s why they need that heparin bridge ● Low Molecular Weight Heparin ○ Also activates antithrombin III BUT only inhibits Factor 10 ○ Factor 10 Inhibitors ■ All have Xa in their name ○ Factor 2 inhibitors ■ Argatroban, Dabigatron, Bivalirudin ■ Pt. presentation ● Platelets drop post giving heparin ○ Stop Heparin, don’t give LMWH ○ Start them on Factor 2 inhibitors ○ Dabigatran is reversible by Idarucizumab Cardio Drugs ○ ACE inhibitors (-pril) ■ Site of action → pulmonary capillaries, endothelial cells ■ Used in hypertensive diabetic patients ■ Improves survival in HF patients (also beta-blockers and potassium sparing diuretics) ■ Contraindicated in patients with bilateral renal artery stenosis (Significant rise in creatinine after ACE inhibitor → renal angiography) ■ Hyperkalemia counteracts the Hypokalemia caused by loop diuretics ■ Scleroderma renal crisis (steroids are never the right answer for scleroderma) ■ Teratogenic → renal dysgenesis, oligohydramnios ○ Cardiac resynchronization therapy ■ EF < 35% + maximal medical therapy ■ Can improve survival ○ Spironolactone ■ Treat Hirsutism associated with PCOS ■ Used to decrease portal HTN in ascites along with Furosemide for chronic use ■ Conn syndrome, bilateral adrenal hyperplasia ■ Improves survival in HF ○ Beta-blockers ■ HCOM, improved survival in HF, migraines, akathisia ■ Initial DOC for thyroid storm is Propanol (peripheral conversion of Thyroid) ■ Labetalol (mixed alpha and beta blocker) ■ Hypertensive moms love Nifedipine ● H = Hydraline, M= alpha methyldopa, L = Labetalol, N = Nifedipine ● Pregnant anti-HTN drugs ■ Don’t give to patients with acute cocaine intoxication ■ Can be used to treat glaucoma (Timolol, etc) ■ Overdose → Atropine then Glucagon 515

○ ○

○ ○ ○ ○

■ Class II antiarrhythmic Alpha-1 blockers ■ Prazosin - PTSD for nightmares (first-line is SSRI and psychotherapy) ■ Useful for BPH ■ Side effect → orthostatic hypotension ■ Tamsulosin (Blocks specifically alpha-1 receptors in bladder) Alpha-2 blockers: Mirtazapine -> used for depression Alpha-2 agonists : Clonidine (leads to rebound HTN) ■ Third-line for ADHD ■ Tourette’s ■ Can be used in opioid withdrawal Loop diuretics ■ Peripheral edema, nephrotic syndrome, CHF ■ Loops lose Calcium → hypocalcemia, hypercalciuria (risk factor for nephrolithiasis) ■ NKCl(x2) transporter in thick ascending loop of Henle ■ Ototoxic (other ones are Cisplatin, Vanc, Aminoglycosides - Gentamicin) ■ Taking these is like having Bartter Syndrome Thiazide ■ Hypercalcemia with HYPOcalciuria ■ Stronger association with hyponatremia ■ Metolazone is an example ■ Taking these is like having Gitelman Syndrome Amiodarone ■ V Tach ■ Class III - K channel blocker Antiarrhythmics ■ Procainamide - Class Ia, used for WPW, can cause drug-induced lupus Verapamil → can cause hyperprolactinemia Nimodipine → post-stroke vasospasm Digoxin ■ Na-K ATPase blocker ■ Normal job - 3 NA leaves cell, 2 K in → hyperkalemia side effect ■ If patient is hypokalemic, more susceptible to Digoxin toxicity ■ Abdominal pain, yellow vision, diarrhea, etc ■ Contraindicated in WPW Causes of dilated cardiomyopathy ■ Trastuzumab ■ Doxorubicin ■ Clozapine → Myocarditis

----------------------------------------------------------------------------------------------------------------------------

Ep 256 [Rapid Review Series 41] 516

African kid, 8 years old, big jaw mass- think Burkitt’s lymphoma. 8:14 translocation- will ultimately lead to amplification and leads to cells continually proliferating. Classically on histology, see starry sky pattern. EBV linked to this EBV is associated with burkitt’s Lymphoma, Hodgkin lymphoma with reed-sternberg cells (lymphocytes with bilobed nucleus), nasopharyngeal lymphoma, primary CNS lymphoma if pt has HIV. Pt works in a barn with nuts, produce, grains- over the last 6 months, they have lost 20 pounds, and have jaundice. On labs- LFTs are high. RUQ- see liver mass. } think HCC. grains have aflatoxins, these are toxic to the liver. Pt over the past 3-4 weeks has joint pain in knees, hips, fingers. See digital clubbing. NBS- CXR - this person has lung cancer, they have hypertrophic pulmonary osteoarthropathy, especially in adenocarcinoma. Small cell- see SIADH (see hyponatremia, hypertonic urine, sp gr > 1.012), ectopic ACTH (high dose dexamethasone will not suppress the next morning’s cortisol), lambert eaton myasthenic syndrome (decreased DTR, proximal muscle weakness, repetitive neurostimulation causes an incremental response. Doesn’t affect the head and neck, more appendicular skeleton than axial skeleton). Txchemo Lung cancer- also can be associated with poly/dermatomyositis. Squamous cell lung cancer- famously cavitates. See air fluid levels in the lungs, abnormal of other lung cancers (especially small cell). Produce of PTHRP-> hypercalcemia, #1 cause of hypercalcemia in the hospital = malignancy Tumor markers - +keratin = squamous cell cancer - +chromogranin = tumor that is endocrine in origin. Common in small cell lung cancer. - +GFEP = + in astrocytoma, glioblastoma multiforme - +desmin - this is a muscle marker. Malignancy is arising from muscle, i.e. tuberous sclerosisrhabdomyosarcoma. - +vimentin - in leiomyosarcomas - +S100 = neural crest derived - melanoma, langerhans cell histiocytoma, schwannoma - +AFP- yolk sac/endodermal sinus tumor, HCC - +Bhcg- choriocarcinoma - +PSA - prostate cancer, more to monitor response to therapy - +calcitonin- medullary thyroid cancer - +CEA- colon cancer, pancreatic - CA 19-9 - pancreatic cancer - CA125- ovarian

517

Person with hyperandrogenism, + adnexal mass- sertoli leydig cell tumor. These are + testosterone. Person with hyperandrogenism, normal testosterone levels- NBS-> get CT abdomen/pelvis to see possible adrenal tumor, would be making DHEAS Granulosa cell tumors- the call exner bodies on histology produce estrogen. See precocious puberty in a kid, especially girls. In a postmenopausal woman, you could see endometrial cancer due to unopposed estrogen (the #1 risk factor for endometrial cancer- unopposed estrogen) Choriocarcinoma- see in a woman who recently had a baby, has hyperemesis gravidarum. Responsive to MTX. very commonly metastasizes to the lung. +Bhcg Pt for the last 3 months has really bad morning headaches, get better throughout the brain. Brain imaging- calcification around vermis- think hemangioblastoma. Can produce EPO-> polycythemia. Calcified masses - Suprasellar, in a child, with bitemporal hemianopsia- Craniopharyngioma, derived from rathke’s pouch (oral ectoderm) - In the frontal lobe- oligodendroglioma. Fried egg appearance on histology. - Adnexal mass, or in anterior mediastinum- think dermoid cyst/teratoma. - Kid hypercalcemia Pt with adnexal mass, lost 15 pounds in the past 3 months, tachy, increased DTR, irregularly irregular on EKG-> think hyperthyroidism. This is struma ovarii- a teratoma with ectopic thyroid tissue. They have afib- most common arrhythmia in hyperthyroidism (but most common risk factor for afib is mitral stenosis) Person with neck mass, prolonged QT - this is hypocalcemia, commonly from calcitonin/medullary thyroid cancer Alk Phos elevation- either from bone or liver process. Check GGT- if elevated, this is a liver thing, i.e. cholestatic process. If normal GGT, it's a bone problem- bone mets, osteoporosis, and paget's disease of the bone. Also can be caused by placental alk phos, i.e., in seminoma, pregnancy. ----------------------------------------------------------------------------------------------------------------------------

518

Ep 257 [Clutch Bilirubin Podcast] Bilirubin Metabolism 1. Bilirubin breaks down from RBC to form hemoglobin (heme + globin) 2. heme + heme oxygenase converted to biliverdin 3. Biliverdin uses biliverdin convertase to convert previous step to indirect bilirubin 4. Indirect bilirubin is not water soluble so it needs to be carried by albumin. 5. Goes to the liver. 6. UDP glucuronyltransferase (UDPGT) → takes indirect bili → conjugates it → To bile canaliculi → intrahepatic ducts → extrahepatic bile ducts → squirt out of sphincter of oddi → helps to emulsify fat etc and some is reabsorbed in the terminal ileum.

Hyperbilirubinemia ● Unconjugated/indirect ○ anything going wrong prior to UDPGT - UDPGT is an enzyme, can be saturated and lead to indirect bili build up ○ Hx of lupus, Hgb 8, total bilirubin 4, indirect bilirubin 3.5 → indirect bilirubin → Hemolytic anemia, i.e. autoimmune hemolytic anemia (a type 2 hypersensitivity reaction) - a.b. against own RBCs ○ Young man has hematuria every morning, arterial and venous thrombosis in the past = paroxysmal nocturnal hemoglobinuria. ■ PIGA mutation- can't make PGI anchors. Can’t put CD55 or CD59 on the surface of RBC- these usually help the cell protect itself from complement mediated damage. Without it, the cells tend to hemolyze → UDPGT is overwhelmed with amount of hemolysis ○ Sickle cell: these RBC hemolyze easily ○ Thalassemia: these RBC don’t last very long because the Hgb is abnormal → splenic macrophages destroys them more often ○ HIV pt: CD4 count is 150, start treating with TMP SMX and weeks after, gets fatigued. See labs: Hgb of 8, coombs test is negative, see indirect hemoglobinuria- this is G6PD deficiency. TMP SMX is a very powerful oxidant and without G6PD, you cannot make 519

○ ○ ○

○ ○ ○

NADPH and you have no reducing power (via pentose monophosphate shunt/pentose phosphate pathway). ■ X linked recessive ■ **women will NOT get G6PD deficiency** European kid with anemia, grandfather had a splenectomy- think hereditary spherocytosis. Problems with spectrin or ankyrin so don’t have enough RBC membrane. When more fluid gets into them, they explode = hemolysis. Also, splenic macrophages destroy them. “Spleen is quality control factory” lol UDPGT enzyme deficiency, i.e. crigler najjar Person with recovering acute illness/undergoes surgery, asymptomatic, see mild jaundice/ indirect hyperbilirubinemia → think Gilbert syndrome. Whenever they undergo stress, their small amount of UDPGT can get overwhelmed. physiologic jaundice: newborn not using lungs → relative hypoxia in utero → increased EPO → very high hematocrit at birth (57%) → higher volume of RBC. ■ When baby is born, they have more oxygen → dont need as many RBC, trying to get rid of fetal hemoglobin. All these RBC need to be removed, and the UDPGT the baby has is not well developed yet anyway → bilirubinemia Rh/ABO incompatibility- these RBC are demolished by antibodies Newborn in the first week of life, mom is feeding the baby every 6 hours → this is breast feeding jaundice Second week of life- breast milk jaundice. Pathophys isnt well known, but breast milk might have a substance that inhibits UDPGT!

● conjugated/direct ○ think obstruction of bile flow ○ Newborn- biliary atresia. Fibrosis of bile ducts-> inability to excrete bile. Do a Kasai procedure in the first few days of life. If you have had biliary atresia for 4+ weeks, baby will need a liver transplant. ■ Ddx- choledochocyst. Choose this if the answer choices do not contain biliary atresia ○ Kid ■ Asymptomatic- may have dubin johnson OR rotor syndrome. Differentiate with core biopsy of the liver. ■ Black = dubin johnson. ■ Not black = rotor syndrome. ■ This is just a mutation of a transporter (MRP1) that moves bilirubin from the bile canaliculi to the biliary tree. ○ Middle aged woman with pruritis, jaundice = primary biliary cholangitis/cirrhosis. Intrahepatic bile duct is affected. See antimitochondrial antibodies. Tx: ursodiol, ursodeoxycholic acid.

520

○ Middle aged male with hx UC = think primary sclerosing cholangitis- see beads on a string (strictures) in intra AND extrahepatic bile ducts. P-ANCA positive. Could try doing an ERCp to lyse the strictures. ○ Acute jaundice, mild RUQ pain, no fever = think choledocholithiasis. ■ If fever+ = is Ascending Cholangitis. Most cholangitis will be caused by untreated choledocholithiasis. ○ Recently had cholecystectomy, signs of peritonitis- think of bile leak peritonitis- bile is very irritating to the peritoneum. Dx- HIDA scan is very good. ■ Differentiating bile peritonitis vs retained gallstone- if stone, don't see signs of peritonitis. ○ Pancreatic cancer- may see double duct sign on u/s = see dilation of common bile duct and pancreatic duct because of pancreas obstructing flow. ○ Person with chronic cholecystitis/penetrating abdominal injury, the blood has very high bile content- this person has a biliary vascular fistula. (btwn bile duct and blood vessel) ■ Blood in the biliary tree- think biliary vascular fistula as well ○ Cholangiocarcinoma ■ Cause of direct bilirubin ○ Cholestatic pattern of liver labs ■ Direct bili is elevated ■ Alk phos is markedly elevated in relation to AST/ALT ● Hepatic pattern of liver labs ○ AST/ALT is disproportionately elevated compared to Alk Phos ● “Alk phos conundrum” ○ Alk phos elevated but GGT is normal- this is a bone problem causing the alk phos elevation. ○ To be a biliary etiology, GGT is typically elevated as well ○ Newish test - 5’ nucleotidase test - if this is elevated, it will say if alk phos is a liver problem or not Cross check: YES

----------------------------------------------------------------------------------------------------------------------------

Ep 258 [Rapid Review Series 42] ● 3 y/o or 23 y/o F or 56 y/o M (in other words any age) w. Fever 102. ○ Dx? Epiglottitis ■ Unvaccinated? H. flu ■ Vaccinated? Strep pneumo (probably) ○ NBS? (in order of importance) 521

■ Endotracheal intubation ■ Medication ● Third generation cephalosporin ○ Ceftriaxone ○ Cefotaxime ■ Lateral neck radiograph ● Thumb sign ● Pt just returned from deployment in some country. Person has had chronic cough, hemoptysis, lost 12 lbs since the onset of sxs. ○ Dx? Tuberculosis ○ How to diagnose? ■ Bronchoalveolar lavage ■ Induced sputum w. Acid-Fast Bacilli state (w. culture) ○ Treatment regimen ■ RIPE + Vit B6 ● Rifampin ● Isoniazid ○ ADE ■ Peripheral neuropathy (if B6 not taken) ■ Seizures (if B6 not taken concurrently) ● Glutamate to GABA pathway ○ Glutamate decarboxylase uses B6 as a cofactor ■ Hepatotoxic (if B6 not taken) ● Also use B6 cofactor for enzymes ■ Anti-Histone Ab Drug-Induced Lupus ■ Can also cause microcytic, Sideroblastic anemia (if not taken with B6 concurrently) ● Due to B6 deficiency ● Synthesis of Heme first step involves glycine and succinyl CoA with ALAS, which uses Vit B6 as a cofactor ● Pyrazinamide ● Ethambutol ● Acid Fast organisms : ○ HIV-patient with Diarrhea. Acid Fast oocyst in patient stools? ■ Dx? Cryptosporidium parvum ■ Tx? Paromomycin or Nitazoxanide ○ Pt with Nocardia? Also has Acid-Fast ○ Tuberculosis induration cutoff ■ 5 mm in person with HIV ■ 10 mm in healthcare workers ■ 15 mm is positive in everyone. 522

● Pt. went to New Hampshire, New York, Connecticut, Boston on a field trick. Person has joint pain and has not noticed a tick bite. (Doesn’t need to) ○ Dx? Lyme disease ○ Organism? Borrelia Burgdorferi ■ What are they? Spirochetes. ○ How to Dx? ■ ELIZA test + Western Blot (to confirm) ○ Treatment ■ 8 y/o ● Doxycycline ○ C/I in pregnant patients. ■ Preggos get amoxicillin ● Assoc of spirochetes in syphilis and lyme disease and leptospirosis? ○ Jarisch–Herxheimer reaction ● Pt has classic erythema chronicum migrans? ○ Dx? Clinical ○ Tx? Doxycycline ● If a patient has Lyme disease that affects the brain? ○ D/C doxycycline ○ Give: ceftriaxone or cefotaxime ● If pt. Has Ascites for any reasons (budd chiari) w. Abdominal pain, AMS. ○ Dx? SBP ■ Translocation of bacteria ○ How to diagnose? ■ Paracentesis ● > 250 mm neutrophil ○ Treatment ■ Ceftriaxone or cefotaxime ● Cefazolin is an ab given 30 mins before incision in surgery is made. ○ If surgery hour/time is changed and ab already given? ■ Give antibiotic AGAIN 30-60 mins before sx ● 22 F for the past two weeks sore throat, joint pain. Show you a picture and you see a big inclusion inside the cell (classic picture u want to recognize) - a big intranuclear inclusion. ○ Dx? CMV ○ Test? ■ Monospot is negative ○ Treatment ■ Ganciclovir ● If resistant to Ganciclovir ○ Tx: Foscarnet ● Kid with congenital CMV? 523

● ● ● ● ● ● ●

● ● ●

○ Microcephaly and periventricular calcification Kid with congenital Toxo ○ Calcification diffuse across cortex Calcifications at frontal lobe - fried egg appearance. ○ Dx? Oligodendroglioma Peds with suprasellar mass w. Bitemporal hemianopsia w. Calcifications of mass ○ Dx.? Craniopharyngioma Calcifications at posterior fossa - Cerebellum w. High Hematocrit? ○ Dx? Hemangioblastoma Calcification around heart ○ Constrictive pericarditis Calcification around pancreas ○ Chronic pancreatitis Stippled, punctate calcifications around the gallbladder? ○ Dx? Porcelain gallbladder ■ Pt may have cholangiocarcinoma or gallbladder cancer ■ TX: ● Elective cholecystectomy Calcified mass at pouch of Douglas, Adnexal mass. ○ Dx? Dermoid Cyst (teratoma) Calcifications around Kidney, HTN ○ Dx? Primary hyperparathyroidism Pt with diabetes and PAD. ○ ABI w. 1.5 (better than good) = PAD ■ Blood vessels have been calcified ■ Dx? Monckerberg calcific sclerosis (Diabetics) ● ABI is “better than good” ○ NBS? ■ Toe-brachial index

----------------------------------------------------------------------------------------------------------------------------

Ep 259 [Clutch Incontinence Podcast] Physiology ● store urine in the bladder which is connected to the urethra that allows the urine to leave ● Continence ○ balance between bladder contracting and urethra closing. ○ When you want to urinate, the bladder contracts via the detrusor muscles. This forces urine out of the bladder and into the urethra → the urethra sphincter muscles (ring of muscle around urethra) have not relaxed yet. When it relaxes, the urine can be excreted. ○ The pressure in the urethra will normally exceed the pressure in the bladder- this keeps us continent. - (THIS is the main takeaway) 524

■ Urethral sphincter is supported by pelvic floor muscles, i.e. levator ani. These help it close as well as it should. Stress incontinence ● Presentation: any increase in intra abdominal pressure (cough/sneeze/valsalva) → pee on themselves ● Cause: something has caused the urethra to lose support. ○ Pregnancy ○ Obesity ○ Menopause- because estrogen was important in these pelvic floor muscles to keep their tone/keep them strong. Decrease in estrogen = increased floppiness ○ In a guy- one who has had prostate surgery, i.e. TERP, removal of prostate, radiation therapy, brachytherapy. ■ May resolve within months ○ So, without support, bladder pressure is unbalanced with urethral pressure, and you will be incontinent (in terms of the main takeaway****) ● Dx: q tip test → 30 degrees of rotation ● Tx: ○ Kegels → will strengthen pelvic floor muscles ○ Mid urethral sling procedure Urge incontinence ● Most common type of incontinence in men (caused by BPH) ● Pathophysiology ○ Detrusor muscles are too excited (hypertonia/hypermobility) → causes overactive bladder. Person has a constant urge to pee. ○ Post void residual volume progress to invasive cancer - any SCC can present with PTH-rp paraneoplastic syndrome = hypercalcemia Cryptorchidism = testis didn't descend (androgen causes descent in 3rd trimester) - seen in androgen insensitivity syndrome - If testis kept in pelvic cavity = cant maintain blood body temp = cant make sperm (but scrotum has

Divine Intervention Step 2CK Podcasts Notes - Read Only File - PDFCOFFEE.COM (2024)
Top Articles
Latest Posts
Article information

Author: Twana Towne Ret

Last Updated:

Views: 6283

Rating: 4.3 / 5 (64 voted)

Reviews: 95% of readers found this page helpful

Author information

Name: Twana Towne Ret

Birthday: 1994-03-19

Address: Apt. 990 97439 Corwin Motorway, Port Eliseoburgh, NM 99144-2618

Phone: +5958753152963

Job: National Specialist

Hobby: Kayaking, Photography, Skydiving, Embroidery, Leather crafting, Orienteering, Cooking

Introduction: My name is Twana Towne Ret, I am a famous, talented, joyous, perfect, powerful, inquisitive, lovely person who loves writing and wants to share my knowledge and understanding with you.